You are on page 1of 119

KROK 2 – Question Bank 1

1.A total of 300 newly diagnosed patients with year-olds were reported to be 9.1 kg and 10.2 kg,
laryngeal cancer are allocated to treatment with and the difference was reported to be "statistically
either surgical excision alone or surgical excision significant (P = 0.02) by a chi-squared test".
plus radiation treatment. Select the study design A. * The wrong kind of statistical test was used.
which most appropriately illustrates. B. There is good evidence that the two groups are
A. * Clinical trial different.
B. Case series report C. One should not compare weights using means;
C. Case report one should always use modes.
D. Case-control study D. One should not compare weights using means;
E. Cohort study one should always use medians.
2.A 39-year-old man with a mild sore throat, fever, E. The confidence interval for the difference in
malaise, and headache. He was treated with means includes zero.
penicillin for presumed streptococcal infection. He 7.Data are available giving the percentage
returns after a week with hypotension, fever, rash, breakdown of a sample of residents of an urban
and abdominal pain. He responds favorably to area into five ethnic groups. These five
chloramphenicol, after a diagnosis of Rocky percentages can be shown graphically in a:
Mountain spotted fever. Select the study design it A. * Pie chart
most appropriately illustrates. B. Histogram
A. * Case report C. Scatter diagram
B. Case series report D. Cumulative frequency plot
C. Clinical trial E. Bar diagram
D. Case-control study 8.Uncontrolled hypertension doesn’t increase the
E. Cohort study risk of developing the following diseases:
3.A total of 3500 patients with thyroid cancer are A. *Diabetes
identified and surveyed by patient interviews B. Cerebrovascular diseases
regarding past exposure to radiation. Select the C. Coronary heart disease
study design it most appropriately illustrates. D. Renal disease
A. * Case series report E. Stroke
B. Case report 9.Select the risk factor most likely to be associated
C. Clinical trial with breast cancer:
D. Case-control study A. * Family history
E. Cohort study B. Dietary habits
4.A total of 10,000 Vietnam veterans, half of whom C. Tobacco smoking
are known by combat records to have been in D. High serum cholesterol
areas where Agent Orange was used and half of E. Uncontrolled elevation of blood pressure
whom are known to have been in areas where no 10.Risk factors for lung cancer include all of the
Agent Orange was used, are asked to give a following EXCEPT:
history of cancer since discharge. Select the study A. * Family history
design it most appropriately illustrates. B. Smoking
A. * Cohort study C. Occupational exposure
B. Case series report D. Air pollution
C. Clinical trial E. Radiation
D. Case-control study 11.A woman whose ectopic pregnancy has ruptured
E. Case report at 18 weeks’ gestation hemorrhages internally,
5.Patients admitted for carcinoma of the stomach are goes into shock, and eventually dies. Select the
age- and sex-matched with fellow patients without classification that most closely represents the case
a diagnosis of cancer and surveyed as to smoking history.
history to assess the possible association of A. *Direct maternal mortality
smoking and gastric cancer. Select the study B. Perinatal mortality
design it most appropriately illustrates. C. Neonatal mortality
A. * Case-control study D. Postneonatal mortality
B. Case series report E. Infant mortality
C. Clinical trial 12.The mean systolic blood pressure in a random
D. Case report sample of 180 men is 141.4 mm of mercury, and
E. Cohort study the 95\% confidence interval is from 139.2 to
6.In the report of a survey of children in a rural 143.6.
community, the mean weights of two groups of 2
KROK 2 – Question Bank 2
A. * We have 95\% confidence that the true = 5.7, P < 0.001). The correlation coefficient is
(population) value lies in this interval. 0.72. We can deduce,
B. The true value is equally likely to be 139.2, A. *The average increment of blood pressure for
141.4 or 143.6. one year of age under the regression model is
C. Everyone makes mistakes in measuring data 1.22 mm.
and carrying out calculations; the confidence B. The mean systolic blood pressure at birth is
interval is to allow this. 81.5 mm.
D. 95\% of population means lie in the interval. C. The blood pressures of two women ten years
E. The confidence interval shows that the mean is apart in age will differ by exactly 12.2 mm.
statistically significant. D. Because the relation seemed linear, regression
13.In a nutrition survey of children in a rural area in a is not an appropriate technique.
developing country, the mean standardised E. One cannot draw any conclusions from such a
weights for age were compared between boys and study
girls and a statistical significance test carried out. 17.In a sample of 38 medical students, the correlation
The test result was P = 0.23. coefficient of the course work marks and final
A. * There is no evidence of a true difference examination marks for pathology was 0.74. The
between boys and girls. coefficient for course and final marks in a
B. There is strong evidence of a true difference different sample of 31 students, in surgery was
between boys and girls. 0.05. From these results we can deduce,
C. P is quite large, so the null hypothesis is true. A. *The association appears stronger for
D. P is not large enough to prove the null pathology than for surgery.
hypothesis as true. B. Surgery is more difficult to learn than
E. P is quite large, so there is probably a true pathology.
difference between boys and girls. C. Pathology is more difficult to learn than
14.In a study to investigate the association of a surgery.
certain alcoholic drink and oesophageal cancer, a D. Course work tests and the final examination
series of cases were interviewed to find out how measure different aspects of knowledge in
many reported regular consumption. A random surgery.
sample of controls from the same community were E. The wrong kind of statistical test was used.
similarly interviewed. The proportions reporting 18.All of the following are criteria for screening a
consumption were higher in cases than in controls, population for a given condition EXCEPT:
with P = 0.04. A. *Screening should be available to and used by
A. * Age and/or sex must be adjusted for before the entire population
any conclusions can be drawn B. The natural history of the condition should be
B. There is strong evidence of a real association. understood
C. There is no evidence of any association. C. The condition must represent an important
D. The wrong kind of statistical test was used. health problem
E. One cannot draw any conclusions from such a D. Suitable screening tests for the condition must
study. exist
15.In a sample of 66 adult women, the mean reported E. Accepted treatment for the condition should be
age of menarche was 12.4 years and the median available
12.7 years. Also, the standard deviation was 10 19.The DRG (diagnosis-related groups) payment
years and the range from 10 to 15 years. system is best described by which payment
A. * The distribution is approximately symmetric methodologies?
B. 75\% of women had reached menarche after A. * Per admission
12.4 years. B. Per diem
C. One is 95\% confident that the true mean lies C. Capitation
between 10.4 and 14.4 years D. Fee-for-service
D. The standard error is 0.3 years E. Risk corridor
E. One cannot draw any conclusions from such a 20.Select the term that best matches the definition
study. “Death of liveborn infant before 1 year of age”:
16.Data from a sample of 33 females from a small A. *Infant mortality
village aged from 22 upwards, gave systolic blood B. Perinatal mortality
pressures and ages. The relationship between the C. Neonatal mortality
two appeared substantially linear in a scatter D. Spontaneous abortion
diagram and the fitted regression equation is BP = E. Postneonatal mortality
81.5 + 1.22 x age, and is statistically significant (t
KROK 2 – Question Bank 3
21.The work in conditions of electromagnetic waves C. Centralized-decentralized
influence can cause the dynamic abnormalities of D. Mixed
certain body systems. Which systems are the most E. Centralized-sectional
sensitive to the action of electromagnetic waves of 27.In drinking water samples selected after
radio-frequency diapason? purification and disinfected by gaseous chlorine
A. * Nervous and cardiovascular systems the following was revealed: chloroform and
B. Digestive and respiratory systems threechloracetic acid in concentration three times
C. Cardiovascular and respiratory systems more than their MAC. What disease may probably
D. Cardiovascular and musculoskeletal systems develop as a result of prolonged intake of this
E. Nervous and digestive systems water?
22.The man with body-mass of 60 kg intakes 600 mg A. *Stomach cancer
nitrates with daily food ration. How many times B. Urolithiasis
does the real intake of nitrates exceed permissible? C. Hypertensive disease
A. * 2 times D. Stroke
B. 5 times E. Anemia
C. 10 times 28.In drinking water samples the following
D. 60 times substances were revealed: copper, manganese,
E. 100 times iron, lead, chlorphenol on the level of their MAC.
23.In the dull December days (calm and mist) the Which chemical substances mentioned above are
respiratory system diseases and diseases limited for sanitary-toxycological index of
accompanied by signs of general intoxication in harmfulness?
the region where thermoelectric power station A. *Lead
works on the solid fuel increased. The mortality B. Iron
among persons over 60 also increased. Which C. Manganese
factor may probably cause the toxic effects? D. Copper
A. *Suspended substances (dust) E. Chlorphenol
B. Lead 29.In blacksmith’s-press’s shop physical work
C. Low temperature connected with unfavourable meteoconditions (air
D. Air humidity to- 40-50oC, intensive infra-red radiation), one
E. Photooxidants worker lost consciousness. Doctor of medical
24.The abnormalities of bone system development, room testified the following: face pallor, wet skin,
the delay of fontanelle over-growth, the delay of intensive perspiration, surface respiration with BR
teeth were observed among children of N-city. of 50 /min, HR of 100/min, delicate filling, body
Which surplus micro-element contained in the to –39,9oC. Which diagnosis is the most possible?
drinking water can cause these changes? A. * Heat stroke
A. *Strontium B. Sun stroke
B. Calcium C. Convulsion disease
C. Cadmium D. Hypertensive disease
D. Arsenic E. Angioneurosis
E. Magnesium 30.In the air-repair work planes fuselage repair is
25.The fulfilment of sanitary-hygienic control of air carried out. The levels of industrial noise on the
medium of hospital wards in somatic department working places constitute 95-97 dBA. The
of town clinic hospital the air samples were taken analysis of noise spectrum showed tones with
by aspiratory method. Which MAC is used for frequency of 100 Hz and higher. Which apparatus
corporation in evaluation of hygienic conditions of was used for the experiment?
patients staying in the hospital? A. *Noisemeter
A. *MAC of atmospheric air B. Audiotester
B. MAC of working zone C. Actynometer
C. Average monthly MAC D. Vibrotester
D. Average annual MAC E. Anemometer
E. Average MAC per working shift 31.What property to typically obligatory medical
26.All departments and subdivisions of a district insurance?
hospital having 300 beds are disposed in the A. *General
separate one- and two-storey buildings. Which is B. System of control of quality of medical
the type of hospital construction? services is carried out by subjects of insurance.
A. * Decentralized C. Guarantor of the payment of expenses - the
B. Centralized commercial organizations.
KROK 2 – Question Bank 4
D. The medical program is determined by the E. The guaranteed volume of the medical and
contract insurer and insured. psychological help
E. Individual or family membership. 38.The treatment – and – prophylactic help to
32.A woman of 34 years old complaints about countrymen at different stages is given with rural
spready bloody discharges from genitals several sites, regional and regional hospitals, clinics, the
days before menses. In anamnesis – diagnostic centers, hospitals at scientific research
dyathermocoagulation 2 years ago. During the institutes. Name the basic establishments of the
investigation of the uterus wall in the mirrors a fourth stage of rendering of medical aid to
cianous cysts in the form of pupilla are found out. agricultural population.
For approving the diagnosis the following should A. *City treatment-and-prophylactic
be done: establishments, the state specialized centers.
A. *Byopsy of the uterus cervix B. The central regional hospital, clinics.
B. Stear for cytological investigation C. Station of blood transfusion, the diagnostic
C. Colposcopy center.
D. Laporoscopy D. Clinics, regional hospital.
E. Ultrasound investigation of the pelvis minor E. Regional hospital, the diagnostic center.
33.Name the most informative parameter of average 39.Of what from elements of carrying out sanitary
duration of stay in a hospital: epidemic supervision has no attitude to
A. On nosological forms. precautionary?
B. On branch as a whole. A. *The control over observance of sanitary-and-
C. On chambers. hygienic norms in establishments
D. On a hospital as a whole. B. A hygienic regulation of potentially dangerous
E. On number of sick-lists beds. factors
34.What factors, from specified, influence as much as C. The account of objects and constructions
possible a level of the general mortality of the D. A choice and tap{removal} of the ground area
population? under construction
A. *Age -sexual structure of the population and a E. The regular control over construction and
condition of an environment reconstruction
B. Quantity of medical workers 40.What methods does a doctor use when estimating
C. Number of medical institutions physiologic development in children and
D. A climate-geographical position teenagers?
E. A level of hospitalization of the population A. *measuring anthropometric data and
35.What criterion testifies to type of age structure of functional indices, estimating the findings
the population in territory? described
A. *A ratio of a share of children and persons of a B. measuring the chest circumference, the form
pension age. of the head
B. Net factor of reproduction. C. measuring arterial pressure and muscular
C. Average duration of a forthcoming life. power
D. Gross - factor of reproduction. D. measuring the concentration of attention
E. A parameter of a natural increase. E. the description of the biological development
36.For comparison of a degree of a variety of weight level
of a body of newborns and 7-years children it is 41.At estimation of physical development a boy is
the most expedient to use: set, that his mass of body is less, than М - 2уR on
A. *Coefficient of a variation by regional standards (tables of regression) for
B. Amplitude lack of chronic pathology. What group of health
C. An average quadratic deviation. does a boy behave to?
D. A mistake of average A. *ІІ
E. Factor of determination B. ІІІ.
37.What from functions does not belong to functions C. IV.
FMSH? D. V.
A. *The guaranteed volume of the specialized E. І.
medical service 42.What is the necessary information for estimating
B. The guaranteed volume of the social help the investigation results after having learnt the
C. Availability of corresponding medical and efficiency of the vaccination against flu in
social services to all population personnel?
D. A continuity of supervision over patients A. * the number of sick persons among 100
people who were inoculated against flu and the
KROK 2 – Question Bank 5
amount of sick persons among 100 people who A. *reanimatology.
were not inoculated B. Infectious
B. the quantity of complications caused by the C. gerontology.
vaccination D. otorhinolaryngology - brigades.
C. the quantity of persons who were inoculated E. gastroenterology.
D. the quantity of persons who were not 49.What establishments provide tertiary medical aid?
inoculated A. *diagnostic and medical centers, dispensaries
E. the number of diseases among 100 persons B. medical centers, dispensaries a child's
who were inoculated against flu and the policlinic
number of diseases among 100 persons who C. specialized and multi profile establishment
were not inoculated against flu, the total (regional, central, city)
amount of workers D. gynecological consultations, permanent
43.Who is responsible for medical - diagnostic establishments, policlinic of city.
section of work in a polyclinic? E. specialized and multi profile hospitals (district,
A. *the head physician, the assistant to the head regional, city)
physician managing branch 50.Purpose of tertiary prophylaxis:
B. The head physician managing branch A. *rehabilitation of patients losing possibility
C. The head physician managing branch, the local complete valuable vital functions
therapist B. forming of adequate relation of population to
D. The assistant to the head physician managing the system of health protection
branch, the local therapist C. Elimination expressed factors of risk, which at
E. The assistant to the head physician managing certain terms can result in the origin,
branch intensification, relapse of disease
44.Name obligatory structurally-functional parts of D. Activation of public funds and organizations
the control system: in regard to medicine
A. *management subject, management object, E. Warning of origin and influences of possible
block of the scientific adjusting. factors of risk of diseases
B. components, communication. 51.Name kinds of medical aid to the population:
C. structure, function, management types. A. *Primary, secondary and tertiary.
D. principles, technique, management methods. B. Out-patient - polyclinic, fast, stationary and
E. laws, technology, management mechanisms. sanatorium.
45.Name component parts of science of management: C. Scheduled, urgent, prime.
A. *theory, technique, organization, methods. D. Hospitalized, dispensary, in-home, preventive.
B. principles, approaches, laws. E. Therapeutic, surgical, traumatology-
C. system, process, functions, technology. orthopedic, pediatric, cardiology, the
D. shots, finances, information. ophthalmologist., etc.
E. time, delegation, organization of labour. 52.S., 59 year old male is ill with generalized
46.In the complement of accreditation commissions, arteriosclerosis with blood circulation
in compliance with the legislation, besides the insufficiency. What ratio between the nutrients do
organs of management representatives must enter you consider to advice?
by the health protection: A. *Proteins : fats : carbohydrates = 1: 0,5 : 3
A. *medical insurance organizations B. Proteins : fats : carbohydrates = 1 : 1,0 : 4
B. fire-prevention security services C. Proteins : fats : carbohydrates = 1 : 1,0 : 5
C. tax service D. Proteins : fats : carbohydrates = 1 : 1,3 : 4
D. services of labour protection E. Proteins : fats : carbohydrates = 1 : 2,0 : 3
E. sanitary-epidemiology service 53.Patient С., 15 year old, consulted his physician
47.Official permission on the right of realization of with complaints of the night blindness.
medical activity at the observance of the set Objectively: rised darkness adaptation time,
requirements and terms, given out to the legal Bitot’s spots on conjuctive. Patient’s skin is dry,
entity is named: desquamatous, folliculitis signs of the face skin.
A. *by a license What foods do you consider should he eat to
B. by a certificate prevent this disease?
C. by admittance to work A. *butter and carrots
D. by a diploma B. lard and garlic
E. by accreditation C. sunflower oil and cabbage
48.What brigades of medical first-aid behave to D. margarine and potato
specialized? E. cocoas milk and pine apples
KROK 2 – Question Bank 6
54.In the surgical unit of the hospital the purulent C. absence of the vegetations at the waste
surgery department is placed on the 1st floor, treatment station
thoracic surgery – on the 2nd floor, general D. improper examining of the station worker
surgery – on the 3 rd floor and gynecological – on E. emergency outlets of the impure wastes
the 4th. Could you find any infringements in the 58.The kitchen of the student hostel received fish: the
department placing? surface is covered with slime, scales are mate,
A. *wrong placing of purulent surgery gills have dirty red color and unpleasant smell,
department eyes are mate and sunken, muscles are soft. Assess
B. wrong placing of thoracic surgery food quality.
C. wrong placing of gynecological department A. *poor quality product
D. wrong placing of general surgery department B. high quality product, can be used without
E. no infringements limitations
55.The worker of battery plant consults physician C. high quality product, can be used with
with complaints of weakness, headache, Status limitations
praesens objectivus: paleness, dark line on the D. conditionally good product
gums, systolic murmur of the heart apex. In blood: E. adulterated product
microcytaric anemia, basophilic grainy RBC, 59.Patient with thyreotoxicisis is in the 2 beds
reticulocytosis. Urine analysis: porfirinuria 0,25 hospital ward of therapeutic department. The area
mg/l. What conditions of occupational of the ward is 18 m2, height 3 m, ventilation rate
environment may the disease cause? 2,5 /hr. Air temperature - 20оС, relative humidity
A. *high lead content in the air of the working 45\%, air movement velocity 0,3 m/s, light
zone coefficient 1/5, noise level 30 dB. Do hygienic
B. high sulfuric acid content in the air of the assessment of the conditions:
working zone A. *discomfortable microclimate
C. high alkaline content in the air of the working B. non-effective ventilation
zone C. poor lighting
D. high benzene content in the air of the working D. high level of noise
zone E. all conditions are OK
E. high iron oxide content in the air of the 60.In the sanitary inspection of labour conditions of
working zone miners they found: air temperature - 26оС, relative
56.On inspection of the laundry of the regional humidity 40\%, dust pollution - 135 mg/m3, silica
hospital it’s defined the relative air humidity is content in the dust 45\%. Miners work by the
99\%, air movement velocity -0,6 m/s, air “dry” technology. What disease can cause
temperature - 15оС. What way to optimize occupational environment conditions?
occupational environment could you advise? A. *silicosis
A. *decrease of humidity and air movement B. urine stones
velocity and increase of air temperature C. gallstones
B. decrease of humidity, air movement velocity D. olivinosis
and air temperature E. pulmonary hemosiderosis
C. decrease of humidity, increase of temperature 61.Name the social - hygienic factor which does not
and air movement velocity influence requirement of the population for
D. decrease of air humidity and temperature of hospitalization:
air, increase of air movement velocity A. *Volume and quality of medical aid.
E. decrease of temperature and air movement B. A level of desease of the population.
velocity C. A way of life of the population.
57.The waste treatment complex is situated 1000 m D. A degree of development of social services.
away from the residential area. The area is E. A level of sanitary culture of the population.
hygienic,the technology of treatment is proper. 62.The 6 year old girl has the low height. Her growth
They use sanitary space for growing of ray grass. development is harmonic. She has premolars and
People living in the nearest households complain incisives of permanent teeth. This year she has
of constant unpleasant smell, plenty of flies. There been ill with measles and infectious hepatitis. She
is high morbidity of intestinal diseases. Specify can calculate and read. She's done Kern-Yiracek's
the probable causes of unsatisfactory test with 6 scores. Why is the girl not ready to be
epidemiological situation. admitted at school?
A. *sanitary space is not used properly A. *infectious hepatitis
B. improper waste chlorination B. low height
C. poor results of the test
KROK 2 – Question Bank 7
D. absence of all permanent teeth 68.In order to prevent the environmental pollution in
E. measles the place N, the artificial methods of sewage
63.Efficiency of activity of family doctor is estimated disposal, biological purification, chlorination and
on the following indexes: compulsory laboratory control are planned. Which
A. *general morbidity one of the main artificial sewage disposal methods
B. quantity of the population served by a family isn’t taken into account?
doctor A. * Mechanical purification
C. general death rate B. Rechlorination
D. full strength by shots C. Filtration
E. hospitalized morbidity D. Irrigation
64.The balance at the market of medical goods is E. Areas of sewage disposal
characterized by: 69.A Land area for building of medical-preventive
A. * balance between the capacity of supply and institutions must meet the requirements which are
demand claimed to the house-building. This concerns to:
B. balance of prices at the market of medical A. * all which are marked
services B. Relief of a land
C. price interaction at the market of medical C. Providing of the flow of the atmospheric
services rainfall
D. excessive demand of medical goods and D. Level of subsoil waters
services at their deficient proposition E. For all which are marked except C
E. the excess of medical services at this certain 70.During an exam, while giving the definition of
market price ''manufacturing microclimate'' a student included
65.The estimation of quality and medical aid the following components to the concept:
effectiveness appears to be realized as such: temperature of the air, moisture of the air, speed of
A. * Structural, processing approach and the one the air, ultraviolet radiation, atmospheric pressure
which is based on the final results the irradiation. Which components named by
B. Structural, organizational and operational student were wrong?
approach A. * Pressure, irradiation, ultraviolet radiation
C. Processing, economical and the one which is B. Irradiation
based on the final results C. Atmospheric pressure
D. The approach to be based on the final result, D. Atmospheric pressure and irradiation
complex and statistical E. All are right
E. Preventive, current and final approach 71.The chief principle of forming the therapeutic
66.A student has got the following examination task. divisions of industrial sections is:
The interlayer waters are known to be so much A. * the number of workers
mineralized that they can’t be used for municipal B. worker’s age
water supply with no treatment. In case of soil C. personnel length of work
pollution by waste products and sewage. There is D. staff’s composition according to health groups
a danger of subsoil water pollution with E. service range
pathogenic microorganisms. Name the source of 72.They damaged the container with alpha-
water supply to be the best for middle and small radioactive substance (solution) with pollution of
water pipes. the work place surfaces. What preventive
A. * interlayer waters measures should be used?
B. subsoil waters A. *desactivation of the premises
C. atmosphere waters B. degasation of the premises
D. surface waters C. respiratory protective equipment
E. springs D. protective shields
67.The rational planning of city districts provides its E. decrees about duration of the working week
division according to the functional signs into 4 73.Choose a method of a graphic representation of
areas:1) industrial are; 2 )suburban are; 3) traffic monthly information about number of the
area. Name the fourth area: registered cases of acute intestinal infection and
A. * selitebouns area their comparisons to the average monthly values,
B. sport area obtained for 5 previous years:
C. cultural area A. *The linear diagram.
D. traffic area B. The radial diagram.
E. new buildings area C. The sector diagram.
D. The figured diagram.
KROK 2 – Question Bank 8
E. Curvilinear. C. rehabilitation of patients losing possibility
74.The 9 years child with diagnosis “chronic complete valuable vital functions
tonsillitis” stands dispanserization control. For 1 D. Activation of public funds and organizations
year of observation there was one exacerbation of in regard to medicine
disease. Physical condition is satisfactory. The E. Prevention origins and influences of possible
general state is not infringed. Define group of factors of risk of diseases
health: 80.The populated area has the water supply from
A. *III (a). artesian chink. Lab analyses data: transparence-30
B. II cm, hardness-5,5 mg eq/l, colority-20о, nitrates-20
C. I mg/l, smell-1 score, fluorine-4,0 mg/l, taste-1
D. III (b). score, coli-tytre-400 ml. What disease would
E. III (c). result water consumption?
75.The child is 6 years old. For one year of A. *fluorosis
observation he has ARD duration 8 days. Physical B. urine stones
worked out satisfactory. Define group of health: C. water-nitrate methemoglobinemia
A. *I-st. D. chronic gastritis
B. II-d. E. syderoachrestic anemia
C. III (a). 81.Define the observation unit, at study the average
D. III (b). duration of hospitalization of patients with
E. III (c). appendectomy, depending on delivery times in a
76.25 unorganized children in the age 2 -3 year will hospital and the clinical form of appendicitis.
be observed on a pediatric district it in the current A. *The patient with appendectomy.
year. What scheduled number of initial visitations B. The patient who is entered in the hospital.
will make to this group of children? C. Average duration of patient's stay in a hospital.
A. *50. D. Delivery times of the patient.
B. 20. E. The clinical form of appendicitis.
C. 40. 82.What methods of the collecting of the information
D. 100. is preferable for study of housing conditions of
E. 200. students of medical HIGH SCHOOL for a training
77.Indicate the registration medical document for the period?
patient Н., that 21.02. was addressed to the doctor A. *Questioning.
with diagnosis ARVD the first time in this year: B. Interviewing.
A. *The statistical coupon is necessary to fill in it C. Selecting of materials.
and it is necessary to deliver on a sign +. D. A method of the directed selection.
B. The statistical coupon for registration of final E. Statistical.
diagnoses is not necessary. 83.Name a statistical observation unit for
C. The statistical coupon to fill in it is necessary, determination of influence amount of blood sugar
but a sign +is not necessary to put in. on the healing of wound's surface in a
D. It is necessary to fill in the emergency notice postoperative period:
on a case of a contagion. A. *The patient in a postoperative period.
E. The necessary registration form is not B. Amount of blood-sugar.
indicated. C. The analysis of a blood.
78.What kind a method is correct to establish force of D. The patient who has a wound surface.
correlation connection between age of men and E. The patient who was discharge on an after-
their mortality from a myocardial infarction? care.
A. *A method of grade correlation (Spirman). 84.The vital capacity of lungs was estimated during
B. A correlation ratio. medical examination of schoolchildren. What
C. The quadrate method (Pirson). device is used for measuring of vital capacity of
D. The Indirect method (Stjudent). lungs?
E. A method of graduated correlation (Armler). A. *A spirometer
79.Purpose of the second prophylaxis: B. The thermometer
A. *Removal of the expressed factors of risk, C. The anthropometer
which at certain terms can result in the origin, D. A tonometer
intensification, relapse of disease E. A centimetric tape
B. forming of adequate relation of population to 85.Estimate the quality of milk: density 1038, the
the system of health protection maintenance of fat 2,8 \%, acidity 18 ( T.
A. * The lowered quality
KROK 2 – Question Bank 9
B. Of good quality C. conditionally edible product, can be used for
C. Of poor quality meal
D. Adulterated D. False spoiled, can be used after boiling
E. Conditionally fit E. adulterated, can be used after boiling
86.Matusis` Device is used for estimation of 92.In order to compare the degree of variety of length
providing of an organism with: of body of new-born pigeon pair it is most
A. * Vitamin C expedient to use the criteria of variety:
B. Vitamin B 1 A. *mean quadratic deviation.
C. Vitamin B 6 B. amplitude;
D. Vitamin B 12 C. limit;
E. Vitamin A D. error of middle
87.Milk contains starch. Solution Lugoli has been E. coefficient of variation
added for definition of starch in milk. What colour 93.What from indexes characterizes natural motion of
will the milk be painted in? population?
A. Dark blue A. *natural increase.
B. Pink B. migratory activity.
C. Green C. mean time of forthcoming life.
D. Yellow D. pendulum migration.
E. Black E. disability.
88.The first-rate wheat bread was brought to the 94.What information is needed for determination of
hospital food-block: with humidity of 45 \%, level of early neonatology mortality:
acidity 3 ( T, porosity of 67 \%. Evaluate the A. *number of the children born by living and
quality of the bread, please. deceased during the first seven mortality.
A. *Of good quality B. number of the children born dead
B. Of poor quality C. such index is not used in practice
C. Adulterated D. number of the children deceased on the first
D. Conditionally fit month of life
E. Of the lowered quality E. number of the children deceased on the first
89.The average body length of neonatal boys is 50,9 year of life.
cm at a sigma 1,66; and average mass - 3432 at a 95.The health of population, most, depends on:
sigma 5,00. What criteria is correct to compare A. *way of life
degree of variability these signs? B. terms of external environment
A. *A coefficient of variation. C. economic line-up
B. A sigma. D. heredity
C. A limit. E. systems of health protection
D. An amplitude. 96.People, which do not have complaints, chronic
E. A coefficient of association. diseases in anamnesis, functional rejections and
90.What from indexes behaves to the performance organic changes in organs and systems, behave to
indicators of policlinic belong: the group:
A. *middle number of visits on one habitant for a A. *healthy (1-2 group)
year. B. it is conditional healthy
B. general morbidity. C. it is potential patients
C. morbidity with the temporal loss of ability to D. patients
work. E. practically healthy
D. frequency of primary output on disability. 97.Give determination of maintenance of concept
E. primary morbidity. “morbidity”:
91.Canned fish “Atlantic hearing” has a paper label A. *sum of cases of the first exposed diseases
with specification of the enterprise-producer (fish- among the population.
processing plant in Yalta), name of the product B. sum of the registered illnesses among the
and others. The surface of the tin is not deformed, population.
the bottom and cover are significantly convex. Tin C. sum of visits of the population MPE.
is hermetic. Tin cover is marked by the stamp: D. sum of being ill persons among the population.
070190 513234 1Р Assess quality of product E. sum of patients of needing treatment.
A. * poor quality product, can’t be used 98.What group of illnesses leads in the structure of
B. high quality product, can be used w/o reasons of mortality of population?
limitations A. *illnesses of the system of circulation of
blood.
KROK 2 – Question Bank 10
B. new formations. ionizing radiation this states included in such a
C. illnesses of organs of breathing. state?
D. illnesses of the nervous system and sense- A. *somato-stochastic
organs. B. somatic effects
E. illnesses of organs of digestion. C. geterosis
99.Name basic principles of science of management: D. genetic
A. *combination of particular a branch and E. gormesis
territorial level, scientific character, economy, 105.Mrs. Т., 33 years old works as the secretary. Her
system, delegation. diet contains 150 g of protein (including 100 g of
B. hierarchical, reliability, operative, reason. animal), 200 g of fat, 600 g of carbohydrates.
C. leadership, responsibility, competence, plenary What pathology can effect this diet?
powers. A. *obesity
D. selection of shots, democratic centralism, B. schizophrenia
specialization and co-operation. C. paradontosis
E. development according to a plan, unity of the D. common cold
personal and collective interests, E. uterine fibromyoma
technological, flexibility. 106.An Odessa inhabitant colected mushrooms in the
100.Name the tasks of head of the department of city forest, fried and ate them. In 12 hours he felt
hospital: severe abdominal pains, vomiting bloody diarrhea.
A. *control after a timeliness and plenitude of In the end of the 1st day of disease jaundice,
inspection and treatment of patients. hepatomegalia and oliguria developed. There were
B. consultation of patients on to the house. convulsions also. He passed on 3rd day of the
C. distributing of acting patients on the disease. What is the cause of the disease?
separations. A. *poisoning with Death Cap
D. calculation of cost of stationary treatment of B. overcooling
patient. C. poisoning with margarine preservants
E. organization of daily permanent establishment. D. poisoning with fly agaric
101.Specific gravity of population which treats oneself E. poisoning with false morel
in ambulatory-policlinic establishments makes 107.In the first stage of coli-index assessment of tap
(in \%): water by the fermentation samples method they
A. *From 70 to 80 accumulate Е. сoli on glucose-peptonic medium.
B. From 40 to 50 In one 100 ml bottle and 1 ml tube indicator
C. From 50 to 60 changed color and gas was produced. What
D. From 60 to 70 differential diagnostic medium should you use for
E. More them 80 2nd stage of the test?
102.Periodic medical surveys are carried out for: A. *Endo medium
A. *Periodic inspection of the certain contingents B. Ressek medium
of workers C. Kitta-Tarozzi medium
B. Inspection of the certain contingents of D. Rappoport medium
workers at reception on the robot E. Bordet medium
C. Revelation of some diseases at early stages 108.The qualifying requirements to the family doctor
D. Directions on MSEC are included in this case:
E. Revelation and formations of contingents for A. *domain by the modern methods of
dispensary supervision diagnostics and treatment
103.Whatever document to the medical worker is B. oratorical art
required to represent at the reception on work? C. ability to conduct the conferences
A. *certificate about registration. D. ability to manage
B. labour book. E. domain by the methods of analysis of the state
C. diploma. of environment
D. military ticket (for liable for military service). 109.A girl 9 years old, has an average height and
E. passport. harmonic growth development. She was ill with
104.Patient С., 48 years old, participated in the acute respiratory infection for five times. Define
liquidation of Chernobyl disaster consequences. the group of her health.
He is treated in in-patient department of municipal A. *2nd group
hospital. His diagnosis: progressing vegetative B. 1st group
insufficiency. What group of biological effects of C. 3rd group
D. 4th group
KROK 2 – Question Bank 11
E. 5th group C. air temperature
110.In the assessment of microclimate they defined D. relative humidity
that in 10 cm above the floor air temperature was E. air movement velocity
17оС, at the 1 m - 19оС, at the 1,5 m - 20оС. 116.While school inspecting they defined that third
What overfall by the vertical is the maximal year form has 26 lessons during a weak; on
admissible? Tuesday the first is Ukrainian lesson, 2nd - music,
A. *2,5 °C 3 and 4th - physical training, 5 lessons at a whole.
B. 1 °C Could you find any infringements in the schedule?
C. 1,5 °C A. *number of classes of physical training
D. 2 °C B. number of lessons per week
E. 3 °C C. number of lesson per day.
111.For controlling of pre-sterilization processing of D. place of music lesson in the schedule
medical instruments they use some tests: E. place of the Ukrainian language lesson in the
phenolphthalein, ortholuidine, azopiram, schedule
benzydine and sodium benzoas for controlling of 117.Student Н. is estimating the noise rate in the
completeness of washing them from blood and punching workshop. What device is he using?
alkaline components of detergents.. What test is A. *Phonovibrometer
the most sensitive for blood traces and the safest? B. Analysator of noise spectrum
A. *Azopiram C. Audiotester.
B. Phenolphthaleine D. Actinometer
C. Benzydine E. Piranometer.
D. Sodium benzoas 118.As preventive means they prescribe UV radiation
E. Ortholuidine in the photarium for adolescent living in
112.Professiogram of assembly shop of the automobile Murmansk region (Russia). What device could be
plant includes the following elements: conveyer, used to define biological and preventive dosage of
assembly works with power 15 W/hr, the number UV radiation?
of the operations – 3 per minute. What is the A. *biodosimeter
criterion for work intensity in these conditions? B. ermeter
A. *monotony. C. ufimeter
B. power of work, W D. uviolmeter
C. emotional pressure E. piranometer
D. intellectual intensity. 119.Mr G. uses untreated feces for fertilizing. He sells
E. operative memory volume vegetables at the city market. What infestation can
113.The seiner crew is under action of low air occur among his customers?
temperatures (from -5°С up to -10°С). What main A. *ascaridosis
organs’ diseases can be prevalent among them? B. trichinosis
A. *respiratory system. C. shistosomatosis
B. blood D. dracunculosis
C. liver. E. opistorchosis
D. gastrointestinal tract 120.The X-ray room of the cardiological center is
E. cardiovascular system placed on the 2nd floor and has 40 m2 total
114.With daily diet in the organism of 5 years old square. The operating control panel is behind the
child (body weight 20 kg) there are 80 mg of shielded screen. The physician’s working place is
nitrates income. Define daily dosage of the nitrate 2 m from the apparatus. No adjoining wards by
A. *4,0 mg/kg of body weight the vertical and horizontal. How many premises
B. 1,0 mg/kg of body weight the X-ray room should consist of as minimum
C. 2,0 mg/kg of body weight A. *3
D. 3,0 mg/kg of body weight B. 1
E. 5,0 mg/kg of body weight C. 2
115.In the thermal workshop of the ship-repairing D. 4
plant the heat amount is 81 kcal/m3 hr. Air E. 5
temperature is 28-33°С, heat radiation – up to 121.Mrs. Y. lives in polluted area in the zone of severe
1800 kcal/m2 hr, relative humidity - 40-60\%, air radioecology. She has own household with the
movement velocity - 0,5-0,7 m/s. What garden and she doesn’t work. She consult
occupational hazard can cataract be caused by? physician about using vegetables from the garden
A. *heat radiation. to rise excretion of the radionuclides. You should
B. convection heat
KROK 2 – Question Bank 12
recommend vegetables with the highest fiber D. anemia
content (more then 1 \%). E. hypermetropia
A. *pumpkin 127.The classroom in the village school has 30 m2.
B. paprika How many 5th year pupils could study in this
C. vegetable marrow premise?
D. carrot A. *20
E. cabbage B. 10
122.In the assessment of microclimate they defined C. 15
that in outer corner at the height of 1 m the D. 25
temperature was 18оС, in the middle - 19оС, in E. 30
inner corner - 20оС. What overfall by the 128.The section of senior creche group is situated on
horizontal is the maximal admissible? the first floor of the kindergarten. Section consist
A. *2°C of a dressing room (18 m2), a dining room (group
B. 1 °C room) (50 m2), bedroom (48 m2), servant room
C. 1,5 °C (14 m2). Section has only one entrance, from the
D. 2,5 °C street and it’s common with middle group. Could
E. 3 °C you define any infringements in the internal lay-
123.Worker С. works at the chemical enterprise N, out?
producing sulfuric acid for 6 year. It was defined A. *infringement of principle of group isolation
that the content of sulfuric gas was in two times B. insufficient area of the dressing-room
more than MAC. What protective equipment do C. insufficient area of the dining room
you advise him to use? D. wrong content of the section premises
A. *gas mask E. wrong placing by the floor
B. hose mask 129.The well is located at the village outskirts. It has
C. ABA the stone walls, cover; the common bucket is
D. mask absent. The nearest area is not covered. The square
E. cotton facial bandage of water mirror is 0,5 m2, depth 20 m. How many
124.In the well situated at the outskirts of the village in grams of chloric lime containing 25\% of active
60 m from the cattle farm they found nitrogen chlorine is it necessary to use for well disinfecting
compounds in concentrations: ammonia - 0,9 if chlorine demand of water is 2 mg/dm3?
mg/l, nitrites -0,1 mg/l, nitrates - 52 mg/l. For A. *80 g
what pollution is this analysis typical? B. 20 g
A. *continuous C. 40 g
B. fresh D. 60 g
C. recent E. 100 g
D. old 130.For assessment of epidemiological danger of air
E. latent environment in the exhibition center they conduct
125.They conduct a survey on the children health in the survey on the air germ pollution. What
city of N. It was defined that in 23\% of cases lead microbe is sanitary indicative?
content of blood was higher than 10 mkg/l. What A. *Str. haemoliticus
was the principal source of pollution of urban B. St. aureus
environment with lead compounds? C. Pseudomonas aeruginosae
A. *ethyl gasoline using D. Escherichia coli
B. using lead-containing paints battery producing E. Clostridia
C. porcelain producing 131.A 66 year old history teacher doesn’t have in his
D. cable producing diet the products having anti sclerotic action.
E. - Correct the teacher’s diet.
126.They sell portable desks in the market. Desks are A. * introduce fish, poultry meat, cheese, fruit,
made from light wooden. They heaven’t sharp vegetables, into diet
edges, the cover has an operated inclination (from B. introduce cereals into diet
1 to 45о), height of the table 0,8 m, sit height 0,5 C. introduce fat meat into diet
m, distance of the sit is positive. What disease can D. introduce floury products into diet
be among the children using the desk for home E. introduce all enumerated products into diet
work? 132.It is necessary to carry out the investigation and to
A. *right sided scoliosis establish the norm of exogenous chemical
B. left-sided scoliosis substance N content in soil. In order to create
C. flat footedness experimental conditions it is necessary to use such
KROK 2 – Question Bank 13
type of soil which has maximal filtrative and 137.Infrared radiation influences the worker of 48
minimal sorption and soaking peculiarities. What years old in the process of production. The depth
type of soil is it necessary to use? of thermical action of the infrared radiation on The
A. *Sandy human organism depends upon:
B. Stony A. *The length of the wave
C. Loamy B. Intensity
D. Peaty C. Action’s direction
E. Brackish D. Wave’s amplitude
133.During his holiday in Ukraine a foreigner felt ill E. Frequent spectrum
with aggravation of gastric ulcere. He sought or 138.Doctor C, in private talk, told about the venereal
medical advice. He was admited to hospital. What disease of his patient M to his neighbour. What
are the documents for his discharging from the kind of responsibility is provided by legislation in
hospital? case of breaking the doctor’s secret?
A. * The extract from a patient’s case history; A. *Discipline or criminal, depending upon the
B. Sick-leave; results
C. Certificate of patient’s hospitalization; B. Administrative
D. А and В variants C. Discipline
E. С and В variants D. Dismissal of the doctor
134.The analysis of population rate of cases is carried E. Financial
out. It is characterized with the following index: 139.Doctor D, an obstetrician – gynecologist has made
A. * В, С, D variants; back-street abortion. In the result of this medical
B. The level of incidence; intervention the patient developed some
C. The structure of incidence; complications which led to sterility. Criminal
D. The dynamics of incidence; abortion according to active legislation provides
E. The registration of incidence. for :
135.A 68 year old pensioner has three meals a day A. *Criminal responsibility
with following distribution of energetic value: B. Administrative responsibility
form breakfast 20\%, from dinner 30\%, from C. Discipline responsibility
supper 50\%. The diet regime shoved be the D. Dismissal of the doctor
following: E. Financial responsibility
A. * 4 meals a day with even distribution of 140.In the workshop of garment factory they supervise
energetic value effectiveness of the ventilation system and control
B. 3 meals a day with superiority of energetic on the microclimate conditions. What device can’t
value for breakfast be used for determining the air movement velocity
C. 4 meals a day with superiority of energetic in the section of ventilation pane?
value for dinner A. *wing anemometer.
D. 3 meals a day with superiority of energetic B. electrical anemometer
value for dinner C. kata-thermometer
E. 4 meals a day with superiority of energetic D. stringed anemometer.
value for supper E. membranous anemometer
136.To determine the sufficient water 141.The premises of medical institutions demanding
decontamination, the amount of residual chlorine the rules of sterility, asepsis and antisepsis be
which is left after chloration is taken into provided with ultraviolet lamp counting:
consideration. What is the method of qualitative A. *1 watt capacity per 1 m3
determination of residual chlorine in drinking B. 1 watt capacity per 10 m3
water after its chloration? C. 1 watt capacity per 100 m3
A. *Adding of potassium iodide and starch to D. 0,5 watt capacity per 1 m3
water samples E. 0,5 watt capacity per 100 m3
B. Fitration of the water sample by up to rosy 142.In the inspection of occupational environment for
colour of solution Greese reagent medical stuff in their operating room they defined
C. Addition of acid to the water sample air temperature – 20°С, air movement velocity -
D. Addition of aluminium sulfuric oxide to the 0,15 m/s, relative humidity - 75\%, carbon dioxide
water sample up to the formation of hydroxide content - 0,3\%, general lightning rate with tube
flakes. lamps - 400 lx, operating field - 7000 lx. What
E. Addition of manganese peroxide to the water indices don’t correspond to the hygienic
sample standards?
A. *relative humidity, carbon dioxide content
KROK 2 – Question Bank 14
B. operating field lighting rate, air movement maize, she rarely eats vegetables and beans and
velocity. never eats meat and fish. What is the disease?
C. carbon dioxide content, general lighting rate A. *pellagra
D. carbon dioxide content, air temperature B. psoriasis
E. relative humidity, general lighting rate. C. scurvy
143.While working a 55 year old man busy in coxo- D. beri-beri
chemical industry can’t do without water. How E. Swift’s dermatitis
showed the drinking of this worker regime be 149.The 5 years old child was brought to the reception
organized? ward with such symptoms as: dryness of skin and
A. * Use 100 ml drinks and water every 25-30 mucosa, short breathing, midriasis, agitation,
minutes. confused consciousness. They defined that before
B. Use drinks and water 10 minutes before meals hospitalization the child had collected berries in
C. Use drinks and water before the beginning of the forest. What plant caused food poisoning?
the work A. *spurge-flax
D. Use drinks and water after break B. fly agaric
E. Use drinks and water after the beginning of the C. green gill
work D. Cikuta
144.The right of establishing the fact and the group of E. Aconitis
stable disability (invalidity) belongs: 150.In the country with decentralized water supply
A. *MFEX (shaft wells) they registered elongation of fonticuli
B. MCC healing, poor bones development, teeth
C. Doctor development retardation among infants. They are
D. The head doctor deputy for disability typical signs for presence in water of
examination. A. *Fluorine
E. The head doctor B. Lead
145.Statistical investigation is carried out. It’s C. Strontium
necessary to formulate totality of choices. Which D. Iodine
of the enumerated methods is not used for E. Arsenium
selecting units of observation into totality of 151.They are building a 2 storied house in the town.
choices? Water pipes and canalization are absent. What
A. * analogous type of lavatory is the best for these conditions?
B. mechanical A. *Lavatory with aerated tank
C. typological B. Chemical lavatory
D. serial C. WC
E. random D. Public lavatory
146.The doctor established the level of flue illness to E. Field lavatory
be increased to 30\% in comparison to last year. 152.Ultra-violet radiation is carried out to prevent in
What index did he use? hospital infections in the premises of the medical
A. *visual institutions. What is its health-improvement action
B. intensive evaluated with?
C. extensive A. *Degree and index of effectiveness
D. correlations B. Multinle of air-exchange
E. non-marked intensity C. Dust content
147.What does a rural hospital consist of? D. Ozone content
A. * a hospital or an outpatient clinic and E. Carbon dioxide content
feldsher-obstetric stations; 153.A 40- year old inhabitant of the city of Kiev
B. feldsher-obstetric stations, a regional hospital; arrived in Abidjan (Ivory Coast) in July of 1998.
C. a regional hospital and female dispensary; In an hour after arriving he lost consciousness
D. a hospital, a regional hospital and feldsher- after leaving the international airport terminal.
obstetric stations; After medical aid he was delivered to the hospital.
E. a regional hospital and a sanitary- His state was characterized with the following
epidemiologic station. signs: expressed adinamia, severe headache with
148.A woman of 55 years old has diarrhea, nausea, the skin was hyperemied, wet; body
desquamation and skin pigmentation (on the face, temperature was rised up to 39оС; short breathing,
neck, feet and hands), irritability and anxiety. We tachicardia. The tourist has never been to hot
know from her anamnesis that her main food is climate countries. What is the cause of syncope?
A. *hot climate impact
KROK 2 – Question Bank 15
B. new impressions D. to use individual protective equipment
C. tourist’s age E. misemploy her
D. Impact of flight 159.The group of Ukrainian tourists visited Malaysia.
E. poor health state of the tourist By WHO statistics this country has a high rate of
154.The worker of the chemical enterprise visited a morbidity owning to water-borne infections. What
health provider with complaints of irritability, way of water disinfecting should they use?
poor workability, insomnia, headache. A. *boiling
Objectively: arhythmical tremor, asymmetry of B. ozonizing
tendous and periostal reflexes, pulse lability, C. freezing
steady red dermographismus. Signs of excretory D. iodination
gingivitis. The worker deals with amalgam E. UV irradiation
producing. What is the disease? 160.The germ pollution of operation room after
A. *chronic mercury poisoning operating was 2500 CFU/m3. They sanitize air
B. chronic lead poisoning with UV-lamps. What percent should germ
C. chronic gasoline poisoning pollution be decreased to consider the sanitation
D. chronic aniline poisoning effective?
E. chronic cadmium poisoning A. *80\%
155.At the market they sell milk brought in the churns. B. 20\%
Smell and taste of milk are usual, color is white C. 40\%
with bluish tint, specific density 1,015 g/cm3, fat D. 60\%
content 2\%. Acidity 15оТ. No admixtures. E. 99\%
Determine milk quality. 161.The cadets of military school were brought to the
A. *milk was adulterated by dilution infection hospital with acute gastroenteritis after
B. milk was adulterated by skimming reception on the graduation day. In vomiting
C. Stale milk masses and rest of the products St. aureus and its
D. milk with suspicious freshness enterotoxine were found out by ELISA. What
E. surrogate milk product have cause food poisoning?
156.During the examination on general hygiene and A. *cream cake
social medicine the student said that the mountain B. sausages
sickness has been caused by the low content of C. fries
oxygen in the highlands air. Could you correct his D. stuffed paprika
response? E. Adulterated cognac
A. *it’s caused by decreasing of partial pressure 162.The man with jaundice was brought to the
of the oxygen hospital. He considered that his state was caused
B. it’s caused by high content of carbon dioxide by roasted peanut. What mycotoxine was found in
C. it’s caused by solar radiation effect peanuts?
D. it’s caused by action of cold air A. *aflotoxine
E. it’s caused by peculiarities of perception of the B. ergotoxine
mountain environment C. botulotoxine
157.While answering the state examination a student D. phasine
of medical faculty said that the sanitary lab E. solanine
decides the problem about suitableness of food for 163.С., 38 year old, has general labor experience of 15
using. Could you correct your college’s answer? years, for 10 years he works as a roentgenologist.
A. *health officer gives final conclusion about By dosimeter data he took 40 rem during these
food quality years Can С. work by his specialty in future?
B. assistant of health officer gives a conclusion A. *yes, he can
C. dietologist gives final conclusion B. no, he can’t
D. cook gives the conclusion C. he can but by the order of Ministry of Health
E. customer gives a conclusion D. he can but after rehabilitation treatment
158.The worker of “Bios" enterprise producing E. he can but after changing dosimeter
laundry detergents consults her health provider 164.The anestesiologist gives narcosis to the patient,
with complaints of rash at the opened areas of the he uses a non-reversive contour. Anesthetic is
face skin, neck and hands. What prevention is halothane. Air temperature in the operation room
necessary? is 21°С, humidity 50\%, level of noise 30 dB.
A. *to use hermetic equipment What occupational hazards is the most significant
B. to change assortment of the production in these conditions?
C. to use air conditioning A. *air pollution with anesthetic
KROK 2 – Question Bank 16
B. improper occupational microclimate workers has hard physical labour. What nutrition
C. high level of noise mode will you advise for them?
D. mental overfatigue A. *3-times meal, total calorie value 3700 kcal
E. compelled working pose B. 3-times meal, total calorie value 3200 kcal
165.The daily diet of a porter contains 110 g of protein C. 4-times meal, total calorie value 3200 kcal
(including 61 g of animal origin), 120 g of fat, 400 D. 5-times meal, total calorie value 5000 kcal
g of carbohydrates. His losses of energy are about E. liberated, total calorie value 3000 kcal
4000 kcal per day. What ways to optimize his 170.Wheat flour came to realization in the trade
nutrition could you advise? network. By the documents its sort was defined as
A. *increase carbohydrates content “extra”. Lab analysis data: color: white with
B. increase quantity of animal protein in the diet yellowish humidity: 17\% smell: acidous acidity:
C. increase quantity of vegetable protein 10о taste: acidous glutens: 15\% crunch: no
D. increase fat content Assess quality of product.
E. decrease quantity of animal protein A. *poor quality product, not suitable for using
166.Family physician advised for patient to rise B. flour of high quality, can be used w/o
vitamin C and calcium supply of the organism by limitations
drinking milk and eating sorrel. What mistake has C. conditionally suitable for using
he done? D. flour has low humidity, not suitable for using
A. *calcium is not absorbed in the presence of E. flour has high humidity, not suitable for using
oxalic acid 171.They inspect the pig corps at the abattoir. The
B. sorrel is poor in vitamin C flesh surface is light-red, section is some how wet,
C. milk does not content calcium spots at the blotting-paper don’t appear, elasticity
D. these foods are allergenic is normal. Fat is white with yellowish tint, solid,
E. alimentary correction hasn’t any advantages in in compression it crumbles, marrow is pink and
comparison with pharmacological one fills the bone channel completely. The meat has
167.Soldier Р,. 19 year old, was brought to the military specific smell of fresh flesh.. In tongue,
hospital with complaints on fatigability, gum diaphragm, intercostals and chewing muscles there
bleeding. Objectively: dryness and paleness of the are trichinas. What should they do with meat?
skin, acrocyanosis. Interdental spaces of the gums A. *use for technical utilization
edematous, gums are loosened, cyanotic. B. use for meal w/o limitations
Folliculitis signs. Vitamin С excretion with urine: C. use for meal after boiling
0,27 mg/hour. Estimate vitamin C supply of the D. use for meal after disinfecting
soldier organism and specify its standards for E. use for animal feeding by the contract with
ascorbic acid excretion with urine veterinarian inspection
A. *low (norm: 0,7-1,0 mg/h.) 172.A student has to measure the speed of air motion
B. low (norm: 2,0-3,0 mg/h.) in experimental laboratory. What device should he
C. low (norm: 1,0-1,5 mg/h) use?
D. high (norm: 0,1-0,2 mg/h.) A. *Cathethermometre
E. normal (norm: 0,2-0,6 мg/h.) B. Asman`s psychrometer
168.In school №75 they conduct the survey of the C. Hair hygrometer
children’s growth. The girl 7 year old has height D. Augustus psychrometer
of 133,3 cm (+2,1(), weight 29,0 kg (+1,0(), chest E. Anemometer
volume 60,1 cm (+0,5(). Estimate her growth 173.For training stamina teenager В., 15 year old,
development started to use a cold shower (water temperature
A. *the child has high height, disharmonic +6оС) for 20-30 minutes. In some days he stopped
development the procedures because of acute tonsillitis. What
B. the child has high height, harmonic water procedures will you advise for strengthening
development (training stamina) at the beginning?
C. the child has low height, harmonic A. *wiping
development B. bathing in the water reservoir
D. the child has average height, harmonic C. Scotch shower
development D. pouring
E. the child has low height, disharmonic E. gargling throat with cool water
development 174.Tooth decay morbidity among people living in N
169.The dockers work at the port in night schift from is 89\%. They drinks water containing 0,001 mg/l
2300 to 700. By heaviness of labour this group of fluorine. What prevention could you advise?
A. *water fluorination
KROK 2 – Question Bank 17
B. brush teeth red, рН of meat juice - 6,0. they found 5 fins at 40
C. inhalate fluorine cm2. Assess meat quality
D. eat more vegetables A. *poor quality product, not suitable
E. use sylantes B. high quality product, suitable w/o limitations
175.For providing health care of people living in a new C. high quality product, suitable with limitations
urban district they planned to build a new hospital D. surrogate product
for 550 beds, situated at the outskirts. The furthest E. adulterated product
buildings will be at 32 km from hospital. What 181.The mountain lake is situated out off the
service radius is the maximum for the urban residential area. The forest is around, shores are
hospitals? sandy and stony. Water analysis: Smell. Taste - 1
A. *15 km score, ammonia and nitrites – no, color –
B. 5 km colorless, nitrates - 40 mg/l, transparence - 40 cm,
C. 10 km chlorides - 50 mg/l, oxidation ability - 3 mg О2/l,
D. 20 km hardness - 10 mg eq/l, coli-titer - 300 ml, microbic
E. 30 km number – 65. Could this water be used for
176.On the 12th April 2000 in Odessa there was drinking?
variable cloudness and in-and-out precipitations, A. *water is suitable for drinking
atmospheric pressure - 755 mm of Hg, wind - 8,5 B. water is suitable only for technical purposes
m/s, daily overfall of temperature -7оС, C. water is suitable after boiling
atmospheric pressure - 10 mm Hg. What is the D. water is not suitable for using
medical type of the weather? E. water can be used after settling
A. *requiring medical control 182.A boy 8 year old is under the dispensary
B. very favorable supervision because of chronic gastroduodenitis.
C. favorable What group of physical education has the boy?
D. severe medical control A. *special
E. acute B. basic
177.The impairment of vision can occur in children C. additional
when carrying out different kinds of work. What D. preparatory
main hygienic conditions of labour can provide E. medical
the prophylaxis of these disturbance? 183.The observation department of maternity house is
A. * ensuring the good illumination of a child’s situated on the first floor under the delivery room.
working place In the department there are 10 2-bed wards and 2 –
B. the time restriction of fulfilling work 1-bed ones. The total amount of the beds is 22.
C. the quality of educational books Reception of pregnant women is conducted via the
D. a blackboard colour isolated unit of the reception ward. What
E. the distribution of children in the classroom infringements of sanitary rules could you specify?
178.The showing of an age-old tendency among the A. *department placement
younger sections of population is acceleration. B. structure of the department
which groups of hypotheses cause this C. total amount of beds
phenomenon D. reception mode of pregnant women and
A. * heliogenic, alimentary, social and economic women in childbirth
B. high-caloric mixtures of children’s nutrition E. No infringements
C. heterolocal marriages, early puberty 184.The worker of a forge shop consults his physician
D. city urbanization and ecology about complaints of poor hearing, fatigability,
E. specific prophylaxis, successes in pediatrics headache. Tonal audiography has been conducted.
179.Which term for the preparation of homework by There are signs of sensitive surditas on the
the pupils of the eighth – eleventh forms will meet audiogram. What prevention could you advise?
hygienic requirements? A. *use antiphones
A. * 3 hours B. hermetize the equipment
B. 1,5 hours C. eat caviar
C. 2 hours D. wear rubber shoes
D. 3,5 hours E. change managers at the enterprise
E. 4 hours 185.In the operating room of the purulent surgery
180.They confiscated 20 kg of pork from a seller at the department they amputated the shin in the patient
market. The surface of flesh is bright, condistence with gangrene. What way is usual for medical
is dense, fat is solid, smell is usual, color is light wastes disposal?
A. *incerination in special stoves
KROK 2 – Question Bank 18
B. dispose to landfills ham, Russian salad, stuffed cabbage-rolls and
C. bury at especial cemeteries other foods. He ate ice-cream with strawberries
D. bury at the hospital territory and rum. From his girlfriend it’s known that ice-
E. reduce to fragments and dispose to cream was melted and frozen again. What food
canalization caused poisoning?
186.For keeping meat soup with salted cucumbers in A. *ice-cream
the workers canteen they used a zinc-coated B. ham
bucket. In 2-3 hours after eating this soup 5 C. alcohol drinks
workers had vomiting. Sick men were D. Russian salad
hospitalized. In a day all signs disappeared. What E. stuffed cabbage rolls
is the disease? 192.The artesian chink is situated out off the
A. *zinc poisoning residential area, the nearest territory is free of
B. botulism pollution and covered. Water analysis: taste - 1
C. overeating score, oxidation ability - 0,5 mg О2/l, coli-tytre -
D. iron poisoning 500 ml, smell - 0 scores, ammonia – traces,
E. staphylococcal intoxication transparence – 40 cm, colority - 10?, microbic
187.After eating of raw duck eggs the family with number – 28, nitrites – traces, nitrates - 92,0 mg/l.
signs of gastroenterocolitis was delivered to What disease can be caused by drinking this
hospital t. What disease can be the most probable? water?
A. *salmonellosis A. *water-nitrate methemoglobinemia
B. erysipelas B. syderoachrestic anemia
C. leptospirosis C. talassemia
D. botulism D. dysentery
E. ergotism E. gastritis
188.Surgeon В., 36 years old, working at the thoracic 193.The parameter of infantile mortality for the last
department of the oncological dispensary. This year was - 16,3, in present year - 15,7. Name a
year he has been subjected to X-ray scopy for kind of the diagram that can be used for a graphic
several times (with esophageal contrasting). What representation of it.
maximum admissible dose is defined for this A. *Stylar.
professional category? B. Linear.
A. *2 mSv C. Intrastylar.
B. 20 mSv D. Sector.
C. 10 mSv E. Radial.
D. 5 mSv 194.Define the basic registration document at the
E. 1 mSv profound study of a case rate with temporary lost
189.For preventive purposes the student drinks mineral labor ability at the industrial enterprise:
waters. What mineral water can be used for A. *A card of the personal account of a case rate.
everyday drinking? B. "The Report on reasons of a temporary lost
A. *potable labor ability".
B. arsenic C. The leaf of disability.
C. boric D. A ambulatory medical card.
D. hydrosulfurous E. The inpatient medical.
E. radon 195.Head of department and a trade-union group have
190.For waste treatment they offer chloric lime with addressed to the head of hospital about dismissal
5\% content of active chlorine. What content of of the senior nurse that works during 17 years.
active chlorine is admissible for disinfecting at The facts of charge were confirmed and
least? recognized as the main nurse. This nurse lives
A. *20\% with a daughter (which does not work, she is
B. 1\% divorced) and 9 month grandson. Make a solution
C. 5\% from items of management.
D. 10\% A. *To keep the worker on a post with the
E. 15\% prevention of dismissal in case of repeated
191.They brought the patient Т., 32 year old, with violation of a labor discipline.
acute gastroenteritis signs to the reception ward of B. To discharge from office the worker with, i.e.
the municipal infectious hospital. He has fallen ill to satisfy demands of collective.
suddenly. Four hours ago he had a lunch with his C. The leaf of disability.
girlfriend. He drank alcohol beverages, ate some D. A ambulatory medical card.
KROK 2 – Question Bank 19
E. To shift a solution of this problem on other Requires prolongation of treatment. Who solves
officials or public organizations. the problem on the further temporary invalidity?
196.On the basis of the application form of parents and A. *Specialized (trоumatologic) MSEC.
explanatory note from the senior nurse of B. DCC.
reception department the chief of medical service C. The head physician of a polyclinic.
sent the official report to the head of hospital D. Interdistrict general MSEC.
about granting for a money by the senior nurse of E. Regional MSEC.
hospital the medicines from humanitarian fundfor 201.What is the maximum duration the leave of
treatment the child. What should make the head of disability during tuberculosis?
hospital in this situation from items of A. *2 months.
management? B. Week.
A. *To conduct service investigation by results of C. 2 weeks.
which to realize the administrative order D. Month.
according to the revealed circumstances. He E. 10 months
should to inform the chief of medical service, 202.The engineer - chemist in age of 47 years often
and in case of acknowledgement of the facts and duratingly is sick of occupational disease of a
inform the collective. skin. Who makes a decision to transfer him to
B. To not accept any activity. other job accepts?
C. To discharge from office the senior sister. A. *DCC.
D. To notify the chief of medical service about B. A head physician.
excess by him the official duties. C. The attending physician.
E. To transmit the official report of chief of D. The chief of shop.
medical service to the chief of a reception E. MSEC.
department for acceptance of effective 203.The patient with high temperature was addressed
measures for elimination henceforth similar on a medical assistant's health center in the
practice. evening. The fact of temporary lost labour ability
197.The child is 6 years old. For one year of was fixed. Indicate the order of examination in
observation he has URI duration 8 days. Physical this case?
worked out satisfactory. Define group of health: A. *The reference on remission of job is issued in
A. *I-st. a night duty, which in the subsequent is used
B. II-d. for issue of a medical certificate by date of the
C. III (a). previous day.
D. III (b). B. The leaf of disability for 1 day is issued.
E. III (c). C. The leaf of disability about 3 days is issued.
198.The employee was invalid during 6 months in D. The leaf of disability for 3 days is issued.
connection with fracture of a hip. Who has the E. What document is not issued.
right to authorize for issue a leaf of disability for 204.They took the sample from 5 tons milk batch. In
the last 2 months? the lab analysis it was defined: fat content 2\%,
A. *MSEC. specific density 1,04 g/cm3, acidity 21оТ,
B. Head physician of the polyclinic. reductase probe – weak positive. What way is the
C. DCC. product to be used in? Please advice.
D. DCC together with the head physician of a A. *sell but inform customers about milk quality
polyclinic. B. write off for animal feeding
E. Deputy a head physician on a working C. technical utilization
capacity. D. sell without limitations
199.The employee 6.03.2001 made abortion under E. do product away
medical signs and she was in a hospital till 205.The student has devices: Geiger counter, Ebert
17.03.2001. On what term the leave of disability is counter, Krotov’s apparatus, Mischuk device,
issued to her? Ebert device. What device can he use to assess air
A. *For 12 days. germ pollution
B. For 3 days. A. *Krotov’s apparatus
C. For 4 days. B. Ebert’s counter
D. For 10 days. C. Geiger’s counter
E. For 11 days. D. Mischuk’s device
200.The man of 38 years was admitted in the hospital E. Ebert’s device
from a place of job in July 19, concerning fracture 206.The student has devices: Geiger counter, Ebert
of a hip. He was invalid till November 19. counter, Krotov’s apparatus, Mischuk device,
KROK 2 – Question Bank 20
Ebert device. What device can he use to assess bilateral wet rales. Choose medicines for
meat quality treatment.
A. *Ebert’s device A. *Morphine, furosemide, nitroprusside sodium
B. Ebert’s counter B. Theophylline, prednisolon
C. Geiger’s counter C. Albuterol, atropine, papaverine
D. Mischuk’s device D. Strophanthine, potassium chloride,
E. Krotov’s apparatus plathyphylline
207.The student has devices: Geiger counter, Ebert E. Cordiamine, isoproterenol
counter, Krotov’s apparatus, Mischuk device, 213.An attack of severe substernal pain developed in a
Ebert device. What device can he use to assess air patient at night. On exam: confusion, pallor of the
radioactivity skin, acrocyanosis, cold sweating, BP of 80/50
A. *Geiger’s counter mm Hg, PR of 120/min, irregular and weak pulse.
B. Ebert’s counter Note, what condition are these features typical
C. Krotov’s apparatus for?
D. Mischuk’s device A. *Cardiogenic shock
E. Ebert’s device B. Acute left-sided heart failure
208.Student В. lives in the canalized house in the flat C. Acute right-sided heart failure
with complete set of sanitary equipment (WC, D. Radicular syndrome
bath, shower, local water heater). How much E. Acute vascular insufficiency
water consumption has he got? 214.A 61-year-old man complained of sneezing and
A. *160-200 l substernal pain on exertion. The prior 2 weeks
B. 10-15 l such pain had appeared at rest, with increased
C. 50-100 l frequency, and failed to respond to 1 tabl of
D. 300-400 l nitroglycerin. What is the most likely diagnosis?
E. 500 l A. *Unstable angina pectoris
209.All enumerated included all kinds of forensic B. Angina pectoris of a new onset
medical documents with the exemption of: C. Myocarditis
A. * Structure D. Radiculitis
B. Primary E. Stable angina pectoris of III functional class
C. Addition, repeated 215.A patient with ischemic heart disease and chronic
D. Committee heart failure develops sudden loss of
E. Complex consciousness; on exam, cyanosis, the widened
210.Classical X-ray image of intestinal obstruction is: pupils, peripheral pulse and blood pressure are not
A. *Gas and horizontal levels defined. On ECG: ventricular complexes are
B. Filling defect absent; instead of them there are waves of
C. High positioned diaphragm different shape and amplitude with irregular
D. Reactive pleuritis rhythm. What is the mechanism of this rhythm
E. Pneumatosis disturbance development?
211.A 15-year-old girl was examined with a history of A. *Multiple microreentry in the ventricles.
gradual onset of fever, malaise, loss of weight. B. Enhanced automatic activity of the ventricles.
There was nothing typical about the kind of fever, C. Disturbances of neurohumoral regulatory
which has been present for more than 7-10 days systems.
and changed quickly. Physical examination was D. Sick sinus syndrome.
unremarkable. What is the single most important E. Accelerated diastolic depolarization, a
examination for excluding miliary tuberculosis? disturbance in electrolyte balance.
A. * Chest x-ray 216.A 41-year-old woman complains of weakness,
B. Liver or bone marrow biopsy fatigue, fever up to 38°C, rash on the face skin,
C. Tuberculin skin testing pain in the wrists and the elbows. On physical
D. Sputum smear and culture of M.tuberculosis exam, erythematous rash on the cheeks with
E. Bronchoscopy “butterfly” appearance, the wrists and elbow joints
212.A 58-year-old man complained of severe are involved symmetrically, swollen, tender on
inspiratory dyspnea and expectoration of frothy motions, friction rub over the lungs, the heart
and blood-tinged sputum. He had been suffering sounds are weak, regular, HR of 88/ minute, BP of
from essential hypertension and ischemic heart 160/95 mm Hg. CBC shows anemia, leucopenia,
disease. On exam, acrocyanosis, “bubbling” lymphopenia; on urinalysis: proteinuria,
breathing, PR of 30/min, BP of 230/130 mm Hg, leukocyturia, casts. What is the main mechanism
of disease development?
KROK 2 – Question Bank 21
A. *Production of antibodies to double-stranded A. *Hodgkin’s disease.
DNA. B. Tuberculosis.
B. Production of antibodies to myocytes. C. Sarkoidosis.
C. Production of antibodies to endothelial cells. D. Tumor metastases.
D. Production of antibodies to myosin. E. Chronic myelocytic leukemia.
E. Production of antimitochondrial antibodies. 221.A 40-year-old man complained of headache in
217.A man with liver cirrhosis complained of nasal occipital area. On physical examination, the skin
bleedings, right subcostal pain, weakness, nausea. was pale; there was face and hand edema, blood
On physical examination: jaundice, hemorrhagic pressure of 170/130 mm Hg. On EchoCG, there
rash, enlarged liver span (of 14 cm), liver edge was concentric hypertrophy of the left ventricle.
irregular. What is the cause of hemorrhagic Ultrasound examination of the kidneys revealed
syndrome in this patient? thinned cortical layer. Urinalysis showed
A. *Decreased liver production of procoagulants. proteinuria of 3.5 g/day. What is the diagnosis?
B. As a consequence of DIC. A. *Chronic glomerulonephritis.
C. Thrombocytopenia. B. Essential arterial hypertension.
D. As a result of portal hypertension. C. Chronic pyelonephritis.
E. K and C hypovitaminosis. D. Polycystic disease of the kidneys.
218.A 56-year-old woman has an acute onset of fever E. Cushing’s disease.
up to 39°C with chills, cough, and pain on 222.A 29-year-old woman is critically ill. The illness
respiration in the right side of the chest. On was manifested by high fever, chills, sweating,
physical examination: HR of 90/minute, BP of aching pain in lumbar area, a discomfort in
95/60 mm Hg, PR of 26 per minute. There is urination, and frequent voiding. Pasternatsky’s
dullness over the right lung. On X-ray: infiltrate in sigh is positive in both sides. On lab exam, WBC
the right middle lobe of the lung. What is the of 20.000/mcL; on urinalysis protein of 0.6g/L,
diagnosis? leukocyturia, bacteriuria. Your preliminary
A. *Community-acquired lobar pneumonia with diagnosis.
moderate severity. A. *Acute pyelonephritis
B. Community-acquired bronchopneumonia. B. Exacerbation of chronic pyelonephritis
C. Acute pleurisy. C. Acute glomerulonephritis
D. Acute lung abscess. D. Acute cystitis
E. Hospital-acquired lobar pneumonia. E. Nephrolithiasis
219.A 36-year-old woman during 6 years has had 223.A 26-year-old man was admitted to the hospital
bronchial asthma. She is sick all the year. She is complaining of stabbing back pain on inspiration
working in the premises where walls are covered and dyspnea. On exam, BT of 37°C, PR of
by mould. She has allergy to aspirin, analgin, and 24/min, HR of 92/min, vesicular breath sounds.
acetaminophen. Now she has four asthma attacks There is a dry, grating, low-pitched sound heard in
per day, especially at night. Nasal breathing is both expiration and inspiration in the left lower
disturbing too. There is wheezing on expiration lateral part of the chest. What is the most likely
over the lungs. Skin tests with feathers, dust mites, diagnosis?
and wood dusts (maple and alder-tree) are A. *Acute fibrinous pleuritis
positive. Order treatment according to the type of B. Myocarditis
asthma and severity of its course C. Pneumonia
A. *Inhaled beclomethason 100 mcg t.i.d. after D. Acute bronchitis
previous inhalation of fenoterol. E. Pneumothorax
B. IV theophylline and clarithromycin P.O. 224.A 58 year-old man complained of pain and
C. Specific desensitization with dust mites and swelling of the left foot small joints. The skin over
feathers allergens. these joints was purple, BT was 38°C. For the last
D. Dexamethasone and theophylline P.O. 6 years he has had a few episodes of such arthritis
E. Ephedrine P.O. and inhaled isoproterenol with duration of each one up to 7 – 10 days. He
during an attack. also suffered from chronic tonsillitis. On exam,
220.A 44-year-old woman complained of weakness, there were abnormal features in the left
subfebrile fever, and pallor of the skin. Physical metatarsophalangeal joints. What is the
examination revealed the enlarged lymph nodes in pathogenetic mechanism of this disease
the right supraclavicular area. On X-ray film, there development?
was enlargement of bronchopulmonary and A. *Increased biosynthesis of uric acid
paratracheal lymph nodes. The liver was enlarged, B. Immune response to streptococcal infection
with increased firmness. What is the diagnosis? C. Hyperproduction of autoantibodies to collagen
KROK 2 – Question Bank 22
D. Decreased amount of chondroitin sulfate C. Clophelin
E. Production of antibodies to native DNA D. Kapoten
225.A 31-year-old man with past history of rheumatic E. Nifedipin
fever was severely ill and complained of fever up 230.A 62-year-old patient complains of rest dyspnea,
to 38 – 39°C, abdominal pain, dyspnea, heart pains. 3 years ago he had myocardial
palpitation; he felt ill 6 days prior. On exam, the infarction. Physical examination: orthopnea,
left heart border was shifted to the left, heart acrocyanosis, swollen cervical veins. Pulse – 92,
sounds were faint, there were systolic and total heart enlargement, the liver is enlarged by 7
dyastolic murmurs at the aortic area, BP of 160/30 cm, shin edema. What is the stage of chronic heart
mm Hg, positive Rumpel-Leede sign, enlargement failure [CHF]?
of the liver and the spleen, diarrhea, and dark urea. A. * CHF-2 B
What is the most likely diagnosis? B. CHF- 1
A. *Infective endocarditis C. CHF- 2 А
B. Rheumatic aortic valve disease D. CHF-0
C. Typhoid fever E. CHF-3
D. Acute viral hepatitis 231.A patient, aged 49, complains of fever of 37,5 0С,
E. Acute nephritis heart pain, dyspnea. S1 is clapping; S2 is
226.A 14 year old patient. He complains of chest pain, accentuated in the aortic area; opening snap,
temperature 38,5, breathlessness. He had acute presystolic murmur are auscultated. What is the
tonsillitis2 weeks ago. He is in a bad state. The most useful investigation for valvular disorder
skin is pale. Heart borders are widened, the tones assessment?
are weakened. Above all heart area you can hear A. * Echocardiography + Doppler–
pericardium friction sound. Electrocardiogramm: Echocardiography
the descent of voltage QRS, the inversion T. The B. Phonocardiography
liver is 3 sm enlarged. ESR – 4mm/h, ASL – 0 – C. Ballistocardiogram
1260, C-reaction protein +++. Your diagnosis: D. Chest x-ray
A. *Rheumatic pancarditis E. ECG
B. Rheumatic pericarditis 232.Physical examination of a person with chronic
C. Rheumatic myocarditis bronchitis reveals expansion of intercostal spaces,
D. Rheumatic endocarditis hyperresonant percussion note, decreased
E. Septic endocarditis whispered voice sounds. Chest x-ray shows
227.A 52-year-old patient with previously functional hyperinflated lungs, low and flattened diaphragm.
Class П angina complains of 5 days of intensified Which of the signs is helpful in diagnosing lung
and prolonged retrosternal pains, decreased emphysema?
exercise tolerance. Angina is less responsive to A. * All of them
Nitroglycerinum. Which of the following B. Low diaphragm
diagnosis is most likely? C. Hyperresonant percussion note
A. * IHD. Unstable angina D. Hyperinflated lungs
B. Cardialgia due to spine problem E. Expansion of intercostal spaces
C. IHD. Functional Class П angina. 233.A patient with nosocomial pneumonia presents
D. Myocarditis signs of collapse. Which of the following
E. Myocardial dystrophy pneumonia complications is most likely to be
228.An ECG of postinfartional [a year ago] patient accompanied with collapse?
shows pathological QS waves in leads VI-V3, I, A. * Septic shock
aVL. Determine the location of old myocardial B. Exudative pleuritis
infarction. C. Bronchial obstruction
A. * Septal and anterior D. Toxic hepatitis
B. Anterolateral E. Emphysema
C. Anterior 234.A 45-year-old man for 1 month has complained of
D. Inferior epigastric and right subcostal aching pain,
E. Postrolateral pruritus, indigestion, dark color of the urine and
229.A 52-year-old patient has hypervolaemic type of acholic stool, fever, and significant weight loss.
essential hypertension. Which of the following is On exam: jaundice, presence of Curvuasier’s sign.
necessary to prescribe either as monotherapy, or in US scan did not reveal stones in the gallbladder
a complex with other antihypertensive remedies? and choledochus. What is the most likely
A. * Hypothiazid diagnosis?
B. Dibazol A. *Cancer of the pancreas head
KROK 2 – Question Bank 23
B. Gallbladder stones B. Chronic type B gastritis
C. Chronic pancreatitis C. Peptic ulcer of the stomach
D. Chronic cholangitis D. Chronic type C gastritis
E. Chronic hepatitis E. Menetrier’s disease
235.A 34-year-old woman fell ill 3 months ago after 240.A 25 -year-old woman complained of edema on
cold exposure. She complained of pain in the hand the face and legs, elevation in blood pressure up to
and knee joints, morning stiffness, and fever up to 160/100 mm Hg, and weakness. She fell ill 3
38°C. Interphalangeal, metacarpophalangeal and weeks after sore throat. On urinalysis, protein of
knee joints are swollen, hot, with decreased ranges 0.5 g/L, erythrocytes of 17 – 20/field, leukocytes
of motions; ESR of 45 mm/hr, CRP (+++), of 2 – 3/field, erythrocyte casts. What treatment
Vaaler-Rouse test of 1:128. What group of should be initiated after establishing of the exact
medicines would you recommend to the patient? diagnosis?
A. *Nonsteroidal anti-inflammatory drugs A. *Penicillin OS
B. Cephalosporines B. Heparin
C. Tetracyclines C. Ceftriaxone
D. Sulfonamides D. Dipyridamole
E. Fluorchinolones E. Ciprofloxacine
236.A 47-year-old obese man complained of periodic 241.A 25-year-old woman complained of fatigue, hair
attacks of acute arthritis in the 1st left loss, and brittle nails. On exam, pallor of skin, PR
tarsophalangeal joint. Lab exam revealed of 94/min, BP of 110/70 mm Hg. On blood cell
increased serum level of uric acid. What is the count, Hb of 90 g/L, RBC 3.5•1012/L, color index
diagnosis? of 0.7, ESR of 20 mm/h. Serum iron level was 8.7
A. *Gout arthritis mcmol/L. what treatment would you initiate?
B. Reiter’s disease A. * Ferrous sulfate orally
C. Rheumatoid arthritis B. Iron dextrin injections
D. Rheumatic arthritis C. Vitamin B12 intramuscularly
E. Osteoarthritis D. Blood transfusion
237.A 22-year-old woman complained of right E. Packed RBCs transfusion
subcostal aching pain, nausea, and decreased 242.A 38-year-old man worked at roofing and drain
appetite. She fell ill 2 months after appendectomy piper production for 15 years. He seeks medical
when jaundice appeared. She was treated in an help for expiratory breathlessness on exertion, and
infectious hospital. 1 year later above mentioned dry cough. On exam, wheezes above both lungs,
symptoms developed. On exam: the subicteric grayish warts on fingers are seen. Factory
sclerae, enlarged firm liver. Your preliminary physician has diagnosed asbestosis. What method
diagnosis is the most important for this diagnosis?
A. *Chronic viral hepatitis A. *Chest X-ray
B. Calculous cholecystitis B. Bronchoscopy
C. Gilbert’s disease C. Blood gas analysis
D. Acute viral hepatitis D. Spirography
E. Chronic cholangitis E. Electrocardiography
238.A 50 -year-old woman for 1 year complained of 243.The patient has worked 13 years as a bulldozer
attacks of right subcostal pain after fatty meal. driver. He complains of dizziness, headache,
Last week the attacks have repeated every day and finger dumbness and pain at night. On exam,
become more painful. What diagnostic study tactile sensivity of peripheral type disturbes him,
would you recommend? ankle muscles are painful, and pulsation on a.
A. *Ultrasound examination of the gallbladder dorsalis pedis is weak. What is the most probable
B. Liver function tests diagnosis?
C. X-ray examination of the gastrointestinal tract A. *Vibration disease
D. Ultrasound study of the pancreas B. Raynaud’s disease
E. Blood cell count C. Syringomyelia
239.A 27 -year-old man complained of aching D. Atherosclerosis obliterans
epigastric pain just after meal, heartburn, and E. Periarteritis nodosa
nausea. Stomach endoscopy revealed a large 244.A worker of chemistry factory had occupational
amount of mucus, hyperemia and edema of exposure to lead for 20 years. Total blood count:
mucous membrane in gastric fundus with areas of RBC of 3.5•1012/L, Hb of 100 g/L, reticulocytes
atrophy. Establish the diagnosis. of 3.3\%, granulated erythrocytes of 40 on 50
A. *Chronic type A gastritis fields of view; lead blood concentration of 0.042
KROK 2 – Question Bank 24
mg\%, porphyrinurea of 0.32 mg/L. What is the shows signs of normocytic and normochromic
probable diagnosis? anemia. What is the preliminary diagnosis?
A. *Chronic lead intoxication, II stage A. *Plumbism, severe form
B. Chronic lead intoxication, I stage B. Mercurialism, severe form
C. Porphyria C. Aluminium intoxication, severe form
D. Hereditary hemolytic anemia D. Asbestosis, severe form
E. Lead carrier E. Cyanides poisoning, severe form
245.A fitter of metallurgic factory with occupation 249.A 45-year-old driver was admitted to the hospital
exposure to high concentrations of mercury fumes with 5 hour substernal pain. Nitroglycerin is not
during 16 years presents instability of pulse and effective. He is pale, heart sounds are regular but
blood pressure, general hyperhydrosis, weak. HR 96 per minute, BP of 100/60 mm Hg.
asymmetric innervations of facial muscles and What is the most probable diagnosis?
tongue, positive subcortical reflexes, hand tremor A. *Acute myocardial infarction
on physical examination. Dentist consultation B. Stable angina
revealed paradontosis, and chronic stomatitis. C. Pulmonary embolism
What is the most probable diagnosis? D. Acute myocarditis
A. *Chronic mercury intoxication. E. Acute left ventricular failure
B. Neuroinfection. 250.A 38 -year-old woman is seriously ill. She
C. Parkinson syndrome. complains of frequent paroxysms of expiratory
D. Acute mercury intoxication. dyspnea. The last paroxysm lasted over 12 hours
E. Mercury encephalopathy. and failed to respond to theophylline. The skin is
246.A 45-year-old coal miner complains of cough with palish gray, moist, RR of 26/min. On auscultation,
black sputum, breathlessness on exertion, which breath sounds are absent over some areas. Your
occurred 4 years before. On physical examination, preliminary diagnosis
wheezes above both lungs, heart sounds are A. *Bronchial asthma, status asthmaticus
without changes, heart rate of 72 beats per minute. B. Chronic obstructive bronchitis
Chest radiography shows multiple, small irregular C. Atopic bronchial asthma, respiratory failure of
opacifications throughout both lungs. What is the the III degree
most probable diagnosis? D. Bronchiectasis, respiratory failure of the II - III
A. *Anthracosis, nodular, slowly progressing degree
form, first stage. E. Ischemic heart disease, pulmonary edema
B. Silicosis, nodular, slowly progressing form, 251.A 46- year-old patient has ischemic heart disease,
first stage. angina on exertion, II functional class. What is the
C. Siderosis, interstitial, slowly progressing form, drug of choice in treatment of acute attack?
first stage. A. *Nitroglycerin sublingually
D. Bissinosis, interstitial, slowly progressing B. Platelet inhibiting agents (aspirin)
form, first stage. C. Spasmolitics (No-spa) IV
E. Asbestosis, interstitial, slowly progressing D. Digitalis IV
form, first stage. E. Sedative agents (Seduxenum) orally
247.A 40-year-old woman who has worked in weaving 252.A 19-year-old girl admitted to the hospital
branch for 10 years complains of frequent complained of pain in the knee and fever of
headache, sleeplessness, irritability, fatigue, 38.6°C. She is ill for 2 weeks after acute
tiredness. Physical examination revealed tonsillitis. On exam, hyperemia and swelling of
instability of blood pressure, internal organs are both knees, BT of 37.4°C, HR of 94/min, BP of
without changes. What is the most probable 120/80 mm Hg, and heart border is displaced to
diagnosis? the left; S1 is weak, systolic murmur is present.
A. *Noise-induced disease. Total blood count shows the following: Hb – 120
B. Hypertension. g/L, WBC – 9.8•109/L, ESR of 30 mm/L. ECG
C. Atopic bronchial asthma. findings: the rhythm is regular, PQ = 0.24 sec.
D. Asteno-vegetative syndrome. What is a causative agent of the disease?
E. Encephalopathy. A. *Beta-hemolytic streptococci.
248.A worker of a printing house complains of B. Viral-bacterial association.
abdominal pain, constipation during last 5 days. C. Autoimmune disorder.
He presents a liliac line at the gingival-tooth D. Staphylococci.
border, tachycardia of 100/min, BP of 160/90 mm E. Ricchetsia.
Hg, painful abdomen on palpation. 253.A 42-year-old woman complains of dyspnea,
Aminolevulinic acid in plasma is elevated. CBC edema of the legs, and tachycardia during small
KROK 2 – Question Bank 25
physical exertion. Heart borders are displaced to of left ventricular hypertrophy on ECG. What is
the left and S1 is accentuated, there is the most probable diagnosis?
protodiastolic murmur on xiphoid process. The A. *Essential hypertension, 2nd stage
liver is enlarged by 5 cm. What is the cause of B. Essential hypertension, 1st stage
heart failure? C. Symptomatic hypertension
A. *Mitral stenosis D. Cardiomyopathy
B. Mitral regurgitation E. Ischemic heart disease
C. Tricuspid stenosis 258.A 39 -year-old woman complained of squeezed
D. Tricuspid regurgitation epigastric pain 1 hour after meal and heartburn.
E. Aortic stenosis She had been ill for 2 years. On palpation, there
254.A 33-year-old man with a history of rheumatic was moderate tenderness in pyloroduodenal area.
fever complains of fever up to 38 - 39°, abdominal Antral gastritis was revealed on gastroscopy. What
pain, dyspnea, tachycardia. Heart borders are study can establish genesis of the disease?
displaced to the left by 2 cm, systolic and diastolic A. *Revealing of Helicobacter infection in gastric
murmurs above aorta, BP of 160/30 mm Hg. mucosa
Petechial rash occurs after measurement of blood B. Detection of autoantibodies in the serum
pressure. Liver is enlarged by 3 cm, spleen is C. Gastrin level in blood
palpable. Urine is brown-yellow. What is the most D. Examination of stomach secretion
probable diagnosis? E. Examination of stomach motor function
A. *Infectious endocarditis 259.A 20- year-old patient with a history of preceding
B. Rheumatic fever streptococcal infection complains of malaise,
C. Acute hepatitis headache, anorexia, subfebrile fever. On exam,
D. Acute nephritis mild generalized edema, BP of 150/90 mm Hg,
E. Aortic regurgitation HR of 100/min, RR of 20/min, the urinalysis
255.A 60-year-old man complains of fever, significant showed increased protein, red cell casts and
weight loss, bone and joint pain, and bleeding hyaline casts. What is your diagnosis?
gums. On exam, paleness, lymphadenopathy, A. *Acute glomerulonephritis
hepato- and splenomegaly. CBC: WBC – B. Acute pyelonephritis
270•109/L with 13\% lymphocytes, 1\% C. Rheumatic fever
monocytes, 21\% basophiles, 29\% neutrophils, D. Essential hypertension
9\% blasts, 12\% promyelocytes, 12\% E. Bacterial endocarditis
myelocytes, 2\% metamyelocytes, 1\% 260.A 32 -year-old welder complains of weakness and
eosinophils. ESR – 22 mm/h. Name the drug for fever. His illness started as tonsillitis a month
treatment. before. On exam, BT of 38.9°C, RR of 24/min,
A. *Myelosan HR of 100/min, BP of 100/70 mm Hg,
B. Prednisolone hemorrhages on the legs, enlargement of the
C. Cytosar lymph nodes. CBC shows Hb of 70 g/L, RBC of
D. Vinblastine 2.2•1012/L, WBC of 3.0•109/L with 32\% of
E. Blood transfusion blasts, 1\% of eosinophiles, 3\% of bands, 36\% of
256.A 54-year-old woman complains of increasing segments, 20\% of lymphocytes, and 8\% of
fatigue and easy bruising of 3 weeks’ duration. monocytes, ESR of 47 mm/h. What is the cause of
Physical findings included pale, scattered anemia?
ecchymoses and petechiae and mild A. *Acute leukemia
hepatosplenomegaly. CBC: RBC – B. Chronic lympholeukemia
2.550.000/mcL; Hb – 73 g/L; HCT 20\%; PLT – C. Aplastic anema
23.000/mcL; and WBC – 162.000/mcL with 82\% D. Vitamin B12 deficiency anemia
blasts, that contained Auric rods; peroxidase stain E. Chronic hemolytic anemia
was positive; What is the most probable 261.A 30- year-old patient complains of
diagnosis? breathlessness, pain in the right rib arc place, dry
A. *Acute leukemia cough and the leg edema. He is ill for 2 months.
B. Chronic leukemia He was treated for rheumatic fever without effect.
C. Thrombocytopenia On exam, cyanosis, edema of the legs, BT of
D. Hemolytic anemia 36.6°C, RR of 28/min, HR of 90/min, BP of
E. Megaloblastic anemia 110/80 mm Hg, crackles above low parts of both
257.A 60-year-old woman has increased BP up to lungs, heart borders are displaced to the left and to
210/110 mm Hg during last 7 years. On exam, the right, weak sounds, systolic murmur above the
heart apex is displaced to the left. There are signs apex. What is the preliminary diagnosis?
KROK 2 – Question Bank 26
A. *Dilated cardiomyopathy D. Gallstones
B. Infectious endocarditis E. Viral hepatitis B
C. Acute myocarditis 267.A 27-year-old man complains of pains in
D. Rheumatic fever, mitral stenosis epigastrium which are relieved by food intake.
E. Acute pericarditis EGDFS shows antral erosive gastritis, biopsy of
262.The risk of cancer being the result of radiation antral mucous presents Hеlicobacter Pylori.
doses of 100 Gray or less to the whole body is Diagnosis is:
difficult to estimate reliably. The reasons for this A. * Gastritis of type B
uncertainty are : B. Gastritis of A type
A. * All of the below mentioned C. Reflux - gastritis
B. The high incidence of cancer of the general D. Menetrier's gastritis
population E. Rigid antral gastritis
C. The low risk of cancer from small doses 268.A man, aged 25, presents with facial edema,
D. The long interval between radiation and moderate back pains, body temperature of 37,5 С,
clinical presentation of cancer. BP 180/100 mmHg, hematuria [ up to 100 in v/f],
E. None of the above mentioned proteinuria [2,0 g/L], hyaline casts - 10 in v/f.,
263.For which of the following diagnostic specific gravity -1020. The onset of the disease is
examinations performed with modern techniques probably connected with acute tonsillitis 2 weeks
on an unknown pregnant patient should ago. The most likely diagnosis is:
therapeutic abortion be considered the result of A. * Acute glomerulonephritis
putative risk from radiation? B. Acute pyelonephritis
A. * None of the below mentioned C. Cancer of the kidney
B. Chest radiography D. Urolithiasis
C. Barium enema study E. Chronic glomerulonephritis
D. IVP 269.A patient had stomach resection a year ago. He
E. Abdominal CT complains of general weakness, giddiness. Blood
264.The highest risk of congenital anomalies probably count: Er 2,6 g/L, Hb 80 g/L, C.ind 0.7, L – 3.7
occurs when human embryos or fetuses are g/L, reticulocytes 1\%, segm 56\%, lymp 34\%,
exposed to ionizing radiation. During which part mon. 6\%, ESR 17 mm/hour. Erythrocytes are
of gestational period does it occur? hypochromic; there are anisocytosis & poikilo-
A. *18-45 days after conception cytosis. Fe of serum 5 mkmol/L. Diagnosis is:
B. The first 7 days A. * Iron-deficiency anemia
C. 10-14 days after conception B. B12-deficiency anemia
D. 90-120 days after conception C. Сhronic myeloleukosis
E. The third trimester D. Aplastic anemia
265.A 38-year-old patient has been treated in a E. Chronic lymphoid leukosis
hospital. A fever of 39 C, chest pain which is 270.A patient of 62 years with DM-2. Diabetes is
worsened by breathing, cough, brownish sputum being compensated by diet and Maninilum. Pаtient
appeared on the 7th day of the treatment. Chest x has to undergo an operation for inguinal hernia.
ray shows left lower lobe infiltrate. Which of the What should be tactics of hypoglycemic therapy?
following is the treatment of choice for this A. * Prescribe the drugs of an insulin of short
patient? activity
A. * Cephalosporins of the Ш generation B. Give Glurenorm in place of Maninilum.
B. Penicillin C. To continue with the current therapy
C. Erythromycin D. Prescribe the drugs of insulin of long activity
D. Tetracycline E. Prescribe guanyl guanidines
E. Streptomycin 271.A 33-year-old lady has been suffering from DM
266.A patient, aged 48, complains of heaviness in the for 5 years. The last 3 years she has taken more
right hypochondrium, itching of the skin. than 100 units of insulin per day. Body weight has
Repeatedly he had been treated in infectious increased up to 10 kg. Fasting blood glucose is 13
diseases hospital due to icterus and itch. mmol /L, glucoseuria - 3\%. Generalized
Objectively: meteorism, ascitis, dilation of microangiopathy. By increasing the dose of
abdominal wall veins, protruded navel, spleen insulin the parameters of glycemia do not change.
enlargement. Diagnosis is: The diagnosis is:
A. * Liver cirrhosis A. * DM 1st type, severe form, decompensation,
B. Cancer of the liver insulin resistant
C. Cancer of the head of pancreas B. DM 2nd type, severe form, decompensation
KROK 2 – Question Bank 27
C. DM 1st type, severe form, subcompensation, 277.In a male aged 25 focal shadowings of small and
Somoji phenomenon medium intensity with unequal contours in the 1st
D. DM 2nd type, moderate form, Zabrodi and 2nd segments of the right lung were revealed
phenomenon during prophylactic photoroentgenography
E. DM 1st type, severe form, decompensation, investigation. Which clinical form can be
allergic response on insulin suspected in this patient?
272.A patient of 32 complains of severe weakness, A. *Focal
tremor of extremities. Objective examination: B. Disseminated
body weight loss, wet & warm skin. The thyroid C. Miliary
gland is enlarged up to the 3rd degree, painless, D. Fibro-cavernous
elastic. Pulse: 108. BP- 160\55 mmHg. Everything E. Tuberculoma
else is normal. The diagnosis is: 278.A woman 26 years old has abused alcohol for 7
A. * Diffuse toxic goiter of the 3rd degree, years. She has psychological dependence on
thyrotoxicosis of the average degree alcohol, but no withdrawal syndrome. Drinks
B. Diffuse euthyroid goiter of the 3rd degree. almost every day approximately 50 – 100 gr. of
C. Chronic autoimmune thyroiditis, hypertrophic wine. Is at her 4-th week of pregnancy. Primary
type prevention of fetal alcohol syndrome requires:
D. Chronic fibrous thyroiditis A. *Treatment of alcoholism and full abstinance
E. Toxiferous adenoma of the thyroid gland from alcohol during all the period of
273.In the development of the inflammation processes pregnancy
glucocorticoids reduce the level of a certain most B. Medical abortion
important active enzyme. It results also in the C. Decrease of alcohol use
reducing of the synthesis of prostaglandins and D. Participation in the A-ANON group
leukotrienes which has a key-role in the E. Gyneacological observation
development of the inflammation processes. Give 279.Interpret GTT. Glycemia: I trial – 5,3 mMol/l, II
the exact term of this enzyme. trial – 8,2 mMol/l, III trial – 4,8 mMol/l
A. * Phospholipase A2 A. *Normal
B. Arachidonic acid B. Impairment of carbohydrate tolerance
C. Lipoxygenasе C. Diabetes mellitus
D. Cyclooxygenase – 1 D. Necessary to repeat test
E. Cyclooxygenase – 2 E. Necessary to order additional laboratory tests.
274.А patient is suffering of a chronic heart 280.What points, concerning myxedema coma are
insufficiency [degree II; phase A]. The patient has correct?
been given a proper therapeutic treatment along A. * Hypothermia is common
with furosemide. Later the patient developed a B. Feeling warm may provoke myxedema coma
lumbosacral nerve root syndrom. To reduce the C. Cause is – increased sensitivity to T 3 and T4
acute pains the doctor prescribed a certain agents, receptors.
which lowered the effect of furosemide. Give the D. Hyperfunction of adrenal glands
name of this medicine. E. Thyroid cancer
A. * Indomethacin 281.Developing of gigantism is conditioned by:
B. Digoxin A. *Overwhelming secretion of GH in
C. Furosemide adolescence
D. Panangin B. Overwhelming secretion of GH in old age
E. Riboxinum C. Overwhelming secretion of GH in adults
275.Which of the following is used for tuberculin D. Overwhelming secretion of somatostatin in
diagnosis in the masses adolescence
A. *PPD-L standard dilution of 2 TU in 0,1 ml E. Inborn sensitivity lack in tissues to GH
B. ATK tuberculin 282.Choose factors, which can cause Addison’s
C. PPD-L diluted in 5 TU in 0,1 ml disease
D. Dry pure tuberculin A. *Tuberculosis
E. Tuberculin in the form of ungutum B. Tumor acting on adrenal hormone
276.Vaccination of BCG should be conducted on: C. Autoimmune destruction of thyroid gland
A. *5 days D. Diabetes Mellitus
B. 3 months E. Brain tumor
C. 10 days 283.To remove onset of tetany you will order
D. 12 months A. *Calcium chloride
E. 5 years B. Benzylpenicillin
KROK 2 – Question Bank 28
C. Prednisolone years. Heard voices commenting on his actions,
D. Potassium citrate had delusions of persecution [was sure that the
E. Magnesium sulphate Mafia wanted to kill him]. After a course of
284.Macroangiopathy, as a symptom of diabetes treatment with neuroleptics was discharged from
mellitus, most often destroy vessels of: hospital with the diagnosis of schizophrenia, state
A. *Brain of remission. The secondary prevention of the
B. Lung relapses of schizophrenia requires:
C. Kidneys A. * Supportive treatment with neuroleptics of
D. Retina prolonged action
E. Intestine B. Long-term hospitalization
285.The treatment of Grave’s disease usually include C. Psychiatric observation
A. *Antagonists of thyroid hormones D. Participation in a self-help group
B. Antidepressants E. Psychoanalytic treatment
C. Narcotic analgesics 291.Patient K, male, 19 years old, has suffered
D. Diuretics moderate mental retardation since childhood. Is
E. Sulfonylureas illiterate, can take care of himself, do simple
286.Choose the correct initial interaction of a photon household work and other kinds of easy work
with an atom in a biological system leading to cell under supervision. His rehabilitation [tertiary
damage prevention] requires:
A. * Interaction of the photon with a subatomic A. * All the above mentioned
particle. For x-ray imaging radiation this will B. Supervision of a social worker
be an electron. C. Physical work under supervision
B. Production of an energetic changed particle D. Supervision of relations [ if any]
usually a photo or recoil electron. E. None of the above mentioned
C. Dissipation of the energy of the electron along 292.Patient F., male, 16 years old was behind other
its path by ionization and excitation. children in development since early childhood and
D. Chemical changes in the ionizing / excited still has moderate mental retardation. He is short,
molecules. has dismorphic body, his face is round, flattened,
E. None of the above mentioned his eyes are narrow and slanted, and there are
287.Which of the following statements concerning epicantial folds in the corners of his eyes. There is
undifferentiated cells in tissues is correct? only one transversal flexor line on his palms.
A. * They are sensitive to radiation but easily What is the probable etiology of this state?
replaced when killed A. * Chromosome abnormality
B. They are very resistant to radiation B. Gene abnormality
C. They are sensitive to radiation and most are C. Maternal alcohol abuse during pregnancy
killed by doses in the diagnostic range D. Pathological delivery
D. They are sensitive to radiation and difficult to E. Infection in mother during pregnancy
replace if killed. 293.Patient A., male, 27 years old came to consult a
E. Cells that survive can't go on to form a tumor psychiatrist with the following complaints: after a
288.Which of the following symptoms would occur bad car accident, the victim of which he was and
only if a total-body acute radiation exposure in which his wife was killed, he suffered
exceeded 5.000 rad (50 Gy) depression, anxiety, flash-backs of the event, sleep
A. * Hallucinations and impairment of vision disorders and nightmares about the accident. He
B. Nausea and Vomiting also had emotional numbness and fatigue. The
C. Diarrhea cause of the disorder was:
D. Bleeding Gums A. * Severe stress
E. Epilation (hair loss) B. Adjustment disorder
289.A total body dose of 200 rad will cause the C. Environmental factors
hematopoietic syndrome. The time for maximum D. Patient’s personality features
hematological recovery will be : E. Endogenic factors
A. * 180 Days 294.A female patient 28 years old, became depressed,
B. 30 days her mood is melancholic; this state is associated
C. 60 days with hypobulia, hypokinesia, slow speed of
D. 120 days thinking. Her attitude towards her past present and
E. 240 days future is pessimistic. The pathogenetic
290.Patient N., 27 years old was hospitalized to the mechanism of this state is supposed to involve
psychiatric hospital for the 4-th time during 2 dysfunction in the:
KROK 2 – Question Bank 29
A. * Hypothalamus E. Syndrome of pulmonary tissues augmented
B. Frontal lobes aerisation.
C. Pituitary 300.The primary bronchopneumonia more often arises
D. Hippocampus as …
E. Corpus callosum A. * Complication of acute bronchitis.
295.A patient, while making repairs at home, suddenly B. Complication of pneumorrhagia.
stood quite still with the painting brush in his C. Complication of stagnation of blood in the
hand. This state lasted for a few seconds. After lungs.
that the patent was rather confused, for some D. Complication of infarct of the lungs.
moments he couldn’t understand what was E. Complication of pneumoconiosis.
happening. He totally forgot the state he was in 301.What auscultative data of the lungs does in lobar
and the events around him, occurring while he was pneumonia exist at stage of hepatization?
in that state. Name the disorder: A. * Bronchial breathing.
A. * Petit mal, epilepsy B. Crepitation.
B. Grand mal, epilepsy C. Moist consonating rales.
C. Jacksonian fit, epilepsy D. Moist non-consonating rales.
D. Disphoria, epilepsy E. Increased vesicular breathing.
E. Twilight state, epilepsy 302.What is the basic of crepitation?
296.The observed patient’s movements are retarded, A. * The separation of alveoli during inspiration
she answers no questions. Sometimes she on walls of which the fibrin has put.
spontaneously stays in strange postures. It is B. Existence of bronchiectasis filled by pus.
possible to set [form] her body and limbs into C. Existence of a cavern containing liquid and air.
different positions artificially. If the psychiatrist D. Stenosis of a clear space of bronchus.
lifts her arm or leg, so that she remains standing E. Friction of the inflamed pleural layers during
on the other leg, the patient can stay in such a respiration.
position for quite a long time. Name the probable 303.Sputum “full mouth” [is more often in morning
disorder: time] is characteristic for:
A. * Catatonic stupor, shizophrenia A. * Bronchoectatic disease.
B. Depressive stupor, bipolar disorder B. Pulmonary tuberculosis.
C. Apathetic stupor, shizophrenia C. Focal pneumonia.
D. Psychogenic stupor, stress disorder D. Acute bronchitis.
E. Dissociative stupor, dissociative psychosis E. Empyema of pleura.
297.Patient T., female, 35 years old, suffers from 304.What is auscultated in syndrome of infiltration of
epileptic fits since she was 15 years old. She has pulmonary tissue?
seizures during which she loses consciousness, A. * Pathological bronchial breathing.
falls down and has tonic and clonic convulsions. B. Intensified vesicular breathing.
The fits occur once a week. What are the main C. Decreased vesicular breathing, dry rales.
principles of treetment in epilepsy? D. Harsh.
A. * All the above mentioned E. Amphoric breathing.
B. Individual approach 305.When can pulmonary bleeding arise?
C. Gradual beginning A. * Bronchoectatic disease.
D. Continuous long-term treatment B. Diffuse catarrhal bronchitis.
E. None of the above mentioned C. Bronchiolitis.
298.What auscultative data are watched in D. Lobar pneumonia in a stage of red
bronchoectatic disease? hepatization.
A. * The clinical picture depends on full or empty E. Bronchopneumonia.
bronchiectasia and on caliber of bronchus. 306.What auscultative phenomenon arises at beginning
B. Bronchial breathing. of acute bronchitis?
C. Amphoric breathing. A. * Harsh.
D. Dry whistling rales. B. Moist fine bubbling rales.
E. Moist fine bubbling rales [non-consonating]. C. Crepitation.
299.What from enumerated syndromes is main in D. Moist medium bubbling rales.
acute diffuse bronchitis? E. Vesiculo-bronchial breathing.
A. * Syndrome of muco-ciliary insufficiency. 307.Symptoms of the effected pallidar system do not
B. Syndrome of bronchial obstruction. include?
C. Syndrome of respiratory insufficiency. A. *Hemibalism
D. Syndrome of pulmonary tissues insufficiency. B. Plastic hypertension
KROK 2 – Question Bank 30
C. Bradikinesia A. * Haemolytic anemia
D. Hipomimia B. Aplastic anemia
E. Micrografy C. Iron deficiency anemia
308.What of the mentioned symptoms isn't D. Hyper sideric anemia
characteristic of poliomyelitis? E. Normochromic anemia
A. *Disturbance of sensitiveness 315.The man, aged 42, applied to the therapeutist with
B. Hypotension complaints of pricking pains in scapulas area,
C. Hyporeflexia dyspnea on physical exertion, cough with
D. Hypotrophy discharge of small amount of sputum. During 10
E. Torpid distal paralysis years he works in coal mining. On percussion-
309.Name the symptom of stretching of the ishiatic box-note sound in the lower parts, on auscultation-
nerve? a harsh breathing. There were no changes in the
A. *Lasseg's heart. Possible diagnosis?
B. Lessage's A. * Silicosis
C. Brudzinsky's B. Tuberculosis of lungs
D. Kernig's C. Silicatosis
E. Vasserman's D. Bronchiectatic disease
310.Name the nuclei of the cranial nerves which are E. Chronic bronchitis
affected Weber's alternating syndrome? 316.A sick man M., aged 52, a street cleaner. He was
A. *Oculomotor ill with pneumoconiosis during 2 years. It’s
B. Accessory nessesary to recommened for the treatment of this
C. Glossopharyngeal disease:
D. Vagus A. * Alkaline inhalations
E. Hypoglossal B. Broncholitics
311.Neuralgia of the trigeminal nerve is characterized C. Oil inhalations
by? D. Sulfonilamides
A. *Transient pain on the face and trigger areas E. Antibiotics
B. Permanent pain in the area of innervation 317.The woman, aged 42, works at the factory on the
C. Loss of sensitivity on the face fabrication of mercury thermometers, complains
D. Trophical disturbances of cornea and loss of of the headache, swoons, reduction of memory,
corneal reflex small and frequent flutter of fingers of drawn
E. Positives effect from sponging by 5\% solution hands, the eyelids and the tongue, bleeding gums,
of cocaine on the posterior surface of the gingivitis. What preparation is it nessesary to use
mucosa of the middle turbinate bone for the elimination of mercury from the organism?
312.Central paresis of the mimic muscles is
characterized by: A. *Unithiol
A. * Affection of mimic muscles of the lower half B. Pentoxil
of the face? C. Magnesium sulphate
B. Affection of masticatory group of muscles D. Sodium hydrate of carbon
C. Prolapse of papillary reflex E. Seduxen
D. Trophic disorders 318.A man,aged 37,working on the collective farm on
E. Affection of mimic muscles of half of the face sowing,was admitted to the infectious hospital
313.The patient H., aged 36,works as a nightman with the clinical symptoms: miosis,labored
during 12 years.He applied with complaints of the breathing,sweating.What kind of poisoning is it
headache, loss of appetite, syncopal conditions, and what is the first aid?
metallic taste in the mouth, sometimes A. *Poisoning by POC. Treatment: atropine
stomachache, dispeptical frustration, diarrhea and B. Poisoning by lead. Treatment: tetacine Calcii
constipation. Objective: fragility of nails, C. Poisoning by the methylic alcohol. Treatment:
hyperkeratosis of palms. Diagnosis? ethylic alcohol
A. * An acute intoxication D. Poisoning by vapours of mercury.
B. Chronic arsenious intoxication from pesticides Treatment:unithiol
C. An acute poisoning by OPS E. Seduxen
D. Carbone bisulfide intoxication 319.A woman of 36 years is on the 12-th week of the
E. Leaden intoxication first pregnancy. We know from the history that
314.A diagnosis of chronic arsenious intoxication was she was treated for infertility. She was in the
defined in a patient Y., a nightman. What form of guests and contacted with child who developed
anemia is characteristic in this disease? rubella in 2 days after meeting. Woman doesn’t
KROK 2 – Question Bank 31
know if she has ever been infected with rubella. especially in calf muscles. The dermal
What is the adequate tactics? integuments and scleras are icteric. There is
A. *Monitory of the specific Ig G Ig M with the hemorrhagic rash on the skin. Urine is bloody.
ELISA The patient went fishing two weeks ago. What is
B. Interruption of the pregnancy the diagnosis?
C. Immune globulin injection A. *Leptospirosis
D. Cyclovin administration B. Yersiniosis
E. Interferon administration C. Salmonellosis
320.Patient L., 50-year old, has been admitted to the D. Brucellosis
clinics with atrophic gastritis. In the blood test: E. Trichinellosis
erythrocytes 3.8 T/L, Hb 68 g/l, c.i. 1, 325.The patient has been in the hospital. The
macroanisocytosis, poikilocytosis. There is a beginning of the disease was gradual: nausea,
megaloblastic type of haemopoesis. A number of vomiting, dark urine, аcholic stools, yellowness of
leukocytes, reticulocytes and thrombocytes is the skin and scleras. The liver is protruded by 3
lowed. Which pathology is suspected?i cm. Jaundice was intensified on the 14th day of
A. *B-12-deficiency anemia the disease. The liver diminished in sizes. Due to
B. Irondeficiency anemia what complication of viral hepatitis, has the
C. Hemolytic anemia patient’s condition worsened?
D. Post-hemoragic anemia A. *Hepatic encephlopathy
E. Thalassaemia B. Meningitis
321.A 32-year old woman developed the Laiel’s C. Relapse of viral hepatitis
syndrome after taking the biceptol. What D. Cholangitis
immunotrope remedies are indicated in this E. Infectious-toxic shock
situation? 326.The patient, 18-years-old was admitted to the
A. *Steroid immunodepressants hospital with complaints of headache, weakness,
B. Non-specific immune modulators high temperature, pain in the throat. Objectively:
C. Specific immune modulators enlargement of all groups of lymphatic nodules
D. Interferons was revealed. The liver is enlarged by 3 cm,
E. Non-steroid immunedepressants spleen - by 1 cm. In the blood - leukocytosis,
322.The patient 25-years-old was admitted on the 1st atypical lymphocytes - 15\%. What is the probable
day of the disease with complaints of double diagnosis?
vision in the eyes, difficult respiration. The day A. *Infectious mononucleosis.
before the patient ate home-made mushrooms. On B. Acute lymphoid leukosis.
objective examination: paleness, widened pupils, C. Diphtheria.
disorder of swallowing, bradycardia, constipation D. Angina.
are marked. What is the diagnosis? E. Adenoviral infection.
A. *Botulism 327.The patient 28-years-old was hospitalized with
B. Yersiniosis preliminary diagnosis "influenza". Roseolous-
C. Leptospirosis petechial rash appeared on the 5th days of disease
D. Salmonellosis, gastrointestinal form on the trunk. The temperature is 41(С. Hyperemia
E. Lambliasis of the face, reddening of scleras, tremor of the
323.The patient Н., of 28 years old, was admitted to tongue, tachycardia, splenomegaly are marked.
the clinic with complaints of the temperature What is the most probable diagnosis?
increase up to 39,0(С, headache, weakness, A. *Epidemic typhus
constipation on the 9th day of the disease. On B. Measles
examination: single roseolas are on skin of the C. Alcohol delirium
abdomen. The pulse rate is 78 per minute. The D. Leptospirosis
liver is enlarged by 2 cm. What is the probable E. Typhoid fever
diagnosis? 328.The patient, 43-years-old was admitted to the
A. *Typhoid fever. hospital with complaints of high temperature of
B. Leptospirosis. the body and severe headache. On examination:
C. Brucellosis. carbuncle is revealed on the forearm. There are
D. Sepsis. intense edema around it, insignificant pain,
E. Malaria. regional lymphadenitis. The patient is a worker of
324.The patient was admitted to the hospital on the 7th cattle-ranch. What disease is it necessary to think
day of the disease with complaints of high about first ?
temperature, headache, pain in the muscles, A. *Anthrax
KROK 2 – Question Bank 32
B. Carcinoma of skin A. *Amoxycillin
C. Erysipelas B. Vancomycin
D. Erysipeloid C. Tetracycline
E. Eczema D. Co-trimoxazole
329.In the patient of 21 years old the disease began E. Imipenem
with increase of temperature to 39,0°С, headache, 334.A patient treated for springtime allergies with
chill, repeated vomiting. Rigidity of occipital terefenadine develops an upper respiratory tract
muscles is determined. The analysis of liquor: problem. He receives an antibiotic and develops a
cytosis - 1237 in 1ml, of them: 84 % of cardiac arrhythmia. What was the likely
neutrophils, 16 % of lymphocytes. On antibiotic?
bacterioscopy gram-negative cocci, are found in A. *Erythromycin
liquor. What is the most probable disease? B. Ampicillin
A. *Meningococcal infection: purulent C. Cefactor
meningitis. D. Doxycycline
B. Meningococcal infection: serous meningitis E. Co-trimoxazole
C. Secondary purulent meningitis. 335.A 75yr. Old man who had developed diabetes
D. Serous meningitis. within the last six months was found to be
E. Infectious mononucleosis. jaundiced. He was asymptomatic except for
330.The theory of self regulation of epidemiological weight loss of 10 pounds in 6 months. On physical
process by V.D. Belyakov includes all mentioned examination he is found to have a nontender,
bellow except globular, right upper quadrant mass that moves
A. * Localization of the parasite in the host with respiration. A CT scan shows enlargement of
organism and the ways of discharging of the the head of the pancreas, with no filling defects in
parasite into the environment the liver. Most likely diagnosis:
B. Geno- and phenotypic heterogeneity of the A. *Carcinoma of the head of the pancreas
populations of a parasite and host B. Infectious hepatitis
C. Mutability of the biological properties of the C. Haemolytic jaundice
parasite and the host populations D. Malignant biliary stricture
D. The phase selfreorganisation of the parasite E. Metastatic disease of liver
populations and a host 336.A 16yr. Old female presents with abdominal pain
E. The regulation role of social and natural and purpuric spots on the skin. Laboratory
conditions in the phase reorganization of the investigations reveals a normal platelet count, with
epidemiological process haematuria and proteinuria. The most likely
331.The diagnosis of the AIDS epidemic initially was diagnosis:
made in the USA by means of A. *Henoch Schonlein purpura
A. * The epidemiological method B. Haemolytic uraemic syndrome
B. The bacteriological method C. Thrombotic thrombocytopenic purpura
C. The virological method D. Heavy metal poisoning
D. The viroscopic method E. Sub acute bacterial endocarditis
E. The serological method 337.A 60yr. Old asthmatic man comes for a checkup
332.The contagious hypothesis by D. Frocastro and D. and complains that he is having some difficulty in
Samoylovich being considered as a causative “starting to urinate”. Physical examination
agent of infectious diseases indicates that the man has blood pressure of
A. * The alive germs, which are transmitted from 160/100mmHg, and a slight enlarged prostate.
one person to another Which of the following medications would be
B. The environmental factors useful in treating both of these conditions:
C. The miasma of cosmotelluric origin A. *Doxazosin
D. The pathological discharges of an organism B. Labetalol
E. Invasion of the body by the evil spirits C. Phetolamine
333.A 70yr. Old alcoholic male with poor dental D. Propranolol
hygiene is to have his remaining teeth extracted E. Isoproterenol
for subsequent dentures.He has mitral valve 338.A 36yr. Old alcoholic patient has cirrhosis and
stenosis with mild cardiac insuffiency and is being pancreatic insufficiency due to recurrent
treated with сaptopril,digoxin and furosemide.The pancreatitis. He complaints of nightblindness,
dentist decides that his medical history warrants decreased ability to taste food, and dry skin with
prophylactic antibiotic therapy prior to the hyperpigmentation. These complaints suggest
procedure and prescribes: deficiency of:
KROK 2 – Question Bank 33
A. *Zinc found to be febrile with temp.of 38.9C, along with
B. Copper right upper quadrant tenderness.Most likely
C. Selenium Diagnosis:
D. Chromium A. *Choledocholithiasis
E. Manganese B. Benign biliary stricture
339.A 47 year old man presents to his physician with C. Malibnant biliary stricture
progressive abdominal swelling. On examination D. Carcinoma of the head of the pancreas
he is found to have ascites and a tender, enlarged E. Choledochal cyst
liver. If the patient describes a chronic course 344.A 30yr. Old man presents with a history of
associated with wasting and low grade fever, the recurrent pneumonias and a chronic cough
differential diagnosis should include everything production of foul smelling, pirulentsputum,
EXCEPT: ocassionally glood tinged,which is worse in the
A. *Chronic Pancreatitis morning and on lying down.on physical
B. Tuberculosis examination, the patient appears chronically ill
C. Cirrhosis with hepatocellular carcinoma with clubbing of fingers, wet inspiratory reils at
D. Hepatitis the base of lungs posteriorly. Most likely
E. Alcoholic liver disease with cirrhosis diagnosis:
340.A 60yr. Old man with unstable angina pectoris A. *Bronchoectasis
fails to respond to heparin, nitroglycerin, beta B. Chronic bronchitis
adrenegic blockers and calcium channel C. Disseminated pulmonary tuberculosis
antagonist. The best management includes: D. Pulmonary neoplasm
A. *Coronary artery bypass grafting E. Chronic obstructive emphysema
B. Intravenous strptokinase 345.The 30-years old patient with the complications on
C. Excercise testing a headache in a nucha, poor dream with
D. Oral aspirin nightmares has addressed to policlinic. A BP was
E. Antihypertensive therapy 150/95 Hg.An item. A boundary arterial
341.A 42yr. Old patient suffering from alcoholism has hypertension was diagnosed. In what dispensary
advanced liver disease with ascites. He is group he must be addresseed for supervision on an
hospitalised for agitation and bizarre behaviour. arterial hypertension?
Examination reveals asterixes on the hands, ankle A. * In the second
clonus, and spider angiomas on the face and B. In a first
chest.Blood ammonia level is twice its C. In a fourth
baseline.Precipitating factors to look for include D. In a third
all of the following EXCEPT: E. In a fifth
A. *Insufficient protien ingestion 346.In the young patient at the reference to policlinic
B. Bleeding esophageal there was diagnosed the 1 stage of hypertension.
C. Excessive diuretic therapy How many times during the year it is necessary to
D. Non compliance with lactulose therapy examine him?
E. Spontaneous bacterial peritonitis A. * Twice.
342.A 45yr. Old man is admitted with his 3rd episode B. Once.
of upper gastrointestinal haemorrhage. He had 2 C. 3 times.
prior ulcer operation. Zollinger-Ellison syndrome D. 4 times.
is suspected. All the following would support your E. 5 times.
suspicions EXCEPT: 347.Patient K.,52 years old, has sustained an acute
A. *Supression of hypergastrinaemia by secretin myocardial infarction 2.01.2001. State of health is
given IV satisfactory. After what time according to the
B. A fasting gastrin level of 450pg/ml. instruction he can be send on sanatorium treatment
C. Post operative notes detailling ulcers in the to specialized sanatorium?
duodenum and jejunum A. * 1.07.2001.
D. Liver metastasis on CT scan B. 1.03.2001.
E. A history of diarrhoea C. 1.04.2001.
343.A 60yr. Old woman, mother of 6 children, D. 1.05.2001.
developed sudden onset of upper abdominal pain E. 1.06.2001.
radiating to the back, associated with nausea, 348.The family doctor diagnosed in a patient an acute
vomitting, fever and chills. Subsequently, she bleeding of an intestine. What is professional
noticed yellow discoloration of her sclera and tactics of the doctor in this situation?
skin. On physical examination the patient was
KROK 2 – Question Bank 34
A. *The urgent hospitalisation in sergical 354.Patient K., laboratory-assistant-radiograph; the
departmewnt. amount of Gamma-radiation per day is 0,65 R.
B. To inject intravenously the aminocapronic Two years later with 60 Co while examining
acid. irritability, headaches, poor appetite have
C. The urgent hospitalization in therapeutic developed. Blood count is normal. While
department. examining 6 years later after cessation of any
D. A day time hospital. contact with ionizing radiation she complained of
E. A hospital at home. severe head - aches, dizziness, cardiac pains,
349.In the structure of death rate in Ukraine the major undue fatigability, weakness; instability of
place is taken by the diseases of? periphery blood indices [especially white] are
A. *Diseases of cirkulatory sistem. being registered. Make up the diagnosis.
B. Oncological disease. A. * Chronic radiation disease of the second
C. Disease of digesstive sysstem. degree of severity
D. Disease of respiratory system. B. Chronic radiation disease of the third degree of
E. Disease of urinary and endocrine systems. severity.
350.The doctor of the city cardiological center solves C. Acute radiation sickness of moderate degree
the problem of the patient after the discharge from D. Acute radiation sickness of light degree
a hospital to the balneal department of E. Acute radiation sickness of severe degree
sanatorium:who cannot be routed? 355.In checking the results of patient?s spinal tap, the
A. *200/110-240/120 CSF glucose level of 1,65 mmol/l with apparently
B. 140/90-160/100 cloudy CSF, is typical of:
C. 90/60-120/80 A. *Meningococcal meningitis
D. 160/90-180/90 B. Meningism
E. 180/110-90/60 C. Cerebral trauma
351.Woman age 40,ill on rheumatic disease with D. Viral meningitis
composite mitral disease with prevalence of the E. Tuberculous meningitis
stenosis of left venous foramen. Complainse on 356.A 26-year-old manual worker complained of 3
the palpitation, fatigability progressing dyspnea, weeks history of fevers and fatigue, weight loss
attacks of a dyspnea and hemoptysis. Now she with no other symptoms. Physical findings:
cannot execute even the mild activities. What Temperature 37,6?C, pulse 88 b/pm, blood
tactics is the most expedient? pressure 115/70 mmHg, superficial lymph nodes
A. *Mitral comissurotomia. (occipital, submental,cervical, axillary) are
B. Conduction of current bicilino-prophilaxis. enlarged, not tender or painful. Rubella-like rash
C. Assiging of anticoagulants. on the trunk and extremities. Herpes simplex
D. Assiging of venous vasodilatators . lesions on the lips. Candidosis of oral cavity.
E. ……….. What infectious disease would you consider?
352.The man,42 years old, has died in a road accident A. *HIV infection
after the hemorrhage on the place ,due to acute B. Influenza
hemorrhagic anemia. What minimum percent of C. Rubella
all volume of blood could result in death at acute D. Infectious mononucleosis
hemorrhage? E. Tuberculosis
A. *25-30\%. 357.Each of the following statements concerning
B. 6-9\%. features of anthrax is correct EXCEPT
C. 10-14\%. A. *The initial lesion in cutaneous anthrax
D. 15-20\%. consists of the black eschar, surrounding
E. 35-50\%. erythema and painful pitting edema
353.In the woman of 42 years ,who suffers from B. Inhalation anthrax results from the alveolar
bronchial asthma ,the acute attack of a bronchial deposition of airborne particles contaminated
asthma has developed. What medication from with spores of B. anthracis.
listed below is contraindicated at granting the first C. Cutaneous anthrax, untreated, results in death
aid to this woman? in 10\% to 20\% of cases
A. * Euphylinum D. The drug of choice in cutaneous anthrax is
B. Izardin. penicillin.
C. Corazolum. E. The preferred serodiagnostic test for anthrax is
D. Morphinum hydrochloride. ELISA
E. Strophanthin hydrochloride. 358.A 27-year old patient with malaria caused by P.
falciparum was treated with Chloroquine (600 mg
KROK 2 – Question Bank 35
base followed by 300 mg base in 6 hours, then 300 362.A 25-year-old man was admitted with 2 months of
mg base a day for 2 days) without clinical and cough and fever. A chest x-ray showed extensive
parasitologic responses to treatment. What is the left upper lobe disease with a 2 cm cavity. All
most likely reason for the failure to respond to three points were strongly positive on direct smear
therapy? and grew M. Tuberculosis, fully sensitive to all
A. *Chloroquine resistant strain of P. falciparum first-line drugs. Patient must be treated with:
B. Glucose – 6 – phosphate dehydrogenase A. * Isoniazid + rifampicin + pyrazinamide
deficiency in patient B. Streptomycin + isoniazid
C. Late recognition of infection due to P. C. Isoniazid + ethambutol
falciparum D. Kanamycin + ethambutol + pyrazinamide
D. Inappropriate route of administration E. P-aminosalicylic acid + streptomycin
E. Hypersensitivity of the patient to Chloroquine 363.A 5-year-old boy was progressively getting more
359.Ten hours before initial observation patient C. had unwell than during the previous 2 months. A chest
frequent feces and vomiting. Fecal and vomiting x-ray had shown right middle lobe collapse. A
masses became like rice-water. Nausea and tuberculin skin test had been strongly positive.
abdominal pain weren’t observed. Hiccup and What is the most characteristic finding in primary
convulsions of lower limbs, temperature 35.4(C, tuberculosis?
hoarse voice, greyish wry face, acrocianosis were A. * Hilar or paratracheal lymph node
usually observed. Breathing 40/min, thread – like enlargement
pulse 120/min, blood-pressure 40/0mm/Hg, B. Atelectasis with obstructive pneumonia
abdomen is drawn. What treating measures should C. Cavity formation
be taken first of all? D. Miliary tuberculosis
A. *Intravenous rehydration E. Hematogenous dissemination leading to
B. Antibiotic therapy extrapulmonary tuberculosis
C. Cardiac glycosides 364.A 50-year-old man was examined in the clinic
D. Disintoxicational therapy with persistant cavitation and sputum. He was
E. Antibotulistic serum injection treated with rifampicin, isoniazid, ethambuthol.
360.A sick person M. is staying in the hospital with the The most common toxic effect of ethambutol is:
diagnosis of abdominal typhus. During the 3d A. * Optic neuritis
week from the beginning of the disease the patient B. Eighth cranial nerve damage
stopped keeping diet and confinement to bed. As a C. Hepatic enzyme elevation
result the body temperature and rapid pulse D. Peripheral neuropathy
decreased and melena appeared. What kind of E. Mental symptoms
complications should we think first of? 365.To reduce the number of adult infection cases in
A. *Intestinal haemorrhage the population it is much more important to:
B. Thrombophlebitis A. * Give good treatment to all sputum positive
C. Meningitis patients
D. Nephroso-nephritis B. BCG vaccination
E. Hepatite C. Tuberculin skin testing
361.A nurse of the kindergarten was taken to the D. Health education campaigns on tuberculosis
hospital with complaints of accute pain in E. Chest x-ray screening
parumbilical part, convulsions of lower limbs, 366.The father had been admitted to hospital with
multiple bile vomiting, frequent watery foul faces pulmonary tuberculosis. He had infected his son,
of green colour in huge amounts. At the same time aged 3 years. If tuberculin skin testing was
all the staff in the childrengarden got ill. Two days positive, with no evidence of illness, the child
ago everybody of them ate cottage cheese with should have been given chemoprophylaxis with:
sour cream. General condition of patients is of A. * Isoniazid
average severity. Temperature 38.2 degrees of C. B. Rifampicin
Heart tones:rhythmic and muted. FHC 95/min, A. C. Streptomycin
Pressure: 160 mm/Hg. Abdomen is slightly D. Ethionamide
swollen, painful. Liver +2 cm. What is the most E. Cycloserine
probable diagnosis? 367.Under what circumstances can preventive therapy
A. *Salmonelosis be given to people who have a negative tuberculin
B. Dysentery test reaction?
C. Cholera A. * High risk close contacts (young children,
D. Food toxic infection HIV-infected persons)
E. Enterovirus infection B. People with chest x-ray findings
KROK 2 – Question Bank 36
C. Low-income groups with poor access to health A. *Raised function of parasympathetic nervous
care system
D. People who inject illicit drugs B. Hypertrophy of myocardium
E. People with symptoms of pulmonary disease C. Frank-Starling’s mechanism
368.The patient, 52 years, show complaints on a D. Vasoconstriction
retrosternal burning pain, appearing at physical E. Activation of renin-angiotensine-aldosterone
loading and disappearing after it. Objectively: a system
condition of the patient is satisfactory, pulse beat - 372.A 56y.o. teacher consulted a department therapist
86 times a minute, during auscultation the tones of about a corresponding diet. Complaints: frequent
heart are rhythmical, are muffled. The arterial constipations [defecation once in 2-3 days],
pressure - 130/80. On the electrocardiogram a flatulence and unpleasant aftertaste; blunt pain in
pathological changes are not revealed. What is the lower half of abdomen after intake of ice
yours preliminary diagnosis? cream or soda drinks. At 17-20 she was treated for
A. *Stenocardia stable on exertion duodenal ulcer. Objectively: height – 164cm,
B. Stenocardia unstable weight – 62kg. Abdomen is oval, palpation
C. Myocarditis doesn’t detect pathological changes. What diet is
D. Q miocardial infarction necessary for prevention of transition of chronic
E. Cardiomyopathy disease into acute?
369.Patient p, 45y.o. was delivered by an ambulance A. *Table #4 by Pevzner
with complaints of pain in the epigastrium, vomit B. Table #4a by Pevzner
at the height of pain with impurity of blood; C. General table
heartburn, weakness, dizziness, “black” stool in D. Table #2 by Pevzner
the morning. He has been ill for 10 years, nutrition E. Table #9 by Pevzner
is irregular. What complication has developed in 373.A 30y.o. patient complains of local pressing
the patient? heartache, dyspnea during physical exercise,
A. *Gastric bleeding memory aggravation and constipation for 3
B. Intestinal bleeding months. Menstruation is small. Objectively: Skin
C. Bleeding from esophageal veins is dry and pale. T- 35.4(C. Expressive edema of
D. Bleeding from hemorrhoidal veins face and limbs. Tones are dull, rhythmic. Ps – 52
E. Thrombocytopenia bpm, AP – 90/60. Hemogram: RBC – 2.8T/l, Hb –
370.A 65 year old patient was admitted in the hospital 92G/l, ESR – 10 mm/hour. I2-absorbtion is15( in
with a complains of fatigue, feeling of “cotton like 24 hours. What medication would you prescribe to
“legs, loss of sensitivitiy of toes. On examination the patient?
pale of the skin, subecteric sclera. Heart sounds of A. *Thyroxin
low intensity, systolic murmur heard over the B. Mercazolin
heart area, liver +2cm, painless, spleen not C. Digoxin
enlarged. Blood analysis: Hb 58mg/l, E- 1.3 x10 D. Furosemid
12/l, color index-1.3, reticulocytes-0.2\%. ESR E. Ferroplex
30mm/hr, megaloblasts-2:100, macrocytosis. Jolly 374.A 35y.o. patient has been admitted to nephrology
bodies, кebots ring. Most informative method of department of OCH with chronic pyelonephritis
investigation. with frequent recurrences. It is necessary to send a
A. *Serum B-12 and folic acid level patient to a sanatorium in 1-2 months after
B. Endoscopic examination of the stomach treatment for prevention of recurrences. What
C. X ray of abdomen health resort is best for the patient?
D. Osmotic resistance of erythrocytes A. *Truskavets
E. Iron content of serum B. Yalta
371.The 63y.o. patient’s complaints are dyspnea C. Morshen
during physical exercise, palpitation and swelling D. Esentuki
of the ankles. He had a transmural myocardial E. Shoyani
infarction a year ago. Objectively: T –36.5(C, BR 375.A 48yr old obse diadetic with impariment of visin
– 22 pm, Ps – 96 bpm, AP – 110/70. for 3 weeks .on examintaion he is found to show
Acrocyanosis. Vesicular respiration is evident; it evidence of peripheral neuropathy and grade of II
is raised in the lower areas with single faint rales. diabetic retinopathy.His blood pressure 160 to
Left cardiac bound is 1.5cm inward of lin. 250mmHg, blood sugar 10 to 250mg\%, while
medioclavicularis. Cardiac tones raised, rhythm is taking 5mg glibenclamide 3 time daily, urine
correct, Liver – 2cm below arcus costalis. What is examination shows sugar +urine, ni ablumin
the mechanism of tachycardia: excretion ans acteone. Urine excretion during
KROK 2 – Question Bank 37
24hrs is 400mg. which is following management cardiac region, palpitation, dyspnea. Objective:
is approprirate? acrocyanosis, heart sounds are reduces, systolic
A. *Enalapril and insulin noise above the apex, extrasystole, blood pressure
B. Phetenoforminand enalapril 90/60 mm Hg. Which among the following is
C. Nefidipine and phenoformin true?
D. Nefidipine and insulin A. *All the given below.
E. Insulin and phenoformin B. Electrophysiological test.
376.A 50yrs ols male has had precaridal pain for 4 hrs. C. Laboratory test.
On examintion his BP 110/80mmhg.pulse D. Hospitalisation.
10beats /mint and respiratory rate is 206/min. His E. None of the above.
ECG maked S-T segement elvation and left 381.A 55 years old man complaints with chest pain for
ventricle ectopies. The initial therepeutoc a duration of 25 min. With radiation to the left
modilaties in his case would include. side which took place 3 days ago at rest and
A. *Lignocaine, streptokinase and morphine dissapeared without any medical treatment. The
B. Lignocanineand streptokineases last day attack took place for 5-6 times per day
C. Morphineand dobtutamine with increases in pain intensity. The results of
D. Streptokineases and morphine laboratory test: Troponin-0,17ng/mg, L-5,6 (109
E. Nitoglcine +Digoxine /l, alaninaminotransferase-100 mmol/l, Lactat
377.A 60 year old woman mother of 6 children Dehydrohenase-350 mmol/l. Which disease do
developed sudden onset of upper abdominal pain you suggested?
radiating to back, associtated with nausea, A. *Non-stable sternocardia.
vomiting, fever and chills. She noticed yellow B. Stable sternocardia.
discoloration of her sclera and skin. On physical C. Myocarditis.
examination the patient was found to be febrile D. Q-Myocardial Infarction.
with temperature of 38.9C, along with right upper E. Cardiomyopathy.
quadrant tenderness 382.A 20y.o. student fell ill with acute
A. *Choledocholithiasis glomerulonephritis. He was ill with angina a week
B. Infectious hepatitis ago. Objectively: skin and mucous layers pallor,
C. Hemoytic jaundice edema of face, limbs and trunk. AP – 180/110.
D. Choledochal cyst Urogram: protein – 6.6 g/l, RBC – 100-120 in f/v,
E. Ascariasis hyaline cylinders – 2-3 in f/v, oligoanuria. What
378.A 40 year old male pesents with headache and infection had in important part in development of
convulisons. His blood pressure is 120\140 mmhg. disease?
Fundus examination revelas papillooedema. A. *Streptococcus
Which of the following drug examination will be B. Virus
most suitable for this patient? C. Staphylococcus
A. *Nitropruside+Furosemide D. Blue-purulent bacillus
B. Diazoxide+trimaterence E. Intestinal bacillus
C. Nitroprusside+Triamterence 383.The patient has peptic ulcer of stomach with
D. Resaprine frequent recurrences. Courses of anti-recurrent
E. Captopril therapy must be:
379.A 75 year old man who had developed diabetes A. *Once in 3 months
within last 6 months was found to be jaundiced. B. Twice a year
He has remained asymptomatic, expect for weight C. Each year
loss about10 pounds in 6 months. On physical D. Each month
examination he is found to have a non- tender, E. After each recurrence
globular, right upper quadrant mass that moves 384.A 65y.o. patient is in the reanimation department
with respiration. A CT scan shows enlargement of with transmural myocardial infarction. He
the head of the pancreas, with no filling defects in complains of abdominal pain, flatulence and
the liver constipation. Objectively: Abdomen is algestic in
A. *Carcinoma of the head of the pancreas the region of colon, with continent inflation; its
B. Metastatic disease of the liver lower part is full. What is the main mechanism of
C. Pancreatitis this condition?
D. Choledocholithiasis A. *Hypodynamia
E. Hemolytic jaundice B. Failure of nervous regulation
380.First patient A. 35 years old with Acute Viral C. Non-adequate alimentation
Respiratory Infection complains of pain in the D. Colon abnormality
KROK 2 – Question Bank 38
E. Failure of aqueous-saline exchange D. Mital stenosis
385.A patient complains of resting dyspnea, high E. A ortic stenosis
temperature and chest pain. Objectively: The right 390.If a 55year male presents with normotension,
half of chest is behind in the act of respiration. nephritic syndrome, azotemia, collateral
During percussion a blunt sound is evident. abdominal veins with upward flow and there is
Sounds are absent in lungs during auscultation. X- history of gross, painless haematuria 6 months
ray: a homogenous opacity 2/3 of right lung. The back, the mostly likely diagnosis is
most informative for diagnosis is: A. *Hypernephroma
A. *Punction of pleural cavity B. Amyloidosis
B. Bronchoscopy C. Papillary necrosis
C. Bronchography D. Periarteritis
D. Pneumotachometria E. Liver Cirrhosis
E. Spirography 391.A 70 yrs old woman is admitted to the hospital
386.A 62y.o. patient is ill with chronic obstructive with increasing fatigabality and dyspone for the
bronchitis. He has smoked for 20 years. Point out several month. Physical examintaion heat rate
the non-medicamentous prevention of recurrences 100/min regular rhythm and BP of 150/100mmhg
in this pathology: engorged neck veins, third heart sound and rales
A. *All measures over lung fields. Skigram on chest show and
B. Training of respiratory muscles prominent vascular markings .This patient is likely
C. Refusal from smoking to be having
D. Optimization of microclimate A. *Congestive cardiac failure
E. Balanced alimentation B. Adult respiratory distress syndrome
387.A 53y.o. woman is suffering from essential C. Atrial fribrillation
arterial hypertension of the I degree combined D. Rebound hypertensive crisis
with obesity of the II degree. Which of the E. Pulumonary edema
following methods assist an accurate decrease of 392.A 45yrs old executive who is a heavy smoker, had
EAH progression risk? severe retrostrenal discomfort while going to the
A. *Therapeutic diet toilet at 7.00 a.m .ECG done immediately showed
B. Psychotherapy that S-T segement elevation in the infrrioe leads
C. Physical therapy which normalised within an hour. This likely
D. Rodone baths mostly diagnosis is
E. Collar zone massage A. *Prinzmetal s angina
388.A 34y.o. patient has been admitted to clinic with B. Acute myocardial infraction
complaints of heaviness in the right subrib, nausea C. Dissecting aneurysm of arota
and emesis after intake of fried pork; dark urine D. Acute pericarditis
and light feces. He was ill with viral hepatitis half E. Stable angina pectilors
a year ago, but wasn’t on a diet. Objectively: 393.A 50 year old patient previously in good health
Scleras are subicteric. Abdomen is with continent and sudden onset of right anterior chest pain
inflation. Liver is +2cm of costal arc margin on without chills, fever or cough. On examination he
lin. medioclavicularis dextra. Its surface is smooth has tanchypneic. Over the right lower rib cage
and it has an elastic consistence. What disease there was an inspiratory rub. Chest X –ray was
would you think about? negative. Which statement is correct
A. *Chronic hepatitis A. *Pulmonary embolus must be cosidered
B. Cirrhosis of liver promptly
C. Active hepatitis B. Pneumococcal pneumonia is mostly and
D. Chronic cholecystitis penicilline should be given
E. Chronic pancreatitis C. Bronchogenic carcinoma is most likely
389.A 40 year patient of rheumatic heart disease D. Bronchoscopy needs to be done promptly
complains of anorexia, weakness and loss of E. Viral pnenumonia is most likely diagnosis
weight, breathlessness and swelling of feet. On 394.A 43 year old man begins to wheeze. There is no
examination temp 39C pulse is 100/mint. way to tell whether this is bronchial asthma or
Auscultation diastolic murmur in the mitral area. cardiac asthma. The best drug to be use would be
Petehical lesion round clavicle and spleen was A. *Aminophylline
palpable, Tooth extraction one month ago. B. Digitalis
A. *Subacute bacteria endocardidtis C. Cortisone
B. Recurrence of rheumatic fever D. Epinephrine
C. Throbocytopenia purpure E. Salbutarole
KROK 2 – Question Bank 39
395.A middle – aged patient presents with a short B. Lidocaine
history of hight fever and rigors. On examination C. Procaineamide
the pulse rate is 140/min and there are signs of D. Digoxin
toxaemia. Chest examination shows restriction of E. Diltiazem
movement over left infraclavicular region and 400.A 40 year old woman in good gernal health
there is dullness on percussion. Vocal resonance is experiences sudden chest pain, fever and shortness
increased with low pitched bronchial breath of breath. She is heavy smoker and takes takes no
sounds over the above area. The mostly likely medication except oral contraceptives. Tachypnea
diagnosis and temperature of 38C are found on physical
A. *Lobar pneumonia examination. Chest auscultation, percussion and
B. Lung abscess radiographic findings are normal. Which of the
C. Bronchial pneumonia following diagnosis is likely
D. Plerual effusion A. *Pulmonary embolus
E. Acute bronchitis B. Atypical pneumonia
396.A 48 year-old man presents with arthralagia, C. Tracheobronchitis
abdominal pain, diarrhoea, progressive weight D. Bacterial pneumonia
loss, low-grade fever, confusion, memoryloss and E. Lung cancer
opthalamoplegia. Investigation reveal 401.A 17y.o. patient complains of acute pain in the
steatorrhoea, imparied xyloseobsorption, abnormal knee joint and T – 38(C. He was ill with angina 3
small - bowel X rays hypoalbuminaemia and weeks ago. Objectively: Deformation and swelling
anemia. The mostly likely diagnosis of the knee joints with skin hyperemia. Small
A. *Whipple s disease movement causes and acute pain in the joints.
B. Coeliac disease Which diagnose is the most correct?
C. Inflammatory bowel disease A. *Rheumatism, polyarthritis
D. Tropical sprue B. Systemic lupus eritematodes
E. Chronic enterocolitis C. Reactive polyarthritis
397.An indian adult who has neve travelled abroad D. Infectious-allergic polyarthritis
come with a history of high fever, headache, E. Rheumarthritis
jaundice, marked oliguria, heptospleenomegaly 402.A 12- year old boy had sever vomiting and
and shock with TLC of 16,000/cumm. The mostly diarrhoea. He was brought to the hosptial because
likely diagnosis is of severe oliguria. He was give IV fluids and
A. *Leptospirosis{weil s disease} furosemide without any diuresis. Blood
B. Viral hepatitis biochemistry revealed : Urea 120 mg\% and serum
C. Yellow fever creatinine 4mg\%.The most likely diagnosis is
D. Haemolytic uraemic syndrome A. *Prerenal azotemia
E. Haemolytic jaundice B. Haemolytic uremic syndrome
398.A patient is noted to have paroxysmal episodes of C. Acute glomerulonephritis
hypertension, tremors, weakness and sweating. D. Acute tubular necrosis
Physical examination reveals tachycardia and E. Acute pyelonephritis
hypertension. The urinary catecholamines and 403.A 56 year old woman has an elevated serum
their metabolites are elevated and a computerised calicum level of 12.2mg/dl. She has no history of
tomography scan detects a mass within the adrenal any illness, or treatment associated with
gland. Which is the most likely diagnosis hypercalcemia. Which of the following studies
A. *Pheocromocytoma would be most helpful in making diagnosis of
B. Cushing syndrome primary hyperparathyriodism
C. Conns disease A. *Serum parathyroid hormone
D. Essential hypertension. Crisis conditions B. Serum ionized calcium
E. Cushing disease C. Serum phosphate
399.A 76-year old woman presents with an acute D. CT scan of the neck
myocardial infarction. Her symptoms stared 4 hrs E. 24 hrs urine calcium excretion
before admission and resolved after 1 sublingual 404.A 13 year old girl with sickle cell anemia presents
nitroglycerine. She has history of hypertension, with pallor, tachycardia, hypotension and massive
pulse is 104beats/min and BP is 180/106mmHg. A splenomegaly. The most likely explanation is
third heart sound is present. No venticular ectopy A. *Splenic sequestration
is observed druning ECG monitoring. Appropriate B. Hemorragic shock
initial therapy may include C. Septic shock
A. *Metoprolol D. Cardiogenic shock
KROK 2 – Question Bank 40
E. Hypovolemic shock C. Uclerative colitis
405.A 30 year old female with rheumatoid arthritis of D. Lactose intolerance
five years duration complains of pain in the first E. Short gut syndrome
three fingers of her right hand over past 6 weeks. 410.A 20- year old woman with 3-4 months history of
The pain seems especially severe at night often bloody diarrhea ,stool examination negative for a
awakening her from sleep. The most likely cause ova and parasites, stool culture negative for
is Clostridium, Campylobacter and Yersinia, normal
A. *Carpal tunnel syndrome bowel series edema, hypermia and ulceration of
B. Atlanto-axial sublaxation of cervical spine rectum and sigmoid colon seen on sigmoidoscopic
C. Sensory peripheral neuropathy examination. Select the most likely Diagnosis:
D. Rheumatoid vacultis A. *Ulcerative colitis
E. Rheumatoid arthrits without complication B. Gastroenteritis
406.A 27 year old woman presents with muscle C. Carcinoid syndrome
weakness, ptosis, has been receiving gentamicin D. Granulomatous colitis
injections for the last 7 days for a urinary E. Lactose intolerance
infection. Thyroid function tests, serum creatine 411.An 80-year old woman with a history of
kinase, eletromyogram and muscle biopsy are Congestive Heart Faliure develops angina
normal. I/V administration of edrophonium results pectoris. Her medications are adjusted to include
in a dramtic improvements in the pateints muscle furosemide, digoxin, nitroglycerine and potassium
strength. Which of the following is most likely supplements. Shortly there after develops
diagnosis intermittent frontal throbbing headache. Which of
A. *Myasthenia gravis the following should the physician do first?
B. Toxic {drug induced}myopathy A. *Discontinue nitroglycerin
C. Duchenne musclar dystrophy B. Perform a temporal artery biopsy
D. Peripheral neuropathy C. Begin propranolol
E. None of the above D. Begin sublingual ergotamine
407.A 56 year old woman is admitted after 2 weeks of E. Obtain a brain CT scan
frequent vomiting. She is dehydrated. Work –up 412.A patient 54 years old, admitted in hospital with
shows complete gastric outlet due to longstanding intensive and tremendous retrosternal pain without
peptic ulcer disease. Her serum due to radiation, pain lasted for 40 minutes, no changes
longstanding peptic ulcer disease. Her serum were observed after taking nitroglycerin.
chloride is 90mEq/dl {low},sodium 134mEq/dl Objectively: pale face, sweatting, hypotonia (pulse
[low},bicarbonate 35mEq/dl [high},and her urine rate 108/min), “gallop”rythm. ECG: elevation of
PH is low {acidic} ST to 6 mm at III and AVF leads. Read the above
A. *Severe Crohns disease given information and select the correct diagnose
B. Sigmoid colectomy from the following:
C. Splenectomy A. *Troponin I and T.
D. Segmental jejunal resection B. Activity of the alaninaminotrasferasa.
E. Sigmoid colectomy C. Activity of the aspartataminotrasferasa.
408.A 47-year old man presents to his physician with D. Activity of the general lactactdehydrogenase.
progressive abdominal swelling. On examination E. Activity of the general creatinphospokenase
he is found to have ascites and a tender, erlarged 413.A patient 60 years old with ischemic heart disease,
liver. If the patient describes the ascites as having post infract cardiosclerosis. After physical
been of abrupt onset and preceded by trauma, the exercise, there is weakness, breathlessness at rest,
most likely diagnosis is strong heart beat, sputum is reddish in colour,
A. *Thrombosis of the hepatic vein {Budd-Chiari cyanotic skin, patient is half seated, half lying,
syndrome} cold sweat. Breathing rate is 30/min, heart rate
B. Congestive heart failure 100/min, blood pressure 100/60 mmHg, different
C. Portal vein thrombosis kinds of rattling, hummid sounds in lungs and
D. Chronic hepatitis heart murmurs. Which mechanism leads to
E. - development of acute heart failure?
409.A 60 year old woman was admitted in the hospital A. *Increase of hydrostatic pressure in small
with abdominal cramps, watery diarrhea and circulating blood vessels.
flushing with episodes of wheezing. Select the B. Decrease of oncotic pressure in the blood
appropriate diagnosis plasma.
A. *Carcinoid syndrome C. Raise in activity of sympathoadrenergic
B. Gastroenteritis system.
KROK 2 – Question Bank 41
D. Increase in volume of circulating blood. B. Disease + high cardiac output + increased
E. Retention of water and sodium in the vascular resistance.
organism. C. Low cardiac output + accumlation of Na+1 +
414.A patient 28 years old, during 10 years diagnosis accumulation of liquid.
of reumatism, failure of aortic valve, admitted in D. Increased resistance of vesseles + increased
the clinic with cardiac deficiency at the II B stage. activity of sympatho adrenergic system.
Which of the following hemodynamics is of E. Increased activity of sympathoadrenegic
importance in the diagnosis of systolic system + increased activity of renin –
dysfunction? angiotensin – aldosteron system.
A. *Dillation of the heart and decrease in heart 419.A patient with hypertension in 2nd stage, has
output. inspiratory breathelessness, cough, weakness after
B. Hypertrophy of the heart and decrease in heart physical exercise. Echo cardigraphy revealed:
output. hypertrophy of left ventricle myocardium. There is
C. Increase in minute volume of blood. a decrease of final diastolic volume with a fraction
D. Decrease in minute volume of blood. of excreation of 58%. Destrution of
E. Decrease of heart output. heamodynamic regulation is the main cause of
415.The patient has had the non-specific hecoid colitis failure of:
for many years. He is in remission now. During A. *Diastolic function of the left ventricle.
dispanserisation the doctor must prescribe B. Systolic function of the left ventricle.
colonoscopy to him: C. Diastolic function of the right ventricle.
A. *Each year D. Systolic function of the right ventricle.
B. Twice a year E. Systolic – diastolic function of the heart.
C. Each month 420.A patient 57 years old, admitted in hospital with
D. Once in 3 months small focal myocardial infarction from the past 3
E. Once in 2 years months is reported to have cardiac insufficiency of
416.Throughout the year after an attack of acute first stage functional class 1 (Fc1). At present he
myocardial infection, a patient complains of complains dyspnea, fatigue, palpitation in normal
periodic pain in the heart area. At the time of physical maintenance. Which of the following is
observation of the electrocardiogram except the best method of prophylaxis for the given
pathological Q in II, III and AVF leads other patient?
changes were not observed. Which investigation is A. *Treatment of the basic disease.
the most apropriate in order to diagnose and treat B. Treatment of the cardiac insufficiency.
the patient? C. Diet with controlled amount of liquid, salt and
A. *Hollter’s monitering electrocardiogram. animal fats.
B. Investigation of electrolytes in blood. D. Satisfactory climatic treatment.
C. Coronarography. E. Control of physical tolerance.
D. Physical test. 421.The patient aged 58 suddenly had headache,
E. Stress-echocardiography. syncope, nausea, fear, “net” glimmering in the
417.A patient with unstable stenocardia was given a visual field, palpitation, cardiac pain. He has been
complex treatment of the following: suffering from hypertension for 18 years. Clinical
anticoagulants, lyins nitrates, (-adrenoblockers. examination revealed cardiac rate 96 per 1 min.,
But on the third day of treatment the pain was BP = 210/115 mm Hg. What preparation should
continous, which tests are carried out on this be prescribed to this patient most currently?
patient to get the proper diagnosis? A. * Corinfar
A. *Coronarography B. Adelfan
B. Stress-ecocardiogram. C. Analgin
C. Test with doses of physical exercises. D. Cristepin
D. Oesophageal electrocardio stimulator. E. Egilok
E. Myocardial syncitigraphy. 422.A 26 years old patient was admitted to the hospital
418.The patient who after ‘Q’ positive myocardial with complains of pain in the right chest, fever up
infarction of left ventricle: experienced dyspnea, to 38-40 C and cough with purulent sputum
edema and hepatomegalia. Echocardiograph: discharge. Before he has been ill within 2 weeks
Increase in end diastole volume & intraventricular with common cold and lower lobe pneumonia on
pressure of left ventricle. What is the main reason the right. He was treated as an out-patient but 3
of congestive heart failure of the patient? days ago his general condition was worsened and
A. *Disease + low cardiac output + accumulation above mentioned complaints was appeared. On
of liquid. physical examination the patient is asthenic,
KROK 2 – Question Bank 42
exhausted, with pale grey skin. An unpleasant C. Chronic myeloleukemia
smell from the mouth is noted. On auscultation: D. Aplastic anemia
the air entering in the lower right lobe is E. Thrombocytopenic purpura
diminished. In other chest there is crepitation. On 426.A man, 25 years, appealed to the policlinic
percussion: there is dullness over the lower lobe concerning a quinsy. Appointed soulfanilamidi
on the right. CBC reveals high WBCs count and aminoglicozidi. On 5-th day a urinary
(leucocytosis) with the formula band shift and low syndrome turned out, arterial hypertension was not
RBCs count (anemia). On the X-Ray graph the present. A to 15 day an infection ended and a
right middle and low lung fields are very dense. In urinary syndrome gradually vanished. What
the lower lobe there is intrapulmonary destructive diagnosis must it be?
cavity with horizontal fluid level surrounded by A. *Infection-tocsical bud.
inflammatory wall. How could you define the B. Sharp glomerulonephritis.
chest disease? C. Sharp pyelonephritis.
A. * Lobar pneumonia complicated by acute D. Sheynleyn’s – Genoh’s illness.
abscess of lung E. Berger’s illness.
B. Lung gangrene 427.Which of the diseases doesn’t trigger cancer of the
C. Non complicated pneumonia rectum:
D. Lobar pneumonia complicated by chronic A. *amebiasis
abscess of lung B. diffusive family polyposis
E. Chronic lung abscess C. Crohn's disease
423.What kind of cereals mentioned doesn’t produce D. ulcerative colitis
worsening the course of celiac disease (non- E. immunodeficiency
tropical sprue): 428.The arterial hypertensive patient aged 30 during
A. *rice last 5 years has had a constant BP index 170/100 –
B. wheat 200/120 mm Hg. On objective examining systolic
C. oats murmur is heard above the navel in the right.
D. rye What’s the reason of hypertension?
E. barley A. *reno-vascular hypertension
424.A 19 years old patient complains of long-term B. hypertension disease
bleeding started after tooth extraction. It is known C. polycystic kidney
from the case history that he has had large D. coarctation of aorta
haematomas in the childhood. His uncle from E. glomerulonephritis
mother’s side died from bleeding. What clinical 429.The patient aged 30 complains of headache,
diagnosis is the most probable? general weakness, nocturia. Proteinuria has been
A. *Hemophilia noted 5 years and arterial hypertention 2 years
B. Hemorrhagic vasculitis ago. Heart rate is 88 beats per min, blood pressure
C. Iron deficiency anemia – 150/100 mm Hg, creatinine – 0.23 mkmole/l,
D. Aplastic anemia urea – 9,1 mlmole/l, glomerular filtration rate 50
E. - ml/min. Diagnosis?
425.A man aged 32 yr notes general weakness, A. *chronic renal insufficiency 1 stage
elevation of the body t°, pains in the throat while B. chronic renal insufficiency 2 stage
swallowing. He is ill 2 months and treated from C. Heart failure 3 stage
tonsillitis by antibiotics without any effect. The D. chronic renal insufficiency 4 stage
main clinical features are body t° 38,4 °C, RR E. acute renal insufficiency 1 stage
(respiratory rate) 22 per min; pulse 108, BP 430.The patient aged 31, called in with complaints of
(blood pressure) 100/60 mm of Hg. The skin is headache, nausea, vomiting. Diagnosis of
pale. There are purpura (hemorrhagic rash) on chronical glomerulonephritis with renal failure
extremities, systemic enlargement of lymphatic was made in stationary. What medical drug again
nodes and hepatosplenomegaly. In CBC RBC are high blood urea nitrogen would you recommend to
2,2*1012/l, Hb 70 g/l; WBC (leuc.) 13,5*109/l; the patient?
atypical cells (blastocytes) 32\%; metamyelocytes A. *Polysorb
1\%; stab neutrophils 3\%; segm. neutr. – 35\%; B. Furosemid
lymph. – 20\%; monocytes – 8\%; thrombocytes C. Hypothiazid
37*109/l; ESR – 30mm/hour. What clinical D. Guttalax
diagnosis is the most probable? E. Verapamil
A. *Acute leukosis 431.A sick man, who is 43, started to complain to his
B. Chronic lympholeukosis therapeutist about the heaviness in the stomach
KROK 2 – Question Bank 43
after a meal, bad belch, vomiting with food, he are leukocytosis, lymphopenia, high ESR. What
had eaten the day before. He has suffered from clinical diagnosis is the most probable?
ulcer of duodenum for 10 years. He has applied to A. * lymphogranulomatosis
the doctor (therapeutist) two times but he started B. chronic lymphatic leukemia
to feel worse last month. What pathology must it C. acute lymphoblastic leukosis
be? D. infectious mononucleosis
A. *Ulcerous stenosis of the exit part of the gullet E. leukomoidal reaction
B. Gullet cancer 436.A 42 years old man suddenly felt worsening of his
C. Stomach cancer general condition and complains of dizziness,
D. Ahalaziya of the gullet weakness and appetite loss. From the clinical
E. Reactive pancreatitis. history it is known that the patient suffers from the
432.The patient, aged 65 rising to the first floor have duodenal ulcer disease within 15 years and it is
felt pain behind the sternum lasting nearly 5 – 8 written down in his individual medical file. In
minutes radiated to the left shoulder and blade. CBC erythrocytes (RBC) are 1,9*1012/l , Hb
For the next 10 days the attacks of pain have 57g/l , KP 0,9, reticulocytes 1\% , thromb.
become more frequent. A feeling as if he lacks air 2,1*109/l , WBC (leuc.) 11,5*109/l , eosinophils-
to breathe (breathleeness) was appeared. The pain 1\%; stabneutrophils-10\%; seg.neutrophils-65\%;
didn’t cease after taking one nitroglycerin tablet. lymph/-19\%; mon.-5\%; ESR-35 mm/hour. What
What is the most probable diagnosis? clinical diagnosis is the most probable?
A. * Progressive stenocardia A. *post-hemorrhagic anemia
B. Prinzmetal’s angina B. hemolytic anemia
C. Angina on exertion III funct.class C. hypoplastic anemia
D. Myocardial infarction D. DVC – syndrome
E. Acute myocarditis E. pernicious anemia
433.Patient after over cooling complained about 437.The therapeutist must prescribe to patients primary
continuos fever up to 38?C, cough with small prophylaxis iron deficiency anemia to prevent its
amount of serous sputum. Was under treatment development. Whо is adminested this
at district policlinic, in ten days hectic fever prophylaxis?
appeared in spite of treatment. In three days after A. *Pregnant women
this suddenly expectorated sputum with “full B. Patients after 60
mouth” and very bad smell. Amphoric respiration C. All children
was auscultated over middle part of the upper lobe D. Patients after operation
from the right side. About what disease is it E. Workers of industrial interprises.
necessary to think? 438.The man of 25 years complains of periodic
A. *Pulmonary abscess. epigastralgia. At inspection the chronic gastritis
B. Bronchiectatic illness. with the increased acidity is revealed. Treatment is
C. Lobar pneumonia. lead. What preparation is expedient for using for
D. Pulmonary tuberculosis. primary preventive maintenance of a stomach
E. Cancer of a lung. ulcer?
434.The patient aged 25 yr has dyspnea, cyanotic skin A. *Famotidin
pallor occurred suddenly. Cardiac rate is 130 B. Cerucal
beats per minute. BP is 85/50 mm. Hg. The heart C. Vicalinum
sounds are significantly quiet. On ECG there are D. Maalox
polytopic premature bites, diffuse myocardial E. Gastropharm
changes. On X-ray there is a cardiomegaly. The 439.A 21 years old patient with history of multiple
onset of illness the patient connects with analgin pneumonias complains of malaise, breathlessness,
intake adverse reaction occurred 15 days ago. palpitation, fillings of noisy ears and “flies” in
What is the most probable diagnosis? front of eyes. The patient is ill within one month.
A. * Abramov – Fiedler’s idiopathic myocarditis On physical examination the skin is pale. In the
B. Hypertrophic cardiomyopathy mouth there is ulceretive necrotizing stomatitis.
C. Exudative pericarditis Also purpura in the femoral and abdominal skin
D. Endocardial fibroelastosis areas is noted. The peripheral lymphatic nodes are
E. Mitral stenosis. not enlarged. CBC: RBC are 1,8*1012/l, Hb
435.A patient aged 26 years complains of the 56g/l K.P-0,9 , WBC 2,3*109/l; eosinohyils-
enlargement of cervical, subclavicular, grain 2\% , stub neutrophils 3\%, segmented neutrophils
lymphatic nodes, skin itching, considerable 65\%, lymphocyes 22\%, monocytes 8\%,
sweating at night, fever up to 390C. In CBC there thrombocytes 16*109/l , ESR 25mm/h. In urine
KROK 2 – Question Bank 44
analysis the macrohaematuria is revealed. What 443.In the patient, aged 45 after physical exertion
clinical diagnosis is the most probable? burning pain near the heart, fear of death, sweating
A. *aplastic anemia, hemorrhagic syndrome occurred. On the ECG there are the QS wave,
B. hemolytic anemia cupola – like ST elevation and negative T in I,
C. iron-deficiency anemia AVL, V1 – V3 leads. What is the most probable
D. B12-dificiency anemia diagnosis?
E. acquired hypoplastic anemia A. * Acute myocardial infarction of the anterior
440.A 70 year old patient has got an unhospital wall of left ventricle
pneumonia of the second category. Prescribe the B. Progressive stenocardia
empiric therapy that is most appropriate for this C. Acute myocarditis
case. D. Lung artery thromboembolism
A. *Spiramycin in combination with doxycycline E. Aortic dissection
per os. 444.Patient A. Complains about the shortness of
B. Penicillin intramusculary breath, pains in area of heart, sense of widespread
C. Ampicillin 0,5 6 times a day pulsation. In anamnesis - frequent quinsies
D. Etazol endovenous. (tonsillitis). Objectively: skin covers are pale;
E. None of the variants is right. positive capillary pulse, “dance of carotids”,
441.Patient P., age 52 has IHD (ischemic heart dome-shaped apex beat displaced downward and
disease) for 5 years. He is worried about the to the left, a pulse is quick, high. The left border of
retrosternal pain that occurs after physical heart is displaced to the left. Weakening I and II
exerxises. The pain lasts for 7-8 minutes and stops tones, diastolic murmur over aorta. BP - 180/40
indepently at rest. The patient doesn’t use nitrats mm Hg. ECG: signs of hypertrophy of left
now. There are no pathologic changes of segment ventricle. What most credible preliminary
ST and wave T on the ECG at rest; during the diagnosis?
coronary angiography, decrease in diameter of the A. *Insufficiency of aortic valve
left coronary arteria by 50\% was found. What is B. Feochromocytoma
the pathogenesis of retrosternal pain development C. Chronic nephritis
of the patient in this case? D. Hypertensive illness
A. *The incapacity of injuried coronary arteria to E. Coarctation of aorta
increase its opening on when oxygen in 445.Patient 47 years is delivered at a clinic due to the
needed. attack of difficulty in breathing. Objectively: skin
B. The presence of the spasm in injured coronary is pallid-cyanotic. Position abed – semisitting.
arteria. Breathing is bubbling, 36 per a minute. The heart
C. Pathological adhesion and aggregation of the sounds are dull, accent of II tone above an aorta.
thrombocytes with the extraction of vasoactive Above lungs different moist rales are hearing to.
substances. The WBC–10,5G/l; ESR 4 mm/hour; AST 1,8
D. Hemodynamic factors of blood flow in the mkmol/l; ALT 1,2 mkmol/l; KFK 2,8 mkmol/l.
time of efforts in aorta. Set a diagnosis:
E. The breach of the coronary blood flow on A. *Acute myocardial infarction
account of arteritis. B. Attack of bronchial asthma
442.The atrial flutter with heart rate up to 130 per min C. Acute bronchopneumonia
firstly appeared in a woman aged 28 yr considered D. Acute pleuropneumonia
before healthy. Physical examination reveals pale E. Pneumothorax
bluish cheeks. The respiratory rate is 24 per min. 446.Patient 28 years complains about the suddenly
Cardiac auscultation is difficult because of high appearing strong and frequent blows of heart. In
rate but S1 tone is loud. The chest is free and anamnesis the cardiac diseases are not present,
abdomen is soft and painless. ECG revealed right was much nervous. Objectively: a patient is
ventricle hypertrophy and atrial flutter. This excited, pulse 160 in a minute, BP-150/30 of mm
woman was not consulted by any doctor because Hg, contractions of heart are rhythmic, sounds
she lives in a small village. What is the most loud, clear. ECG: heartbeats 160 per a minute, the
probable diagnosis? QRS complex is stored, the P wave is not
A. *Mitral stenosis determined. In the chest leads the voltage of the T
B. Myocarditis wave is megascopic. Your diagnosis:
C. Thyrotoxicosis A. *Paroxysmal tachycardia
D. Artherosclerotic cardiosclerosis B. Atrial flutter
E. Myocardiodistrophy C. Atrial fibrillation
D. Atrioventricular block of the III degree
KROK 2 – Question Bank 45
E. Respiratory arrhysmia sweat, BP-80/50 mm Hg, heart sounds very dull.
447.The ECG of 10 years old child demonstrates high About what it is necessary to think:
cardiac rate up 180 per minute. Р wave is A. *Diaphragmal myocardial infarction
accumulated on Т and deforms it. The PQ interval B. Peptic ulcer
is slightly prolonged. QRS complex doesn’t C. Acute gastritis
changed. What cardiac disorder has this child? D. Solyarit
A. *Narrow complex supraventricular tachycardia E. Diaphragmal hernia
B. Atrial hypertrophy 452.Patient 30 years with complaints about the
C. Ventricular hypertrophy expressed shortness of breath and palpitation
D. WPW (Wolf – Parkinson – White) syndrome during the last year. On X-ray of the chest
E. Premature bites considerable expansion of heart is exposed. On an
448.Patient 50 years, after heavy physical loading electrocardiogram – decline of amplitude of the R
substernal pain of cutting character appears wave in all leads and negative T. On
suddenly. BP-140/80 of mm Hg; pulse-80 in a echocardiogram – dilatation of ventricles of heart
minute. The rhythm of heart regular, heart sounds with the decline of fraction of the ejection to 35 \
clear. Pain disappeared in 10 minutes %. A valvular apparatus not changed. It is possible
independently. It is known from anamnesis, that to suppose:
except attack of ARVI a patient was nothing ill. A. *Dilated (congestive) cardiomyopathy
About what is it necessary to think? B. Insufficiency of aortic valve
A. *First arising angina pectoris up C. Insufficiency of mitral valve
B. Diaphragmal hernia D. Myocarditis
C. Prinzmetal’s angina pectoris E. Innate heart disease
D. Myocardial infarction 453.Woman, 32 years, during 2 years was treated by
E. Acute pericarditis internist because of neurocirculatory dystonia and
449.At a patient 58 years, 2 years being ill of stable by gynecologist because of menorrhagia.
angina pectoris of tension, the character of clinical Complaints about the promoted fatigability,
displays of angina pectoris changed suddenly, muscular weakness, predilection for strong smells
more often and making of attacks heavier (petrol, ether), the desires eat chalk. Common
appeared, began anymore to accept nitroglycerine, analysis of blood: RBC- 3,5 T/l; Hb – 105 g/l; C.i.
an effect from acceptance diminished. About what – 0,8; hypochromiya, anisocytosis; WBC. - 6,8
is it necessary to think? g/l; B-0\%; E-2\%; St-4\%; S-60\%; L-26\%; M-
A. *Progressive angina pectoris 8\%; ESR 12mm/ch. Whey iron 7-8 mkm/l. What
B. First arising angina pectoris up is it necessary to appoint?
C. Stable angina pectoris of tension A. *Peroral preparations of iron
D. Myocardial infarction B. Vitamins of B group
E. Acute myocarditis C. Folic acid
450.A patient 56 years is at a clinic concerning the D. Parenteral preparationsof iron
acute myocardial infarction the third week. He E. Vitamin C
complains about the increase of temperature, pains 454.The woman aged 30, is suffering from polycystic
in area of heart of a different intensity and kidney. She was admitted to the hospital because
character, pains in area of humeral joint and defeat of weakness thirst, nocturia. Diuresis was about
of skin. There is leukocytosis in a blood, 1800 ml per day. Blood pressure 200/100mm Hg.
eosinophiliya, the ESR, level of (-2 and (-globulin, Blood test: erythrocytes: 1,8*109. Hg-68 g/l.
C-reactive protein increased. About what is it Urine specific gravity is 1005, leucocytes- 50-60,
necessary to think? erythrocytes in microscopic fild - 3-5, creatinine-
A. *Heart postinfarction syndrome of Dressler 0,82 mmole/l, potassium – 6,5 mmole/l, provision
B. Heart postinfarction cardiac insufficiency urine filtration rate 10 ml/min. What’s the
C. Tromboemboliya of pulmonary artery immediate task in the case treatment?
D. Aneurysm of heart A. *Hemodialysis
E. Pleurisy B. antibacterial therapy
451.A patient 65 years during a few days disturbs pain C. sorbents
in an epigastric region, once or twice was D. blood transfusion
vomiting. Appealed to the doctor. It was E. Hypotensive therapy
diagnosed acute gastritis. Got medical treatment. 455.A patient is 60 years. In anamnesis - hypertensive
However much being of patient continued to get illness, angina pectoris. Felt off suddenly
worse – weakness, pallor of skin covers, sticky palpitation, heaviness in area of heart, difficult
breath appeared. At examination: it is pale, it is all
KROK 2 – Question Bank 46
of a cold sweat, lips are cyanotic. A pulse is difficult inhalation. Objectively: orthopnoe. Pulse
frequent, it is difficult to count up. BP 100/50 mm 108 per/min, rhythmic. The heart sounds are dull,
Hg. The heart sounds are dull. A patient is quickly on an apex the rhythm of gallop. In the lower parts
hospitalized. How do you think, with what of lungs moist wheezes. What complication of
diagnosis? hypertensive illness arose up at a patient?
A. *Myocardial infarction A. *Cardiac asthma
B. Hypertensive crisis B. Myocardial infarction
C. Attack of angina pectoris C. Acute violation of cerebral blood circulation
D. Fainting-fit D. Fainting-fit
E. Cardiac asthma E. Pulmonary edema
456.The sick man aged 40 has been suffering from 460.Patient M., 52 years. Complains about headache,
glomerulonephritis. He complains of vomiting, palpitation. About 5 years suffers of hypertension.
muscle cramps in the distal extremity. Blood Had the myocardial infarction one year ago, 6
pressure 180/120 mm Hg, creatinine in blood months ago - cerebral stroke. Objectively: being of
serum – 770 mkmole/l, glomerular filtration – 5 middleweight. The left border of heart is displaced
ml/min. What treatment methods are indicated to laterally on 2,5 sm. Accent of II tone on an aorta,
this patient? systole murmur on an apex. Ps - 96 in a minute,
A. * hemodialisis arrhythmic. BP 190/105 mm Hg. Your
B. sorbents diagnosis?
C. hemo – absorption A. *Hypertensive illness of the III degree
D. plasmapheresis B. Stenosis of bulb of aorta
E. hemofiltration C. Insufficiency of aortic valve
457.A patient 55 years complains about pain, slight D. Coarctation of aorta
swelling in the joints of brushes, long constraint at E. Atherosclerosis of aorta
mornings, limitation of mobility. 1 year is ill. 461.Man 40 years complains about gradually
Treated oneself with ibuprofen with a small effect. increasing headache, nausea, inclinations to
Objectively: swelling of metacarpal-phalange, vomiting, somnolence, pains in a heart, fog before
proximal interphalange joints of the II-III fingers eyes. Suffers with hypertensive illness 12 years.
of both brushes with pain limitation of mobility. Closing dates much worked, got tired.
ESR 37 mm/hour. What researches are most Objectively: it is pale, a face is puffy, Ps-62 /min,
informing for clarification of diagnosis of is hard. BP 200/120 mm Hg. The left border of
rheumatoid arthritis? heart is displaced to the left on 2 cm. Accent of II
A. *Rheumatoid factor, X-ray of brushes joints tone above an aorta, slight edema of shins.
B. Determination of uric acid in blood Specific gravity of the urine is 1015. WBC -
C. Titers of antichlamidias antibodies single in f/vision. ECG: signs of hypertrophy of
D. Presence of LE-cells left ventricle. Your diagnosis?
E. Determination of blood lipids level A. *Hypertensive illness of the II degree, crisis
458.A patient 24 years complains about nausea, B. Ishemic heart disease
vomiting, headache, shortness of breath. In 10 C. Chronic pyelonephritis
years had an acute nephritis. Proteinuria was D. Itsenko-Kushing Illness
found out in urine. Objectively: a skin is grey- E. Coarctation of aorta
pale, the edema is not present. Accent of II tone 462.Patient I., 50 years, complains about headache,
above an aorta. BP 140/100 - 180/100 mm Hg. flashing of beauty spot before eyes, excitation,
Blood level of residual N2 6,6 mmol/L, creatinine trembling in all body. Suffers of hypertensive
406 µmol/L. Day's diuresis 2300 ml, nicturia. illness 5 years. Arterial pressure 140/90 - 180/100
Spac. gravity. of urine is1009, albumen 0,9 g/L, mm Hg. Objectively: skin covers are moist, red
WBC-0-2 in f/vis. RBC.-sin. in f/vis., hyaline color. Ps is tense, 96 per/min. BP - 190/105 mm
casts single in specimen. Your diagnosis? Hg. Left border of cardiac dullness - 1 cm laterally
A. *Chronic nephritis with violation of kidney from a left middle clavicular line. Accent of II
function tone on an aorta. ECG has the signs of
B. Feochromocitoma hypertrophy of the left ventricle. Spec. grav. of
C. Hypertensive illness of the II degree urine 1019. WBC. - 2-4 in f/vis. RBC. - 0-1 in
D. Nephrotic syndrome f/vis. Your diagnosis?
E. Stenosis of kidney artery A. *Hypertensive illness of the II degree, crisis
459.A patient 70 years suffers of hypertensive illness B. Feochromocitoma
more than 20 years. Woke up at night due a C. Acute glomerulonephritis
suddenly arised attack of shortness of breath with D. Atherosclerosis of aorta
KROK 2 – Question Bank 47
E. Insufficiency of aortic valve disappears in 2 minutes. What is the main
463.A patient has complaints about a tormental pathogenic mechanism of the edema in this case?
(agonizing) cough with expectoration to 600- A. *Decrease of difference between the oncotic
ml/daily purulent chocolate color of sputum with a and the venous hydrostatic pressure.
decay smell. Onset of illness was abrupt, Secondary hyperaldosteronism.
temperature 39(C, fever of irregular type. There is B. Decrease of difference between the oncotic
the area of darkening with a cavity in a center on and the venous hydrostatic pressure. Primary
X-ray film, with irregular contours and level of hyperaldosteronism.
liquid. What disease is the question? C. Exceeding hydrostatic over the oncotic
A. *Gangrene of lung. pressure in arterioles. Secondary
B. Tuberculosis. hyperaldosteronism.
C. Bronchiectatic illness. D. Exceeding hydrostatic over the oncotic
D. Abcetic pneumonia. pressure in arterioles. Primary
E. Lobar pneumonia. hyperaldosteronism.
464.A female patient complains of weakness, E. Increase of the left ventrical of the heart filling
dizziness, breathlessness on physical exertion and, pressure. Primary hyperaldosteronism.
diarrhea. In history the patient started to be ill 3 467.Woman, 56 years, complaints about the skin
years ago after operative stomach resection by B- itching, sweating, subfebrile fever with the
II done against a chronic gastric ulcer disease periodic increases of temperature to 38-39 C(, loss
complicated with severe bleeding. On physical of weight, heaviness in right and left
examination the skin is pale and tongue brightly hypochondria, dyspeptic disorders. Objectively: a
“lacquered” red. The CBC reveals a low HB and skin is pale, some icteric. A liver palpate at the
hyperchromic enlarged in size RBCs, their level of umbilicus, a spleen on 2 cm is below than
degenerative forms and poikilocytosis, Zholy’s umbilicus, dense consistency. Common analysis
intra cell inclusions. She was treated by iron of blood: RBC -3,0 T/L, Hb-110 g/L, C.I.-0,8;
preparations and vitamin B12 and then her WBC-28 G/L, bas. – 4\%, eos.-5, myel.-6, St.-8,
condition became much better. What is the type of seg.-70, L-5, M-3, trom.180g/L, ESR mm/hour.
anemia? Your diagnosis:
A. * Anemia due to gastricectomy A. *Chronic myeloleucosis (subleucemic form)
B. Iron deficient anemia B. Chronic myeloleucosis (leucemic form)
C. В12 – folic acid deficient anemia C. Chronic myeloleucosis (Aleucemic form)
D. Coexistence of Iron deficient anemia with D. Acute myeloblastic leucosis
hereditary В12 – folic acid deficient anemia E. Chronic lympholeucosis
E. Aplastic anemia 468.Rectoromanoscopy showed a 1 cm polyp in the
465.Disease began abruptly with shaking chills, severe patient. On histological examination there was
headache and fever up to 39-40?C. Pain appears found an adenomatous polyp. The further
from the left side of the chest in 4 hours during reasonable step in treating this patient can be?
respiration. Cough at once was dry, but in 4 days A. *irrigoscopy
rusty sputum was expectorated. Percussion B. rectum resection
revealed hip sound over left low lobe, vocal C. endoscopy polypectomy
fremitus was intensified, during auscultation was D. repeated analysis of the occult blood in feces
listened bronchial respiration over affected lobe. E. repeated rectoromanoscopy
What percussion sound will be over space of 469.The patient, 45 years, show complaints on severe
Traube? retrosternal pains, not decreasing after usage of
A. * Tympanic sound. nitroglycerine. Objectively: integument’s are pale,
B. Bandbox sound there is cold perspiration, the respiration is
C. Clear lung sound. vesicular and weakened, the tones of heart are
D. Dull-tympanic. rhythmical, pulse beat - 78, arterial pressure -
E. Hip sound. 110/70. On the electrocardiogram: a rhythm is
466.Patient C., age 32, is admitted in a hospital for a right sinus, there is pathological wave Q in the
long time due to toxico-allergic myocorditis. V1-V4, depression of segment ST in III AVF.
There is dyspnea at rest, heart rate 104 per min, What is your preliminary diagnosis?
arrhythmia was also observed. The pulsation of A. *Acute miocardial infarction anterior-septal-
cervical veins occured in bed. There is an edema apex.
in the lower limbs, the skin is pale, and cold B. Attack of a stenocardia.
palpation on skin. The cap-shaped depression C. Thromboembolism of the pulmonary artery.
KROK 2 – Question Bank 48
D. Acute miocardial infarction of posterior wall E. Cholangitis conditioned by the presence of
of the left ventricular. stones in the bile tract
E. Acute pericarditis. 474.The serologic profile of the patient having been
470.Of the patient of nephrological department, who is inoculated with recombinant vaccine against
ill on chronic glomerulonephritis, changed for the hepatitis B (HBV includes the positive result of
worse. The patient is delayed, it is impossible to the test for:
get in touch with him, painful sensitivity is kept, A. *anti-НВс
integument’s are pale and dry, the face is swollen, B. HBsAg
there is a fibrillary twitching of muscles, pupils C. anti-HBe
are narrowed, the respiration is vesicular and D. anti-HBs
weakened, the tones of heart are rhythmical, pulse E. anti - HBc and anti -HBs
beat - 96, arterial pressure - 190/115. The blood 475.Which of the following substances (drugs) can
test: creatinine - 0,352 mmol/liters. What cause granulomatous hepatitis?
condition was developed at the patient? A. *allopurinolum
A. *Uremic coma. B. alchohol
B. Hyperosmotic coma. C. paracetamol
C. Toxic coma. D. methyldopha
D. Coma, which conditioned by damage of E. cordaron
central nervous system. 476.The patient is 36. For a few years he has
E. Hepatic coma. complained of heaviness and a dull ache in the
471.Patient C aged 47 yr complains of intensive skin abdomen. The liver is enlarged, hard and has an
itching, jaundice, bone pain. The skin is uneven surface. CT (computer tomography) has
hyperpigmentated. There is multiple xanthelasma shown multiple pathological changes in the organs
palpebrarum. The liver is +6 cm enlarged, hard of abdominal cavity. The correct diagnosis is:
with acute edge. The blood analysis revealed total A. *polycystosis of the liver;
bilirubin 160 mkmol/l, direct – 110 mkmol/l, AST B. cirrhosis of the liver;
(asparate aminotransferase) 2,1 mmol/l per hour, C. multiple metastases of liver cancer;
ALT – 1.8 mmol/l, alkaline phosphotase 4,6 D. echinococcosis of the liver;
mmol/l per hour, cholesterol – 9,2 mmol/l, E. acute hepatitis.
antimitochondrial antibodies M2 in a high titer. 477.In the patient with the pulsating formation in the
What is the probable diagnosis? epigastrium where systolic murmur has been heard
A. *primary biliary liver cirrhosis a sudden acute pain in the abdomen has occurred.
B. primary liver cancer After the shock condition with loss of
C. chronic viral hepatitis B consciousness was developed. The diagnosis is:
D. acute viral hepatitis B A. *aneurysm rupture
E. alcoholic liver cirrhosis B. ulcer perforation
472.The patient aged 60 yr feels dyspnea on even C. acute appendicitis
slight physical exertion and complains of lower D. peritonitis
limbs swelling in the daytime. He is suffering E. acute pancreatitis
from Ischemic Heart Disease and persistent atrial 478.The 48 years old patient has the left ventricle wall
flutter within 2 years. On physical examination the hypertrophy according to the data of
patient appears acrocyanotic. Heart rate is 120 per echocardiography. What disease has no such a
min., PS 68, BP 105/63 mm Hg. Heart sounds are symptom?
loud, arrhythmic. Chest auscultation reveals moist A. *Dilatative cardiomyopathy
rales in the lower lung regions. What treatment is B. Ischemic heart disease
the most advisable for this patient? C. Hypertonic disease
A. * Digoxin D. Hypertrophic cardiomyopathy
B. Egilok E. Myocarditis
C. Isoptine 479.Splenomegaly, the low RBC (red blood cells)
D. Corinfar count, the high content of urobilinogen in feces of
E. Preductal the patient with jaundice are the symptoms of:
473.The complications of acute cholecystitis which A. *hemolytic jaundice
require surgical intervention are as follows except: B. extrahepatic cholestasia
A. *Jaundice C. intrahepatic cholestasia
B. Empyema of the gall-bladder D. sepsis
C. Emphysematous gall-bladder E. mechanical jaundise
D. Gall-bladder perforation
KROK 2 – Question Bank 49
480.The patient, 32 years, complains of a constant necessary short acting β2-antagonist but not
aching lumbar pains, frequent painful urination, more than 3-4 times per day and at night
subfibrile temperature, a headache. Urine test: B. β2-antagonist of the short action during the
relative density - 1015, protein - 0,066 gr/liters, attack of breathlessness
leukocytes on all field of vision, erythrocytes - 4- C. beclometasone 500 mkg /day, salmeterol 50
5, oxalates. On the roentgenogram: expansion of mkg /day.
kidney plelois. What is your preliminary D. beclometasone 2000 mkg/day, oral
diagnosis? prednisolon 10 mg/day, salmeterol 100
A. *Chronic pyelonephritis. mkg/day.
B. Acute pyelonephritis. E. β2-antagonist of the short action 6 times per
C. Acute glomerulonephritis. day, oral prednisolon 20 mg/day (2 times in
D. Chronic glomerulonephritis. the morning).
E. Paranephritis. 484.A patient aged 43 years was admitted to the
481.A patient K. aged 35 years complains of hospital. She has been suffering from the
weakness, dizziness, headache, palpitation, attacks bronchial asthma for 7 years. The main clinical
of pain in umbilical, sacral areas, fever up to features are following. The patient`s position is
38,20C, urine discoloration resembled smoky orthopnoic and she speaks only with separate
brown and normal stool. The condition started words. The additional muscles take part in the act
after bee sting. On clinical examination the of respiration. Respiratory rate is 32 per min.
jaundice is noted. BP (blood pressure) is 100/60 Wheezes are heard in all lung’s surface. The heart
mm of Hg, spleen is enlarged. RBC rate 125 per min. Expiratory peak volume after
( erythrocytes) are 2,3 * 1012/l, НB 72 g/l, K - short acting B2 antagonists intake is 55\% from
0,9, reticulocytes 13,1\%, WBC (leucocytes) 6,2 the normal value. PaO2 is 58 mm of Hg, PaCO2
* 109/l, ESR -25 mm/hour. In urine urobilin 42 mm Hg, SaO2 – 90\%. Haw should be
reaction is strongly positive, bilirubin pigments classified the bronchial asthma observed in this
are absent. The total bilirubin is 60 mkmol/l, case?
indirect 40 mkmol/l. Serum iron is 20,4 mkmol/l. A. *sever
What clinical diagnosis is the most probable? B. Moderate to severe
A. * Acquired hemolytic anemia C. mild
B. Acute liver insufficiency D. Moderate
C. Iron-deficiency anemia E. Life threatening
D. В12- deficiency anemia 485.A 52 years old hard smoker patient complains of
E. Viral hepatitis persistent cough with purulent sputum discharge
482.A 30 years old woman complains of dizziness, especially in mornings, dyspnea provoked even by
weakness, increased nails fragility, alopecia and slight physical exertion, wheezing chest, tahypnoe,
pica. On clinical examination body t° is 36,7°, the general weakness. He considers himself to be ill
skin is pale, RR (respiration rate) is 20 per min; Ps during 12 years. The overwritten conditions
98, BP (blood pressure) 100/60 mm of Hg. In appear 3-4 times per year usually after common
CBC erythrocytes are 2,8*1012/l, Hemoglobin cold and have tendency to progress. About what
(Hb) 85 g/l, KP-0,9, reticulocytes-2\%; WBC disease do you think first of all?
(leucocytes) 4,7*109/l; eosinophils-2\%; stub A. *Chronic obstructive lung disease
neutrophils-3\%, segmented neutrophils-62\%, B. Bronchial asthma
lymphacytes-26\%, monocytes-5\%, ESR- C. Mucoviscidosis
20mm/hour. Serum iron is 6,8 mkmol/l. What D. Bronchoectatic disease
preparation is necessary to prescribe to a patient? E. Aspergillosis
A. *Iron per os 486.Patient P. aged 62 yr was diagnosed non-hospital
B. Fresh blood bilateral localized in lower lobes pneumonia
C. Erythrocytal mass group III and risk class IV with exacerbated
D. Iron pareuterally chronic obstructive lung disease in III stage. What
E. Vitamin B12-intramuscularly antibacterial preparation should be chosen for this
483.Patient P, 34 years old was diagnosed long-lasting patient for treating in the in-patient department?
bronchial asthma 3rd type and 3rd type of A. *Protected aminopenicillin intravenously or
exacerbation. Pulmonary insufficiency is of the I cephalosporin of the II-III generation plus
type. What therapy should be administered in this macrolid
case? B. Fluoroquinolones of the III-IV generation
A. *beclometasone 1000 mkg/day and at night, intravenously
salmeterol 100 mkg/day and at night, if C. Aryfromycin or quarithromycin
KROK 2 – Question Bank 50
D. Clindamycin C. ІІІ
E. Amycacin D. IV
487.Patient F. aged 23 yr complains of the BT E. Progressive ischemic heart disease
elevation up to 380 C, marked itching of the skin, 492.A patient T. 33 years was admitted to a hospital. A
an appearance of rash on the whole body. She ate patient is pale, at an attempt to stand up complains
strawberries in the evening. What preparations about strong dizziness. There was approximately
should be administered first of all? hour ago vomiting like coffee-grounds. BP - 90/60
A. *Antihistamine preparations of the 3-rd mm Hg., pulse - 120 b/min. It is known from
generation. anamnesis, that a patient during 4 years suffers
B. Antibiotics with ulcerous illness of the stomach, painless
C. Histaglobulin form. An ulcer was exposed at gastrofiberoscopy.
D. Autoserum. Your diagnosis:
E. Antipyretic drugs. A. *Ulcerous illness of stomach, complicated
488.A boy aged 15 yr periodically has been with bleeding
complaining of the epigastric pains, nausea and B. Ulcerous illness of duodenum, complicated
heartburn. Gastroduodenoscopy revealed the sings with bleeding
of gastroduodenitis and ulcer impairment of the C. Erosive gastritis
duodenal mucous membrane. What drug will be D. Acute pleurisy
the most effective in the treatment of this patient? E. Acute myocardial infarction, abdominal form
A. *De-nol 493.Patient T., 48 years. Complaints about the decline
B. Nos-pa of appetite, heaviness in an epigastria region after
C. Papaverin a meal, pains in right hypochondria, aching
D. Atropin character, increasing after a meal, especially fat.
E. Almagel Disturbs nausea, flatulence, disorders of stool, loss
489.3 weeks ago the patient was ill with tonsillitis. of weight. At examination – jaundice of skin,
Clinical examination revealed edema, arterial vascular spiders, gynecomastia, ascites, “caput
hypertension, hematuria, proteinuria (1,8g/per medusae”. A liver at palpation is painless, dense,
day) granular and erythrocital casts. What is the an edge is sharp, a spleen is enlarged. Your
suggestive diagnosis? preliminary diagnosis:
A. *Glomerulonephritis A. *Liver cirrhosis
B. Cystitis B. Chronic active hepatitis
C. Pyelonephritis C. Acute hepatitis A
D. Intestinal nephritis D. Acute cholecystitis
E. Renal amyloidosis E. Acute pancreatitis
490.The 52 years old patient has been admitted to the 494.Patient complains about significant dyspnoe.
hospital. He complains of vomiting, soft black During inspection has forced posture (orthopnea).
repeated stools (melena) during the day. Such Percussion of the lungs revealed hip dullness to
cases have never been before. The pulse rate is 96 the 2-nd rib (backside) with upper boundary as
beats per minute, the blood pressure 100/70 mm parabolic line on the left side. Respiratory
Hg. On finger examination of the rectum there sounds and vocal fremitus over dullness are
was found black feces. The appropriate examining absent. What breathing will be in the area of
is: dullness?
A. *esophagogastroduodenoscopy A. *Will be absent.
B. rectoromanoscopy B. Amphoric.
C. irrigoscopy C. Vesicular.
D. roentgenoscopy of the stomach D. Bronchial.
E. laparoscopy E. Cogwheel.
491.The 58 years old patient complains of the burning 495.A patient is ill 20 years with chronic bronchitis,
pain behind her breastbone which appears when complains about cough with purulent sputum up to
she walks more than 300 m of distance and when 200-300 ml/daily. Lately discharge about 2 liters
she rises on the second floor. The stereotype pain of purulent sputum per day with the biggest
propagated into the left shoulder disappears within amount of sputum in a morning time, fingers as
1-2 minutes after sublingual nitroglycerin intake. “drumsticks”, nails have type of “watch glasses”.
Identify the functional class of ischemic heart About what disease is it necessary to think?
disease: A. *Bronchiectatic illness
A. *ІІ B. Abscess of lung
B. І C. Pulmonary tuberculosis
KROK 2 – Question Bank 51
D. Lobar pneumonia plane abdomen X-ray does not reveal disorders.
E. Gangrene of lungs What is the diagnosis?
496.Patient L., 38 years. Complaints about dull, aching A. *Thromboembolism of superior mesenteric
pains in area of right hypochondria, permanent or artery
arising up in 1-3 hours after the reception of B. Perforated duodenal ulcer
abundant and especially fat food and fried dishes. C. Acute pancreatitis
Pain radiates upwards, in the region of right D. Acute cholecystitis
shoulder and neck. In addition, often disturbs E. All mentioned above
feeling of bitter taste and metallic taste into the 500.Point out the sign which is not a criterion for the
mouth, belch with air, flatulence. At palpation of cessation of a physical load test in patients
abdomen tenderness in area of projection of gall suffering from cardiovascular pathology:
bladder. A liver is not enlarged, a spleen not A. *Increasing the cardiovascular frequency more
palpate. Your preliminary diagnosis: than 30\% as compared with the initial value
A. *Chronic cholecystitis B. The segment ST depression more than 2 mm
B. Cirrhosis of liver from the midline
C. Acute hepatitis C. The development of the ECG complete or
D. Chronic hepatitis partial heart block
E. Acute cholecystitis D. The paroxysmal ventricular tachycardia
497.The 62 year old patient complains of the 10 kg development
weight loss within last 2 months, sometimes dark E. The signs of inadequate cerebral blood supply
red urine voiding occurs and pain in the middle 501.A 70 year old patient is on residential follow-up
back. The patient is feverish up to 390C. On with the diagnosis: ischemic heart disease, stable
palpation there is the left kidney tenderness. In angina of effort, II functional class. Ciliary
blood: hemoglobin is 90 g/l, erythrocytes – arrhythmia, tachysystolic form. Сhronic heart
2,8*1012/l, leukocytes – 8,8*109/l, erythrocytes failure II. Secondary diagnosis: diabetes mellitus,
sedimentation rate – 42mm/h. In urine: specific type II.
gravity is 1018, protein – 0,.66 g/l, A. *Antagonists Ca++, verapamil group
mycroerytrocyturia . The previous diagnosis is: B. Antagonists Ca++, nifedipine group
A. *kidney tumor C. Nitrates of durable action
B. kidney carbuncle D. α-adrenoreceptor blocking agents, selective
C. kidney tuberculosis E. α-adrenoreceptor blocking agents, with
D. acute glomerulonephritis intrinsic sympathomimetic activity.
E. urolithic disease 502.Which of the following antiarhythmic preparations
498.The patient aged 38 within 2 weeks fills pain in is contraindicated in treatment of ventricular
the upper one third part of the sternum that occurs premature bites:
usually when he rises to the 2nd floor. The pain A. *verapamil
lasts up to 5-10 minutes. BP is 120/80 mm Hg. B. metoprolol
ECG T-waves are smoothed out in V1- V4. What C. ethacizine
is the most probable diagnosis? D. amyodaron
A. * The first attack of angina E. propaphenon
B. Myocardial infarction 503.The patient, 67 years, shows complaints on
C. Angina on exertion III ph. class palpitation, dizziness, noise in ears, feeling of
D. Angina on exertion IV ph. class shortage of air. Objectively: the patient is pale,
E. Vegetative vascular dystonia integument’s are damp, respiration is vesicular,
499.A patient aged 45 yr long time suffers with mitral frequency of respiratory movements - 22, pulse
stenosis and arrhythmia joined it last year. Acute beat - 200 times a minute, arterial pressure -
burning pains in the epigastrical region, weakness, 100/70. On the electrocardiogram: frequency of
dizziness appeared suddenly 2 hours ago. The heart beat - 200, ventricular complexes are
main clinical features of the case are following. widened, deformed, the arrangements of segments
The patient’s general condition is severe. Pulse is ST and of wave T is discordant. The wave Р is not
120 beats per minute and weak. BP (blood changed accumulates on QRST, conformity to
pressure) is 90/40 mm Hg). Abdomen is paifull natural laws between Р and QRS is not present.
and rigid (does not take part in respiration). The What infringement of a rhythm was developed at
symptoms of peritoneum irritation are observed. the patient?
Peristaltic sounds are not heard. Usual percussive A. *Paroxismal ventricular tachycardia.
liver’s dullness is preserved. But there is not B. Sinus tachycardia.
dullness in the sloping parts of abdomen. The C. Thimmer arrythmia.
KROK 2 – Question Bank 52
D. Ventricular extrasystole. E. Prazozin.
E. Atrial tachycardia. 508.The patient complains of syncope
504.The patient, 32 years, complains of the general (unconsciousness fits) that started to occur after
weakness, loss of appetite, thirst, dryness in a myocardial infarction which he had 6 months ago.
mouth, a headache, decrease of urinary excretion, On clinical examination the heart sounds are quiet,
aching lumbar pains, edema on legs and the face. heart rate is 40 per min. and BP 110/70 mm Hg.
Arterial pressure - 165/80. The blood test: On Holter ECG monitoring second degree Mobitz
creatinine - 0,18 mmol/liters, hypoalbuminemia, type 2 (3:1, 4:1) A – V block was established.
hyperlipidemia. Urine test: density - 1026, protein Asystolic periods long up to 3-4 sec. What is
- 3, 5 gr/liters. Daily proteinuria - 4. What clinical current treatment?
syndrome conducting at the given patient? A. *artificial pacemaker implantation
A. *Nephrotic. B. prescription of nitrates
B. Uric. C. atropine regular intake
C. Acute nephritic. D. prescription of preductal
D. Hypertensive. E. neopinephrine regular intake
E. Edematous. 509.The female patient aged 28 had had a flu. Two
505.The patient, 40 years, with combined mitral defect weeks later she felt pain behind the breastbone.
with prevalence of a stenosis suddenly felt a sharp On clinical examination there are hoarse voice,
retrosternal pain, difficulty of breath (dyspnea). neck vein’s swelling, absence of relative cardiac
Objectively: a condition of the patient is grave, dullness. ECG voltage is low and segment ST
cyanosis, swelling of cervical veins, tachipnea - 40 rises above the midline in all leads. What is the
times a minute, vesicular respiration, tachycardia - most probable diagnosis?
120 times a minute, arterial pressure - 80/50. On A. * Pericarditis
the roentgenogram: an enlightenment of B. The second myocardial infarction
pulmonary fields on the limited site, expansion of C. Acute muocarditis
a shadow top of superior cava. What is yours D. Idiopathic cardiomyopathy
preliminary diagnosis? E. Angina Pectoris
A. *Thromboembolism of the pulmonary artery 510.The female patient aged 25 had had a flu. Two
B. Acute miocardial infarction. weeks later severe weakness, mild fever,
C. Attack of bronchial asthma. breathlessness and constant aching cardiac pain
D. Attack of heart asthma. there occurred. On clinical examination the first
E. Attack paroxysmal tachycardias. heart sound (S1) is quite (s1). Body temperature is
506.The 46 years old patient complains of the 37,3°C. The ECG voltage is low and first degree
anginous attacks development at night chiefly AV block noted. ESR is 30 mm/h. What is the
from 3 to 4 AM without apparent cause. Within most probable diagnosis?
valoergometric examination at time of 50 w A. * Acute myocarditis
physical load the slight cardiac pain was noted by B. Pneumonia
the patient and this pain disappeared at 75 w load C. Dilatation cardiomyopathy
moment. Make a diagnosis: D. Infectious endocarditis
A. *Prinzmetal’s angina E. Unstable stenocardia
B. Progressive stenocardia 511.According to the New-York Cardiologists
C. Early postinfarction stenocardia Association Recommendations the 55 years old
D. Angina on exertion patient has been made the diagnosis ischemic
E. Angina at rest heart disease, angina on exertion functional class
507.The patient aged 60 had myocardial infarction 10 II, heart failure functional class I, arterial
years ago. He has been suffering from arterial hypertension of the second degree, hypertensive
hypertension for about 15 years. Now he feels heart. Point out the preparation of choice in
palpitation and pain behind the breastbone. BP is treatment of this case:
190/100 mm Hg, heart rate 100 per min. On ECG A. *metoprolol
there are supraventricular premature bites, left B. clophelinum
ventricular hypertrophy and scar on the posterior C. doxasozine
myocardial wall. PQ interval longs 0,18 s. What D. nitroglycerin
preparation is indicated to the patient? E. indapamide
A. * Egilok 512.The 42 years old woman suffers from viral
B. Corinfal hepatitis B. On examination there has been
C. Clophelin determined the absence of comprehensive contact,
D. Triampur confusion (patient`s disorientation in place and
KROK 2 – Question Bank 53
time), the mild smell of acetone out of the mouth. aminotranspherase) fall from 8.0 mmole/l to 3.8
What is your immediate action? mmole/l. What cause triggered the patient’s
A. * admission to the intensive care unit (ICU) deterioration?
B. the blood prothrombin control A. * acute encephalopathy
C. the aminocapronic acid preparation infusion B. joining of the secondary infection
D. the antibiotics prescribe C. biliary ducts obstruction
E. the dynamic observation of vital functions D. acute hemolytic anemia
513.In a 6 year-old child infiltrate of 18 mm in E. infective toxic shock
diameter was detected as a result of the Montoux 518.A child from tuberculosis place was admitted to
test. A 1 year ago Montoux test showed infiltrate the children's department of tuberculosis hospital
of 10 mm. Determine the result of Montoux test. with primary tuberculous complex. On the X-ray
A. *Hyperergic there is shadowing of medium intensity with poor
B. Hypoergic defined outline in 2-nd segment of the right lung,
C. Normergic bound "track" with the root, increased root
D. Anergic lymphonodes. Determine the stage of primary
E. Postvaccinal allergy tuberculous complex.
514.18 years old patient complains of cramping A. Bipolar
abdomen pain, weakness, body temperature up to B. Pneumonic
38,5 for 2 days, scanty portions of stool with C. Consolidation
mucous up to 10 times per day. The patient`s D. Calcification
tongue is white coated and dry. The abdomen is E. Fibrous
painful in the left iliac region. Sigma is 519.The 30 years old patient just now arrived from
spasmodic. What is the suspected diagnosis? Tajikistan. There are complains of fever up to
A. * Shigellosis 40°С, which is accompanied by chills and
B. Thyphoid fever sweating. There has been revealed
C. Food toxicoinfection hepatosplenomegaly in this case. Blood analysis
D. Salmonellosis shows the red cells count (RBC) 3.0 x 10e12/l,
E. Escherichiosis Нb 80 g/l, white blood cells count (WBC) 4.0 x
515.19 years old patient has an acute onset of disease. 10E9/l, Еos 1%, Neutro 65%, Limph 24%, Мon
The body temperature is 36. He complains of 10%, ESR- 25 mm/hr. The previous diagnosis is:
weakness, vomiting without any nausea and A. * malaria
abdomen pain. The large amounts of stool like rice B. adenoviral infection
-water with smell of grated potatoes is the feature C. sepsis
of this case. Diagnosis? D. typhoid
A. * Cholera E. leptospirosis
B. Shigellosis 520.A pain in the axillary area, increase of the
C. Campilobacteriosis temperature developed 10 hours ago in the patient.
D. Botulism On exa-mination: shaky gait is marked, the tongue
E. Salmonellosis is coated by white fur. The pulse is frequent. The
516.Within 20 days the 20 years old man has had a painful lymp-hatic nodules are determined in the
fever and productive cough. By auscultation there axillary area. The skin is bloodshot, glistening
has been detected bilateral diminished air entry above the lymphatic nodules. What is the most
and moist rales. The X-Ray showed volatile probable diagnosis?
(migrants) infiltrates. Point out the diagnosis: A. *Plague
A. * ascaridiasis B. Acute purulent lymphadenitis
B. bronchitis C. Lymphogranulomatosis
C. bronchial asthma D. Anthrax
D. embolism of pulmonary artery E. Tularemia
E. lobar pneumonia 521.In a surgical department appendectomy was
517.The 19 years old patient was admitted in the in- performed on the patient Н. of 30 years old.
patient department with viral hepatitis B. On the Subfebrile temperature persisted operation.
5th day of the disease the patient become to be Periodical watery stool developed. Since the 10th
excited, speaks much. The jaundice is day of the disease the temperature has been 39,5С,
deteriorating. There is tachycardia. The size of with chill. Eruption, enlarged liver, moderate
liver is decreasing up to 2.5 cm per day. The total jaundice, polyarthralgia are marked. What is the
serum bilirubin increased 2 times comparison with diagnosis?
day of admission and ALAT (alanine A. * Yersiniosis
KROK 2 – Question Bank 54
B. Dysentery 526.The patient consulted to the doctor with
C. Typhoid fever complaints of high temperature to 37,8°С,
D. Amebiasis moderate pain in his throat for 3 days.
E. Salmonellosis Objectively: the submandibular lymphatic nodules
522.The patient S. was admitted to the hospital with are enlarged. The palatine tonsils are hypertrophy,
complaints of general cramps, anxiety. He fell ill covered by gray, smooth fur, which is spread to
two days ago. During the last 6 hours the attacks the uvula and forward palatine arches. What is the
became more frequent and prolonged. On most probable diagnosis?
examination: sardonic smile, rigidity of occipital A. * Diphtheria of stomatopharynx
muscles, trunk and extremities are marked. The B. Candidiasis of stomatopharynx
patient injured his foot by rusty wire a month ago. C. Ulcer-necrotic angina of Symanovsky-Viensan
What is a diagnosis? D. Agranulocytic angina
A. * Tetanus E. Infectious mononucleosis
B. Rabies 527.A patient T. fell ill acutely with headache, chill,
C. Meningitis pain in her throat, temperature 38°С. The patient
D. Poisoning with Strychninum was examined by the doctor the same day. The
E. Epilepsy doctor revealed gray firm membranes on the
523.The patient E. has complaints of rough barking tonsils, enlarged and painful lymphatic
cough, loss of voice, difficult respiration. He has submandibular nodules. Diphtheria was suspected.
been ill for 4 days. The temperature is 38°С. What is the most effective method for verification
Objectively: the patient is restless, irritable, of the diagnosis?
stenotic respiration, the tonsils are enlarged, A. * Bacteriological method
covered by dense, pale-gray coats, which are B. Puncture of submandibular lymphatic nodes
spread to the soft palate. The mucous membrane C. Serological method
of the pharynx is cyanotic. What is the preliminary D. Intracutaneous allergic test
diagnosis: E. General blood analysis
A. *Diphtheria of stomatopharynx and larynx, 528.The patient С. fell ill 3 days ago. The disease
croup began by increase of the temperature up to 39,5(С,
B. Foreign body of larynx pains in the muscles of the legs and trunk. There
C. Wide – spread form of diphtheria of are weakness, bleeding from gums and nose. The
stomatopharynx skin and sclera are yellow. What is the diagnosis?
D. ARVI - acute laryngotracheitis A. *Leptospirosis
E. Localized form of diphtheria of B. Viral hepatitis A
stomatopharynx C. Influenza
524.The patient of 20 years old complained of dermal D. Trichinosis
itching, bright reddening and edema in 4 days E. Typhoid fever
after microtrauma in the area of nail phalanx of 529.The patient С. fell ill acutely: a high temperature
the 3-rd finger, received due to preparation of pig (38,0°С), pain in the epigastric area, vomiting,
meat. What is it necessary to think about first? plentiful watery stool without admixtures and
A. *Erysipeloid tenesmus in 3-4 hours after using insufficiently
B. Erysipilatous inflammation fried meat. Objectively: the paleness of dermal
C. Phlegmon integuments, acrocyanosis, tachycardia,
D. Panaritium hypotonia. The stomach is painful on palpation.
E. Paronichy What is the preliminary diagnosis?
525.The patient of 42 years old had attacks of the fever A. *Salmonellosis
every 48 hours during one week. The temperature B. Lambliasis
of the body increased to 40°C and decreased in 3- C. Dysentery
4 hours and accompanied by excessive sweating, D. Escherihioses
general weakness developed. The liver and spleen E. Acute pancreatitis
are enlarged and firm on palpation. What is the 530.The patient М., was admitted with complaints of
most effective method for verification of the nausea, vomiting, spastic abdominal pains, chill.
diagnosis? She be-came ill acutely 2 hours ago after taking
A. *Microscopy of blood smear and thick drop the pasta with meat. In objective examination:
B. Virological method temperature - 38,5°C, Ps-110, BP-110/60 mm Hg,
C. General analysis of blood the tongue is coated and dry. Stool was one time
D. Bacteriological method without admixtures. What investigation is
E. Immunoenzyme analysis necessary for making a diagnosis?
KROK 2 – Question Bank 55
A. *Culture of feces and wash waters A. * Brill’s disease
B. Culture of blood on a bile broth B. Meningitis infection
C. Culture of duodenal contents C. Typhoid fever
D. Virological blood analysis D. Flu
E. Biochemical blood analysis (bilirubin, E. Epidemic Typhus
amintransferases, prothrombin) 536.A patient with the body temperature 38,4°C
531.The patient С. of 18 years has complaints of sore developed severe barking cough, respiratory
throat, headache, weakness, fatigue. On distress with intercostal chest retractions. The
examination: enlarged cervical, axillary and patient looks moderate cyanotic espesially in
inguinal lymphatic nodules up to 1-3 cm in appearance of the nasolabial triangle. What illness
diameter, of dense consistence, non-adhered, provokes this symptoms more frequently then
motile are revealed. There are hyperemia of the others?
fauces and purulent coats on tonsils. A. * Parainfluenza
Hepatosplenomegaly. What is the diagnosis? B. Adenovirus infection
A. *Infectious mononucleosis C. Pneumonia
B. Typhoid fever D. Grippe
C. Sepsis E. Enterovirus infection
D. Diphtheria 537.On the 18th day of the disease in the patient
E. Lacunar angina suffering from typhoid fever the sudden body
532.The preliminary diagnosis was made to the patient temperature fall down up to 37°C, the rapid
– meningococcal disease: meningitis, pulse, thirst and hypotension were noted. The
meningococcemia. The condition is grave. The stool assay (Gregersen) for occult blood is
patient is pale, adynamic. There are many positive. What complication was occurred?
hemorrhagic elements on the skin. Meningeal A. * hemorrhage
signs are positive. The pulse is frequent, weak, BP B. infective toxic shock
– 80/40 mm Hg. The temperature is 40°С. What C. perforative peritonitis
etiotropic therapy would you prescribe? D. anemia
A. * Chloramphenicol intramuscularly E. joining the secondary infection
B. Quinine intravenously 538.The patient complains on high temperature
C. Etazolum intravenously (38,5°С), chill, severe headache, dry cough. The
D. Penicillin intramuscularly skin of his face and conjunctiva are bloodshot. His
E. Streptomycin intramuscularly throat is bloodshot, granular enanthema and
533.The 32 years old man-fisherman has fever up to punctate hemorrhages are on the soft palate. The
400 C, headache and calf muscles pain. At the 5 th pulse rate is 90 beats per minute, rhythmical, of
day of disease onset the depleted urine output satisfactory properties. What is the probable
and increased blood urea and creatinine are noted. diagnosis?
Point out the cause of this condition: A. * Influenza
A. *Leptospirosis nephrosonephritis B. Epidemic typhus
B. The increasing of protein catabolism C. Typhoid fever
C. The high aldosterone secretion D. Q fever
D. Serum prothrombin deficiency E. Acute tonsillitis
E. Leptospirosis liver affection 539.In a 32-year-old patient with infiltrative right
534.A man-hunter complains of a headache, severe upper lobe pulmonary tuberculosis tuberculoma of
weakness, fever about 39°C and right upper limb 3 cm in diameter was formed in 6 months after
edema. On the hand there are an painful ulcer hospital treatment. What radical method of
covered by dark crust on the bottom and small treatment may be used?
vesicles around it. A. *Segmental lung resection
A. * anthrax B. Artificial pneumothorax
B. tularemia. C. Thoracoplastic surgery
C. brucellosis D. Pulmonectomy
D. plague E. Extrapleural pneumolisis
E. forelosis 540.A 19-year-old patient was admitted to the hospital
535.A 70-year old woman has fallen ill acutely. The with the diagnosis: meningitis of the unknown
fever is about 39 0C. The patient is excited, out- etiology. She was investigated. Blood test:
self reflexive, talkative. The other features of the leucocytes- 10,0 x 109/L, neutrophile shift to the
case are scleral injection, facial hyperemia, left, lymphopenia, monocytosis, ESR-19 mm/hr.
Rosenberg’s anathema. Diagnosis? CSF: increase in protein value, decrease in glucose
KROK 2 – Question Bank 56
and chloride counts, leucocytes-130 in 1 ml., 545.A 3-year-old girl was admitted to the hospital in
100\% lymphocytes, MBT (-). In 48 hrs the fibrin critical condition with temperature 39.60C. She
layer was formed. What etiology of meningitis can has been sick for the last 10 days: she has
be suspected in this patient? dyspnea, severe headache. Her mother says she
A. *Tuberculous has been in contact with her uncle who is suffering
B. Viral from tuberculosis. Muscles of nucha are rigid.
C. Pyrulent Sharp dyspnea. What changes in liquor are
D. Insult characteristic for tuberculosis meningitis?
E. Tumor A. *Decreased glucose and chlorides contents
541.A 42 year-old patient with the diagnosis: B. Increased glucose and chlorides contents
Infiltrative pulmonary tuberculosis in the phase of C. Normal glucose and chlorides contents
disintegration, МBТ (+), was hospitalized to D. Decreased protein contents
specialized hospital. Who should conduct final E. Huge quantity of cells
disinfection at the place contaminated by 546.The patient D. was admitted to the hospital on the
infection? second day of the disease with complaints of
A. *Workers of sanitary epidemic control temperature increase up to 39(C, paroxysmal pains
B. Patient in the lower part of the abdomen, frequent watery
C. Members of patient’s family stool with admixture of the mucus. On
D. Medical personnel of a polyclinic examination: the stomach is painful in the left iliac
E. District doctor area. The sigmoid is spasmodic. What is the
542.The disease in a child’s institution began suddenly preliminary diagnosis?
in 2-3 hours after having the cottage cheese, which A. *Acute dysentery
didn’t undergo thermal treatment. All patients had B. Cholera
plentiful repeated vomiting, abdominal pain, C. Esherichiosis
watery stool, paleness of the dermal integuments. D. Toxic food-borne infection
The body’s temperature is 37,3°С. The clinical E. Typhoid fever
manifestations passed during a day. What is the 547.A patient is suffering from disseminated lung
probable diagnosis? tuberculosis. What is the most common
A. *Toxic food infection complication of this process?
B. Poisoning with salts of hard metals A. *Pleurisy
C. Bacterial toxicosis B. Cardio-pulmonary insufficiency
D. Acute intestinal infection C. Kidney insufficiency
E. Mycotoxicosis D. Amyloidosis
543.A patient N. was admitted to the hospital in the E. Pulmonary bleeding
extremely serious condition: cold extremities, 548.Tuberculosis of the right intrathoracical lymph-
acro-cyanosis, dry skin and mucous membranes, nodes complicated with bronchonodular fistula
dark circles around eyes. The pulse rate is was diagnosed in a 9-year-old child at a
frequent, thread. BP - 40/0 mm Hg. The disease tuberculous dispensary, confirmed by
began 10 hours ago: plentiful repeated watery bronchography. A sputum culture examination for
stool without admixtures and repeated vomiting. finding mycobacte-rium tuberculosis was
The severity of the condition is caused by: performed. In what period of time it will be
A. *Hypovolemic shock possible to get the result?
B. Infectious-toxic shock A. *In 6 -8 weeks
C. Distress syndrome B. In a weeks
D. Acute renal insufficiency C. In 3 days
E. Mixed shock D. In 3 months
544.The patient C. fell ill acutely 12 hours ago: E. In a day
temperature 38°C, chill, general weakness, spastic 549.Virage of tuberculin test is found in a 7 year-old
pains in the low parts of the abdomen. 5 hours child from a tuberculous epidemic place. The lung
later frequent scanty stool with admixtures of X-ray reveals enlargement of the right lung root
mucus and blood occurred. The patient drank and dilated contours. Preliminary diagnosis:
unboiled milk. What is the preliminary diagnosis? Tuberculous bronchoadenitis. What form of
A. *Acute dysentery tuberculous bronchoadenitis this child has?
B. Typhoid fever A. *Infiltrative
C. Acute amebiasis B. Small
D. Cholera C. Tumorous
E. Salmonellosis D. Fibrous
KROK 2 – Question Bank 57
E. Cirrhotic A. *Chronic amebiasis, abscess of the liver
550.A 6-year-old child has virage of the Montoux test. B. Chronic relapsing dysentery
He is healthy. What dose of isoniazid must be C. Helminthiasis (diphyllobotriasis)
prescribed to this child for prophylaxis of D. Balanthidiasis
tuberculosis? E. Hypovitaminosis
A. *5 mg/kg 556.The disease began acutely: frequent watery stool
B. 12 mg/kg developed 6 hours ago. The body temperature is
C. 10 mg/kgs normal. Then the vomiting developed. On
D. 25 mg/kg examination: his voice is hoarse, eyes are deeply
E. 15 mg/kg sunken in the orbits. The pulse is frequent. Blood
551.CSF is taken in the patient with suspected pressure is low. There is no urine. There are
tuberculous meningitis. Which kind of cytosis is cramps of the low extremities. What is the
characteristic in tuberculous meningitis? preliminary diagnosis?
A. *Lymphocytous A. * Cholera
B. Neutrocytous B. Typhoid fever
C. Monocytous C. Dysentery
D. Eosinophilous D. Salmonellosis
E. Basophilous E. Toxic food-borne infection
552.A 6-year-old child complain of weakiness, bad 557.In a 3 year-old child with right-sided tumorous
appetite and cough. He has virage of tuberculin bronchadenitis dyspnea, cyanosis and extensive
test. He is pale, malnourished, has dry cough appeared on the ground of receiving
micropolyadenia. BCG cicatrice is absent. On the specific therapy in tuberculosis hospital. During
X-ray there is shadowing in the upper right lobe X-ray control upper lobe of the right lung had
associated with infiltrated lung root. What disease shadowing and reduced in volume, organs of
can be thought of in this patient? mediastinum are displaced to the right. What
A. *Primary tuberculous complex complication had arisen in the child?
B. Pneumonia A. *Atelectasis
C. Lung cancer B. Pneumonia
D. Eosinophilous infiltrate C. Apical pleurisy
E. Pulmonary echinococcus D. Miliary tuberculosis
553.A 37-year-old patient with disseminated E. Abscess of the lung
pulmonary tuberculosis was treated by HRZS in 558.A patient was admitted with the loss of
tuberculous hospital, in three weeks after begining consciousness. He has history of diabetes mellitus
of treatment he felt tinnitus aurium, giddiness, type 1 for 12 years and acute gastroenteritis during
decrease of hearing. Which medication can lead to the last week. There are dry skin, soft eyeballs,
such symptoms? shallow breathing, no acetone smell. What is the
A. *Streptomycin most probable coma?
B. Rifampicin A. *Hyperosmolar
C. Isoniazid B. Hyperlactacidotic
D. Ethambutol C. Ketoacidotic
E. Pyrazinamid D. Hypoglycemic
554.In a 25-year-old student during prophylactic E. Alcoholic
examination foci in the right lung are found. Call 559.A patient presents with the loss of consciousness,
the most common localization of focal dry hot skin, facial hyperemia, Kussmaul’s
tuberculosis in the lung segments? breathing, acetone smell. Blood gluse level - 33
A. *1, 2 mmol/l. Ketoacidotic coma was diagnosed. What
B. 2, 3 is to be done first of all?
C. 1, 5 A. *Intravenous insulin injection
D. 9, 10 B. Intravenous injection of glucose
E. 7, 8 C. Intravenous injection of glutamic acid
555.The inhabitant of India periodically had sharp D. Intravenous injection of sodium chlorine
abdominal pains, indulgence of stool. Stool is solution
sometimes viscous of brown color, with admixture E. Hydrocortisone + thyrotropin
of mucus. The patient lost weight significantly. 560.A 41 years old woman complains of infertility,
The pains in the right hypogastrium, chills, and memory disturbance, appetite loss, skin dryness
high temperature developed two weeks ago. What and mild swelling (pastosity).On physical
is the preliminary diagnosis? examination there are evident bradycardia and
KROK 2 – Question Bank 58
dull heart sounds. Thyroid gland is diffusely D. Hyperlactacidotic coma
enlarged and very hard. The serum thyreoglobulin E. Stroke
antibodies are in 1: 1000 titer. What thyroid gland 565.A 67 years old female with diabetes mellitus type
disease is presented most probably in this patient? 2 presents with severe pain and soreness of the
A. * Ridel's thyroiditis legs, most of all of hips. The touch is extremely
B. Autoimmune thyroiditis painful. What is the most probable diagnosis?
C. Nodular goiter A. *Diabetic acute painful neuropathy
D. Acute purulent thyroiditis B. Autonomic neuropathy
561.A 45 years old patient long time suffers from C. Central chronic neuropathy
kidney polycystic disease with 4th degree of D. Radiculoneuropathy
chronic renal failure and is treating with chronic E. Asthenic syndrome
hemodialysis. She complains of skin itching, 566.A 32 years old patient was presented with primary
lower extremities joints pain. The X-ray of feet chronic adrenal insufficiency of moderate severity.
shows the metatarsus - phalangeal junctions How will be changed the level if 17-KS after the
resorption. The patient takes calcium preparations. test with ACTH (synacthen)?
Serum calcium is 2,2 mmol/l, phosphorus 0,95 A. *No changes
mmol/l, parathormone 554 ph/ml. What diagnosis B. 50 \% increase
is proposed? C. 100 \% increase
A. * Secondary hyperparathyroidism D. 50 \% decrease
B. Multiple myeloma disease E. 100 \% decrease
C. Primary hyperparathyroidism, osteopenic 567.A 42 years old female complaints of nausea,
form frequent defecation, weakness, dizziness, body
D. Neurofibromatosis weight loss. Skin colour is similar to intensive tan.
E. Siple’s syndrome Hyperpigmentation is the greatest on joints, palms.
562.A 38 years old patient was urgently admitted to BP 86/60 mmHg. What is necessary to be
the hospital with complaints of sudden weakness, performed to define the cause of the disorder?
dizziness, loss of consciousness, body weight loss, A. *Determination of urine 17-КS and 17- OHCS
, nausea, vomiting, severe pain in epigastric area, B. Hands X-ray
diarrhea, skin hyperpigmentation. What is the C. Urinalysis
most probable diagnosis? D. Determination of urine catecholamines
A. *Addisonic crisis E. Determination of urine calcium
B. Acute gastroenteritis 568.A 26 years old patient was revealed on a street
C. Meningoencephalitis with the loss of conscioucness. He was admitted to
D. Scleroderma the hospital with provisional diagnosis of
E. Pellagra alcoholic intoxication. Episodes of cramps were
563.A 33 years old female has diabetes mellitus for 5 registered. The skin is dry, skeletal muscles tonus
years. She uses more than 100 insulin units per is increased. Pathological reflexes are positive.
day for the last 6 months. Her body weight gained The smell from his mouth is normal. There are
for 10 kg. Fasting glucose level is 13 mmol/l, signs of injections on the abdomen and hips. Pulse
daily urine glucose – 3\%. Generalized 90 per min, BP 104/75 mmHg. What is the most
microangiopathy. Hypoglycemia was reported probable diagnosis?
after attempts of insulin dosage increase. What is A. *Hypoglycemic coma
the diagnosis? B. Ketoacidotic coma
A. *Insulin resistance C. Alcoholic coma
B. Dawn phenomenon D. Drugs intoxication
C. Diabetic nephropathy E. Cerebral Hemorrhage
D. Diabetic neuropathy 569.A 28 years old patient had used simple insulin for
E. Insulin allergy 2 years. Two days ago he started to use new
564.An unconscious patient presents with moist skin, scheme of therapy with Protaphan. Previous
shallow breathing. There are signs of previous evening he was presented with sudden weakness,
injection on the shoulders and hips. BP 110/70 increased sweating, loss of consciousness. What is
mmHg. Tonus of skeletal muscles and relexes are the most probable diagnosis?
increased. Cramps of muscles of the extremities A. *Hypoglycemic coma
are seen. What is the most likely disorder? B. Hyperosmolar coma
A. *Hypoglycemic coma C. Uremic coma
B. Hyperglycemic coma D. Ketoacidotic coma
C. Hyperosmolar coma E. Stroke
KROK 2 – Question Bank 59
570.The 56 years old patient has worked at the are palpated. Chest is clear and a vesicular type of
aluminium plant more than 20 years. Within 3 respiration is heard in the lungs. Heart sounds are
last years he has developed loosening of teeth, clear, rhythmical. X-ray examination: roots of the
bone and joint pains, piercing pains in heart area, lung are of normal configuration, lung fields are
vomiting. The previous diagnosis was: transparent. Sinuses are free. The CBC:
A. * fluorine intoxication leucocytes are 7,5*109 /L, ESR 17 mm/hour.
B. mercury intoxication Clinical diagnosis is:
C. lead intoxication A. *tuberculosis intoxication
D. phosphorus intoxication B. TB contamination
E. manganese intoxication C. Chest tuberculosis
571.A sick 37 years old woman was admitted to the D. primary tuberculosis complex
district hospital with symptoms of mercury- E. vegetovascular dystonia
organic pesticide poisoning (she ate bread made 575.18-years-old man complains of pain in the right
from pickled seeds). Chose the antidote therapy in hip joint. Contracted year and a half ago.
this case: Movement in the affected joint is restricted, right
A. *Unitol leg is short-cut and on the right hip there is a cold
B. Atropin abscess. Blood test: leucocytes–11,5(109; slight
C. Dipiroxim neutrophilia and lymphopenia. X-picture: signs of
D. Izonitrazin osteoporosis, joint space constriction, destruction
E. Mecaptid of the pelvic bones and the head of thigh bone.
572.A 55 years old patient was diagnosed infiltrative Choose the most likely diagnosis.
tuberculosis of the right lung upper lobe with A. *Tuberculosis if the hip joint.
destructive lesion. AFB (acid fast bacilli) in B. Arthritis deformans of the hip joint.
sputum are positive. The history: 5 years ago he C. Gouty arthritis.
was ill with focus lung tuberculosis. In the in- D. Gonorrheal arthritis.
patient department he was administered anti TB E. Epiphyseal aseptic necrosis of upper end of
(tuberculosis) therapy recommended DOTS as for femur.
the patient of the 3rd category. After the month of 576.A six-year-child whose father suffers from
treatment the mild jaundice is appeared. bacterial lung tuberculosis shown negative
Biochemical blood analysis reveals the increased Mantoux skin test. In what term he has to be
activities of AST and ALT. What anti TB BCG-revaccinated?
preparation caused such complication? A. *not earliar, than in 3 days and not later, than
A. *rifampicyn in 2 weeks.
B. Isoniazidum B. in a month.
C. pyrazinamide C. in two days.
D. ethambutol D. during a year.
E. streptomycin E. in 3 weeks.
573.A 45-year-old woman complains of periodical 577.A patient aged 52 years works at the chicken farm.
painless bleeding during urination, weakness, The main complains are slight fever up to 380C
hyperhidrosis, losing flesh (about 10 kg for last six and a cough with mucopurulent sputum discharge.
months) and continual fever with ague attacks. He associates the disease with the commoncold.
Blood and urine test haven’t revealed any The last fluorography examination was done 2
peculiarities. Tuberculosis of kidney had been years ago.. Now the X-ray examination shows in 6
suspected. Choose the most expedient programme segment projection non-homogenous infiltrative
of additional examination. density with destructive lesion up to 3 cm in size.
A. *Culture urine test for micobacteria, excretal The ESR is 13 mm per hour. The WBCs
urography. (Leucocytes) are 6,8*109 /L in blood. AFB (acid
B. Ultrasound examination of the kidneys, fast bacilli) are found in the sputum. What disease
biochemical blood test is the most probable?
C. Direct smear test of the sputum A. *infiltrative tuberculosis of the lung on the
D. X-kidneys, Mantoux skin test. right
E. Laparoscopy, direct smear urine test B. right-side abscess formation pneumonia
574.A 7 years old boy began to complain of the body C. right lung cancer
t0 elevation up to 37,50C in the evenings, D. pneumoconiosis
weakness, loss of appetite. Mantoux test is 12 mm. E. infarction – pneumonia of the right lung
Last year it was 5 mm. On clinical examination
the skin is pale and enlarged groin lymph nodes
KROK 2 – Question Bank 60
578.Choose the minimal duration of treatment course 583.After delivery a women felt weakness,
in the case of destructive and bacterial lung hyperthermia. On the skin of the nose, cheeks,
tuberculosis effective treatment? breast there are rosy-red spots with grayish scales.
A. *6 months In the center of the foci there’s a scurry atrophy.
B. 4 months Small joints of the extremity are edematic, the
C. 8 months skin around them is red, their function is
D. 9 months destroyed. What’s the clinical diagnosis?
E. 3 months A. *Systemic lupus erythematousus
579.A 47 years old man after lifting a considerable B. Mycosis of the skin
heavy weight suddenly felt an acute pain in the C. Psoriasiform syphilid
right side of the chest and breathlessness. A D. Ruber planus lichen
symptoms increased and mucous and skin became E. Toxiderma
cyanotic. An emergency doctor learned that 7 584.A worker of the cement plant visited doctor with
years ago the patient was ill with an infiltrative the complaints on itch, edema and weeping of the
chest tuberculosis of the right lung, had recovered skin near the fistula from osteomyelitis on the
and was not examined within the last 3 years. On right cruse. Margins of the focus are clear. What’s
percussion exaggerated tympanitis was revealed the most possible diagnosis?
over the right part of chest and auscultation A. *Microbus eczema
revealed the sharply weakened breath sounds here. B. True eczema
What disease should be the first thought? C. Professional eczema
A. *spontaneous pneumothorox D. Allergic professional dermatitis
B. pulmanonary thromboembolism E. Staphylococci sycosis
C. exudative pleurisy 585.A 34-year-old driver saw the doctor because of the
D. right lung atelectasis lesion of the skin of the extremity significant itch
E. croupous pneumonia weeping. He has been ill for 5 months. During
580.A 2 months old child (an infant) was born in term examination of the skin of the back of the hands
with 3550 g of weight and had had the symptoms and lower one-third part of the forearm a lot of
of hemolytic disease of the new - born appeared in papules, vesicles, pustules, scales, erosions, with a
the first days after delivery. No vaccinations were large amount of exudates were revealed. What
made in maternity home. The infant is healthy disease can it be?
now. What vaccine should be inoculated firstly? A. *Chronic eczema
A. *BCG-M B. Acute eczema
B. Diphtheria, tetanus toxoids and pertussis C. Pyoderma
vaccine D. Scabies
C. BCG E. Neurodermatitis
D. ADP 586.In patient B, 34 years old on the skin of the back
E. ADP-M of the hands, abdomen and thigh there’re
581.Laboratory assistant of the bacteriological symmetrical apparent foci of the lesion of
laboratory contracted tuberculosis. What different sizes with unclear borders, where on the
mechanism of contamination is the most probable erymatous background one can see small papulas,
in this case? vesiculas, single pustules and in the centre –
A. *immediate contagion erosion, scabs, weeping. Make your diagnosis?
B. alimentary infection A. *True eczema
C. inhalant infection B. Streptodermia
D. unknown C. Microbic eczema
E. intrauterine infection D. Neurodermitis
582.Patient K, 26 years old, has rosy-red spots, E. Scabies
plaques with distinct borders on the skin of his 587.Primary seronegative syphilis has been established
face, hairy part of the head. On the surface of most in the patient [28 y.o.] on the basis of
foci there are closely set scales. If the patient confrontation and laboratory researches. However
scratch them off, he feels pain. Subjectively: terms of the incubatory period are reduced
moderate itch. What is the diagnosis? considerably. In what case it can be?
A. *Disseminated lupus erythematousus. A. *Tuberculosis pulmonary, oncological
B. Tuberculosis of the face skin. problems, narcotics
C. Photo dermatitis. B. Reinfection
D. Mycosis of the face skin. C. Treatment of the intercurrent illnesses by
E. Face eczema. antibiotics
KROK 2 – Question Bank 61
D. Massiveness of the infection B. Urticaria
E. Bad sanitary and hygienic conditions C. Duhring’s dermatitis
588.The patient [29 y.o.] has consulted a venereologist D. Diffuse neurodermatitis
on the occasion of ulcers on the penis. He had a E. Skin itch
casual sexual connection with the unfamiliar 592.Very strong itch at the area of pubis and inguinal
woman for 1,5 months ago. The failure of penis folds disturbers the patient, 42 years old, who has
has found for 2 weeks ago. Two ulcer elements returned from a foreign mission. During the
have been found out at survey. They had the round examination it was revealed many scratchings,
form, equal edges and they were painless. The haemoragical scabs and spots on the skin of the
lymphadenitis of dense-elastic consistence is lower part of abdomen and internal surfaces of
palpated in the inguinal region. Lymphatic nodes thighs. What is the diagnosis?
are painless and mobile, the skin above them is A. *Pediculosis
without an inflammation, they are not connected B. Scabies
with each other and neighbouring tissues. The skin C. Neurodermitis
of the trunk and limbs has no rash. What about D. Epidermofitia
diagnosis is it necessary to think? E. Erythrasma
A. *Primary Syphilis 593.It was known that woman in childbirth, being in
B. Acute Gonorrhoea pangs, had known gonococcus from the urethra
C. Soft Chancre and vagina. What have to be a treatment of
D. Chancriform Pyoderma newborn child?
E. Scabies A. *To treat of all natural foramens by the 30\%
589.The obstericain-gynecologist has found out the solution albucidi
ulcer with the round form and the size about 10 B. To treat of all natural foramens by manganese
mm on the small sexual lip in the student [19 y.o.] 1:10000
at medical survey. The ulcer did not worry the C. To treat of all natural foramens by penicillin
patient. Objectively: the edges are precise and D. To treat of all natural foramens by acid borici
equal, its colour meat-red, the condensation is in E. To treat of all natural foramens by strong tea
the basis of the ulcer. The priority in the plan of 594.The man, 55 years old, complaints for appearance
patient’s inspection will be: of many rashes in the oral cavity. They are very
A. *Inspection on T.pallidum and serological sickly, especially during the eating. During the
reactions of blood on the syphilis examination there are spread red round erosions,
B. Finding of herpetic antibodies in the whey of without inflammation. What kind of diseases you
blood must think at first?
C. Inspection of sexual partners A. *Pemphigus vulgaris
D. Inspection on the agent of Soft Chancre B. Duhring’s dermatitis
E. Inspection on the Gonorrhoea, Trichomoniasis C. Candidiasis
and other infections transmitted by the sexual D. Aphthous stomatitis
way E. Toxiderma
590.The women, 45 years old, has many bullas in here 595.The patient, 65 years old, has big bullas with
skin. The bullas resemble the peas in dimension. flabby tegmen, red erosions in the mucosan
The rest skin is free of spots, without membrane of the oral cavity, in the limbs and
inflammation. The red erosion are in the oral inguinal area, without any reason. She has a pain,
cavity. They are very sickly. The patient notices especially during the eating. What is the
the put of weight, feels herself bad. What is the diagnosis?
diagnosis? A. *Pemphigus
A. *Pemphigus vulgaris B. Toxiderma
B. Vesical toxidermia C. Duhring’s dermatitis
C. Vesical toxidermia D. Epidermolisis
D. Vesical streptodermia E. Candidiasis
E. Vesical epidermolisis 596.The patient, 46 years old, has inflammatory spots
591.The patient [19 y.o.] has consulted a dermatologist and papules at the skin of nose and cheeks. They
with complaint of the strong inch, which amplifies after being in the skin over a long time. There is
appreciably at night. He has steam papulo- burning at the areas. The symptom of Benje-
vesicular elements of excoriation on the skin of Mesherskiy is positive. What kind of pathological
the trunk, superior limbs and internal surface of appearance was formed this symptom?
the hips. What is preliminary diagnosis? A. *Follicular hyperkeratosis
A. *Scabies B. Ballonic degeneration
KROK 2 – Question Bank 62
C. Vacuolar degeneration sexual contact. During the examination the skin
D. Parakeratosis over the right ankle joint, the left knee joint and
E. Spongiosis the right hip joint was hyperemic, hot to the touch.
597.The patient has many uncavitary infiltraty A. *Reyter’s disease
elements, increasing in the surface of skin. The B. Psoriatic arthritis
elements resemble the grain. There are red, flat, C. Behterev’s disease
poligonalis, with impression in the center. The D. Lupus aerytematosis
network of Wikchem is present. What kind of E. Arthritis rheumatoidis
morphological elements and disease the patient 602.The chlamydiosis (clamidiosis) infection is due to
has? A. * Elementary bodies
A. *Papules by lichen rubber planus B. Reticular bodies
B. Papules by psoriasis C. Anaerobic bacteria
C. Papules by secondary syphilis D. L – form bacteria
D. Papules by eczema chronicle E. А, B are correct
E. Papules by toxiderma 603.What pathologic process in the skin define
598.The patient, 39 years old, has a big pink Nikolsky symptom:
papilomatosic growths on the skin near the anus. A. * Acantholysis
The growths resemble the cauliflower and hold on B. Acanthosis
the thin stalk. The surface of growths is red and C. Spongiosis
macerated. The pain and burning at the area of D. Hydropic degeneration
affection trouble the patient. What is the E. Papillomatosis
diagnosis? 604.To confirm a diagnosis of the tinea versicolor
A. *Condilomas acuminates (branny lichen) what diagnostic test is used?
B. Condilomas lata A. * Baltser’s test
C. Varicose of haemorrhoidal veins B. Three – glass test
D. Pyodermia C. Salt – solution test (blister test)
E. VICH D. Nikolsky symptom
599.The boy 3 years old, fell ill in the first years of E. Benye – Meshchersky symptom
life. There are bullas at the place of traumatism of 605.After the 10th days at the accidental sexual
skin. The same diseases has his mother. During of connection the women had purulent secretion from
the examine it was reveal serous bullas at the skin the genitals, burning. Syringing, genital-bath with
of hands and feet. In the place of opening bullas soda, chamomile, didn’t give any effects during
we can see erosions and serous crusts. What is the one week. During of the examine by gynecologists
diagnosis? /after two weeks from the beginning of disease/ it
A. *Simple bulla epidermolysis was reveal for erosions the big genitals sexual lips,
B. Pemphigus at the smears-gonococcus. Control serological
C. Polimorfic exudativum eritemae reaction /CSR/ for syphilis is negative. What will
D. Duhring’s dermatitis a doctor do?
E. Vesical pyodermia A. * Repeated examination for syphilis
600.Some foci of failure have appeared on the head in B. The treatment of gonorrhea
the patient during last 2 months. The skin has a C. Examine for chlamidiosis
light pink colour, it is covered by grey-white D. Examine for chlamidiosis
scales. The loss of hair is marked as a result of E. Examine for candidacies
their break-down on 2-3 mm above a level of the 606.The patient, 28 years old, living impropriety
skin. Grey-white prickles are found among the sexual life, it has diagnosed-trichomoniasis. What
scales. Separate black points are observed in a kind of preparations must be use for it etiology
place of exit from the follicle. By your diagnosis treatment?
is: A. *Metranidozoli
A. *Trichophytose Superficial B. Sulfanilamide
B. Alopecia Areata C. Penicillin
C. Alopecia Syphilitica D. Abactali
D. Lupus Erythematosus [discoid form] E. Cyprinid
E. Leprosy 607.The women, 29 years old, applied to the doctor
601.The patient, 32 years old, came to the doctor with with completions for massive secretion from the
complaints for the increase of body temperature vagina, itch, burning, pain during urination.
up to 38 degrees Celsius, pain in his joints, During the examination it was reveal hyperemia
conjunctivitis. He felt ill acute after the accidental external sexual organs, vagina. Much quantity of
KROK 2 – Question Bank 63
yellow foamy discharge, urethritis, endocervicitis. A. *Clinical and serological control during 6
What is the diagnosis? months
A. *Trichomoniasis B. Clinical and serological control during 3
B. Gonorrhea months
C. Candidacies C. Clinical and serological control during 1
D. Chlamidiosis month
E. Bacterial vaginosis D. Clinical and serological control during 12
608.The patient, 27 years old, single, living months
impropriety sexual life. During the last 2 month E. Removal from the register
looks unimportance mucous, sometimes purulent 613.The women, 60 years old, hospitalized with
secretion from urethra, subjective-inconstant itch complaints for strong pain at the right side and
or sensation burning from urethra. He didn’t apply rash on skin. She having ill during 5 days. During
to the doctor. What is the diagnosis? of the examine it was reveal grope vesiclaes with
A. *Chronicle gonorrhea serous exudation at the erithematosis skin of
B. Acute gonorrhea intercostals space. During the palpation she has
C. Trichomoniasis pain. What is the diagnosis?
D. Candidacies A. *Circular lichen
E. Herpes B. Pemphig
609.The pregnant is in her second half pregnancy C. Pemphigus
diagnosis chronicle aendocerviciti and urethrity. D. Toxiderma
What prescribe, to the patient? E. Duhring’s dermatitis
A. *Penicillin 614.The pregnant women applied to the doctor with
B. Tetracycline completions for pain during the urination and
C. Riphampicini secretion from the vagina. She is ill during 4 days.
D. Pirogenali During examine it was the clinic of acute
E. Gonovaccini vulvovaginitis. In the smears – are gonococcus.
610.The man, 27 years old, applied to the doctor with What must be a treatment of the patient?
completions for pain in appended right testis, have A. *Penicillin
a higher temperature until 40 degree Calcium, B. -
headache, indisposition. At the anamnesis had C. Metranidazol
gonorrhea. During examine-appendage of right D. Tetraciclin
testis is hyperemier, increase, during of the E. Рirogenal
palpation it was reveal compact sickly infiltration. 615.The woman, who has just returned from a foreign
What is the diagnosis? mission, has the bladders on the skin of her neck
A. *Gonorrhea aepididimitis and shoulder-blade. Very strong itch
B. Tuberculosis epididimitis inconveniences the patient. At the examination we
C. Cancer of testis can see many folliculates, excoriations near the
D. Varicocele of testis bladders. This disease the woman connects with
E. Abscess of testis the wearing orhers clothes. What is your
611.At the 16th day after the abortion the women had diagnosis?
abdomen pain, indisposition, the higher of A. *Pediculosis
temperature until 39,5 degree Cesium. It was B. Scabies
reveal from the anamnesis- accidental sexual C. Toxicodermia
connection with unknown man. She treated D. Pyodermia
independently, only by the external preparations. E. Neurodermitis
Rate erythrocyte sedimentation \ESR\ - 44 mm/h. 616.The women, 28 years old, applied to doctor with
What is the diagnosis completions for limited falling out the hairs. In the
A. *Gonorrhea anamnesis – she had frequent headache
B. Syphilis indisposition, artromyalgia, fever, impropriety
C. Candidiasis sexual life, using narcotics. RW is negative. What
D. VICH of examines must be the first?
E. Herpes A. *Examine for VICH
612.The women suffering from the chronic gonorrhea B. Examine for at neuropathology
finished the complete course of treatment. The C. Examine for gonorrhea
source of the infection was not establish. What D. Examine for fungi
will a doctor do? E. Examine for trichomoniasis
KROK 2 – Question Bank 64
617.The patient has the spots of contagious mollusk pain in throat, diffusion increase of lymphatic
at the face, hairiness part of head, sexual organs. nodes. He has disease during 2 weeks. In
Rash steady to the therapy, with often anamnesis is narcotism. During of the examine
recidivations. Beside, the patient has often fevers there are wide spread rash pink color (spots and
infections, height temperature, until 38 degree of papules), on the head and body, (having peripheral
Celsius, pain in the right side. During the examine increase, violet and brown colors, (reminding
it was reveal is anemia, leukopenia, importance haemangiomae). During of the examine it was
lowering of T-helpers. What is the diagnosis? importance lowering of index T-cells immunity.
A. *VICH What is the diagnosis?
B. Syphilis A. *VICH
C. Lupus tuberculosis B. Secondary syphilis
D. Lupus erithematodes C. Eczema
E. Pyodermia D. Toxiderma
618.The women 32 years old, has periodical E. Psoriasis
hyperthermia, feels bad, put of weight. She has 622.The women 28 years old, complaints for
systemic candidiasis, nothing of any appearance of vesical rashes at the red margin in a
anticandidiasis preparations doesn’t give effects. low lip, after over a long time stay at the cold.
During of the examine it was limphocytopenia, During of the examine – the skin is free from the
anemia, leucopenia, lowering of T-helpers. What rashes. What is the diagnosis?
is the diagnosis? A. *Vesical lichen
A. *VICH B. Duhring’s dermatitis
B. Pemphigus C. Pemphigus
C. Duhring’s dermatitis D. Toxiderma
D. Dermathomyazitis E. Candidiasis stomatitis
E. Lupus erithematodes 623.After the 5 days the accidental sexual connection
619.The man, 40 years old, appealed to the doctor with the man had purulent secretion from the urethra,
the complaints for defeats and pain at sexual sick at the end of urination, discomfort. During of
organs. He is ill during one year. He received the examine of the smears – the leukocytosis in all
some treatment, with out any results, more of it field of vision, insude and extracellular the are
hearth begins to transformate to ulcers being over many gonococcus. What is the diagnosis?
a long of period of time. During of the examine it A. *Gonococci acute back urethritis
was ulcer with not right margins, considerable B. Gonococci acute front urethritis
appliances of immunodeficiency. During the C. Gonococci acute totals urethritis
palpation were pain, polyadenitis. What is the D. Gonococci acute total prostatitis
diagnosis? E. Gonococci acute epididimitis
A. *VICH 624.The patient [56 y.o.] with the chronic furunculosis
B. Pyodermia has consulted a dermatologist with complaints of
C. Syphilis the deterioration of his condition during last year.
D. Psoriasis What researches should be prescribed to this
E. Candidiasis patient first of all?
620.The patient, 27 years old, has erythematous spots, A. *Research on the maintenance of sugar in
plates with fat hyperkeratotic scales yellow color, blood
which had appeared one week ago on the face, B. Wassermann’s reaction
hairiness part of head, extensor surface of C. Research on HIV
extremities. He had this disease some month, after D. Hepatic tests
blood transfusion. The patient complaints of a bad E. Research of the immune status
state, put of weight, diarrhea, the treatment result 625.The young man [25 y.o.] has consulted a
didn’t give. During of the examine it was reveal dermatologist with complaints of the appearance
importance lowering of index of immunity. What of a rash on the skin of the face, trunk and limbs.
is the diagnosis? It disappears quickly and it is accompanied by
A. * VICH intensive itch. Separate non-cavitary elevated
B. Syphilis porcelain elements are marked on the skin. What
C. Toxiderma is the primary morphological element make a
D. Psoriasis clinical picture?
E. Reyters diseases A. *Wheals
621.The patient complaints of bad state, increase of B. Papules
temperature of body until 38-39 degree of Celsius, C. Nodules
KROK 2 – Question Bank 65
D. Pustules 631.The patient has consulted a dermatological clinic
E. Vesicles for consultation. His clinical lesions have caused
626.The patient, 62 years old, who is being treated by of carrying out of special researches. The fusion of
the neuropathologist for the neurithis of neuros intercellular epithelial bridges was found in the
facialis, the doctor noticed on his body, face and prickle-cell layer at histological research of the
limbs the achromic spots of different sizes and the struck epidermis. These changes are signs of:
gradual disappearance of sensuality: at first of A. *Acantholysis
temperature, then of pain and at last tactile and B. Acanthosis
also falling of hair of his eyebrows and eyelashes. C. Dyskeratosis
What kind of examination must be prescribe for D. Granulosis
the patient at the first time? E. Vacuole degeneration
A. *For the mycobacterium of lepra 632.The signs of acanthosis are marked at the patient
B. For the mycobacterium of tuberculosis with the psoriasis at histological research of the
C. For the LE-cells struck skin. For this process is typically:
D. For the treponema pallidum A. *Amplified duplication of prickle-cell layer
E. For the RIF, RITP B. Disturbance of keratinization of the epidermis
627.The patient, 49 years old, has the erythematosic C. Thickening of the granular layer of the
spots and plates on the skin of her face, body and epidermis
limbs, consist of the dense small knots. On the D. Necrobiotic and degenerative changes of the
places of some of them there are hearths of Malpighian layer
atrophies. During the examination it were revealed E. Fusion of intracellular epithelial bridges in the
the breach of temperature, painful and tactile prickle-cell layer
sensuality. What do you think the disease is? 633.Wickham’s positive symptom [that is the small
A. *Lepra netting and web-shaped lines on a surface of the
B. Tuberculosis papules as a result of focal granulotosis] is found
C. Toxiderma in the patient [23 y.o.] by the
D. Sclerodermia dermatovenereologist. By your preliminary
E. Herpes diagnosis is:
628.Nikolsky's positive symptom is found in the A. *Lichen Ruber Planus
patient [66 y.o.] by the dermatovenereologist. By B. Psoriasis
your preliminary diagnosis is: C. Molluscum Contagiosum
A. *Pemphigus vere D. Tuberculosis Cutis Papulonecrotica
B. Solar burn E. Syphilis Secundaria
C. Duhring’s hepetiform dermatitis 634.Tarnovsky’s hypertrichosis is found in the patient
D. Herpes Zoster [26 y.o.] by the dermatovenereologist. It can speak
E. Lupus Erythematosus about:
629.Ardy’s positive symptom [that is the presence of a A. *Late Congenital Syphilis
rash of impetigo’s character in the region of the B. Hirsutism
both ulnar shoots] is found in the patient [23 y.o.] C. Itsenko-Cushing’s syndrome
by the dermatovenereologist. By your preliminary D. Chronic Trichophytosis
diagnosis is: E. Psoriasis
A. *Scabies 635.After the trauma of the right cruse on external
B. Psoriasis surface the man of 27 years old had nodes, which
C. Avitaminosis A later opened with secretion which had a lot of glue
D. Hyperkeratose substance yellow color. The ulcer had appeared,
E. Lichen Ruber Planus which was spherical shape with hand over margins
630.The man [23 y.o.] has small pustules on the skin and necrotic masses on the fundus. The patient
of the face. They are located in the centre of the doesn’t have subjective sensations. What is the
hair follicle and surrounded by a narrow diagnosis?
hyperaemic band. Subjective sensations are A. *Syphilis tertiary
absent. By your clinic diagnosis is: B. Tuberculosis of skin
A. *Ostial Folliculitis C. Furunculosis
B. Chancriform Pyoderma D. Phlegmonous
C. Parasitic Sycosis E. Vasculitis of nodus
D. Furuncle 636.The mother with her doter applied to the
E. Pseudofurunculosis dermatologist with complains for the itch of
hairiness part of head and the back surface of the
KROK 2 – Question Bank 66
neck. During the examine it was reveal the white D. Vesicles.
and gray dandruff, fixating strong to the hair. The E. Papulo-vesicles.
girl has the excoriations, single pustules at the 642.On the trunk of the patient a lot of white and
neck. What is the diagnosis? yellowish-brown elements of various
A. *Pediculosis configuration and size, situated apart on the
B. Seborea sunburned skin have been revealed. Relief and
C. Trihophytia picture of the skin in the foci haven’t changed.
D. Psoriasis About what morphological element can we think?
E. Microsporia A. *Spots without inflammation.
637.The man, 23 years old, came to the doctor with B. Papulas.
complaints for onset the ulcus near the anus 1 C. Spots with inflammation.
month ago. The patient doesn’t have subjective D. Blister.
sensations. He is bisexual. What is the diagnosis? E. Lichenification.
A. *Syphilis primaria 643.Spherical skin defect, size: under 3 cm in the
B. Pyodermia diameter, with gentle margins, necrotic profound
C. Periproctitis fondues, easily, bleeding has been revealed on the
D. Ecthyma scabiosus patients skin of the lateral surface of the lower part
E. Hemorrhoids one-third part of the left crush. What morphogical
638.Neoplasm of red colors, has been revealed in the element can we think about?
region of the patient’s anus. It reminds color A. *Ulcer.
cabbage or a cock’s comb, in palpation it’s soft. B. Erosion.
About what morphological element can we speak? C. Fissure.
A. *Vegetations D. Profound pustule.
B. Warts E. Superficial pustule.
C. Scars 644.The patient, 28 years old, came to the doctor with
D. Papules complaints for the discomfort during the urination,
E. Nodes itch and scanty secretions from his urethra, mainly
639.In patient, 56 years old, on the internal surface of in the morning, pain in the lower part of his
the lower one-third of the left cruse there’s an abdomen. He has been ill for about one month.
apparent ulcer of three-kopecks-coin size with During the examination there was found
even raised margins and flat fundus. The skin hyperemiae near the external outlet from his
around it is red and there’re a lot of vesicles, urethra and leycocitosis in the smears. What is the
erosions, massive scabs. What’s your diagnosis? diagnosis?
A. *Microbus eczema A. *Chlamydeous urethritis
B. Vasculitis of the skin B. Trichomonadia urethritis
C. Ulcer pyoderma C. Gonorrhoeal urethritis
D. Tuberculosis of the skin D. Candidal urethritis
E. Tertiary syphilis E. Bacterial urethritis
640.On the patients elbows and knees there’re rosy-red 645.The boy, 12 years old, has numerous small
elements; size: under 2 cm. Picture of the skin on papules (up to 2 mm in diameter) on the skin of
their surface is clear. In palpation there is some forehead and limbs. They have the color of normal
infiltration, in vitropressure elements get pale. skin and shine. Papules have the hollows is the
What morphogical element can we think about? center. If we try to press the elements, some white
A. *Papulas. crumlike mass secretes. . What is the diagnosis?
B. Turbcles. A. *Contagious mollusk
C. Pustules. B. Streptodermia.
D. Nodes. C. Toxicodermia.
E. Spots. D. Simple verruca.
641.A patient saw the doctor because of the lesion of E. Acne vulgaris.
the face skin. Examination showed grouped small 646.The patient, 35 years old, had a treatment 5 years
elements with transparent contents on the cheeks ago for syphilis secundaria recidiva. He didn’t
and skin of the upper lip on the eritemathosus complete his treatment (he changed his home
background. What are the morphological element address). Now he has tuberculums on his nose and
here? forehead. RW, RITP and RIF are positive. What is
A. *Vesicles. the diagnosis?
B. Pustules. A. *Syphilis tertiary
C. Blisters. B. Furunculous
KROK 2 – Question Bank 67
C. Tuberculosis of skin insufficiency of muscle tonus, bilateral positive
D. Nodules vasculitis Babynsky’s sign.
E. Phlegmonous A. * Dushenne myopathy
647.The patient, 57 years old, has some plates on the B. Multiple sclerosis
skin of the extension surfaces of his limbs, a lot of C. Verdnig-Goffmann spinal amyotrophy
squamules, which are easily shelled away. D. Schtrumpel spastic paraplegia
Isomorphic reaction of Kebner and triad of Auspic E. Erb-Rott miopathy
are positive. What is the diagnosis? 652.Man 27 y.o. has been outside, on the cold air, for
A. *Psoriasis some time. On the following day, after waking up,
B. Lichen rubber planus he has found inability to close left eye, running
C. Eczema tears, hyperacusia, and change of sensation of
D. Neurodermitis frontal 2/3 of the tongue. Objectively: flattening of
E. Mycosis naso-labial fold, left corner of the mouth drop and
648.The women, 33 has scanty secretions, itch in the its paralyses, the “sail” symptom is positive, left
external genitals, unpleasant sensations during corneal reflex is missing. Your diagnosis:
urination, sometimes she feels pain in the lower A. * Neuritis of left n. facialis
part of her abdomen. It was diagnoses B. Neuritis of left n. trigeminus
chlamidioses. What kind of preparations are more C. Neuralgia of left n. trigeminus
effective? D. Tumour of the left pondo-cerebellar corner
A. *Tetracycline E. Neuritis of right n. facialis
B. Penicillin 653.Patient 38 y.o., suddenly started to feel headache,
C. Biseptol accompanied by nausea, vomiting and loss of
D. Retarpen consciousness. Objectively: АP 190/30 mmHg,
E. Cephazolin pulse 94/min, tense; skin is hyperemic.
649.The positive symptom of “the handles of a Craniocerebral nerves are normal, changes in
suitcase” [that is the presence of bridge-shaped movement and coordination are not seen. There
cicatrices with a free space] is found in the patient are positive meningial signs (rigidity of nack
[22 y.o.] by the dermatologist. By your muscles, Kerning’s and Brudsinski symptoms).
preliminary diagnosis is: Liquor is blood stained. Your primary diagnosis:
A. *Tuberculosis Cutis Colliquativa A. * Subarachnoid haemorrhage
[Scrofuloderma] B. Trombosis vessels of the brain
B. Syphilis Tertiaria C. Brain vessels embolism
C. Leprosy D. Haemorrhage on the brain
D. Leishmaniasis E. Meningitis
E. Bejel 654.A 43-year-old woman complains of the low back
650.A 50-years-old man experienced the severe low pain irradiating to the left lower extremity which
limbs pain more often at night. The localization of become worse in movements. Simultaneously the
the pain was not constant. Short time later the patient notes numbness in the leg and cricks
ataxia developed and became worse at bad light. (crumps). The palpation of shank and hip muscles
On ophthalmologic examination the weakness of is tenderness. The hip flexion with knee extension
pupils reaction to light and eyes convergence are is painful and can not be done. There is not
noted. The accommodation is good. On the sensitivity and reflexes loss disorders.
fundus of eyes the disks of optic nerve are of grey A. *Vertebrogenic lumboischialgia on the left
colour. Also the nevrological disorders of deep B. Vertebrogenic radicular syndrome L 5 –S1 on
sensitivity were revealed. The Wassermann’s the left
reaction assay of Treponema pallidum C. Arthritis of the left hip joint
immobilization are positive. D. Endarteritis of the lower extremities
A. * Tabes dorsalis (locomotor ataxia) E. Spinal stroke
B. Myelitis 655.Girl 10 y.o. last three years often suffers from
C. Acute encephalomyelitis otitis. Has had rheumatic endocarditis. Vague,
D. Disorder of spinal blood circulation easy to get tired. Objectively: Patient constantly
E. Syringomyelia moving. All involuntary movements are quick,
651.A 4 years old boy developed the impairment of broad and constantly changing. Your diagnosis:
free motility, he began walk badly, the difficulties A. * Chorea minor
in getting up appeared. The main clinical features B. Parkinsonism
are absence of muscle hyportrophy, weakness of C. Huntington's chorea
deep tendon reflexes, mild tetraparesis with D. Viral encephalitis
KROK 2 – Question Bank 68
E. Wilsons – Konovalov disease manifestations disappeared after proserin
656.Patient 23 y.o. – general constriction, arm tremor, injection.
that gradually developed one year after a serious A. *myasthenia
respiratory viral infection. Objectively: greasiness, B. Parkinson disease
mask-like face, quieting speech, hypersalivation, C. encephalitis
shuffling gait. Your diagnosis: D. ischemic insult in the brainsteam
A. * Postencephalic parkinsonism E. multiple sclerosis
B. Wilsons-Konovalov disease 660.On patient’s examination it was revealed that he
C. Huntington's chorea understands speech addressed to him but he
D. Convexital arachnoiditis cannot speak but there were no motor impairments
E. Basal arachnoiditis of the vocal apparatus. What is your diagnosis?
657.After sleep in the morning a 60 years old patient A. *Motor aphasia
noticed that his left extremities are stillness and B. Sensor aphasia
left-sided sensitivity of skin is impaired. The C. Amnestic aphasia
patient has had a history of two same attacks. The D. Semantic aphasia
conditions are the humid and pale skin, BP 100/65 E. Dysarthria
mm Hg, pulse rate 60 beats per minute. The left 661.The patient V., aged 32, complains of staggering
nasolabial fold is smoothed. The active gait in the dark and in good light. Determine the
movements in the left extremities are absent but kind of ataxia.
reflexes from the ones are higher. Positive A. * Cerebellar
Babinsky’s sign on the left foot and unilateral left B. Sensitive
sided hyperesthesia there are. The meningeal C. Vestibular
irritation tests are negative. D. Functional
A. *Ischemic insult in the right hemisphere of the E. Astasia-abasia
brain 662.The patient has a tumour of hypophysis. What
B. Hemorrhagic insult in the right hemisphere of kind of hemianopsia will develop in it?
the brain A. *Bitemporal hemianopsia
C. Multiple sclerosis B. Quadrant hemianopsia
D. Encephalomyelitis in the brainstem C. Binasal hemianopsia
E. Infarction in the brainstem D. Hononimic right-sided hemianopsia
658.During 3 weeks a 27 years old woman suffers E. Hononimic left sided hemianopsia
from common weakness, sleep disturbances, 663.Dissociated anesthesia develops in damage of …
anxiety, subfebrility up to 37,6 0C, numbness and A. *Posterior horns of the spinal cord
weakness of legs mostly on the left, some times B. Anterior horns of the spinal cord
urine retention. The main neurological features are C. Radices of the spinal cord
following. Cranial nerves function isn't impaired. D. Plexuses
Tendon reflexes are equable from the upper limbs E. Peripheral nerves
and S>D from lower limbs. The exaggerated knee 664.A great spastic epileptic episode in a patient began
reflexes and absent left Achilles’ reflex are noted. when he turned his head and eyes to the right.
The sensitive surfaces on the feet are reduced. The Determine the localization of the pathological
muscle hypotrophy of the lower extremities focus.
attaches the attention. A. * Middle part of the middle frontal gyrus
A. *Acute disseminated myelitis B. Posterior part of the lower frontal gyrus
B. Spinal form of multiple sclerosis C. Upper part of the postcentral gyrus
C. Acute focal myelitis D. Middle part of the postcentral gyrus
D. Tumor of spinal cord E. Posterior part of the upper temporal gyrus
E. Cystic arachnoiditis 665.A 52 year-old patient has damage of the Broca’s
659.Within the last 3 months a 22 years old women centre after hemorrhage in the brain. What speech
complains of the common weakness. In the disorder occurs?
evenings she notes a doubling of vision. In the A. * Motor aphasia
morning she insists that her general condition is B. Sensor aphasia
good. The active movements in extremities are in C. Amnestic aphasia
a full volume but the muscle strength in the D. Semantic aphasia
extremities is depleted. The clinical examination E. Dysarthria
revealed eye lids ptosis, divergent squint, mask- 666.Which type of sensitivity is the most affected
like face and quiet voice. All clinical during early stages of multiple sclerosis?
A. * Vibration
KROK 2 – Question Bank 69
B. Pain polyneuritic type. Which disease would you
C. Stereognosis suspect first?
D. Temperature A. * Diphtherial polyneuropathy
E. Tactile B. Neuropathia of n. sublingualis
667.A tumour of the left frontal lobe base compresses C. Neuropathia of n. glossopharyngeus
the olfactory bulb (bulbus olfactorius). What D. Truncal encephalitis
symptoms are developing? E. Pseudobulbar syndrome
A. * Left -sided anosmia 673.Patient C. 68 y.o. has general and cerebral
B. Olfactory hallucinations arteriosclerosis, arterial hypotonia. Woke up in the
C. Right-sided anosmia morning, can’t move left extremities. There is no
D. Hyperosmia feeling in the left side of the body. Objectively:
E. Bilateral anosmia pale skin, covered with cold sweat. AP 100/65
668.A patient has spastic left-sided hemiplegia which mmHg, HR 60/min, weak. Respiration is weak.
is accompanied by impairment of all kinds of Active movement in the left extremities is not
sensitivity on the same side. Where is the focus of present. Tendinous reflexes are more vivid on the
affection? right when on the left. Babinski symptom on the
A. *The inner capsule left. Meningial signs are not present. Primary
B. The anterior central gyrus diagnosis?
C. The posterior central gyrus A. * Iscemic stroke
D. Cerebral peduncle B. Subarachnoid haemorrhage
E. Medulla oblongata C. Hemorrhagic stroke
669.A patient had thrombosis of the cortical branches D. Transient iscemic attack
of the middle cerebral artery that resulted in E. Acute encephalytis
destruction of the middle part of the right 674.Male 22 y.o. has returned from military service.
precentral gyrus. What motor disorders occurred? Started to feel transient changes of eyesight
A. *Central monoplegia of the left arm sharpness. Weakness of the right foot has
B. Central monoplegia of the right arm appeared one year later. Objectively: horizontal
C. Central monoplegia of the left leg nystagmus, absence of abdominal reflexes,
D. Central monoplegia of the right leg hypereflexia of tendentious and periostal reflexes,
E. Left-sided hemiparesis right foot clonus, Babinski, Chaddock and
670.A child is observed to have arrhythmic Rossolimo symptoms, bilateral Zhukovski
involuntary movements of the extremities and symptom. Primary diagnosis?
trunk. He makes faces, smacks his lips, and often A. * Multiple sclerosis
pulls out his tongue. The muscular tonus of the B. Cerebral arteriosclerosis
extremities is reduced. What character are the C. Lateral amyotrophic sclerosis
forced movements in this patient? D. Acute disseminated encephalomyelitis
A. * Chorea E. Acute transversal myelitis
B. Hemiballism 675.Patient N. 45 y.o., stoker, complains of taking too
C. Myoclonias long to heal burns, he can’t feel. First time
D. Torsion spasm painfree burn had place 10 years ago. On
E. Athetosis examination: sluggish paraparesis of upper
671.A 42-year-old patient is troubled by difficulty in extremities and segmental dissociated bilateral
walking. Objectively: Hypomimia, slowing down loss of sensitivity on the level C4-T2 segments.
of movements, increased muscular tension of Your diagnosis:
plastic type; his steps are slow and shuffling. His A. * Siringomyelia
speech is quiet and monotonous. Determine B. Multipl sclerosis
localization of the process. C. Lateral amyotrophic sclerosis
A. *Pallidonigral system D. Poliomyelitis
B. Hypothalamus-hypophysis zone E. Encefalomyelitis
C. Cerebellum 676.Patient 28 y.o. complains of headache, nausea.
D. Striatum system Has been punched to face a day ago. Loss of
E. Pyramidal system conscience lasted for 3-5 minutes. Objectively:
672.Patient, 35 y.o. has had an illness with fewer and bloodshot left eyeball. Pronounced vegetatic
sore throat. Shortly after he has developed stigmas. Your diagnosis:
dysphagia, dysarthria, weakness and changes of A. *Brain concussion
movement in feet and hands, hyporeflexia, B. Brain contusion
changes of sensitivity of extremities by C. Brain compression
KROK 2 – Question Bank 70
D. Subarachnoidal hemorrhage D. Iscemic spinal stroke
E. Soft tissue injury of the head E. Thorax hemathomyelia
677.Patient has had craniocerebral injury, with 682.Patient 27 y.o. 5th day of respiratory disease,
satisfactory condition after it. However, 10 days appeared strong headache, accompanied by
later, the patient has become weak and drowsy nausea, repeated vomiting, hyperestesia,
with following development of comatose state. photophobia. On examination: lying down with
Neurological status: broad right pupil, left sided thrown back head, legs are brought to trunk,
hemiparesis. Which diagnosis has to be suspected? pronounced rigidity of neck muscles, positive
A. * Subdural haematoma Kernig and Brudsinski symptoms. There are no
B. Epidural haematoma paresises extremitis. Select the main neurological
C. Subarachnoid hemorrhage syndrome:
D. Сerebral ventriclar hemorrhage A. *Meningial syndrome
E. Introcerebral haematoma B. Liquor hypertension syndrome
678.Ambulance was called to a 48 y.o. man. From the C. Liquor hypotension syndrome
words of relatives he has had three episodes of lost D. Radicular syndrome
conscience and fitting during the day. On E. Vegetative crisis
examination: the following fit is observed: patient 683.The 52-years woman suffering from obesity,
lost conscience, fell on the floor, tonic and then complaints to bloody discharges from sexual paths
clonic cramps of trunk and extremities happened. during 4 days. Last normal menses was 2 years
The fit lasted for 4 minute, ended by involuntary ago. Histological investigation of biopsy of the
urination. What type of fit was observed? endometrium has revealed adenomatous
A. * Major epileptic fit hyperplasia. Which reasons from listed below
B. Vegetatic crisis promoted the development of disease?
C. Absence A. *Excessive transformation of preandrogens
D. Episode of hysteria from fatty tissues.
E. Fainting B. Hypersecretion of estrogens by tissues of the
679.Female 29 y.o. complains of head and extremities organism.
tremor, poor memory. On examination: choreotic C. Poor aromatization of preandrogens owing to
hyperkinesis, Kaiser-Fleishner’s rings on the hypothyroidism
eyball, reduction of ceruloplasmin blood level. D. The increased contents of FSH
Your diagnosis: E. Supersecretion of androgens by the cortex of
A. * Hepatolenticular degeneration paranephroses.
B. Huntington's chorea 684.The data of a separate diagnostic curettage of the
C. Pier –Marie’s ataxia mucous of the uterus’s cervix and body made up
D. Friedreich's ataxia in connection with bleeding in a postmenopausal
E. Chorea minor period: in the scraping of the mucous of the
680.Male 30 y.o., noted growing fingers and facial cervical canal no pathology is revealed, at
scull, changed face. Complains of poor eyesight, endometrium - the highly differentiated
tiredness, skin darkening, loss of body weight. X- adenocarcinoma is found. Metastasises are not
ray shows broadening of sella turcica, thinning of found. Which method of treatment is the most
tuberculin sphenoidale, signs of increased correct?
intracranial pressure. What diagnosis can you A. *Surgical treatments and hormonetherapy
make? B. Surgical treatment + chemotherapy
A. * Adenoma of hypophys C. Surgical treatments and radial therapy
B. Encephalitis of truncus D. Radial therapy
C. Optico - hiasmatic arachnoiditis E. Surgical treatments
D. Adrenal gland tumor 685.The woman of 27 years complaints of the disoders
E. Tumor of pondo-cerebellar corner of menstrual function for 3 months, irregular pains
681.Patient, complains of pains in the mid thoracic in the abdomen. In bimanual investigation: in the
part of spine for last 3 months. Recently, gradually dextral appendages range of a uterus there is an
increasing weakness of the right leg is noted. elastic spherical formation, painless, diameter of
Objectively: central paresis of the right leg and 7 sm. USI: in a right ovary –a fluid formation,
loss of pain sensitivity from the nipple level on the diameter of 4 sm, unicameral, smooth. What
left leg. Which disease can be suspected? method of guiding is the most preferable?
A. * Extramedular spinal tumor A. *Administration of an estrogen–gestogen
B. Intramedular spinal tumor complexes within 3 months with repeated
C. Acute thorax myelitis survey
KROK 2 – Question Bank 71
B. Operative treatment B. Hysteroscopy.
C. Dispensary observation of the patient C. Hromogidrotubation
D. Anti-inflammatory therapy D. Colposcopy
E. Chemotherapeutic treatment. E. Cystoskopy.
686.The patient of 40 years complaints of colic pains 690.The woman was hospitalised with full-term
in the lower abdomen and abundant bloody pregnancy. In survey: the uterus is morbid, the
discharge from sexual paths. Last 2 years she had abdomen is tense, cardiac tones of the fetus are not
menses for 15-16 days, abundant, with clots, auscultated. What is the most probable
painful. In anamnesis – 2 medical abortions. In complication of pregnancy?
bimanual investigation: from the canal of the A. *Premature detachment of the normally posed
cervix of uterus - a fibromatous nodes, 3 sm in placenta.
diameter, on the thin crus. Discharges are bloody, B. Labor before term.
moderate. Choose correct tactics: C. Back occipital presentation.
A. *Operation: untwisting of the borning nodes D. Acute hypoxia of a fetus.
B. Hormonal hemostasis E. Hydramnion.
C. Fase by fase vitamin therapy 691.By the end of the 1st period of physiological labor
D. Supravaginal ablation of the uterus without the clear amniotic waters were given vent.
appendages. Contractions lasted 35-40 sec every 4-5min.
E. Hysterectomy without appendages. Palpitation of the fetus 100 beats per minute. The
687.The patient of 40 years complains of discharges AP is 140/90 mm Hg. Diagnosis.
from the vagina of yellow colour. Bimanual A. *Acute hypoxia of the fetus.
investigation: without pathological variations. In B. Labors before term.
smear – Trichomonas vagynalis and blended flora. C. Premature detachment of normally posed
Colposcopy: two hazy fields on the front labium, placenta.
with a negative Iodum probing. Your tactics: D. Back occipital presentation
A. *Treatment of specific colpitis and with the E. Hydramnion
subsequent biopsy 692.In the 40 weeks pregnant woman in intrinsic
B. Diathermocoagulation of the cervix of the obstetric investigation: the cervix of a uterus is
uterus undeveloped. The oxytocin test is negative. Upon
C. Specific treatment of Trichomonas colpitis inspection at 32 weeks it is revealed: AP 140/90
D. Cervixectomy mm. Hg, proteinuria 1 g/l, peripheric edemas.
E. Cryolysis of cervix of the uterus. Reflexes are normal. Choose the most correct
688.The 32 year old woman consulted a gynecologist tactics of guiding the pregnant.
concerning abundant long menses within 3 A. *Labor stimulation after preparation
months. Bimanual investigation: the body of the B. Strict bed regimen for 1 month.
uterus is enlarged according to about 12 weeks of C. Complex therapy of gestosis for 2 days
pregnancy, distorted, tuberous, of dense D. Cesarian section immediately.
consistence. Appendages are not palpated. E. Complex therapy of gestosis for 7 days
Histological investigation of mucosa of the body 693.The 26-year old woman had the second for the last
of the uterus: adenocystous hyperplasia of 2 years labor with Oxytocin application. The
endometrium. Optimum medical tactics: child’s weight - 4080 gr. After the placent birth
A. *Surgical treatment there was a massive bleeding, signs of
B. Hormonetherapy hemorrhagic shock. Despite the introduction of
C. Phytotherapy contractive agents, good contraction of the uterus
D. Radial therapy and absence of any uterus cervix and the vagina
E. Fase by fase vitamin therapy failures, the bleeding proceeds. Choose the most
689.The woman complaints of slight dark bloody probable cause of bleeding.
discharges and weak pains in the bottom of A. *Atony of the uterus.
abdomen within several days. Last menses were 7 B. Failure of cervix of the uterus
weeks ago. The test for pregnancy is positive. C. Hysterorrhesis.
Bimanual investigation: the body of the uterus is D. Delay of the part of placenta.
about 5-6 weeks of pregnancy, of softish E. Hypotonia of the uterus
consistence, painless. In the left appendages - a 694.The woman is admitted to the maternity home
retortlike formation, 7х5sm, mobile, painless. with discontinued patrimonial activity and slight
What it is necessary to do to determinate the fetal bloody discharges from the vagina. The condition
eggs localization. is serious, the skin is pale, consciousness is
A. *USI. confused. AP 80/40 mm Hg. The palpitation of the
KROK 2 – Question Bank 72
fetus is not determined. In anamnesis there was a E. Determination of the contents of Testosteron-
Caesarian section a year ago. Establish the Depotum in Serum of blood.
diagnosis: 699.The primapara M., 20 years, is in in time -labors
A. *Hysterorrhesis. proceeding for 4 hours. Light amniotic waters
B. Presentation of the cord. were given vent. The fetus’ head is pressed to the
C. Placental presentation orifice in the small pelvis. Prospective mass of the
D. Abjointing the mucous fuse from cervix of the fetus 4000,0 ± 200,0. Palpitation of the fetus is in
uterus norm. Intrinsic investigation: cervix is absent,
E. Premature expultion of the amniotic waters. disclosure – 2 cm, the fetal bladder is not present.
695.In the woman of the first day after labor the rise of The head is in 1-st plane of the pelvis, a sagittal
temperature up to 39°С was registered. The juncture is in the left slanting dimension. A
breakage of the fetal membranes has taken place glucose-calcium-hormone - vitaminized
36 hours prior to labors. The investigation of the background was conducted with the purpose:
bacterial flora of cervix of the uterus revealed – A. *Prophylaxes of delicacy of patrimonial
hemocatheretic streptococcus of a group A. The activity.
uterus body is soft, tender. Discharges are bloody, B. Labor inducing
with a mixing of pus. Establish the most probable C. Prophylaxes of hypoxia of the fetus
postnatal complication. D. Antenatal preparation
A. *Metroendometritis E. Treatments of delicacy of patrimonial activity
B. Thrombophlebitis of veins of the pelvis 700.The Primapara of 22 years, was hospitalised for
C. Infected hematoma ante partum preparation concerning the pelvic
D. Infective contamination of the urinary system presentation. The position of the fetus is
E. Apostatis of junctures after the episiotomy. longitudinal, breeches are pressed to the orifice of
696.The woman from a groop of risk (chronic a small pelvis. Palpitation of the fetus is clear,
pyelonephritis in anamnesis) had labor through rhythmical, 140 impacts per minute. Patrimonial
natural patrimonial pathes. In day after labors she activity is not present. What it is necessary to
complains of fever and loin pains, often urodynia. include into antenatal preparation
Establish the most probable complication. A. *Folliculinum
A. *Infection contamination of the urinary B. Oxytocinum
system. C. Partusistenum
B. Thrombophlebitis of veins of the pelvis. D. Prednisolonum.
C. Infected hematoma. E. Zinci sulfas of magnesium
D. Endometritis. 701.The 24 years old primapara, was hospitalised with
E. Apostatis of junctures after episiotomy complains on expultion of the amniotic waters.
697.The 24-years patient in 13 months after the first The uterus during palpation is tonic. The position
labors has addressed with the complaint on of the fetus is longitudinal, is pressed with the
amenorrhea. Pregnancy has concluded by a head to an orifice in a small pelvis. Palpitation of
Caesarian section concerning to a premature the fetus is rhythmical, 140 beats/min, auscultated
detachment of normally posed placenta at the left below a belly-button. Intrinsic
hemorrhage has made low fidelity 2000 ml owing investigation: cervix of the uterus is 2,5 cm long,
to breakdown of coagulability of blood. Choose dense, the externum os is closed, leak light
the most suitable investigation: amniotic waters. Point a correct component of the
A. *Determination of the level of Gonadotropins diagnosis:
B. USI of organs of a small pelvis A. *Antenatal expultion of the amniotic waters.
C. Progesteron assay B. Early expultion of the amniotic waters.
D. Computer tomography of the head C. The beginning of the 1-st term of the labor
E. Determination of the contents of Testosteron- D. The end of the 1-st term of the labor
Depotum in Serum of blood. E. The pathological preliminary term.
698.In the woman of 24 years about earlier normal 702.The 29 year old patient has had a surgical
menstrual function, cycles became irregular, treatment concerning the benign serous epithelial
according to tests of function diagnostics- tumour of the ovary. The postoperative term has
anovulatory. The contents of Prolactinum in blood passed without complications.What is it necessary
is boosted. Choose the most suitable investigation. to prescribe in the rehabilitational term:
A. *Computer tomography of the head. A. *Hormonetherapy and proteolytic enzymes.
B. Determination of the level of Gonadotropins. B. Antibacterial therapy and adaptogens
C. USI of organs of a small pelvis C. Lasertherapy and enzymetherapy
D. Progesteron assay. D. Magnitotherapy and vitamin therapy
KROK 2 – Question Bank 73
E. Does not demand the further observation morbidness. What is the most correct tactics of the
703.The 34-years old woman on the 10-th week of doctor?
gestation /the second pregnancy / has consulted A. * Surgical dissecting, a drainage of an abscess
the doctor of female consultation with the purpose of the gland, antibiotics
of statement on the dispensary record. In the B. Antibiotics, Sulfanilamidums
previous pregnancy there took place hydramnion, C. Surgical dissection, drainage of the abscess of
the child was born with mass of the body of 4086. the gland
What method of investigation is necessary for D. Antibiotic therapy
carrying out, first of all? E. Antibiotics, detoxication and biostimulants.
A. *The test for tolerance to glucose 707.At the woman of 33 years during carrying out tool
B. Determination of the contents of ?? revision of the uterus cavity concerning
fetoproteinum incomplete infected abortion perforation of a wall
C. Bacteriological investigation of discharge of the uterus at a bottom is made. What is the
from the vagina tactics of guiding?
D. A cardiophonography of fetus A. * Suturing of a punched foramen after cutting
E. USI of the fetus. of the edges of the wound
704.The puerpera is on the 4-th day after the normal B. Suturing of a punched foramen
labor. The common state is satisfactory, there are C. Hysterectomy
no complaints. The body temperature 36,5 °С; D. Antibiotics reducing agents, observation
sphygmus is of 80 beats / minutes, satisfactory E. Strict confinement to bed, observation.
properties, rhythmical; AP of 120/80 mm.Hg on 708.The primagravida with pregnancy of 37-38 weeks
both humeral arterias. Mammas are mild, painless, complaints of headache, nausea, pain in
papillas are safe. The uterus is dense, painless, its epigastriums. Objective: the skin is acyanotic.
bottom is 6-8 cm higher than the bosom. The Face is hydropic, there are short fibrillar bounces
lochia is serously-bloody. The diagnosis " a of blepharons, muscles of the face and the inferior
subinvolution of the uterus " is made.It is extremities. The look is fixed. AP 200/110 mmHg;
necessary to manufacture the following: sphygmus of 92 beats / minutes, intense.
A. *To appoint the agents stimulating reductions Respiration frequency 32/min. Activity of the
of the uterus heart isrhythmical Appreciable edemas of the
B. Endometrial instillation of antiseptics inferior extremities. Urine is cloudy.
solutions. A. * Droperidolum of 0,25 \% - 2,0 ml
C. Supravaginal ablation of the uterus B. Dibazolum of 1 \% - 6,0 ml
D. Tool revision of the cavity of the uterus C. A papaverine a hydrochloride of 2 \% - 4,0 ml
E. HBO. D. Hexenalum of 1 \% - 2,0 ml
705.The 26 years old woman complaints of the E. Pentaminum of 5 \% - 4,0 ml.
subitaneously arisen pains in the bottom of the 709.In the primapara, 30 years, intensive attempts with
abdomen, irradiating to the anus, nausea, an interval of 1-2 min, duration 50 sec have
giddiness, bloody dark discharge from sexual tract begun. In time of inclination of the head of the
within one week, the delay of menses for 4 weeks. fetus in the parturient woman complaints on the
Signs of boring of the peritoneum are positive. severe pain in the perineum have developed. The
Bimanual investigation: borders of the body of the perineum, height= 4 sm, has turned pale. What is
uterus and its appendages are not determined it necessary to perform:
because of sharp morbidness. The diverticulum A. *Perineotomy
and morbidness of the back and dextral vaults of B. Epiziotomija.
the vagina are marked. What is the most probable C. Protection of the perineum.
diagnosis? D. Vacuum - extraction of the fetus.
A. *Broken tubal pregnancy E. Waiting tactics.
B. Apoplexy of the ovary 710.The pregnant woman of 29 years old has been
C. Acute right-hand adnexitis suffering from urolithiasis, secondary-chronic
D. Torsion of the leg of the tumour of the ovary pyelonephritis during 8 years. What group of risk
E. Acute appendicitis the occured complication of pregnancy should be
706.At the gynecology department there is a patient of related to?
32years with the diagnosis: "the acute A. *Threat of development of a gestosis
bartholinitis". The body temperature is 38,2 B. Threat of patrimonial traumatism
degrees, leucocytes = 10,4 Т/l, the ESR = 24 mm / C. Threat of bleeding
hour. In the area of big gland of the vestibulum - a D. Threat of delicacy of patrimonial activity
dermahemia, the sign of the fluctuation, sharp E. Threat of isosensibilisation
KROK 2 – Question Bank 74
711.The patient of 23 years old has been registered in E. The aspiration of contents out of respiratory
female consultation with 4 years ago undergone tract and oral cavity
contagious hepatitis, chronic 718.Name the possible pathogenic mechanisms of
cholecystopancreatitis. Point one of the most prenatal fetal contamination:
probable complication of pregnancy in the first A. *Everything that is enumerated below:
trimester. B. Transplacental
A. *Hepatosis pregnant C. Uprising
B. Early gestosis D. Transdecidual
C. Spontaneous abortion E. Descending
D. Anemia pregnant 719.Clinical manifestations of antenatal infection
E. Dermatosis of pregnant depends on:
712.The primagravida of 20 years old which has A. * Everything enumerated below
diabetis for 8 years is hospitalised into the B. Term of pregnancy when infection took place
gynaecology department with early gestosis of C. Virulence as a causative agent
average gravity. Point the most probable D. Channels of infection
complication. E. Species of a causative agent
A. *Hyperglycemic coma 720.The teratogenic effect at time of early terms of
B. Spontaneous abortion pregnancy shows:
C. Hypoglycemic coma A. * All enumerated viruses
D. Bronchial asthma of the pregnant woman B. The Virus of rubella
E. Anemia of the pregnant woman C. Cytomegalovirus
713.Stool in children with salmonellosis is: D. The virus of II type herpes simplex
A. *Green colored, diarrheal, watery and profuse E. The chickenpox virus
with admixture of mucus 721.For treatment of colpitis caused by Candid's
B. Normal fungus it is used:
C. Pea-soup A. *All enumerated below:
D. Discolored without mucus B. "Polyjnuks"
E. Foamy, profuse, foul-smelling and yellow C. "Clotreamazol"
colored D. "Pymaphucin"
714.In case of appearance of bleeding caused by E. The given answers are wrong
infiltrable cancer of a uteral cervix , it's necessary 722.These are characteristic signs of gonococcus
to conduct: A. * The intracellular position in a cytoplasm of
A. * A tight tamponade of vagina mononuclears
B. An amputation of a uteral cervix B. The positive painting by Gramm
C. Clampation of side fornixis (parametriums) C. A rounded shape
D. Laparotomy, ligation of internal iliac artery D. Tropism for the laminated flat epithelium
E. Laparotomy, widened extirpation of an uterus E. Everything enumerated above
[Wertheim's hysterectomy] 723.In case of tuberculosis of genital organs a primary
715.The clinical manifestations of the HELLP- focus is located most often in:
syndrome are: A. * Lungs
A. *Everything that is enumerated below B. Bones
B. Icterus C. Urinary excretory system
C. Pains in the right hypochondrium D. Lymph nodes
D. Blood vomiting E. At the peritoneum
E. Bleeding at the spots of injections 724.In case of gonorrheal salpingitis, the following
716.Possible complications of gestosis can be: listed below are correct except:
A. *Everything that is enumerated below: A. * The lack of an activation of sharp phased
B. Anuria blood indices
C. Cerebral hemorrhage B. A debute of disease in an early follicle stage of
D. Exfoliation of retina a menstrual cycle
E. Blindness C. Double affection of uterus' adnexa
717.To render urgent help in time of an eclampsia D. Of a quick effect of antibiotic therapy
attack it's necessary to accomplish: E. Of a multiple affection
A. *Everything that is enumerated below: 725.In order to define the true conjugate it is
B. Introduction of a gag necessary:
C. The fixation of a tongue with tongue forceps A. * From the value of the external conjugate to
D. Handling a patient in a horisontal state take away 9 cm
KROK 2 – Question Bank 75
B. From the value of the external conjugate to A. * Histological investigation of the biopsy
take away 11 cm material
C. To the value of the external conjugate to add 9 B. Dilated colposcopy
cm C. Cytological investigation of the smears
D. To the value of the external conjugate to add D. Hysterocervicoscopy
11 cm E. Echography of the organs of a small pevis
E. An external conjugate is equal to the real 732.The majority of innocent ovarian tumours are
conjugate transformed:
726.In order to define the size of true conjugate it is A. * Serous cystoadenoma
necessary: B. Fibroma
A. * From the value of the diagonal conjugate to C. Mucinous cystoadenoma
take away 1,5 cm D. Thecoma
B. To the value of the diagonal conjugate to add E. Teratoma
1,5 cm 733.To the retentional cysts of the ovaries is attributed
C. From the value of the external conjugate to everything that is enumerated, except for:
take away the value of the diagonal conjugate A. * Dermoid cyst
D. To the value of the external conjugate to add B. Follicular cyst
the value of the diagonal conjugate C. Yellow body's cyst
E. The diagonal conjugate is equal to the real one D. Paraovarian cyst
727.The big slant size of a fetal head is defined E. "Chocolate cyst"
between : 734.As premorbid background for development of a
A. * An occiput hump and chin gestosis are all enumerated pathologic conditions,
B. An occiput hump and a bridge of nose except:
C. A suboccipital fossa and a front angle of a A. * Lung pathology
great fontanel B. Diabetes mellitus
D. A suboccipital fossa and the middle of a great C. Hypertensive diseases
fontanel D. Fatness
E. A hypoglossal bonelet and the middle of a E. Renal pathology
great fontanel 735.A true or an obstetric conjugate is a distance
728.The straight size of a fetal head is defined between:
between: A. * An inner surface of the upper edge of a pubis
A. * An occiput hump and a bridge of nose and a promontory
B. An occiput hump and a chin B. The middle of the pubis and the place of
C. A suboccipital occiput fossa and the middle of junction of the 2nd and the 3rd lumbosacral
a great fontanel vertebras
D. A suboccipital fossa and a front angle of a C. Lower edge of the pubis and promontory
great fontanel D. Lower edge of the pubis and the pelvic bone
E. A hypoglossal bonelet and the middle of a E. Upper and lower edge of the pubis
great fontanel 736.A diagonal conjugate is measured between:
729.What disorders of a menstrual function are the A. *The lower edge of the pubis and the
most characteristic of an uterus myoma? promontory
A. * Hyperpolymenorrhea B. The upper edge of the pubis and promontory
B. Oligomenorrhea C. The lower edge of the pubis and pelvic bone
C. Algomenorrhea D. The middle of the pubis and the place of
D. Amenorrhea junction of the 2nd and the 3rd lumbosacral
E. Metrorrhagia vertebras
730.Which of the methods of examination is the most E. Ischiococcygeal spines
informative in the diagnostics of a tube infertility? 737.A parturient woman aged 32. After 3d fullterm
A. * Laparoscopy with chromosalpingoscopy delivery, in early afterbirth period there appeared
B. Pertubation bleeding caused by fetus, weigh 4,3 kg. To stop it,
C. Hysterosalpingography the external massage of uterus, introduction of
D. Transvaginal echography uterotonics, manual examination of uteral cavity
E. Bicontrast pelviography walls and a massage on a fist, an ester tampon into
731.Making out the diagnosis of uteral cervix cancer the back fornix of vagina were fulfilled. The result
with the detection of an initial stage is possible in was absent, bleeding lasted, hemorrhage ran up to
time of: 1200 ml. Patient’s condition was bad: she was
pale, heartbeat reached 140b\min, AP 80\40 mm.
KROK 2 – Question Bank 76
Hg. Show the most acceptable method of bleeding right slant size, a small fontanel is left closer to a
termination in the case. sacral bone. Point to the position and the view of
A. * To fulfill the laparotomy with the the position of the fetus
hysterectomy of the uterus without adnexa A. * The 2nd position, the back view
B. To repeat the fist masage of the uterus B. The 1st position, the front view
C. To fulfill the uterus tamponade C. The 1st position, the back view
D. To fulfill the clampation of the parametriums D. The 2nd position, the front view
by Henkel-Ticknadze E. The high-riding sagittal suture
E. To introduct the methylergometrine 741.A patient aged 35 with the presence of the right
intravenously ovarian cyst after a sharp bending had strong pains
738.A pregnant woman (35 weeks), aged 25, was in an abdomen,on the right. At the moment of
admitted to the hospital because of bleeding admission to the hospital the condition is
discharge. In her past history there were two satisfactory, the pulse is 86 b/min, AP – 115/80
artificial abortions. In a period of 28-32 weeks mm Hg, to – 37,2oC. Abdomen is soft, painful at
there was noted the onset of hemorrhage and USD the right illiohypogastric area where are noted
showed a placental presentation. The uterus was in intension of muscles. A menstrual function is
normotonus, the fetus position was transversal (Ist normal. In the time of vaginal investigation the
position). The heartbeats were clear, rhythmical, vagina is bulky, fornixes are free and painless. The
140b\min. Show the further tactics of managing uterus has normal sizes, is densed and mobile. The
the pregnant woman. left adnexa are without any peculiarities. In the
A. * To fulfill a delivery by means of Cesarean area of the right adnexa is palpated a tumor-like
section. mass of size 12x8x8 cm, of elastic consistency,
B. To fulfill the hemotransfusion and to prolong painful. In time of investigation there were noted
the pregnancy increasing pains. Which complication was it in
C. To introduce the drugs increasing blood that case?
coagulation and continue observation A. * Torsion of cyst
D. To fulfill the stimulation of delivery by B. Rupture of cyst
intravenous introduction of oxytocin C. Infection of cyst
E. To fulfill the observation for the intensity of D. Malignant degeneration
hemorrhage and at the moment of stopping the E. Bleeding into cyst
bleeding to prolong the pregnancy 742.The 3rd full-time pregnancy, the 2nd delivery. The
739.The first full-term delivery, II period. The fetal anamnesis showed a spontaneous abortion
position is longitudinal, I position, a frontal view. complicated by metroendotermitis. Following 26
The heart beat is clear, rhythmical, 140 b/min. The weeks some bloody discharge was noted which
head presents, that it can't be determinated by was estimated as a threat of abortion. The
external manner. Outflowed liquid is clear. In time beginning of delivery labor caused some bleeding.
of an internal inspection: the uteral cervix is The uterus had clear circuits, the position of fetus
effaced, dilatation is full, membranes are absent. was longitudinal, the fetal head being slightly bent
The vertex presents sagittal suture which is in a to the opening into a small pelvis. The fetal
straight size, a small fontanel is under the pubis. heartbeats were clear and rhythmical 140 b\min.
The head of the fetus is disposited below the lower At the time of inner inspection bleeding increased,
edge of the pubis, a coccygeai and ischial hums.In common loss of blood equalled to 300 ml. The
the time of labor the fetal head appears from a diagnosis that was made: partial placental
pudendal fissure. What area of pelvis occupies a presentation. Choose the most rational tactics.
fetal head? A. * To fulfill a delivery by means of the
A. * An area of small pelvis outlet cesarean operation
B. That is pressed to the opening into a small B. To fulfill amniotomy
pelvis C. To fulfill amniotomy with the following
C. The area of opening into a small pelvis applications of skin-headed forceps
D. The area of wide part of a cavity of a small D. Accelerate a delivery by intravenous
pelvis introduction of oxytocin
E. The area of narrow part of a cavity of a small E. Fulfill an observation of character of delivery
pelvis activity
740.In time of the vaginal examination of a parturient 743.A patient aged 75 complains of appearing a tumor-
women it was determined: the cervix is effaced, like mass from vagina at the time of small
dilatation is 5 cm, the fetal head is occlused to a physical exertion and suppression of urine. In a
small pelvis' opening. A sagittal suture is in the lying position the above-mentioned tumor can be
KROK 2 – Question Bank 77
easily set into the vagina after which the urination A. *Glucosuria of pregnants
renews. Objectively: pudental fissure gapes at the B. Diabetes mellitus
time of exertion, the whole uterus and vagina C. Disturbances of tolerants to glucosae
walls with a urinary bladder and a rectum come D. Obesity
out, mucosa of the uterus is dry, atrophic. The E. Diabetes mellitus of the 1st type
patient has severe somatic pathology – an 749.A pasurient has atonic bleeding in early
ischemic heart disease, cardiosclerosis, in postpartum period. Postpartum hemorrhage is
anamnesis – double myocardial infarction. 1500ml (1,8\%). Status is hard, the consciousness
Surgical treatment is offered. Which volume of is confusional, stupor, motor anxiety, temperature
surgical intervention is the most rational that the of 37C, pail skin, periphery cyanosis. Tachycardia
patient has: of 130-140 b/min , AP 70/40 mm. Hg., CVP –
A. * Median colporrhaphy 20mm. Hg, dyspnea – 40 per min, hour diuresis
B. Frontal and back colporrhaphy 15-20 ml/hour, haemotocrytis 0.25 , shock index
C. Ventrosuspension 1.4, hemoglobin concentration 70 g/l. Doctor’s
D. Vaginal Hysterectomy tactics?
E. Manchester operation A. *Laporotomy. Extirpation of uterus without
744.The 25 years old patient has complaints about adnexae uteri. Infusion therapy
suppression of mensis during 3 months, vomiting. B. Manual control of cavum uteri and massage on
According internal genicological investigation – the fist after Baksheyev
cyanosis of vaginae and cervix uteri, body of C. Applying the clames to parametrium, claming
uterus is increased. Fundus of the uterus on the 3 the cervix uteri
sm under the symphisis, consistens of uterus is D. Putting the tampone with etherus in the back
soft, during the internal investigation it is fornix
tioghtly.Adnexa uterus are not palpable. E. Cold on beneath the abdomen
Discharges – is mucosae. What is the most 750.A woman in birth for the first time of 29 years old.
probably diagnosis? Pregnancy of 40 weeks. The sizes of the pelvis
A. *Pregnancy in gestation time – 12 weeks are: 25 – 28 – 30 – 20 cm. The presumable weight
B. Fibromyoma of uterus of the fetus is 4.900 +- 200 g. Position of the fetus
C. Horionepithelioma is longitudinal. Head presentation. Heart beating
D. Disturbances of the menstrual cycle of the fetus is 140 per minute. Rhythmical.
E. Amenorhea Vaginal investigation: cervix uteri is shortened. It
745.On the fifth day after the normal delivery the let’s 2 fingers go through. The water bag has not
height of the fundus uteri above pubis equals been open harmed. The head balloting above the
A. *7 – 8 cm aditus ad pelvis minor. Promontorium of the
B. 5 – 7 cm sacrum is not reachable with the finger. What is
C. 9 – 10 cm the tactics of leading the delivery.
D. 11 – 12 cm A. *Cesarian section
E. 12 - 14 cm B. Conservative leading of the childbirth
746.The beginning of delivery is signed with C. Stimulation of the delivery activities
A. *The beginning of regular birth contraction D. Obstetrics forceps
B. Rupture of water bag E. Amniotomy
C. Discharges of mucus from the vagina 751.Patient after the operation of the caesarian section
D. Desccending of the uterus fundus has sharp pains in abdomen, vomiting, nausea,
E. Placing the fetus head or the pelvis in the diarrhea, tachycardia, tachypnoe, enteroparesis,
aditis ad pelvis minor symptoms of irritation of peritoneum are positive.
747.Delivery is finished Temperature is 39 C. In the blood formula is
A. *With the birth of placenta leucocytosis. What is the main difference between
B. Rupture of water bag pelvioperitonitis and parametritis?
C. Complete opening of the ostium of the uterus A. *Positive symptoms of irritation of peritoneum
D. The birth of the fetus B. Type of the pulse
E. Separation of the placenta C. Type of the temperature
748.A 28years-old pregnant at 20 weeks of gestation. D. Type of the pain
In analysis of urine - level of glucosuria is 1.5\%, E. Clinical analysis of blood
diuresis – 2 l. Fasting blood sugar : 5,2 mmol/l; 2 752.The first and the second periods of delivery – are
hours after 75 g glucose load – 6,2 mmol/l. without complications. It is an uterus bleeding
Height is 160 sm, weight is 78 kg. What is the after 10 minutes after childbirth, hemorrhage is
most probable reason of glucosuria? 200 ml. After separation and birth of placenta by
KROK 2 – Question Bank 78
Krede-Lazorevitch’s method bleeding is not 756.On the second day after the normal delivery the
stopping it is increasing. Maternal passages are height of the fundus uteri above pubis equals
not traumated. According examination of placenta A. *12 – 15 cm
it is a defects of placental tissue with size 3x4sm. B. 15 – 16 cm
Uterus bleeding is continuing. What is the most C. 9 – 11 cm
probably reason of the bleeding? D. 7 – 9 cm
A. *Defect of the placental tissue E. 5 – 7 cm
B. Hypotonia of the uterus 757.A woman of 18 years old applied to the female
C. Adherent placenta dispensary at the term of 11 – 12 weeks. One
D. Vegetative placenta month ago a woman suffered from rubella. What
E. Ingrowing placenta influence on the fetus is the most probable in this
753.22 years old patient has complaints about profuse woman?
bloody discharges with clots from genitals, hard A. *Congenital embryopathy
pains in abdomen, like contractions. Last mensis B. Hemolytic disease of the newborn
was 3 months ago. According bimanual C. Delivery trauma of the newborn
investigations : in vagina - a lot of clots of blood, D. Disease of the gyalynum membranes
cervix uteri is open at 2 sm, in the canal of cervix E. Chromosomal abnormalities of the fetus
uteri – elements of fetal ovum, uterus is enlarged 758.A woman is in delivery for the first time. Arrived
to 12 weeks of pregnancy, adnexa uterus are not to hospital in the second period of the delivery in
palpable. What is the most probably diagnosis? the term. Wasn’t observed in the famine
A. *Incipient abortion dispensary during pregnancy. The head of the
B. Treatened abortion fetus is in the thin part of the cavity minor pelvis.
C. Complete abortion The state of the fetus is satisfactory. Small febrilar
D. Late abortion trembling of eyelids , which spreader to the face
E. Incomplete abortion and upper extremities. What is the tactics of
754.33 years-old woman in childbirth (2nd labors). leading the delivery.
Sizes of pelvis 25-28-31-20. Fundal height – 40 A. *Obstretrics forceps
sm, circumferentia of abdomen – 100 sm. Stage of B. Cesarian section
pushing. Passive segment of the uterus is hard C. Fetus destructive operation
painful. Uterine contractions are painful, edema of D. Conservative leading of delivery with
external genitals and muscles of the pelvic floor. epysiotomy
Contrant ring on the line of umbilicus. The sagital E. Vacuum extraction of the fetus
suture of the fetus head is in the direct size at the 759.On the seventh day after the normal delivery the
first plate of the pelvis. Symptom of Henkel- height of the fundus uteri above pubis equals
Vasten is positive. What must be a doctor’s A. *5 – 7 cm
tactics? B. 7 – 8 cm
A. *Anaesthesia, cesarian section C. 9 – 10 cm
B. Injection of spasmolytics D. 11 – 12 cm
C. Obstetric forceps E. 13 – 14 cm
D. Vacuum-extraction of fetus 760.From what time of pregnance fetal heart beating
E. Infusion of uterotonics starts to be heard.
755.The 30 years-old pregnant arrived to the maternity A. *25 weeks
home in 37’s gestation term, head presentation of B. 18 weeks
the fetus. She has complaints about headache, C. 20 weeks
vomiting, pains in the epigastric area, disturbances D. 22 weeks
of vision. Artherial blood pressure 180/110 mm E. 30 weeks
Hg, proteinuria – 1,66 g/l. The tonus of uterus is 761.A 23 years-old pregnant at the 10 weeks of
normal. Fetal heart sounds – 140 beat/min, rhythm gestation has complaints about enlarged thyroid
is normal. What is the doctor’s tactics in this gland. Ps is 72 b/min, artherial blood pressure
clinical case? 110/70 mm/Hg. Thyroid gland is enlarger in all
A. *Intravenous injection of Magnesii sulphatis, parts, painless, mobile, is not fused with
Droperidol, Euphillin underlying tissue. What analysis are necessary to
B. Gastric lavage do for marking of thyroid gland function?
C. Intravenous injection of Dibasol, Papaverin A. *Registration of the TTG content in the blood
D. Anaestethia with Ether B. Registration of T 4 in the blood
E. Intravenous injection of Analgin C. Uptake II 31 by the tyroid gland
D. Ultrasound examination
KROK 2 – Question Bank 79
E. Nucleo-magnetic resonans examination A. *12 – 14 cm
762.During investigation of the 25 years-old pregnant B. 5 – 7 cm
in the term of gestation 34 weeks by the doctor of C. 7 – 9 cm
the female dispensary are registrated edema of D. 9 – 11 cm
legs, AP on the right hand 140/90mm Hg , on the E. 14 – 16 cm
left – 130/90 mm Hg, proteinuria – 0,66 g/l , level 768.The woman in birth for the first time of 20 years
of blood protein – 56 g/l. State of the fetus is old with plenty of amniotic fluid is being in
satisfactory. What is the most probably diagnosis? delivery for 3 hours. Delivery activities are active.
A. *Preeclampsia of light degree In uterus there is one fetus with head presentation.
B. Preeclanpsia of the hard degree Heartbreaking of the fetus is normal. Opening of
C. Edema of pregnants the cervix uteri is 4 cm. Water bag is strained
D. Hypertonic diseases between the contractions. Appoint the tactics of
E. Pyelonephritis of pregnants leading the childbirth.
763.In a woman in delivery with preecplampsia of the A. *Amniotomy
middle degree occasionally dyspnea and dry B. Waiting tactics
cough, motor anxiety, pain in thorax, hemoptysis, C. Cesarian section
tachypnoe, tachycardia, cianosis of the lips D. Sleep, relaxation
appeared. During the aucultation: accent of the E. Stimulation the childbirth with prostaglandins
second tone above the lung artery, multiple rales 769.A woman is in the third in - time childbirth. The
in lungs. What is the most probable complication weight of the woman body is 80 kg. A boy with
appeared? 4.200 kg, and the length of the body of 50cm was
A. *Tromboembolia of the lung artery born. General blood loss is 450 ml. Physiological
B. Eclampsia volume of blood loss for this woman equals :
C. Embolia with amneotic liquid A. *400 ml
D. Edema of the lungs B. 450 ml
E. Attack of the bronchal asthma C. 500 ml
764.Pregnant of 23 years old applying to female D. 550 ml
dispensary complained of infections hepatitis, E. 600 ml
holycistopancreatitis, which she suffered from 2 770.A woman of 30 years old in birth for the first time
years ago. Appoint the most probable has the beginning of intensive pushing with the
complication of pregnancy in this woman: interval of 1 – 2 minutes, lasting 45 seconds.
A. *Hestosis During the birth of the fetal head, extraordinary
B. Preliminary childbirth pain in the perineum appeared. The perineum 4
C. Hemorrhage cm high, is pail. Heart beating of the fetus is not
D. Rupture uteri affected. What is to be done:
E. Weakness of the delivery activities A. *Perineotomy
765.The 19 years -old woman is in the delivery first B. Episiotomy
time. It is the begining of the first period of labors. C. Protection of the perineum
In her anamnesis – metral cardiac defect in the D. Waiting tactics
stage of compensation. Her state is satisfactory. E. Pudendal anesthesia
Amniotic bag is not break. What is the doctor’s 771.When can a woman stand up and walk after
tactics in this delivery? childbirth?
A. *Active-waiting tactics with presents of A. *In 6 – 8 hours
therapeutist and maximum analgesia B. In 24 hours
B. Cesarean section C. In 2 hours
C. Obstetrics forceps D. In 3 days
D. Vacuum-extraction of fetus E. Right away after childbirth
E. Stimulation of the labor activity 772.The woman of 25 years has addressed to the
766.On the fourth day after the normal delivery the doctor with complaints of a nausea, vomiting 1
height of the fundus uteri above pubis equals times in a day, a sleepiness, a delay of a menses
A. *9 – 11 cm for 2 months. At bimanual inspection: cyanosis of
B. 5 – 7 cm the mucosa of the vagina and uterine cervix. The
C. 7 – 9 cm uterus is enlarged, softened, especially in a region
D. 12 – 14 cm of an isthmus, however during inspection began
E. 14 – 16 cm more dense, painless. Appendages are not
767.On the third day after the normal delivery the palpated. Discharge are mucous. The most relible
height of the fundus uteri above pubis equals diagnosis.
KROK 2 – Question Bank 80
A. *Pregnancy functions are normal What Hormon secreted by a
B. Myoma of uterus pituitary gland, stimulates reductions of a uterus in
C. A salpingocuesis a puerperal period?:
D. Infringement of a menstrual cycle A. * Oxytocinum
E. Ovarian tumor B. Folliculinum
773.The 24-years old woman, earlier not pregnant, C. Progesteronum
terminated to accept oral contraceptives. After last D. Chorionic Gonadotropinum
reception of a drug she had one menses, and then E. Prolactinum
within 6 months the amenorrhea was observed. 777.At the puerpera of 26 years old, for 4 day after
Choose the most suitable investigation: labour the incessant parent bleeding began. The
A. *USE of the organs of the small pelvis haemorrhage has made 400 ml. The common state
B. Determination of the level of Gonadotropins is worsened - a body temperature 36, 7о С, pulse
C. Determination of the level of Progesteron of 94 beets / mines, the AP of 90/70 mm.Hg. The
D. Determination of the level of Testosteron uterus is intense, morbid, its bottom is at a level of
E. Computer tomography of the head a umbilicus. The diagnosis is: “Delivery in time. A
774.The puerpera of 24 years old, the 2d day after bleeding of the 4th day of puerperal term.” It is
labor. A common state is satisfactory, a body necessary:
temperature is 36,6 degrees , sphygmus 82 beets / A. *Tool revision of a cavity of the uterus
mines, satisfactory properties. There is no B. Manual inspection of a cavity of the uterus and
pathology from internals. Mammas are enlarged in erasion of the delayed parts of a placenta
the dimensions, mild, papillas are whole. The C. Outside massage of a uterus after bleeding
uterus is dense, painless, a bottom of it is on 5 sm urinary bladder
above a bosom. A lochia are serouse-bloody, in a D. To enter drugs reducing a uterus
small amount. Physiological functions are normal. E. Supravaginal ablation of a uterus
It is enough for determination of character of 778.A 5 years old girl who attends a kindergarden has
lochia: an atypical pneumonia. What antibacterial drug
A. * A sighting should be prescribed?
B. Microscopies lochia A. * Macrolides
C. A luminescent microscopy of lochia B. Quinolones
D. Bacteriological investigation of lochia C. Cephalospores of the 1-st generation
E. Vaginoskopy. D. Cephalosporines of the 2-st generation
775.The puerpera of 22 years old for 2 day after E. Cephalosporines of the 3-st generation
normal laborC. A common state is satisfactory, a 779.The pregnant woman, aged 25 yr with 36 weeks
body temperature is 36,5 degrees, puls of 80 beets gestational term complains of upper abdomen
/ mines, satisfactory properties. Mammas are pain, nausea, vomiting and blurred vision. There is
enlarged, papillas are whole. The uterus is dense, also generalized edema. BP on both arms is
painless, a bottom is on 8 cm above a bosom. A 170/100 mm Hg. What is the most probable
lochia are bloody, in a small amount. diagnosis?
Physiological functions are normal. In a puerperal A. *severe preeclampsia
period it is enough for determination of dynamics B. moderate preeclampsia
of reduction of a uterus: C. eclampsia
A. *Palpation to define a level of standing of a D. retinal detachment
uterine fundus E. epileptic coma
B. USE 780.A 25 years old woman in labor has the second
C. X-ray inspection timely labor. The contractions appear every 5-6
D. To meter a circle of an abdomen. minutes and last 20-25 seconds. The longitudinal
E. Palpation to define a level of standing of fetal position and occipital presentation of the fetal
contractive rings of a uterus head are found against the small pelvis entry.
776.The puerpera of 25 years old, 7 day after labor. What is the stage of labor?
The common state is satisfactory, there are no A. *Opening of the uterine cervix
complaints A body temperature is 36,6 degrees, B. Expulsion of the fetus
puls is 76 beets / mines, satisfactory properties. C. Afterbirth period
The BP of 120/80 mm.Hg on both humeral D. Preliminary period
arterias. Mammas are mild, painless, papillas are E. Postnatal period
whole. The uterus is dense, painless, a bottom of it 781.A 20 years old 36 weeks of gestation pregnant
is defined at symphisis level. A lochia of serous- woman was admitted to the obstetrical hospital
mucous character, scanty. . Physiological with complains of the pain in the lower abdomen
KROK 2 – Question Bank 81
and bloody vaginal discharge. The general 785.The 27 years old PG (prima gravida) woman
condition of the patient is good. Her blood delivered the full term newborn with big weight.
pressure is 120/80 mm Hg. The heart rate of the The placenta was separated spontaneously. Within
fetus is 140 beats per minute and rhythmic. By 20 minutes after labor the patient lost 300 ml of
vaginal examination the cervix of the uterus is blood clots from vagina. During the external
formed and closed. The discharge from vagina is massaging the dough-like hypotonic uterus and
bloody up to 200 ml per day. The head of the fetus additional 200 ml blood loss were noted. The
is located high above the minor pelvis entry. A correct subsequent treatment is:
soft formation was defined through the anterior A. *manual examination of the uterine cavity
fornix of the vagina. What is the probable B. curettage of the uterine cavity
diagnosis? C. cold on the lower portion of abdomen
A. *Placental presentation D. uterine extirpation
B. Premature placental separation E. supragroinal uterine extirpation
C. Uterine rupture 786.At term of a gestation of 40 weeks height of
D. Preterm labor standing of a uterine fundus is less then assumed
E. Incipient abortion for the given term. The woman has given birth to
782.The woman on the seventh day after delivery the child in weight of 2500 g, a length of a body
complicated by uterine bleeding and manual 53 cm, with an assessment on a scale of Apgar of
investigation of the uterine cavity started to be 4-6 points. Labor were fast. The cause of such
feverish up to 38,70C and complains of the state of the child were:
bloody purulent vaginal discharge with A. * Chronic fetoplacental insufficiency
unpleasant smell. The uterine upper border is B. Delay of an intra-uterine fetation
located between the pubis and omphalus. The C. Placental detachment
uterus is tenderness and has a dough-like D. Infection of a fetus
consistency. What is the most probable diagnosis? E. Prematurity
A. *Puerperal (postpartum) endometritis 787.A 26 year old woman complains of a mild bloody
B. Uterine postpartum subinvolution discharge from the vagina and pain in the lower
C. Pelvioperitonitis abdomen. She has had the last menstruation 3,5
D. Sepsis months ago. The pulse rate is 80 per min. The
E. Physiological puerperal (afterbirth) period blood pressure (BP) is 110/60 mm Hg and body
783.18-years-old PG (prima gravida) woman is in fully temperature is 36,6°C. The abdomen is tender in
term labor lasting 12 hours. The woman fills her the lower parts. The uterus is enlarged up to 12
self tired and dozes between the contractions. The weeks of gestation. What is your diagnosis?
general conditions of woman and fetus are good. A. *Inevitable abortion
The obstetric sizes of pelvis are 25-28-31-20 cm. B. Incipient abortion
The labor activity is mild to moderated. On C. Incomplete abortion
vaginal examination the uterine cervix is smooth D. Complete abortion
and up to 4 cm dilated. Amniotic sac is intact. E. Disfunctional bleeding
Fetal head is presented against the small pelvis 788.Primapara, 24 years with a normal pelvis. After 8
entry in sacrum right posterior position (SRP). hours of normal patrimonial activity spasms
What is the correct management of labor? became weak, ineffective. Vaginal research:
A. * Obstetrics sleep-rest disclosing of a uterine cervix is 8 sm, the head of a
B. Amniotomy fetus is with its the big segment in an inlet into a
C. Oxytocin test small pelvis. The amniotic membrane is absent.
D. Labor stimulation Palpitation of a fetus a clear, rhythmical 130 heart-
E. Cesarean section. rate. Tactics?
784.After the labor the vaginal examination did not A. *Stimulation of patrimonial activity
reveled the uterine cervix damage. In the perineal B. Obstetric forceps
region a deep skin laceration up to the anus and C. Medicament dream - rest
muscular breakage of vagina posterior wall D. Caesarian section
including external sphincter were noted. What E. Spasmolyticses
kind of complications was happened? 789.A 2 years-old child suffering from food allergy
A. * II degree perineal rupture became acutely ill. The spasmodic cough,
B. I degree perineal rupture prolonged expiration and expiratory wheezing are
C. III degree perineal rupture presented. The respiratory rate is 60 per min. The
D. Groin rupture of the III degree mild respiratory distress mast be noted because of
E. - chest additional muscles participation in breathing.
KROK 2 – Question Bank 82
The wheezing chest is hyperinflated. Choose the leukocytosis 13,5(10 9/l, ESR - 32 mm/h.
treatment which must be done to the patient by Establish the preliminary diagnosis.
intravenous infusion. A. *Postnatal acute endometritis
A. *Euphyllinum, predisolon B. A postnatal thrombophlebitis
B. Penicillin C. A postnatal adnexitis
C. Berodual D. A postnatal parametritis
D. Lasolvan E. A postnatal pelviperitonitis
E. Calcium gluconate 794.Menstrual function of the 42 years-old woman as
790.At multipara, in a duration of gestation of 40 hyperpolymenorrhea and progressiv
weeks, at survey a spherical, balloting formation is algodysmenorrhea withing last 10 years. Vaginal
defined to the left of an umbilicus, the palpitation dascharge like chokolade, uterine size is enlarged,
of a fetus is auscultated closer to an umbilicus, tuberous, little morbid, uterine adnexa are not
about one 140/minutes. It is possible to think of palpable, fornix are not painful, deep. What is the
what position of a fetus in this case? most probably diagnosis?
A. *Transversal position of a fetus, left position A. *Endometriosis of uterus
B. Transversal position of a fetus, right position B. Cancer of uterus
C. Longitudinal position of a fetus, a head C. Submucous fibromyoma of uterus
presentation D. Endometritis
D. Pelvic presentation E. Endometriosis of adnexa uterus
E. Slanting position of a fetus 795.In gynecological department a woman
791.At a vaginal examination at primapara: disclosing complaining about acute pains beneath the
of a cervix of a uterus up to 8 sm, vertex abdomen, increased body temperature up to 38
presentation, a sagittal suture is in the transversal deg., complaining about purulent discharges from
size of the inlet plane, is posed closer to a the vagina, arrived. Sexual life is discursive.
promontorium, the big and small fontanels are at During the bimanual investigation: adnexae uteri
the same level. What insertion of the head of a are painful, purulent leucorrhea. For making a
fetus takes place in this case? diagnosis it’s necessary:
A. *Anterior asynclitism A. *Bacteriological investigation of discharges
B. Average transversal standing of head from the genitals
C. High direct standing of head B. Colposcopy
D. Low transversal standing of head C. Probing the uterus
E. Posterior asynclitism D. Curettage of the uterine cavity
792.The data of a functional condition of the child E. Cytological investigation of the vaginal
after delivery: palpitation is 134 heart-rate; leucorrhea
respiration is independent, but without the first 796.A woman of 27 years old leading an active sexual
cry; a skin of the face and a trunk is pink, life, complaining about numerous vesicules on the
extremities - dark blue; movements are active, cry right major lip, scratching and irritation. Rash
is loud. A condition of a newborn according to a appears periodically before mensis and finished in
scale of Apgar? 8 – 10 day. Possible diagnosis.
A. *8 points A. *Virus of Herpes Simplex
B. 5 points B. Bartolinitis
C. 6 points C. Primary syphilys
D. 10 points D. Genital condilomatosis
E. 9 points E. Cytomegalovirus infection
793.Puerpera, the 3-d day of the postnatal period, 797.A 60 years old and 10 years menopausal woman
complaints to a fever up to 38,2°С, a pain in the complains of the abdomen enlargment, weight
lower part of abdomen, purulent discharge from lost, asthenia, bloody discharge from vagina. The
vagina, delicacy have appeared. Objectively: pulse uterus is enlarged like up to 16 weeks of gestation
- 98 heart-rate, the BP – 120/80 mm.Hg, a skin is and hard. The salpinxes are not defined. What
pale. The abdomen is soft, painless. The uterine method of diagnostics is it necessary to carry out?
fundus is lower than umbilicus up to one A. *Consequent diagnostic curettage of the
transversal finger, has a soft consistence, morbid. uterine cavity and cervical canal
At vaginal research: the uterus enlarged till 17 B. Ultra sound investigation
weeks of pregnancy, softened, painful. C. Laporoscopia
Appendages of a uterus and vaults of the vagina D. Hysterosalpingographia
are without features. Discharge are purulent, in E. Sexual hormone investigation
moderate quantity. The analysis of a blood: a
KROK 2 – Question Bank 83
798.During the colposcopia endometriosis of the C. Ultra sound investigation of the abdominal
vaginal part of the cervix uteri was found out in a cavity
patient M. Choose the method of treatment. D. Roentgenoscopy.
A. *Laser vaporisation E. General blood analysis.
B. Dyathermocogulation 803.In a woman of 28 years old with complaints about
C. Еxterpation of uterus infertility during 3 years. In the laporoscopy
D. Supervaginal amputation internal endometriosis was found. In anamnesis is
E. Antiinflamation treatment chronical salpingoophoritis. The most probable
799.A woman of 35 years old complaints of pathogenital cause of infertility in this woman is:
amenorrhea during a year after a hard fright. A. *Local secretion of prostoglandines
Diagnosis. B. Disturbance of synthesis of cortisolum in
A. *Psychogene amenorrhea adrenal glands
B. Hypomenstrual syndrome C. Thickening of the albumine coverage of
C. Hyperpolymenorrhea ophorons
D. Proyomenorrhea D. Congenital hypoplasia of genitals
E. Algodysmenorrhea E. Increasing of viscousity of cervical mucus
800.11 years -old girl arrived to the hospital with 804.A woman of 18 years old complaints on the
uterus bleeding after suppression of mensis. absence of pregnancy during regular genital life
Anaemia is absent. Genital formula is Ax2 P3 within 1 year. Contraception is not being used.
Mg2 Me+. Virgin. Investigation per rectum – no Pregnancy never took place. During the bimanual
pathology. The treatment of this patient should be investigation no pathology is found out. With
start from what method should the investigation of this
A. *Infusion therapy with using of uterotonics couple be started?
B. Abrasio cavi uteri A. *Spermogram
C. Hormonal homeostasis B. Hysterosalpingography
D. Antyanaemical therapy C. Laporoscopy
E. Intramuscular injections of uterotonics D. Test of the functional diagnostics
801.A woman of 45 years old is working as a painter E. Bacteriological inoculation
and a builder. During bimanual investigation 805.A mother with a girl of 6 years old with pruritus in
elytroptosis of the second degree was find out. perineum area and outer genitals applied to the
What is the reason of the disease? doctor. Objectively: general condition is
A. *Hard physical work satisfactory. Multiple of scratches of vulva and
B. Delivery trauma perineum, around the anus, hyperemia, edema.
C. General disease of the vagina What investigation should be carried out?
D. Erosion of the cervix uteri A. *Analysis of feces for helmints
E. Disturbance of the bowels function B. Bacteriological inoculation
802.A 20 years old woman was admitted to the C. Stear for flora
reception room of the central district hospital. The D. General analysis of urine
patient complains of a lower abdomen cramping E. Glucose of the blood
pain ascending upward, dizziness and vomiting 806.In a girl of 3 weeks of age fountain vomiting not
within painful attack. She recalls the last menses connected with feeding is registered many times a
haven 6 weeks ago. On clinical examination a day. Periodical liquid defecations. Decrease of the
cheeks skin hyperpigmentation is noted. The body weight. During the examination the child is
mucous membranes are pale. Chest is clear. The rigid and has dehydratation. Hypertrophy of
heart beats are rhythmic. The pulse is soft and clytoris is obviously seen. What disease takes
weak 108 per minute. The body temperature is place?
36,3°C. The extremities are cold in touch. A. *Congenital adrenogenital syndrome
Abdomen is tense, considerably distended and B. Pylorosthenosis
painful. On percussion in the lower abdomen C. High bowel impaction
dullness is found. There is not any vaginal bloody D. Real hermophroditism
discharge. What method of clinical investigation E. Occute bowel infection
could help the surgeon to make a correct 807.A 13-years old girl complains of vaginal bleeding.
diagnosis? The last menses was 4 months ago. The patient
A. *Abdominal cavity punction through the complains of weakness and dizziness. Her blood
posterior fornix of vagina analysis shows the hemoglobin 64 g/l,
B. Laporoscopia erythrocytes count 1,8 x 1012/l, platelet count 280
KROK 2 – Question Bank 84
x 109/l. By vaginal examination the uterus is not D. Clinical and bacteriological analysis of a blood
enlarged, the salpinxes are not affected. E. Bacteriological research after provocation
A. *Juvenile uterine bleeding 812.In a gynecology department the woman with
B. Incipient abortion complaints on bloody allocation from a vagina has
C. Willibrandt’s disease addressed. Last menses was 3,5 months ago.
D. Thrombocytopenic purpura Ultrasonic did not pass. A uterus at a palpation of
E. Vulvovaginitis a paste consistence, painless, height of standing of
808.In a 32-year old woman during the bimanual a uterine fundus - 24 sm. Parts of a fetus are not
examination a tumor-like firm to hard consistency palpated, palpitation is not listened, the movement
swelling was revealed on the left of the uterus. It does not feel. From both sides of a uterus are
is displaceable and painless. Its diameter is about palpated the ovoid form of formation 5 – 6 cm.
10 cm. The uterus and adnexia are not changed. What is the most authentic diagnosis?
The menstrual function is not impaired. The A. *Hydatiform mole
patient looks healthy. What is the probable B. Threat of abortion
diagnosis? C. A placental presentation
A. *cyst of the left ovary D. A premature placental detachment
B. Cystoma of the left ovary E. Antenatal destruction of a fetus
C. Dermoid cyst of the left ovary 813.In a gynecology department there is a 47 years old
D. Ovarian cancer woman with dysfunctional uterine bleeding after a
E. Extrauterine pregnancy delay of a menses of about 10 days. With what it
809.A 42-year old woman consulted by a doctor is necessary to start the treatment?
complais of the gnawing-like pain in the lower A. *A fractional medical-diagnostic curettage of a
abdomen, weakness, long abundant menses. The mucous of uterus
general condition is good. By bimanual B. Hormonal hemostasis
examination the enlarged uterus sized about 10 C. Nonhormonal hemostasis.
weeks of gestation being hard, painless and having D. Cyclic hormonal therapy
irregular contours was revealed. What is the E. Ablations of the uterus
probable diagnosis? 814.On the 3rd day after the artificial abortion the
A. *Uterine myoma patient felt a dull pain in all abdomen, fever up to
B. Dysfunctional uterine bleeding 39,9°C, nausea and one time vomiting . The
C. Ovarian cystoma pulse is 120 beats per minute and weak. A marked
D. Uterine body cancer abdomen distention and painful voiding are
E. Uterine pregnancy defined. Morton, Schyotkin – Blumberg’s
810.A patient complaints of the pains beneath the symptoms are not clear positive. After 24 hours of
abdomen, radiating to the vagina. strengthening observation the patient’s condition become better
during menses, sexual intercourse. Anamnesis: but lower abdomen distention and tenderness are
two years ago there was a suspect for still presented. The pulse and body temperature
endometriosis. Vaginal investigation: to the back are normal. On vaginal examination there is any
of uterus a painful nodular conformation is found effusion defined in the space between uterus and
out. Diagnosis. rectum. The uterine cervix upward shift provokes
A. *Retrocervical endometriosis severe pain. Salpinx is enlarged and also painful.
B. Adenomyosis An abdominal cavity puncture through the
C. Chronically inflammation of the adnexae uteri posterior vaginal fornix did not reveal a blood.
D. Parametritis What is the diagnosis of this woman?
E. Endometriosis of the cervix uteri A. *Pelvioperitonits
811.A 27 years old woman being treated concerning a B. Diffuse peritonitis
chronic adnexitis for 5 years, is delivered in a C. Extrauterine pregnancy
gynecology department with attributes of D. Acute appendicitis
pelviperitonitis. Her husband is suffering from E. Ovarian apoplexy
chronic urethritis. What researches should be 815.The patient of 24 years was admitted into a
appointed for precising of treatment tactics? gynecology department due to a disharmony of the
A. *Bacteriological research of a vaginal content sizes of a uterus to a duration of gestation. Last
and determination of sensitivity to antibiotics menses were 4 months ago. At vaginal
B. Bacterioscopic research of a vaginal content examination: the uterus is enlarged according to
C. A puncture of an abdominal cavity through a 10 weeks of pregnancy. What diagnosis the most
back vault of the vagina, bacteriological probable?
sowing of an obtained punctate on gonococcus A. *Missed abortion
KROK 2 – Question Bank 85
B. Beginning abortion were not used. Her husband’s analysis of semen is
C. Abortion at a course without pathology. From what diagnostic method
D. Incomplete abortion will you start the workup in this case of sterility?
E. Menacing abortion A. *Hysterosalpingography
816.At vaginal examination: the uterus is enlarged by B. Hormone investigation
a chorionepithelioma up 12-13 week pregnancy, C. Ultra sound investigation
the patient complains of a strong uterine bleeding; D. Diagnostic scraping out of the uterine cavity
a haemoglobin is 52 g/L. What treatment should E. Hysteroscopia
be appointed to this patient? 821.A patient complaints on dark spready discharges
A. *Hysterectomy with appendages and the from genitals before and after the mensis. During
subsequent chemotherapy the colposcopy cianosis inclusions are visualised
B. Ablation of a uterus on the cervix uteri. Diagnosis.
C. A chemotherapy A. *Endometriosis of uterus wall
D. Anemia treatment B. Adenomiosis
E. A hormonetherapy C. Erosion of the uterus wall
817.47 years- old patient is complainting to bloodish D. Endocervicitis
discharge from genitals within 3 weeks after E. Polyp of the cervical canal
suppression of menses during 2 months. 822.The 31 year old female patient complains of the
According bimanual vaginal investigation sise of lower abdomen pain, fever up to 38,8°C. The
uterus is nornal, it is’nt painful, fornix are deep, patient is ill within 4 days. She has had her last
palpatia is painless. Uterine adnexa are not menstrual period one week ago. Besides it is
palpable.What is the most probably diagnosis? known that she has a history of chronic salpingitis
A. *Dysfunctional uterine bleeding (pelvic inflammatory disease – PID). The patient`s
B. Adenomyosis skin is pale, the tongue is dry, the blood pressure
C. Uterine body cancer is 120/80 mm Hg, the pulse rate is 100. The
D. Vaginal tumour Schyotkin’s sign is (+)-ve. By bimanual
E. Submucosal fibromyoma of uterus examination the 10 cm sized painfully sharp
818.Patient has complaints about pain in her abdomen, swelling has been noted in the right side from
metheorism, nausea, and vomiting, nervous uterus. The WBC (leukocyte count) is 16*109/l.
irritation. Symptoms are registered on the 3rd day What is the previous diagnosis?
after abolishing of clomiphen-cytratis. Objective A. *pyosalpinx
state – bracing of musk’s of the abdominal wall, B. Acute salpingo-oophoritis
adnexa uterus are enlarged, painful. The level of C. Parametritis
AP is low, tachycardia. What is the pathogenesis D. Appendicitis
of such complication? E. Endometritis
A. *Hyperstimulaton of ovaries 823.A 26 years old woman is delivered in a
B. Inflamation of ovaries gynecology department with complaints on a
C. Yellow body persistency sharp pain in right ileac range. Last menses 16
D. Ovulation days ago, in time. At survey in specula: a vagina
E. Bleeding to the intestinal cavity and uterine cervix are without changes. At a
819.12 years -old girl is complaining to the strong vaginal examination the body of the uterus and
pains in the right iliounguinal area of abdomen appendages are not accessible to a palpation
after physical tension, vomiting, body temperature because of sharp morbidity and a strain of muscles
is 38 C. Palpation of abdomen - symptom of of a frontal abdominal wall. The back vault is
Schotkin is positive in the right iliounguinal area. morbid. What it is necessary to make for
What is a main type of investigation in this case? specification of the diagnosis?
A. *Ultrasound investigation A. *Puncture of an abdominal cavity through a
B. Clinical blood examination posrerior vault of the vagina
C. General analysis of urea B. Colposcopy
D. Rectal examination C. Laparotomy
E. X- ray investigation of the organs of D. Define horionic gonadotropinum level
abdominal cavity E. Hysteroscopy
820.In the gynecologic office a 28-years old woman 824.A 18 years old woman complains of acute pain in
complains of sterility within three years. The the external genitals particularly in walking and
menstrual function is not impaired. There were labia majora swelling. Body temperature is 37°C,
one artificial abortion and chronic salpingo- pulse rate is 98 beats per minute. The examination
oophoritis in her case history. Oral contraceptives revealed the thickness of the right labia majora
KROK 2 – Question Bank 86
and severely painful swelling 5,0x4,5 cm in it The 828.18-year old woman complains of the pain in the
local skin and mucous membranes are hyperemic. lower abdomen. Some minutes before suddenly
There is an abundant smelly discharge. What is she was appeared unconsciousness at home. The
the most probable diagnosis? patient has not had a menses within last 3 months.
A. *acute bartholinitis The main features of this case are pale skin, the
B. furuncle of the major genital lip pulse rate 110 beats per minute, Hb (hemoglobin)
C. acute vulvovaginitis 76 g/l, BP 80/60 mm Hg. The Schyotkin’s sign is
D. cyst of the Bartholin’s gland (+)-ve. The vaginal examination revealed
E. acute vulvitis following features. The uterus is a little bit
825.A woman of 28 years old complains about enlarged. Its displacement is painful. There is also
periodical aching pains beneath the abdomen, any lateral swelling with indistinct size. The
subfebral temperature. Night sweating, posterior fornix of the vagina is tenderness and
olygomenorrhea, algodysmenorrhea, infertility overhangs inside. What is the most probable
during 7 years. Woman lost 7 kg of weight. diagnosis?
Bimanual investigation: uterus is not enlarged, A. *impaired extrauterine pregnancy
mobile, painless, adnexae uteri have numerous B. ovarian apoplexy
bands, painless. What primary investigation C. twist of cystoma of right uterine adnexa
should be carried out for making a diagnosis? D. acute salpingoophoritis
A. *Inoculation of menstrual blood three times E. acute appendicitis
during one menstruation 829.The 36 weeks of gestation pregnant woman was
B. Inoculation of the urine admitted to the obstetric in-patient department.
C. Hysterosalpinogography She has previous history of arterial hypertension,
D. Analyses of discharges now complains of a headache, aching pains in the
E. X-ray investigation of lunges lower abdomen and bloody discharge from vagina.
826.Secundapara of 25 years. In the third period of The main clinical features are blood pressure
delivery the bleeding has appeared. The attributes 180/100 mm Hg and hypertonic uterus. During
of placenta’ detachment are absent. At manual investigation about 300 ml of dark blood was
detachment of a placenta it was revealed that a discharged from vagina. The fetal heartbeats are
placenta is fixed, with growing into a not heard. What is the diagnosis?
myometrium. Tactics of the doctor? A. *premature placental separation
A. Laparotomy, a hysterectomy B. Placental presentation
B. Application of uterotonic agents C. premature delivery threat
C. A hemotransfusion D. uterine rupture
D. Laparotomy, supravaginal amputation of E. Embolism caused by amniotic fluid
uterus 830.The woman delivered twins has early postnatal
E. Tool secretion of an afterbirth hypotonic uterine bleeding reached 1.5\% of her
827.A 37-year old woman was admitted to the bodyweight. The bleeding is going on.
gynecological department by the ambulance. She Conservative methods to arrest the bleeding have
complains of acute pain in the lower part of been found ineffective. The conditions of patient
abdomen, weakness, nausea, vomiting. Menstrual are pale skin, acrocyanosis, oliguria. The woman
and genital functions are not impaired. Ultra is confused. The pulse rate is 130 beats per min,
sound investigation a week ago revealed the cyst BP – 75/50 mm Hg. What is the further treatment?
of the right ovary. The patient`s conditions are the A. *Uterine extirpation
body temperature 36,6 °C, the pulse 88 beats per B. Supravaginal uterine amputation
minute, the blood pressure 90/60 mmHg. The C. Uterine vessels ligation
acute tenderness in the lower abdomen and D. Inner glomal artery ligation
positive Schyotkin-Blumberd’s symptom are E. To put clamps on the uterine cervix
noted. Uterus and left adnexia are normal. A 831.Primapara with a preeclampsia of a serious degree.
displaceable firm to hard consistency swelling was The sizes of a pelvis: 23-25-29-19 cm. The
defined on the right side from the uterus. It is position of a fetus is longitudinal, vertex
severely painful on palpation. What is the presentation. Palpitation of a fetus is not
diagnosis? auscultated. An attack of an eclampsia. Vaginal
A. *Twist of ovarian cyst pedicle examination: disclosure of uterine cervix is
B. Acute salpigooophoritis complete, the head is in a narrow part of a pelvic
C. Ovarian apoplexy cavity. Tactics of delivery’s conducting?
D. Cyst rupture A. *Embryotomy
E. Pyosalpinx B. Applying of obstetric forcepses
KROK 2 – Question Bank 87
C. Vacuum extraction of a fetus. abortions and caesarian section concerning a
D. Caesarian sections. placental presentation in anamnesis. Suddenly the
E. Skin-head forcepses by Ivanov parturient woman complaints of strong abdominal
832.Primapara 26 years, in-time labor, began 8 hours pains, weakness have appeared. The BP - 80\50
ago. Amniotic fluid has flown away 3 hours ago. mm Hg. From a vagina - moderate bloody
Spasms for 30-40 sec., in 5-6 minutes, regular, discharge. Palpitation of a fetus is not auscultated.
painful. The position of a fetus is longitudinal, Parts of a fetus are palpated to the left of an
head presentation; fetus’ head is pressed to an middle line of abdomen. Patrimonial activity is
inlet into a small pelvis. Palpitation of a fetus absent. The preliminary diagnosis?
precise, 136 hits in one minutes. At a vaginal A. *Rupture of uterus
examination: disclosure of uterine cervix is 8 sm, B. Premature detachment of normally posed
a forehead, superciliary arches of a fetus are placenta
palpated, a frontal suture is in the right slanting C. Placental presentation, bleeding
size. The amniotic membrane is not present. D. Uterine’ cervix rupture, ІІІ degree
What from the listed below is the most rational? E. Embolism with amniotic fluid
A. *Caesarian section 836.A pregnant woman having 20 weeks gestation
B. Embryotomy operation term began to complain of weakness, fatigue,
C. Cavitary obstetric forcepses dizziness. The hemoglobin is 80 g/l, pulse rate 86
D. Uterotonics beats per minute, blood pressure 120/80 mm Hg.
E. Dream - rest, spasmolyticses The skin and mucous membranes are pale. What is
833.The 20 years old patient shows complaints on a the diagnosis?
delay of menses to 10 days. Infringement of A. *Anemia of pregnant woman
menstrual function marks for the first time. Sexual B. Neurocirculatory dystonia
life is regular, is not preserved from pregnancy. At C. Endocrinopathy
survey: a state is satisfactory, an abdomen is D. Chronic arterial hypertension
painless, the AP of 120/80 mm.Hg. The pulse is E. Preeclampsia
72 per minute, of satisfactory properties. At 837.Patient has complaints about non-regular manses,
transvaginal echograpphy: progressing tubal obesity, girsutism, she suffers from sterility.
pregnancy is suspected. Correct tactics of the According bimanual investigation – size of the
doctor? uterine body is smaller than normal one. The left
A. *Urgent hospitalization and right ovaries are mobile, painful. Sizes of
B. General laboratory examination in polyclinic ovaries are 4 x 5 x 4 sm. What can be a pathology
C. Ultrasound scanning in one week in this case?
D. Examination according to the tests of function A. *Disease of polycystose ovaries
diagnostics B. Chronical adnexitis
E. Puncture of an abdominal cavity through a C. Tuberculosis of adnexa uterus
posterior vault of the vagina D. Cysts of ovaries
834.In a gynecology department there is a 34 years old E. Endomerriosis of ovaries
patient with complaints of acute pains in the 838.A 26 years old woman, delivery has taken place
bottom of abdomen on the right which have begun 12 months ago, complicated by hemorrhage and
suddenly, nausea, vomiting. Bimanual research: DIC-syndrome in an early puerperal period. A
the uterus of the normal sizes, its shifting is patient complaints on a headache, giddiness,
morbid, the right ovary a little bit enlarged, sleepiness, abasement of hair on a head and pubis.
spherical, morbid. Vaults of the vagina are The BP is 90/50 mm. Hg. Menstrual function after
protruding; the palpation on the right is morbid. In delivery is not iterated. The uterus is diminished,
specula: the uterine neck and mucous of vagina ovaries are not palpated and dryness of mucous of
are not changed. Discharges are not present. What vagina is marked. The basal temperature is
method of research is most informative? monophasic, lower than 37(C, signs of pupil and
A. *Laparoscopy fern are negative. What is the pathogenesis of
B. A puncture of an abdominal cavity through a disease?
back vault of the vagina A. * A depression of function of a pituitary gland
C. Ultrasonic investigation of organs of a small B. Excessive production of prolactinum
pelvis C. Depression of function of ovaries
D. A colposcopy D. Infringement of function of paranephroses
E. Bimanual research E. Hyperproduction of androgens
835.Multipara, 35 years, 1 period of the second labor. 839.The 46 years-old woman has complaints about
Spasms are of average force. Three medical prolonged menstruation, polymenorrhea within 2
KROK 2 – Question Bank 88
years. She has hypertonic diseases, obesity of 2nd B. Draw up a statement about forensic medical
degree. In the hystological investigaion in the examination
scrap of endomethrium – numerous big glands C. Conduct an inquiry by preliminary investigator
with cists.What is the link of the pathogenesis of of police department
such state? D. Utilisation copy of medical documents
A. *Aromatisation of androstendiol in estrone E. Conduct forensic medical examination by
B. Hyperprolactinaemia district forensic medicine expert
C. High secretion level of adrenocorticopripical 844.Patient M., 37 years old. He came to the clinic
hormone wounded after an hour.On the abdominal skin
D. Low level of glucocorticoids there is pain around umbilicus due to prick and cut
E. Hyperandrogenia wound of about 0,5 x 1 sm with slight bleeding.
840.28 years-old patient has complaints about absense What help would you provide to the patient?
of menorrhea, growth of hair as of men’s type. A. *Laparotomy, suevey organs of abdominal
Married. Menarche since 14 years- age – cavity. Primary surgical processing of the
hypomenorrthea and finished after 2 years. Height wound.
is 160 cm, weight is 65 kg. Female constitution. B. Drainage of the wound with rubber strip.
Colour and humidity of skin are normal. External C. Survey the wound with canal probe.
genitals of the female are normal. Uterine size is D. Aseptic bandage.
normal to. Uterine adnexa are enlarged according E. Suture the wound
palpation. What is the most probably diagnosis? 845.The patient K. is 23 years old. He has wounded
A. *Syndrome of Shtein-Levental left arm due to the gunfire. The bones of the arm
B. Disease of Itsenko-Kushing are undamaged. What appropriate surgical help
C. Adreno-genital syndrome must be provided to such a patient?
D. Adrenosteroma A. *Primary surgical Processing with a flowing
E. Corticoadrenosteroma suction.
841.30 years old patient has complaints about B. Suture wound.
amenorrhea during 2 years after delivery. Labor C. Suture wound and dry it.
was complicated with massive bleeding. After D. Dry wound with towel gauzes.
delivery woman lost a body weight, has alopecia. E. Aseptic dressing of the wound.
According of bimanual investigation – the size of 846.If a child has attached fingers in his right hand,
uterine body is small, hypoplasia of minor and then what will be your diagnosis?
major lips. What is the etiological factor of this A. *Synductylia
changes B. Polyductilia
A. *Necrosis of adenohypophis after C. Macroductilia
haemorragical shock D. Ectroductylia
B. Stimulation of ovulation E. Ectromelia
C. Momental turning of the ovarial function 847.Patient K., 34 years old. A dog bit him 3 hours
D. Involutia of hypothalamus before. In the left arm there is wound by the tooth
E. Pathological clymacs of dog`s bitc without bleeding . What surgical help
842.The adolescent of 15 years old was brought to the would you provide to such a patient?
hospital with complaints on poor night vision. A. *Wash wound with detergent water and apply
Objectively: increased darkness adaptation time, anti-septic.
Bitot’s spots on conjuctiva. The patient skin is B. Aseptic bandage.
dry, scales off, folliculitis signs of the face skin C. Cream bandage.
are present. What is a cause of thedisease? D. Complete suture of the wound.
A. *retinole deficit E. Incomplete suture of the wound.
B. Thiamine deficit 848.Patient K., 37 years old complains of pain in the
C. Biotin deficit right arm, which increases during motion, increase
D. Folic acid deficit body temperature upto 39° C. In the right cubital
E. Napthtochynones deficit fossa there is a frace of injection, hyperemia and
843.What guarantees from the preconceived attitude to thickening along the vein. Your diagnosis?
the physician in cases of professional law A. *Phlebitis
violations do you know? B. Phlegmon
A. *Sanction of public prosecutor, conduct an C. Abscess
inquiry by preliminary investigator of D. Inflammation of lymph
prosecutor’s office, committee of experts E. Arizipeloid
KROK 2 – Question Bank 89
849.Purulent medisatinitis is diagnosed on a 63 year 855.What from the listed operations in treatment of
old patient. What of the below listed diseases are ulcer of stomach and duodenum is carried out only
not the cause of purulent mediasdtinitis? under emergency indications?
A. * Cervical lymphadinitis. A. * Sewing up perforative ulcer
B. Deep nech phlegmon. B. Resection of stomach by Bilrot I
C. Perforation of the cervical part of the C. Resection of stomach by Bilrot II
easophagus. D. Selective proximal vagotomy
D. Perforation of the thoracic the easophagus. E. Truncal vagotomy with pyloroplastic
E. Iatrogenic injury of the trachea. 856.With the restrained umbilateral hernia which has
850.The diagnosis – melanoma was made to a 16 year become complicated by phlegmon hernial, it is
old patient after examination with complaints of necessary to make for patient:
frequent pain in the abdomen, pigmentation of the A. * Herniotomy by Meio-Sapegko
mucosa and skin, polyp in the stomach and large B. Herniotomy by Meio
intestine was found. It is know that the mother of C. Herniotomy by Sapegko
the patient analogous pigmentation and was D. Herniotomy by Lekser
treated often for anemia What disease is E. Herniotomy by Grenov
suspected? 857.What treatment is shown for the patient of 63
A. * Peytz – Egers’s polyposis. years of age with an attack of bilious colic, caused
B. Chron’s disease. by gall-bladder stones?
C. Tuberculosis of the intestine. A. * Conservative treatment
D. Adolescent polyposis. B. Emergency operation
E. Hirschprung’s disease. C. Urgent operation after cupping an attack
851.What developes in cases with decompensated D. Antifermental therapy
pyloric stenosis: E. Laparoscopic cholecystotomy
A. * Isotonic dehydration. 858.Choose correct tactics at the patient of 53 years of
B. Hypertonic dehydration [eksikosis]. age at the initial stage of obturative intestinal
C. Hypotonic dehydration. obstruction
D. Intoxication. A. * Operative treatment at an inefficiency of
E. Renal insufficiency. conservative actions
852.A 35 year old woman was admitted to thoracic B. Only conservative treatment
surgery department with elevation of body C. Emergency operation
temperature upto 40 0 C, onset of pain with deep D. Scheduled operation
breath in the side, cough with big quantity of E. Nasogastral intubation
purulent sputum and blood with bad smell. What 859.52-year-old man have recurrent transient ischemic
disease causes these symptoms? attacks. Auscultation of the carotid arteries
A. * Abcsess of the lungs detected murmur. What diagnostic method is
B. Complication of liver echinococcosis necessary to apply the first?
C. Bronchectatic disease A. * Ultrasound dopplerography
D. Actinomycosis of lungs B. CT of the brain
E. Tuberculosis of lungs C. MRI of the brain.
853.Which of the listed below opertion are not done in D. Cerebral angiography
cases of perforative duodenal ulcers? E. Electroencephalography
A. *Gastrostomy 860.On dispensary supervision at theraputist are the
B. Resection of 2/3 - 3/4 of the stomach patients with diabetes mellitus, hypertonic disease,
C. Vagotomy + Pyloroantrumectomy obesity. The cancer with what localizations in this
D. Vagotomy + resection of the ulcer group is necessary to expect with a high degree of
E. Suturing of the ulcer risk?
854.At the patient of 34 years of age you suspect an A. *Uterus body
abscess of Douglas spaces. What method of B. Gulet
research is preferable to diagnostics: C. Liver
A. * Digital research of rectum D. External sexual organs
B. Rectoromanoscopy E. Uterus neck
C. Laparoscopy 861.27 years old woman complains on photophobia,
D. Percusion and auscultation of a stomach watering and dimness of vision on right eye.
E. R-scopy of abdominal cavity During examination of this eye ciliary congestion
and branch-like infiltration of the cornea, which
KROK 2 – Question Bank 90
stains with fluorescein, were observed. What eye- A. * Poisonind of CO
drops are contra-indicated? B. Burns
A. * Dexamethazon 0,1\% C. Fractures
B. Tropicamid 1\% D. C.F.T. [cranio-facial trauma]
C. Laevomicetin 0,25\% E. Crash - sundrome
D. Glucosa 40 \% 868.The most available and informative diagnostic
E. Acyclovir 3\% methods for closed trauma of the urinary bladder:
862.Within three weeks a patient had violation of nasal A. *Retrograde cystography
respiration, mucopurulent discharges from a nose, B. Pelvic artheriography
headache. At anterior rhinoscopy in middle nasal C. Cystography
meathus the stria of pus, edema, hyperemia of the D. Sonography of the urinary bladder
mucosa of the nose have been determined. What E. Palpation and percussion of abdomen
diagnostic method is necessary for assigning first 869.In order to differentiate diagnosis of anuria and
of all? ishuria you should accomplish:
A. *X-ray of paranasal sinuses A. *Catheterisation of the urinary bladder
B. Computer tomography of a skull B. Analysis of urine
C. Bacteriology analysis of the nasal mucous C. Analysis of the blood
D. Punction of the maxillar sinus D. Excretory urography
E. General blood analises E. None of the above
863.The patient with an acute middle purulent otitis 870.All undermentioned is correct with respect to the
complicated by mastoiditis has been admitted to tumors of renal pelvis and ureter besides:
hospital. On roentgenogram of mastoid processes A. *Postoperative radiation and chemotherapy
the shading of the cellular system on the lesion, improve survival
absence of bone septs have been marked. What are B. Hematuria and colic take place
medical actions in the second stage of mastoiditis? C. Tumor is often multiple one
A. *Mastoidotomy D. Men are ailing more frequently
B. Paracentesis of the drum E. Treatment – nephrurenerectomy [ with
C. Radical operation on the middle ear resection of the urinary bladder]
D. Tympanoplasty 871.Female-patient Z.,of 27 years old, was admitted to
E. Cateterization of the Eustachian tube the clinic with complaints of pain in right ankle
864.A woman, 35 years old, admitted to reanimation joint, non-bearable extremity. On the clinical
department in asthmatic state. What is most examination: the patient had a fracture of anterior
trustworthy criterion of breath effectively? part of distal metaepiphisis of tibia [Pott-Desto`s
A. * РаСО2 and РаО2 fracture]. What is the mechanism of such an
B. Respiratory volume injury?
C. Respiratory volume A. * Supination of the foot.
D. Minute respiratory volume B. Load along the axis with planter flexion of
E. Determination of “dead” space foot.
865.Increasing of central venous pressure is marked on C. Direct blow.
the base of decreasing arterial pressure during the D. Pronation of the foot.
dynamic investigation of it at victim with plural E. Load along the axis with dorsal flexion of foot.
combined injuries. What is this combination 872.Patient K.,of 22 years old, was admitted to trauma
evidence of? center with complaints of pain in left ankle joint,
A. * Addition of cardiac insufficiency which increased while movements and weight
B. Blood deposition in venous channel bearing. On the clinical examination it was found,
C. Shunting that the patient had the closed fracture of medial
D. Hypervolemia malleolus without displacement. In which position
E. Growth of bleeding speed the foot has to be fixed in plaster cast?
866.What kind of injures is offen happened while A. * At right angle with varus positioning of the
carthquakes? foot.
A. * Crash syndrome. B. In position of planter flexion of foot.
B. C.F.T. [cranio-facial trauma]. C. In position of pronation
C. Fracture of upper extremities. D. In position of supination.
D. Fracture of lower extremities. E. In position of dorsal flexion of foot.
E. Abdominal traumas 873.Female-patient K, of 45 years old, was admitted to
867.What kind of injures is often happened while the traumatological ward with the closed fracture
explosions at mines [pits]? of the medial malleolus with its displacement up
KROK 2 – Question Bank 91
to 3 mm. In which position the foot has to be fixed B. Operation on thyroid gland
with a plaster cast? C. Krail’s operation
A. * At right angle with varus positioning of the D. Subtotal resection of thyroid gland and
foot. fascicular resection of limphatic nodes
B. In position of planter flexion of foot. E. Vanach’s operation
C. In position of pronation. 880.To the doctor 28 years old a woman has addressed
D. In position of supination. with the complaints to increase of the size of
E. In position of dorsal flexion of foot. pigment nevus which was available from birth, get
874.What wall of the inguinal canal is strengthened in wetting, itch. What method of research should not
case of indirect inguinal hernia? be applied to diagnostics in this case?
A. *Anterior A. * Incision biopsy
B. Posterior B. Yaks’s reaction
C. Inferior C. Radioisotope diagnostics
D. Superior D. Termography
E. Posterior-inferior E. Glass-print
875.Triangl Callot is orientation for exposure of: 881.A boy 10 years old complains on pain of left eye
A. *Cystic artery and strong photophobia after trauma his left eye
B. Cystic duct by a pencil at the school. Left eye examination :
C. Common bill duct blepharospasm, ciliary and conjunctival
D. Common hepatic duct congestion, cornea is transparent, other parts of
E. Hepatic artery eyeball – without changes. Visus 0,9. Right eye is
876.A 19-year-old girl was admitted to Emergency health, Visus 1,0. What additional method do you
Department: unconsionsness state, cyanosis, choose first of all?
myotic pupil, shallow breathing – 12/min. BP A. *Staining test with 1\% fluorescein
90/60 mm Hg, 78 pulses/min. Choose the action B. X-rey examination of orbit
corresponding to this clinical situation: C. Tonometria
A. * Controlled respiration D. Gonioscopia
B. Gastric lavage E. Cornea sensation-test
C. Oxygen inhalation 882.What does it mean “the Monteggia fracture -
D. Caffeine injection dislocation?”
E. Cordiamine injection A. * Combination of fracture of the upper half of
877.A 47 year-old man with gastric cancer, the ulna with dislocation of the head of the
complicated by profuse hemorrhage was radius
successfully treated by surgical intervention. After B. Dislocation of the head of the radius
the operation the native blood plasma transfusion C. Dislocation of both forearm bones
was performed. Whereupon developed: D. Combination of fracture of the radius with
acrocyanosis, hypersalivation, hyperpnea, BP dislocation of the head of the ulna
80/40 mm Hg, 120 pulses/min. The initial drug E. Fracture of both forearm bones
therapy had to begin with injection of: 883.What is the external rotation of the lower limb
A. * Epinephrine common for?
B. Prednisolon A. * Fracture of the femoral neck
C. Diphenylhidramin B. Fracture of the pubic bone
D. Dopamine C. Fracture of the ischial bone
E. Aminophilline D. Fracture of the tibia
878.For the persons who are taking place in a zone of E. Fracture of the femoral shaft
failure on nuclear object, the greatest risk of 884.What choice for the treatment of the femoral neck
development within the first decade is represented fracture is incorrect?
by cancer: A. * Skeletal extension
A. * Thyroid gland B. Internal fixation by the three - flange nail
B. Skin. C. Internal fixation by parallel screws
C. Reproduction system organs D. Fixation by the dynamic hip screw
D. Breast E. Prosthetic replacment of the femoral head
E. Lungs 885.What most common complication like avascular
879.40 year old a patient is diagnosed: 1. Medular necrosis of the femoral head occurs after?
thyroid gland cancer. 2. Feochromatcitoma. What A. * Subcapital fracture of the femoral neck
operation should be made at first? B. Transcervical fracture of the femoral neck
A. * Operation concerning feochromatcitoma C. Fracture of the base femoral neck
KROK 2 – Question Bank 92
D. Intertrocharteric fracture E. Thermography
E. Pertrochanteric fracture 891.The method of treatment of the sthenotic cancer of
886.What is the typical displacement in Colles’s the rectum with local spread is…
fracture? A. * Colostomy
A. * The distal fragment of the radius is tilted B. Chemotherapy
backwards C. Phytotherapy
B. The distal fragment of the raduis is tilted D. Hormonal therapy
anteriorly E. Phytotherapy + chemotherapy
C. The distal fragment of the raduis is deviated 892.The method of treating the unresectable sthenotic
externally cancer of the pyloric area of the stomach is…
D. The distal fragment of the raduis is deviated A. * Sidetracking anastomoses
internally B. Chemotherapy
E. The distal fragment of the raduis is rotated C. Radiotherapy
outside D. Phytotherapy
887.A growth of the right side of the scrotum was E. Phytotherapy + chemotherapy
found at the examination of a 3-month-old child. 893.In a patient with a stab injury to the femoral
This formation has elastic consistency, its size artery, the treatment of choice is:
decreases in sleep and increases when the child is A. * End to end anastomosis
crying. What investigation will be helpful for B. Interposition of autogenous vein graft
make a correct diagnosis? C. Ligation of both transected ends
A. * palpation of the thickened cord, crossing the D. Interposition of homologous arterial graft
pubical tubercule [ sign of the silk glove]; E. Interposition of Dacron graft
B. diaphanoscоpy 894.A 28-year-old woman comes to the Emergency
C. palpation of the external inguinal ring Room with a slightly reddened, painful “knot”, 8
D. punction of the scrotum cm above the medial malleolus. Examination in
E. examination of the formation in the standing position demonstrates a distended
Trendelenburg’s position vein above and below the mass. There are no other
888.A rounded shadow with well defined outlines was abnormalities on physical examination. The most
found at the costo-vertebral angle on the chest X- likely diagnosis is:
ray roentgenogram of the otherwise healthy 9- A. * Superficial venous thrombosis
year-old girl . Make a preliminary diagnosis. B. Early deep vein thrombosis
A. * Ganglionevroma C. Insect bite
B. Sympatoblastoma D. Cellulitis
C. Ganglioneuroblastoma. E. Subcutaneous hematoma
D. Sympatogonioma. 895.The majority of arterial emboli originate from
E. Sarcoma of the vertebra. which one of the following sites?
889.A baby was born at 36 weeks of gestation. A. * Left atrium
Delivery was normal, by natural way. The baby B. Left ventricle
has a large cefalohematoma. The results of blood C. Cardiac valves
count are: Hb 120g\l, Er 3,5T\l, total serum D. Thoracic aorta
bilirubin 123mmol\l, direct -11mmol\l, indirect - E. Abdominal aorta
112mmol\l. What are causes of hyperbilirubinema 896.A patient with a history of coronary artery disease
in this case? and atrial fibrillation develops sudden pain and
A. *erythrocytes hemolisis weakness of the left leg. Examination reveals a
B. intravascular hemolisis cool, pale extremity with absent pulses below the
C. disturbance of the conjugative function of the groin and normal contralateral leg. The most likely
hepar diagnosis is:
D. condensing of bile A. * Arterial embolism
E. mechanical obstruction of the bile fall. B. Arterial thrombosis
890.A patient, male, 50 years old, has a black flat mole C. Acute thrombophlebitis
on the skin of the leg for 10 years. Since 4 months D. Cerebrovascular accident
the shapes of the mole become irregular. What E. Dissecting aortic aneurysm
diagnostic methods should be used? 897.Which of the following is the most common
A. *Excision biopsy clinical manifestation of an abdominal aortic
B. Fine needle biopsy aneurysm?
C. Smear for microscopic examination A. * Most are asymptomatic
D. Incision biopsy B. Peripheral embolization
KROK 2 – Question Bank 93
C. Acute rupture D. Іnfectious mononucleosis
D. Back pain E. Pharyngeal candidosis
E. Chronic peripheral ischemia 904.A 30 year old woman complains of nasal
898.Which of the following best summarizes blockage, sneezing, watery nasal discharge. The
indications for operation on an abdominal aortic body temperature is normal. Objectively – edema
aneurysm? of nasal mucous membrane of both sides,
A. * Any aneurysm greater than 5 cm in diameter especially on lower turbinate (with cyanosis).
B. Any abdominal aortic aneurysm Blood tests dates – mild leukocytosis and
C. Only symptomatic aneurysm eosinophilia. What is the diagnosis?
D. Only symptomatic aneurysm greater than 5 cm A. * Allergic rhinitis
in diameter B. Foreign body of nose
E. Only ruptured aneurysm C. Infective rhinitis
899.Which one of these antibacterial drugs shows an D. Atrophic rhinitis
ototoxic action? E. Acute sinusitis
A. *Gentamicine 905.What is the most frequent cause of acute urinary
B. Amoxycilline retention in men over 60 years old?
C. Cefalexine A. *Benign prostatic hyperplasia
D. Оfloxacine B. Prostate cancer
E. Ciprofloxacine C. Urinary bladder cancer
900.Which one of those symptoms is the most typical D. Urethral stricture
for the adenoid hypertrophy? E. Urethral stones
A. * Nasal obstruction 906.The method of treating a small cell of anaplastic
B. Swallowing disturbance lung cancer T2NxM1, with bone methastases is…
C. Olfaction disturbance A. *Chemotherapy
D. Giddiness B. Pneumonectomy
E. Headache C. Lobectomy
901.What must be done first in case of animate foreign D. Radiotherapy
body presence in the external auditory meatus E. Immunotherapy
(insect)? 907.The male patient, 60-year-old, tobacco smoker for
A. * Oil or alcohol instillation in the ear 30 years, alcoholic, has dysphagia and weight loss
B. Syringing with warm water since 4 months. Suggested diagnoses?
C. Removing by use of the crocodile forceps A. *Cancer of the esophagus.
D. Removing by use of a small hook B. Esophageal achalasia.
E. Analgetic drugs designation C. Hanter`s disease
902.A 30-year-old man complains of a sharp pain in D. Esophagitis
the right ear, hearing loss, high temperature during E. Esophageal diverticulum
three days. Objectively – right ear whispering 908.A 50-year-old male patient was brought to
language – 0,5 m, external ear is intact, Emergency Department with diffuse abdominal
otoscopically - eardrum protrusion, hyperemia and pain and signs of cardiovascular collapse. Upon
swelling, loss of landmarks. What disease is it? arrival he had BP 95/60 mm Hg, 120 pulses/min,
A. * Acute purulent otitis media diuresis 20 ml/h, HgB 100 g/l, red blood cells
B. Acute mastoiditis count 2.1•1012/l. The patient needs infusion of:
C. Chronic secretory otitis media A. * Crystalloid and colloid
D. Chronic purulent otitis media B. Crystalloid and red blood cells
E. Eustachian tube disfunction C. Crystalloid and 5\% dextrose
903.A 22 year old man complains of acute throat pain, D. 5\% dextrose and red blood cells
increasing upon swallowing during 3 days. Body E. 5\% dextrose and colloid
temperature 38,3(С, neck lymph nodules are 909.A 33-year-old male patient was brought to
slightly enlarged and painful. Pharyngoscopically Emergency Department with the signs of
– tonsilar hyperemia, enlargement and edema, cardiovascular collapse: BP 60/30 mm Hg, 140
tonsils are covered by round yellow fibrinous pulses/min, the skin is pale and moist, diuresis 20
patches around crypts openings. Beta-haemolytic ml/h, HgB 80 g/l, red blood cells count 2.5•1012/l.
streptococcus in swab analysis. What is the The reduction of blood volume averages:
diagnosis? A. *30-40\%
A. * Acute membranous tonsilitis B. 10-15\%
B. Acute follicular tonsilitis C. 15-20\%
C. Pharyngeal diphtheria D. 20-25\%
KROK 2 – Question Bank 94
E. 25-30\% leukocytosis, normal serum levels of amylase,
910.A 45-year-old male patient was admitted to the lipase, and bilirubin. The most likely diagnosis is:
intensive care unit because of myocardial A. * Acute cholecystitis
infarction. An hour later the ventricular facilitation B. Perforated peptic ulcer disease
occured. What from following should be C. Myocardial infarction
administered: D. Sigmoid diverticulus
A. * Defibrillation E. Acute pancreatitis
B. External chest compression 917.During an operation for presumed appendicitis,
C. Lidocaine injection the appendix is found to be normal; however, the
D. Adrenalin injection terminal ileum is markedly thickened and feels
E. Cardiac pacing rubbery to firm, its serosa is covered with a gray-
911.What is the most common histological type of white exudate, and several loops of apparently
malignant tumors of the prostate? normal small intestine are adherent to it. The most
A. *Adenocarcinoma likely diagnosis is:
B. Transitional cell carcinoma A. * Crohn’s disease of the terminal ileum
C. Seminoma B. Perforated Meckel’s diverticulum
D. Rhabdomyosarcoma C. Ulcerative colitis
E. Squamous cell carcinima D. Ileocecal tuberculosis
912.What is normal daily diuresis in adult? E. Acute ileitis
A. *1500-2000 ml 918.Which of the following are absolute indications
B. 200-500 ml for common bile duct exploration without
C. 500-1000 ml preliminary cholangiography?
D. 2000-2500 ml A. * Toxic cholangitis
E. 2500-3000 ml B. Pancreatitis
913.What is the treatment of choice for the patient C. Bilirubinate stones in the gallbladder
with 1,5 cm uncomplicated, asymptomatic, D. Multiple small stones in the gallbladder
calcium oxalate renal stone? E. Dilated common bile duct
A. *Extra-corporeal shock wave lithotripsy 919.In patients who have sustained blunt trauma to the
B. Percutaneous nephrostolithotomy chest, which of the following would most likely be
C. Open pyelolithotomy the cause of acute cardiopulmonary collapse?
D. Descending litholisis A. * Pneumothorax
E. Watchful waiting B. Hemothorax
914.What is the treatment choice for 60-year old male C. Pulmonary contusion
with renal cell carcinoma T2N0M0 and normal D. Rib fractures
contralateral kidney? E. Acute adult respiratory distress syndrome
A. *Radical nephrectomy (ARDS)
B. Resection of the kidney 920.A 50-year-old man comes to the emergency room
C. Nephroureterectomy with a history of vomiting of 3 days’ duration. His
D. Simple nephrectomy past history reveals that for approximately 20
E. Radiation therapy years, he has been getting epigasric pain, lasting
915.Following bilateral femoral embolectomy, a for 2 to 3 weeks, during early spring and autumn.
patient with atrial fibrillation becomes oliguric He remembers getting relief from pain by taking
and passes red-colored urine. All the following milk and antacids. Physical examination showed a
measures may be appropriate in this patient’s fullness in the epigastric area with visible
management EXCEPT peristalsis, absence of tenderness, and normal
A. * Cessation of heparin active bowel sounds. The most likely diagnosis is:
B. Administration of sodium bicarbonate A. * Gastric outlet obstruction
C. Administration of mannitol B. Small bowel obstruction
D. Determination of serum potassium C. Volvulus of the colon
E. Aggressive hydration D. Incarcerated umbilical hernia
916.A 45-year-old woman, mother of four children, E. Cholecystitis
comes to the emergency room complaining of the 921.A 50-year-old woman presents with a 2-year
sudden onset of the epigastric and right upper history of mild, diffuse, tender thyroid
quadrant pain, radiating to the back, associated enlargement with a 10-pound weight gain and
with vomiting. On examination, tenderness is fatigue. Of the following which is the most likely
elicited in the right upper quadrant, bowel sounds diagnosis?
are decreased, and laboratory data shows A. * Hashimoto’s thyroiditis
KROK 2 – Question Bank 95
B. Riedel’s thyroiditis mm.Hg, pulse 100 beats per minute. Which one
C. Subacute thyroiditis from below listed methods allows to define
D. Suppurative thyroiditis character of wound with the greatest accuracy?
E. Papillary thyroid carcinoma A. *Primary surgical cleaning and exploration of
922.Which of the following is the most appropriate in Wound.
the initial management of a patient with confirmed B. Chest X-ray
esophageal corrosive injury? C. Ultrasound of thoracic Cavity
A. * Steroids for 3 weeks D. CT scan.
B. Prompt institution of serial esophageal E. Bronchoscopy
dilatation 927.The patient K. 42 years old, presented with the
C. Parenteral hyperalimentation and prohibition diagnosis of “Acute iliofemoral vein thrombosis
of oral intake for 2 to 3 weeks (1st. Day), Pulmonary artery thromboembolism”
D. Esophagectomy with gastrostomy, cervical and admitted in vascular department of the
esophagostomy, and delayed reconstruction Hospital. What is your tactics?
E. Esophagectomy with immediate reconstruction A. * Thrombolytic therapy plus implantation of
923.A 24-year-old law student is brought into the Cava filter.
emergency room complaining of severe abdominal B. Surgical treatment.
pain of 6-8 hours duration. He had been to a party C. Introduction of low-molecular wt. Heparin.
the night before. The pain is in the epigastrium D. Elastic Bandage of legs.
radiating to the back and is associated with E. Bronchoscopy.
nausea. The patient vomited twice prior to coming 928.Patient K. 35 years old, after abortion developed
to the emergency room. Clinical examination deep veins thrombosis of leg and on the 3-day
reveals an anxious, acutely ill youth with a regular cough and a retrosternal pain developed with
pulse rate of 100/min, blood pressure og 100/68 hemoptysis. Which investigation is necessary at
mm Hg, and a temperature of 38,1 (C. The most first to make the correct diagnosis?
likely diagnosis is: A. * Electrocardiogram, Chest X-Ray.
A. * Acute pancreatitis B. Phlebography, Dopplerography.
B. Acute cholecystitis C. Ultrasonogram of Abdomen.
C. Acute appendicitis D. Palpation of stomach.
D. Acute diverticulitis E. Auscultation of Lungs.
E. Mesenteric adenitis 929.73 years old patient hospitalized with the
924.In which of the following disorders does the diagnosis of “Tumour of Abdominal Cavity”. On
pathophysiology of portal hypertension involve examination: On the right side of the abdomen a
presinusoidal intrahepatic obstruction? mass of 10х15 cm size is palpated. Patient is
A. * Congenital hepatic fibrosis suffering of ischemic heart diseases, Hypertension
B. Alcoholic cirrhosis ІІ-ІІІ stages. It is suspected an aneurysm of an
C. Hemochromatosis abdominal aorta. For the verification of the
D. Budd-Chiari syndrome diagnosis it is necessary to execute:
E. Cavernomatous transformation of the portal A. * Aortoarteriography.
vein B. X-ray of Abdomen.
925.A nonalcoholic 45-year-old woman presents with C. Diagnostic puncture.
an asymptomatic, septated 10-cm cystic mass in D. Laparosynthesis.
the body of the pancreas. She should be advised E. Irrigoscopy.
that: 930.25 years old patient presented to emergency
A. * Complete excision of the lesion is indicated department after 40 minutes of stab injury of chest
B. Drainage by Roux-en-Y jejunostomy is in a projection of heart in a critical condition.
indicated Confused, cold sweating, Blood Pressure 60/20
C. The lesion is probably benign and requires mm.Hg, Pulse on peripheral arteries was absent.
observation only What is the most probable diagnosis?
D. It is cancerous and she will likely die from it A. *Cardiac Injury.
in 12 months B. Lung Injury.
E. Total pancreatectomy is the preferred surgical C. Pneumohemothorax.
management D. Bleeding from soft tissues of chest wall.
926.40 year old patient presented to Emergency E. Injury of intercostals vessels.
department with the cut injury on the right side of 931.53 years old patient, complains of a heartburn,
the chest wall. Profuse bleeding from the wound regurgitation of air, vomiting. In
but the patient is in conscious, B.P – 120/60 esophagodudenoscopy: - Marked prolapse of
KROK 2 – Question Bank 96
squamous mucous of stomach into the esophagus. Fatigue, Dizziness. Cyanosis. Unstable
In radiogram marked protrusion of 1/3 stomach Hemodynamic. On Chest X-ray-Fracture of Right
into the posterior mediastinum. Provisional four Posterior-Lateral ribs, Collapse of right lung
Diagnosis – A sliding hiatal hernia, ІІІ degree. 2\3 Volume. What is the possible diagnosis?
What is the tactics of treatment? A. * Right sided Post-traumatic Pneumothorax.
A. *Plastic of a diaphragm according to Belsey. B. Right sided post-traumatic excudative
B. Lewis's operation [Transhiatal resection of pleuritis.
esophagus]. C. Abruption of Main Bronchus.
C. Vandal’s operation [plastic of lower third of D. Right-sided Hemothorax.
esophagus.] E. Hematoma Mediastinium.
D. Heller’s Operation. 936.Patient 19 years old, Presented to Emergency
E. Esophagoectomy, Abdomino-cervical Method. department in critical condition after Trauma of
932.Patient В, 64 years, complains of difficulty in Chest with chief complain of Left sided chest
swallowing solid food, vomiting, weakness, loss pain, Dyspnea, Fatigue, left sided massive
of weight. In esophagodudenoscopy on a subcutaneous emphysema of chest wall. On chest
posterior-lateral wall, sub mucous layer tumour X-ray Atelectasis of left lung, Shift of mediastinal
with the precise contours, easily movable is organs to left. Cardiac cavity not enlarged. Your
determined. The diagnosis: Benign tumour of Diagnosis?
lower third esophagus [Leiomyoma]. Your A. *Abruption of Left main bronchus.
tactics? B. Left sided total Hemothorax.
A. *Endoscopic removal of a tumour. C. Fracture of left Ribs, left sided Pneumo-
B. Heller’s Operation. hemothorax.
C. Esophagoecotmy, Abdomino- cervical D. Left sided Post-traumatic pnemothorax.
Method. E. Left sided post-traumatic pleuritis.
D. Operation Vandal’s [plastic of the Lower third 937.36 years old patient presented with complains of
of esophagus]. dyspnea, dizziness. History of Thoracic trauma 2
E. Lewis's operation [Transhiatal resection of days back. On examination decrease movement of
esophagus]. the left side of the chest wall. On chest X-ray –
933.Patient [Female] 48 years old, chief complain of Collapse of the 1/3 of left lung. Fracture of left 4-6
dysphasia for solid and liquid food, nausea, and ribs. What are the possible complication patients
fatigue. In radiographic examination of has developed?
esophagus- stricture of lower third esophagus and A. *Pneumothorax.
dilatation of upper third esophagus. Positive B. Posttraumatic Hemothorax.
Symptom [ ] Diagnosis-Cardiospasm C. Empyema of pleura.
III stage. What is the volume of necessary D. Pleuritis.
treatment needed? E. Posttraumatic pneumonia.
A. * Heller’s Operation. 938.Patient К. 19 years old admitted with the diagnosis
B. Conservative treatment: Cerucal, Rantac, No- “Chest Wall Trauma (Thoracic Trauma)” with
spa, Intravenous infusion. Complain of difficulty in expiration and
C. Vandal’s Operation [Plastic surgery of lower inspiration. On examination patient is pale. Blood
third esophagus]. Pressure 90/50 mm.Hg. On auscultation: Silent on
D. Esophagoectomy, Abdomino-cervical Method. left side (no breathe Sound). On chest X-ray:-
E. Lewis Operation [Transhiatal resection of Shift of mediastinal organs to right, atelectasis of
esophagus with gastroplasty]. left lung, your diagnosis?
934.Patient К, 65 years old, inpatient of surgical A. *Left sided Tension Pneumothorax.
department of hospital after hernioplasty on the 6 B. Fracture of Ribs.
day suddenly lost consciousness; there was C. Injuries of a chest wall.
cyanosis of the upper part of a thorax and the face D. Cardiac Injury.
and dyspnea. What is the diagnosis? E. Hemothorax.
A. * Pulmonary artery thromboembolism. 939.45 years old patient admitted in a clinic in a
B. Heart attack of a myocardium. critical condition. Before admission patient was
C. Hypoglycemic Coma. suffering from pneumonia for 3 weeks. On
D. Hyperglycemic Coma. examination: - Skin and mucous membrane dark -
E. Perforation of stomach Ulcer. earthy color, a body temperature 38c, Dyspnea on
935.Patient А., 44 Years old, presented to Emergency rest, decrease breathe on the left side. Productive
Department after 3 hours of trauma with chief Cough with large amount of sputum. On chest X-
complain of Right sided Chest pain, Dyspnea, ray. What is the most probable diagnosis?
KROK 2 – Question Bank 97
A. * Empyema of pleura. E. Isotope Phlebography.
B. Bronchitis. 944.A 35 years old patient complains of a difficult
C. Pleuritis. swallowing, pain behind the breastbone. He can
D. Pneumonia. eat only liquid food. While swallowing sometimes
E. Pneumothorax. he has attacks of cough and dyspnea. Above
940.32 years old patient presented in a hospital in a mentioned complaints is progressing. It is known
critical condition with chief complain of acute that the patient has had a chemical burn of
retrosternal chest pain with radiation to back. On esophagus one month ago. What complication
examination:- skin and mucous are pale, t-38,8 does the patient have?
С. Marked subcutaneous emphysema of soft A. *Corrosive Esophagitis and strictura
tissues of a neck, puffiness of face. On the eve ate B. Esophagitis
fish. On Chest X-ray expansion of mediastinum is C. Esophageal diverticula
revealed. What is the most probable diagnosis? D. Cardia Achalasia
A. * Mediastinitis. E. Cardia insufficiency
B. Heart attack. 945.A 42 years old man with long history of disease
C. Abscess of lung. complains of a frequent heartburns, moderate pain
D. Pneumothorax. in epigastrium and behind breastbone propagated
E. Pneumonia. in the back in point between shoulder blades. Pain
941.48 years old patient, suffering from postphlebetic appears with meals or just after meals and can be
syndrome of the left leg since 2 years. On provoked by physical exertion. Also he has had a
examination: Dilated superficial veins of left leg relapsed bronchopneumonia earlier and events of
and thigh, and pubic region, a significant swelling melena. The CBC reveals anemia. On X-Ray film
of the left leg. Light physical exertion aggravates there is a bubble of gas in the posterior
pain. What kind of treatment should be mediastinum. ECG documents an arrhythmia.
recommended to patient? What is your diagnosis?
A. *Reconstructive operation on deep veins of the A. * Hiatus hernia of esophagus
left thigh. B. Chronic pancreatitis
B. Conservative therapy. C. Ischemic heart disease
C. Compression treatment. D. Gastric ulcer
D. Phleboectomy. E. Mediastenitis
E. Phlebo-sclero-obliteration. 946.A 70 years old woman had had a planned
942.52 years old patient admitted in vascular laparoscopic cholecystectomy done according
department of the hospital with sever edema and biliary calculi. Six months later the patient again
pain of holding apart character in the right leg has attacks of severe pains in the right
and thigh, aggravated by passive movements. On hypochondrium accompanied by jaundice and
examination: On the right leg sever edema starting dark urine and stool discoloration. The total serum
from the foot till inguinal ligaments are observed bilirubin is increased up to 60 mcmol/l, direct 40
cyanotic skin. What is the most probable mcmol/l. What disease does the patient have?
diagnosis? A. *residual choledocholithiasis
A. *Acute iliofemoral vein thrombosis. B. Papillostenosis
B. Erysipelas of the right leg. C. tumour of the pancreas head
C. Acute thrombophlebitis of superficial veins. D. tumour of the large duodenal papilla
D. Lymphostatsis. E. choledochus stricture
E. Phlegmon of the right Leg. 947.A 60 years old woman has been ill with chronic
943.67 years old patient Hyperstenic features, calculous cholecystitis for 10 years. During the
suffering from varicose veins of both legs since 18 treatment in sanatorium the patient had had a
years. During last 2 years three times had hepatic colic with jaundice. Ultra sound
thrombophlebitis of superficial veins of the right investigation revealed a lot of calculi sized 5-6
leg. 4 months back on the lower third of right leg mm in the gallbladder. Choledochus is widened to
trophic ulcer developed. What method of 15 mm and contains concrements up to 6 mm in
investigation is informative for specification of the diameter in the distal part. What method of
diagnosis of the patient? treatment is the most adequate and current?
A. *Ultrasonic duplex scanning. A. * endoscopic papillosphincterotomy,
B. Functional tests to determine the condition of laporoscopic cholecystectomy
Valve. B. cholecystectomy, choledocholithotomy,
C. Phlebography. external choledochus drainage according to
D. Dopplerography of deep veins. Kerr
KROK 2 – Question Bank 98
C. cholecystectomy, transduodenal B. Acute appendicitis
papillosphincterotomy C. Acute salpingoophoritis
D. cholecystectomy, choledochoduodenostomy D. Pyosalpinx
E. cholecystectomy, choledochojejunostomy E. Tubo-ovarian abscess
948.A 58 years old woman with overweight right 951.A pregnant woman with 24 weeks gestation term
before has had an attack of right hypochondrium has felt a cramping pain in low abdomen. Nausea
pain and jaundice with dark urine and stool and vomitting are absent. She looks for a medical
discoloration appeared. On clinical examination aid in the gynecologic out-patient office. On
the abdomen is distended and painful on palpation clinical examination the abdomen is soft and
in the right hypochondrium, The mild liver tender on the right. The Schyotkin – Blumberg,
enlargement there is. In blood the total bilirubin is Rovzing, Koap’s symptoms are slightly positive
90 mkmol/l, direct (conjugated) 60 mkmol/l. What and Brendo, Michelson’s signs are strongly
investigation is the most informative to clarify the positive. What is the most adequate tactics of the
diagnosis? doctor in the situation?
A. * retrograde cholangiopancreatography A. *To send the patient to the in-patient
B. intravenous cholegraphy department at once to solve the problem of
C. infusional cholegraphy urgent surgical operation
D. intracutaneous intrahepatic cholegraphy B. To observe the patient for the next 24 hours at
E. ultra sound investigation of the home to clarify the condition
hepatopancreatobiliary zone C. Medication therapy
949.A 62 years old woman complains of severe D. Emergent diagnostic abdominal cavity
constant pain in the right hypochondrium, puncture through the posterior vaginal fornix
jaundice, discoloration of stool and dark urine, in this female dispensary office
mild fever up to 37,5оС. Above mentioned E. Urgent interruption of pregnancy
complaints were appeared after an attack of severe 952.A 45 years old woman was operated because of
abdomen pain connected with fatty food intake. biliary calculi and obstructive jaundice. A two
On clinical examination the abdomen is soft. A months later after operation there is continuing
painful enlarged gall bladder is palpated. The bile discharge up to 500,0-600,0 ml per day
Orthner, Kerr’s symptoms are positive. What is through the Kerr`s external choledochus
the probable diagnosis? drainage. On fistulography using the drainage in
A. * Acute cholecystitis, choledochus calculi and the distal part of the choledochus “a forgotten
obstructive jaundice stount” up to 8 mm in diameter was identified.
B. Infectious hepatitis The choledochus is dilated up to 16 mm. The most
C. Liver cancer correct surgeon treatment in this case is:
D. Liver abscess A. * Endoscopic papillosphincterotomy and
E. Liver cirrhosis removing a concrement from choledochus;
950.A 22 years old woman was admitted to the B. Choledocholithotomy with close seam on
reception department. She complains of severe choledochus;
cramping lower abdomen pain occurred C. Choledocholethotomy
unexpectedly, general weakness, sleeplessness, choledochoduodenostomy;
appetite loss and fever up to 39,9° C. At first the D. Choledocholithotomy choledochojejunostomy;
pain was appeared in point between umbilical E. Choledocholithotomy and drainage of the
region and epigastrium and then it was localized choledochus.
in the in the right iliac region. The patient recall 953.A 19 years old man was admitted to the reception
the last menses 8 weeks ago. On clinical department in 20 minutes after a knife wound of
examination the abdomen is soft, painful in the the left chest. The patient is confused. The heart
right iliac. The Schyotkin – Blumberg’s symptom rate is 96 beats per minute and blood pressure
is slightly positive, Michelson’s symptom is clear 80/60 mm Hg, The dilated neck veins, sharply
positive. On bimanual gynecological examination diminished apical beat and evident heart
the soft uterus is enlarged according pregnancy enlargement there are. What penetrative chest
onset. Near the uterus there is a soft swelling wound complication is described?
identified as a separated ovary. In CBC the WBCs A. *Pericardium tamponade
(leucocytes) are 15x109 /l. Their formula shows B. Massive hemothorax
bandemia. There is high ESR up to 65 mm/h. C. Open pneumothorax
What is the most probable cause provoked above D. Closed pneumothorax
written condition? E. Valve-likes pneumothorax
A. * Acute appendicitis and ectopic pregnancy.
KROK 2 – Question Bank 99
954.54 years old patient, presented with dizziness, an months. Objectively: dry, muddy colored skin, On
episode of decreased brain blood circulation, palpation On the right iliac fossa – infiltration
complains of a pain over the umbilicus after meal (mass) 8х10 sm. Size. Which is almost not
,sometimes very sharp, is accompanied by displacing (Immovable), on percussion dull sound
vomiting, a episode of diarrhea. History of Blood above the mass. On auscultation peristalsis is
in stool sometimes. Cardiac activity arrhythmic, increased. Нв blood - 86 g/l. What is the most
extra systole. Moderate tenderness around probable pathology that might have causes such
umbilicus. What is the most probable diagnosis? clinical picture?
A. *Non- specific ulcerative colitis. A. *Carcinoma of Caecum.
B. Crohn’s Diseases. B. Cancer of the right kidney.
C. Acute intestinal ischemia. C. Appendicular Infiltrate.
D. Chronic cholecystitis. D. Crohn’s Diseases.
E. Duodenal Ulcer with penetration. E. Retroperitoneal Tumour.
955.45 years old man presented with chief complains 959.Patient K, 42 years old, is hospitalized in surgical
of rise in temperature up to 38°C, pain and department with complaints of acute sharp pain in
swelling in lumbar region and painful mass 5х6 the stomach, vomiting. Suffering from a duodenal
sm. in size, crimson color of skin over the mass, in ulcer for last 8 years. Suspected as a Duodenal
the center purulent - necrotic fistulas which is Perforation, however free fundus gas in abdominal
secreting pus. What is the most probable cavity is not revealed. The ulcer is suspected as
diagnosis? covered perforation. What method of diagnosis
A. *Carbuncle of lumbar region. should be applied for correct diagnosis?
B. Abscess of lumbar region. A. *Pneumogastrography.
C. Erysipelaous inflammation. B. Pneumoperitoneum.
D. Para nephritis. C. Laparosynthesis.
E. Renal Colic. D. Contrast (dye) investigation of stomach and
956.Patient К, 43 years old hospitalized in surgical duodenum.
department of the hospital with the diagnosis of E. Fibrogastroscopy.
Mechanical jaundice, cholangitis. During 960.Patient B. 74 years old is hospitalized in surgical
echographic researches found out Huge hydatid department with the diagnosis of perforated
cyst of liver (echinoccocus of liver), dilatation of stomach ulcer. In the anamnesis heart attack of a
CBD(Common Bile duct) and intrahepatic ducts. myocardium, diabetes, Hypertension. The patient
What is the mechanism of jaundice in was advised for Operation, which patient
echinoccocus of liver? categorically refused. How to treat the patient?
A. *Rupture of contents of cysts into hepatic A. *Taylor’s Method.
ducts. B. Infusion therapy.
B. Compression of portal vein with occurrence of C. Antibacterial therapy.
portal hypertension with jaundice. D. Start Ulcer Therapy
C. Occurrence of a viral hepatitis. E. Discharge the patient.
D. Intoxicytic hepatitis due to absorption of 961.A 32 years old patient presented with sudden rise
ecchinococcus fluid (Hydatid cyst fluid). in temperature, High grade fever, headache, pain
E. Suppuration of cyst with occurrence purulent in stomach and lumbar region, yellowish
cholangitis. discoloration of skin. Urine output of the patient
957.Patient K, 54 years old operated for hydatid cyst is 100 ml dark muddy colored. Later with these
of liver, during operation found two cysts instead symptoms Muscles pain is added. One week ago
of three, as it has been diagnosed in the the patient went for fishing. What is the probable
preoperative period. Which methods of diagnosis?
investigation will be accurate to locate the third A. * Leptospirosis.
cysts? B. Viral hepatitis A
A. *Intraoperative echography. C. Viral hepatitis E
B. Intraoperative Cholangiography. D. Acute pyelonephritis
C. Intraoperative Choledochoscopy. E. Food poisoning
D. Intraoperative Retrograde 962.28 years old patient presented with history of 14
Cholangiopancreatography. hours constant pain in right iliac fossa. In last 2
E. X-ray Abdomen and Pelvis. hours the pain has decreased. Objectively: Local
958.65 years old patient complains of a pain in the guarding of abdominal muscles. Diagnosed as
right iliac fossa, loss of weight, decrease appetite, acute appendicitis. What histological form of
weakness, and history of constipation more than 6
KROK 2 – Question Bank 100
acute appendicitis could result in reduction of in the abdomen, delay in passes of gases and stool.
intensity of a pain of a stomach? History of the patient - appendectomy.
A. * Gangrenous. Objectively: Position of the patient -lying, pale
B. Cataral. skin. Pulse 90/ minutes. Blood Pressure - 110/80
C. Phlegmonic. mm. Hg, t - 37,2°C, Moderately distended
D. Perforated abdomen, asymmetric, rigidity on the lower part
E. Empyema of the appendix of the abdomen. Increased peristalsis. Rebound
963.What preparations are used for prevention of tenderness- negative (Shetkina- Blumberg).
fungal infection? Manual per rectum analysis of rectum- empty
A. *Fluconozol, Orungol, Nisoral. ampoule. Your diagnosis?
B. Rubomycin, Bleomycin, Mytomycin C. A. * Acute intestinal obstruction.
C. Cytosar, Cormyctin, Lomycitin B. Food poisoning
D. Captopril, Enalapril. C. Hepatic Colic.
E. Isoniazid, Ftibazid, Pyrazinamid. D. Acute pancreatitis
964.Patient Н, 44 years old, is hospitalized in surgical E. Hepatic Colic.
department with the diagnosis – of 968.A 41 year old patient was admitted to the intensive
postcholecystectomic syndrome, residual care unit with hemorrhagic shock due to gastric
choledocholithiasis, cholangitis, and mechanical bleeding. He has a history of hepatitis B during the
jaundice. Operated 8 months back, done last 5 years. The source of bleeding are esophageal
cholecystectomy, Choledocholithotomy, drainage veins. What is the most effective method for
of abdomen according to Keru. What from of control of the bleeding?
below-mentioned procedure would be appropriate A. * Introduction of obturator nasogastric tube.
to avoid occurrence of postcholecystectomic B. Intravenous administration of pituitrin
syndrome? C. Hemostatic therapy
A. *Intraoperative cholagiogrpahy. D. Operation
B. X-ray of Abdomen. E. Administration of plasma
C. Intravenous Cholecystocholangiography. 969.What developes most often after accidental intake
D. Per oral Cholecystography. of Hydrochloric acid:
E. Echography. A. * Cardiac insufficiency.
965.30 years old woman, 15 days ago had mild trauma B. Cushing’s syndrome.
of 5th finger of the left hand. Treated herself at C. Kutling’s syndrome.
home independently, Due deterioration of a D. Deylads's syndrome.
condition she visited hospital for medical advice E. Acute pancreatitis.
with rise in temperature up to 36 0c. Objectively: 970.Patient С, On chest X-ray found collapse of the
Hypermia and swelling on the ventarl surface of right lung, dislocation of the mediastinum on the
finger. Restricted Movements of the finger. X-ray left. During puncture of the pleural cavity 2.5 L.
of the left hand: It is impossible to exclude an of air is allocated. What is your diagnosis?
early stage of development оsteomyolitis of the A. *Right sided Pneumothorax.
fifth finger. The diagnosis: Panarchy of 5th finger B. Left-sided Pneumothorax.
of the left hand. What form of Panarchy has C. Empyema Pleura.
occurred in the patient? D. Mediastinitis.
A. *Bony. E. Pneumomediastinium.
B. Hypodermic 971.Patient of 23 years old suffering from acute
C. Paronychia glomerulonephritis with nephrotic syndrome,
D. Tendon Type. Initial Phase with normal renal function. What is
E. Joints Type. the baseline treatment?
966.Contraindications for operation in acute A. *Antibiotics
pancreatitis are: B. Saluretics.
A. *Hemodynamic unstability and pancreatogenic C. Kurantil
shock D. Heparin
B. Functional insufficiency of the E. Prednisolone.
parenchymatous organs 972.65 years old patient had been on observation for 5
C. Purulent and septic complications years concerning an ulcer of antral part of a
D. Peritonitis stomach. Patient refused operation. Since last 6
E. Erosive bleeding months patient is having constant pain in the
967.The patient, 43 years old is hospitalized with epigastric region. Disgust to meat products has
complaints of repeated vomiting, spasmodic pain appeared. Working capacity has decreased. The
KROK 2 – Question Bank 101
patient has become thin. In contrast examination 975.The diagnosis of Right sided pnuemothorax is
of the stomach circular form of defect of a mucous made to a 36 year old patient. What method of
membrane up to 5 sm. in diameter and aperistaltic treatment is indicated to the patient?
zone is revealed. What is an effective method of A. *Surgical treatment: Drainage of the pleural
verification of the diagnosis cavity.
A. *Fibrogastroduedenoscopy with biopsy. B. Antiinflammation therapy.
B. Ultra sonogram. C. Symptomatic therapy.
C. Pneumoperitoneum. D. Pleural puncture
D. Roentgenoscopy of Stomach. E. Thoracotomy.
E. ERCP 976.A 33 years old patient was admitted to the
973.38 years old man suffering from duodenal ulcer reception room of the Central District Hospital. He
for long time, patient start feeling constant complains of a severely painful swelling localized
heaviness in a stomach after meal, regurgitation, on posterior neck, fever up to 38,4oC and general
vomiting food contains which he had in the weakness. It is known that the patient suffers from
evening of the previous day, weight loss. diabetes mellitus within 5 years. On physical
Objectively: Relatively satisfactory condition of examination on the posterior neck surface there is
the patient, appetite not changed, Turgor of skin is an infiltrate elevated above surrounded skin. The
reduced. On palpation the stomach is soft, tissues affected by swelling are tens and blue
symptoms of irritation of abdomen is not present, reddish discolored in central area. There are also
“noise of splash “in epigastria region. Urinations several purulent necrotic pustules which are
normal. Stool once in 3 days. What complication connected with each other and formed a large skin
has occurred in the patient? necrosis. A thinned necrotic skin of this swelling
A. * An ulcerative stenosis of pyloric canal. has a holes look like sieve and a pus is discharging
B. Acute pancreatitis. through out. What disease should a doctor
C. Achalasia, esophagitis. consider first of all?
D. Cancer of a stomach. A. *Carbuncle
E. The covered perforation of an ulcer. B. Furuncle
974.A 60 year old patient complains of the weakness, C. Acute skin cellulitis
loss of appetite, periodic fever up to 38-40°C, loss D. Carbuncle associated with anthrax
of body weight, cough with a purulent sputum in a E. Skin abscess
small amount on daytime and large up to 300- 977.Patient B, 63 years old is hospitalized in thoracic
400ml sputum discharge with stinking smell on surgery department with complaints of nausea,
morning. He is chronic patient suffering from vomiting after taking food, weakness, loss of
chronic lung emphysema within 10 years. At the weight. After radiological investigation the
past he had had an acute left sided pneumonia of diagnosis is as follows: - “Achalasia Cardia”.
the lower lobe 8-10 weeks ago. After that he What from below-mentioned is the reason of this
noticed a mild mainly on evening fever and night disease?
sweats. The above mentioned complaints was A. *Insufficient development of Auerbach’s
appeared 4 days ago. On physical examination the plexus.
patient looks toxic. There are severe underweight, B. Cicatricial stenosis of esophagus.
grey skin, unpleasant small from the mouth, finger C. Hiatal Hernia.
clubbing, asymmetric chest secondary to the air D. Varicose of Esophageal vein (Esophageal
entry limitation on the left. On auscultation the Varices).
breathing sounds are diminished in the lower chest E. Tumour of lower third of esophagus.
on the left and pleural rub phenomenon is 978.A 38 year old woman was hospitalized to the
defined here. Over other chest surface a moist surgical unit with acute abdominal pain irradiating
rales are heard. The chest X-Ray reveals a to the spine and vomiting. On laparocentesis
pneumosclerosis and lung cavity with liquid level hemmorhagic fluid is obtained. What disease is
and thick walls sized 10x7cm in diameter in the suspected?
upper lobe on the left. What is the diagnosis of the A. * Acute pancreatitis
patient? B. Renal colic
A. *Chronic lung abscess with in bronchus C. Acute enterocolitis
drainage D. Perforative gastric ulcer
B. Acute abscess of the left lung E. Acute appendicitis
C. Left sided destructive pneumonia 979.A woman born in 1952 consulting by a doctor in
D. Left sided chest TB the out-patient office complains of a reddish
E. Bronchiectasis bordered swelling in the low back skin appeared 3
KROK 2 – Question Bank 102
days after branch tree prick. The fever is mild up without considerable cause. O-shaped deformity
to 37,9°C. Other complains are the general of the legs in the knee joints was appeared. The
weakness, headache, malaise and appetite loss. On skull, pelvis and lower extremities X-Ray films
physical examination on the loin skin a swelling shows the thickening of flat bones. In the long
and hyperemia are revealed. On palpation there is bones there is a hyperostosis along the bone axis.
a positive fluctuation symptom. What is the most The blood tests does not reveal any inflammation
probable diagnosis? activity. Serum calcium is normal. What disease
A. * Acute abscess of the loin skin do you consider in this case?
B. Acute cellulitis of the loin skin A. *Paget’s disease
C. Hematoma B. hyperparathyoid dystrophy
D. Carbuncle C. chronic osteomyelitis
E. Furuncle D. myeloma
980.A 42 years old patient consults by a surgeon with E. mottled disease (marble disease)
complains of the painful, severely itching and 984.45 years old woman complaints of pain and
hyperemic thumb of the right hand. It is known movement restriction in the right hip joint. The
that the patient has pricked his finger with a fish disease is in progress. The history of trauma is
bone one week ago. On examination the affected negative. The X-Ray does not reveal malignancy
thumb is rosy red and painful on touch. There is a or inflammatory disease but only shows an angled
red bordered and elevated above the surrounding disproportions and ostephytes. What is the
skin spot. The chest and heart are symptomatic diagnosis?
free. The heart rate is 80 per min. Blood pressure A. * The deforming arthrosis of the right hip joint
is 130/90 mm Hg, Body temperature is 36,7°C. B. Non-specific arthritis
What’s the diagnosis? C. Specific arthritis
A. *erysipeloid D. Polyarthritis
B. Erysipelas E. Radiculitis
C. acute lymphangitis 985.The 45 years old man locksmith complains of
D. acute panaritium poor fourth and fifth fingers straitening in the right
E. Paronychia hand. He is ill whithin 6-7 years. Every year the
981.Patient Е, 51 year old is hospitalized in disease worsens. On examination the fourth and
gastroenterology department with complaints of the fifth fingers are flexed and cannot be even
jaundice, loss of weight, weakness, dark color passively extended. The X-Ray does not reveal
urine, and light colored stool. Diagnosis: any bone damage. What kind of contracture do
Mechanical jaundice, Cholangitis. Disease began you consider in this case?
gradually. Suspected as Cancer of ampullae’s of A. *Dupuytren’s contracture
vater. What diagnostic method should be applied B. Myogenic contracture
for confirmation of the diagnosis? C. neurogenic contracture
A. *Fibroduedenoscopy with biopsy of ampulla D. Ischemic contracture
of Vater. E. tendinous contracture
B. Echography. 986.The 35 years old patient has severely restricted
C. X-ray Abdomen. movement ability in the vertebral column. Within
D. Pneumogastrography. 3 years the patient has had a persistent pain and
E. Computer tomography. progressive stiffness in the low back later spread
982.A 15 years old teen complains of high fever up to out into the thorax and cervix. The patient did not
39,5 – 40°C and a local metaepiphesal localized look for medical help before. The history of back
in low one third of hip pain. There are local skin trauma or acute disease is negative. The laboratory
hyperemia, soft tissues swelling and knee tests are normal. What disease do you consider in
movements restriction secondary to the pain. The this case?
patient denies the trauma. Blood WBC A. *ankylosing spondylarthritis
(leucocytes) are 15x10E9. X-ray reveals hip bone B. osteochondrosis
destruction and sequestration. C. tuberculous spondylitis
A. *Haematogenic osteomyelitis D. polyarthritis
B. Bone TB (tuberculosis) E. radiculitis
C. Paget’s disease 987.The patient man-welder (profession related with
D. Osteosarcoma long standing on knee position) was consulted by
E. Myeloma a doctor because of development knee joint
983.The 67 years old patient within 5 years had had 5 swelling and knee pain at working time. On
recurrent fractures of the lower extremities examination there has been found a soft bordered
KROK 2 – Question Bank 103
swelling localized lowly from patella with normal 991.A patient complains of a general weakness, fever,
color and callous skin. There is not local muscle and joint pains and sore throat. The pain is
hyperthermia. The X-Ray does not reveal any increasing on swallowing. Throat examination
destructive impairment of the bones. What is the reveals pink mucous membranes of the pharynx.
treatment? The tonsils are congested and swelled. There is
A. *operative bursectomy membranous exudate in crypts. This membranes
B. ultrahighfrequency (UHF) aren’t spreading out of the tonsils border and can
C. tight bandage be removed easily. What is the previous
D. puncture diagnosis?
E. magnetotherapy A. * Membranous (lacunar) streptococcal
988.The sick woman complains of fever up to 38,2°C, tonsillitis
severe earache reflected into the left temple and B. Follicular streptococcal tonsillitis
persistent headache. Also there is hearing C. Acute viral pharyngitis
depletion. She fall in illness 3 days ago after D. Diphtheria
common cold. Otoscopy shows normal auricle and E. Hypertrophic pharyngitis
external auditory meatus without pathological 992.The patient factory worker has been brought in the
features. Palpation of trugus and papillae - like department emergency by ambulance. The
spout is painless. Tympanic membrane looks red admission diagnosis is the penetrating cornea
and bulged with indistinct landmarks. Whisper is injury of the right eye. On the slit lamp
perceived by the patient from 0,8m of distance and examination the low intraocular pressure, corneal
colloquial speech only from 3 m. What’s a swelling and adhesion of injured corneal margins
probable diagnosis? in paraoptical zone have been detected. The depth
A. *Acute otitis media of anterior chamber is 2,5 mm. What method of
B. Furuncle of the external auditory meatus the following investigations must be carried out
C. Acute mastoiditis first?
D. Acute external otitis A. *Roentgenography of the orbital cavity by f
E. Exacerbation of chronic otitis media Komberg – Baltin
989.A 38 years old woman complains of a purulent B. Roentgenography of the orbital cavity in two
discharge from the left nostril. The body projections
temperature is 37,50C. The patient is ill during a C. boneless roentgenography by A.Vogt
week and associates her illness with common D. eye electroplatismagraphy
cold. pain on The palpation of her left cheek E. eye ultrasonography
reveals tenderness.. The mucous membrane in the 993.The patient complains of eyelids redness and
left nasal cavity is red and turgescent. The swelling, troublesome itching of the eyelids
purulent exudates is seen in the middle meatus in margin and eyelashes loss. He is being consulted
maxillary. What is the most probable diagnosis? by an ophthalmologist in the local public health
A. *Acute purulent maxillary sinusitis center. The doctor prescribes various eye drops
B. Acute purulent frontitis preparations with relapsed effect. What kind of
C. Acute purulent ethmoiditis investigation should be carried out?
D. Acute purulent sphenoiditis A. *investigation for demodicidosis
E. Purulent rhinitis B. conjunctival sac bacteriological smear
990.34 years old patient, during tooth filling C. checking up the refraction
accidentally inhaled a dental pine. Referred to D. consulting by an allergologist
emergency department of Hospital. Complain of E. testing blood glucose
moderate dyspnea, dry cough, dizziness, and 994.Diarrhea is not typical but still often symptom of
disturbed. On Chest X-ray on the hilar region of acute appendicitis in children. In what case
right lung identified radio opaque subject. What diarrhea is exact sign of appendix inflammation:
volume of the help is necessary in this case? A. * in case of pelvic appendices location
A. * Urgent Fibrobronchoscopic removal of the B. in case of peritonitis
foreign body. C. in infants and early aged children
B. Urgent Diagnostic Fibrobronchoscopy. D. in case of retrocecal appendicitis
C. Urgent Rigid Bronchoscopic removal of the E. when acute appendicitis is secondary to acute
foreign body. enterocolitis
D. Thoractomy, Bronchotomy, removal of 995.The child with the symptoms of acute appendicitis
foreign body. has been brought to the in-patient department by
E. Antibacterial therapy, Cough expectorants, ambulance. Examination is impossible because of
Control Chest X-ray.
KROK 2 – Question Bank 104
his negative contact faulted behaviour. What are primary rheumo-carditis, polyarthritis; acute
you to do? course, cardio-vascular insufficiency IIA. Which
A. *to examine the child under general anesthesia of medicines should be prescribed?
B. to examine the child in spite of his temper A. *Prednisone
C. to have laparoscopy taken B. Cefazolin
D. to wait for child`s physiological sleeping C. Delagil
E. to admit the child to a hospital for D. Diprazinum
observation by children’s doctor and the E. Erythromycin
surgeon 1001.The 10 years old boy suffered from angina 2
996.On the second day after birth the newborn has weeks ago has complaints on joint pain and
multiple duodenal content vomiting. Meconium impossibility of movement in left knee and right
didn’t pass away. The abdomen is soft and elbow. There was fever [38,50] and ankle
distended in the upper region but retracted in the dysfunction, enlargement of cardiac dullness on 2
lower one. The correct diagnosis is: cm, tachycardia, weakness of 1st sound, gallop
A. * development of congenital ileus; rhythm, weak systolic murmur near apex. Which
B. resolution of congenital ileus; diagnosis corresponds to such symptoms?
C. pylorostenosis; A. *Acute rheumatic heart disease
D. Ledd’s syndrome; B. Systemic lupus erythematosus
E. congenital diaphragm hernia. C. Juvenile rheumatoid arthritis
997.The 5 month old child has become uneasy after D. Reiter’s disease
first time carrot puree feeding. There is multiple E. Reactive arthritis
vomiting. The general condition is moderate. The 1002.The child is 1,5 years old. Symptoms: chronic
abdomen is not distended and soft. By rectal cough with purulent sputum, dyspnea, retardation
examination there has been found that the feces of physical development, large amount of stool.
contain much mucus with bright blood admixture Sweat chloride 150 mEq/l. The child has been ill
and looks like red currant jelly. What disease does since 2nd month of age. Diagnosis: cystic fibrosis.
the child have? Choose the best therapy.
A. * intussusception A. *Enzymes + antibiotics
B. intestinal infection; B. Cholepoietic+adaptogenetic medicines
C. dyspepsia; C. Н2-blockaders + hepatoprotectors
D. gastrointestinal hemorrhage (bleeding); D. Vitamins+antibiotics
E. acute ileus. E. Vitamins + mucolytics
998.The symptoms and signs of acute appendicitis 1003.The 10 years old boy has complaints on headache,
depends on the anatomical location of appendix. weakness, fever [40С], vomiting, expressed
What kind of location promotes signs of urine dyspnea, pale skin with flush on right cheek, lag
tract irritation and the diarrhea? of right hemithorax respiratory movement,
A. *descending dullness on percussion over low lobe of right lung,
B. Medial weakness of vesicular respiration in this zone. The
C. Retrocaecalis abdomen is painless and soft under palpation.
D. typical Which disease lead to these symptoms and signs?
E. left- hand side location A. *Pneumonia croupousa
999.The 10 years old child has complaints on fever B. Intestinal infection
[39°C], frequent painful urination [pollakiuria]. C. Acute appendicitis
Urinalysis: proteinuria [0,066 g/l], leukocytouria D. Acute cholecystitis
[entirely within eyeshot], bacteriuria [105 colony E. Flu
forming units/ml]. Which diagnosis is the most 1004.The patient with acute respiratory viral infection
probable? [3rd day of disease] has complaints on pain in
A. *Acute pyelonephritis. lumbar region, nausea, dysuria, oliguria.
B. Acute glomerulonephritis Urinalysis – hematuria [100-200 RBC in eyeshot
C. Dysmetabolic nephropathy spot], specific gravity – 1002. The blood creatinin
D. Acute cystitis level is 0,18 mmol/l, potassium level - 6,4 mmol/l.
E. Urolithiasis Make the diagnosis.
1000.The 8 years old boy has suffered from angina. In 2 A. *Acute interstitial nephritis
weeks he has complaints on migratory joint pain, B. Acute renal failure
joint oedema and restriction of movement, fever. C. Acute glomerylonephritis
After examination there was diagnosed acute D. Acute cystitis
rheumatic heart disease, activity of III-rd degree, E. Acute renal colic
KROK 2 – Question Bank 105
1005.The baby boy was born at term from 1st 1010.5 year old boy fell ill abruptly: fever up to 39,8 C,
pregnancy. The jaundice was revealed at 2nd day recurrent vomiting, severe headache. Convulsions
of life, then it increased. The adynamia, vomiting occur in 3 hours. Physician found out positive
and hepatomegaly were observed. The indirect meningeal sign. Pleocytosis of 2500 cells chiefly
bilirubin level was 275 mcmol/l, the direct polymorphonuclear cells, elevated protein
bilirubin level -5 mcmol/l, Hb - 150 g/l.. Mother’s concentration and normal glucosa concentration
blood group - 0[I], Rh+, child’s blood group- was found in cerebrospinal fluid examination.
A[II], Rh+. Make the diagnosis. What is your diagnosis?
A. * Hemolytic disease of newborn [АВО A. * Purulent meningitis
incompatibility], icteric type B. Serous meningitis
B. Jaundice due to conjugation disorder C. Tuberculous meningitis
C. Hepatitis D. Subarachnoidal hemorrhage
D. Physiological jaundice E. Encephalitis
E. Hemolytic disease of newborn [Rh - 1011.7 –year – old girl has mild form of varicella.
incompatibility] Headache, weakness, vertigo,tremor of her limbs,
1006.The 3 months old infant who is suffering from ataxia, then mental confusion occur on the 5th day
acute segmental pneumonia reveals dyspnea of illness. Meningeal signs are negative.
[respiration rate – 80 per minute], paradoxical Cerebrospinal fluid examination is normal. How
breathing, tachicardia, total cyanosis. Respiration / can you explain these sings?
pulse ratio is 1:2. The heart dullness under normal A. *Encephalitis
size. Such signs characterise: B. Meningitis
A. *Respiratory failure of III degree C. Meningoencephalitis
B. Respiratory failure of I degree D. Myelitis
C. Respiratory failure of I degree E. Neurotoxic syndrome
D. Myocarditis 1012.7 year old girl fell ill abruptly: fever, headache,
E. Congenital heart malformation severe sore throat, vomiting. Minute bright red
1007.3 – year – old child has had fever, cough, coryza, rash appear in her reddened skin in 3 hours. It is
conjunctivitis for 4 days. It took sulfadimethoxine. more intensive in axillae and groin. Mucous
Today it has fever up to 39 C and maculopapular membrane of oropharynx is hyperemic. Greyish
rash appears on its face. The rash is on normal patches is on the tonsills. Submaxillary lymph
background of the skin. What is your diagnosis? nodes are enlarged and painful. What is your
A. * Measles diagnosis?
B. Allergic rash A. * Scarlet fever
C. Rubella B. Measles
D. Scarlet fever C. Rubella
E. Pseudotuberculosis D. Pseudotuberculosis
1008.2 – year – old girl has been ill for 3 days. Today E. Enteroviral infection
she has low – grade fever, severe catarrhal signs, 1013.8 year – old boy fell ill acutely: fever, weakness,
unabudant maculopapular rash on her buttocks headache, abdominal pain, recurrent vomiting,
and enlarged occipital lymph nodes. What is your then diarrhea and tenesmus. Stools occur 12 times
diagnosis? daily, are scanty, contain a lot of mucus, pus,
A. * Rubella streaks of blood. His sigmoid gut is tenderness and
B. Scarlet fever hardened. What is your diagnosis?
C. Measles A. * Dysentery
D. Adenoviral infection B. Samonollosis
E. Pseudotuberculosis C. Cholera
1009.3-year – old boy fell ill abruptly: fever up to 39 C, D. Staphylococcal gastroenteritis
weakness, vomitng. Haemorrhagic rash of various E. Escherichiosis
size appear on his lower limbs in 5 hours. 1014.The 7 months old infant is suffering from acute
Meningococcemia with infective – toxic shock of pneumonia which was complicated by
1 degree was diagnosed. What medications should cardiovascular insufficiency and respiratory
be administered? failure of II degree. The accompanied diagnosis is
A. * Chloramphenicol succinate and prednisone malnutrition of II degree. Choose the best variant
B. Penicillin and prednisone of therapy.
C. Penicillin and immunoglobulin A. *Ampiox+amicacin
D. Chloramphenicol succinate and interferon B. Macropen + Penicillin
E. Ampicillin and immunoglobulin C. Penicillin + Ampiox
KROK 2 – Question Bank 106
D. Gentamycin + Macropen length 52 cm at birth. Now many times per day the
E. Ampiox + polymixin supplement [up feeding] should be given?
1015.The child has complains of the “night” and A. *2
“hungry” abdominal pains. At fibroscopy in area a B. 3
bulbus ofa duodenum the ulcerrative defect a dia C. 1
of 4 mms is found, the floor is obtected with a D. 0
fibrin, [H.p +]. Administer the optimum schemes E. 4
of treatment: 1021.A 2 months old healthy infant with good appetite,
A. *De-nol – Trichopolum – Claritromicin. is given artificial feeding since 1–st month. When
B. De-nol is it advised to start the corrective feeding [fruit
C. Maalox-ranitidin juice]?
D. Vicalinum-ranitidin A. *4,0 months
E. Trichopolum B. 1,5 months
1016.The child was born from 5th pregnancy and 1st C. 2,0 months
delivery. Mother’s blood group - A[II] Rh-, D. 3,0 months
newborn’s -A[II] Rh+. The level of indirect E. 1,0 months
bilirubin in umbilical blood was 58 mcmol/l, 1022.Infant was born with body mass 3 kg and of length
hemoglobin - 140 g/l, RBC-3,8 T/l. The level of 50 cm. Now he is 3 years old. His brother is 7
indirect bilirubin in 2 hours was 82 mcmol/l. The years, suffers from rheumatic fever. Mother
hemolytic disease of newborn [icteric-anemic requested the doctor for a cardiac check up for the
type, Rh-incompatibility] was diagnosed. Choose 3 years old son. Where is the left relative heart
the therapeutic tactics. border located?
A. *Replacement blood transfusion ( conservative A. *1 cm left from the left medioclavicular line
therapy B. 1 cm right from the left medioclavicular line
B. Conservative therapy C. Along the left medioclavicular line
C. Blood transfusion ( conservative therapy D. 1 cm left from the left parasternal line
D. Symptomatic therapy E. 1 cm right from the left parasternal line
E. Antibiotics 1023.A 7-year-old girl suffers from bronchial asthma.
1017.Mother with infant visited the pediatrician for In spring, usually she has a bronchial attack. What
expertise advice. Her baby was born with body was the conclusion after auscultation of the lungs?
mass 3,2 kg and of length 50 cm. He is 1 year old A. *Both types of rales
now. How many teeth the baby should has? B. Fine budding rales
A. * 8 C. Sebelent dry rales
B. 10 D. Coarse bubling rales
C. 12 E. Crepitation rales
D. 20 1024.Boy, 7 year old, had an attack of asphyxia and
E. 6 distant whistling rale after playing with a dog. In
1018.Mother visited the pediatric for expertise advice. the anamnesis: atopic dermatitis caused by eating
Her son was born with body's mass 3 kgs and eggs, chicken, beef. What group of allergins is the
length 48 cm. He's 1 year old now. What is the reason of the development of bronchial astma
required normal mass? attacks?
A. *10,5 kg A. *Epidermal
B. 9,0 kg B. Dust
C. 11,0 kg C. Pollen
D. 12,0 kg D. Itch mite
E. 15,0 kg E. Chemical
1019.6 months infant was born with body's mass 3 kg 1025.A 14 year old boy has rheumatism. During 2 years
and length 50 cm. He is given natural feeding. he has transfered 3 rheumatic attacks. What course
How many times per day the infant should be fed? of rheumatism does the patient have?
A. *5 A. *Prolonged
B. 7 B. Acute
C. 6 C. Subacute
D. 8 D. Latent
E. 4 E. Persistent-Reccurent
1020.Infant is 6.5 months now and is given natural 1026.The patient with aquired heart failure has diastolic
feeding since birth. Body mass was 3.5 kg, with pressure 0 mm Hg. What heart failure does the
child have?
KROK 2 – Question Bank 107
A. *Aortal stenosis epicanthal folds. In epithelial cells of buccal
B. Mitral stenosis scrape X-chromatin [Barr body] is absent.
C. Aortal insufficiency Diagnosis is
D. Mitral insufficiency A. *Shereshevsky-Turner syndrome
E. Rheumatism B. Klinefelter syndrome
1027.Child [12 years old] has the ulcer disease of C. Down syndrome
stomach. What is the etiology of this disease? D. Edwards syndrome
A. Helicobacter pylory E. Patau syndrome
B. *Intestinal bacillus 1033.What signs are not typical for mumps?
C. Salmonella A. *Redness of the tissue around the ear
D. Lambliosis B. Painful chewing
E. Influenza C. Red and edematous Stensen’s duct
1028.A nine year old child is in hospital with acute D. The earlobe is pushed upward and out ward
glomerulonephritis. Clinical and laboratory show E. Swelling of the tissue round the angle of the
the acute condition. What food is not limited mandible
during the acute period of glomerulonephritis? 1034.The diphtheria pseudomembrane is commonly:
A. *Carbohydrates A. *Whitish or dirty gray, tough and adhered to
B. Salt the mucous membrane
C. Liquid B. Yellow, friable, localized in the gland crypt
D. Proteins C. White colored, friable, can be removed easily
E. Fats D. Yellow, localized beneath tonsillar mucous
1029.An 18 month child, taken to hospital on the 4-th membrane
day of the disease. The disease began acutely with E. Black, friable
temperature 39, weakness, cough, breathlessness. 1035.The typical signs of infectious mononucleosis are:
He is pale has cyanosis, febrile temperature for A. *Acute tonsillitis, lymphadenopathy and
more than 3 days. There are crepitative fine hepatosplenomegaly
bubbling rales at the auscultation. Percussion B. Acute tonsillitis and swelling of the neck
sound is shortened in right under scapula area. X- tissue
ray picture: unhomogenius segment infiltration 8- C. Acute tonsillitis and strawberry tongue
10 in the right, the increase of vascular picture, D. Acute tonsillitis and coryza
unstructural rools. Your diagnosis: E. Acute tonsillitis and conjunctivitis
A. *Segmentary pneumonia 1036.Measles in children is characterized by:
B. Grippe A. *Maculopapular rash, conjunctivitis, rhinitis,
C. Bronchitis high fever
D. Bronchiolitis B. Vesicular rash and high fever
E. Interstitial pneumonia C. Tonsillitis and conjunctivitis without rash
1030.9 year old patient. She has fitlike abdominal pains D. Hemorrhagic spots and meningeal signs
after fried food. No fever. She has the pain in E. Tonsillitis, strawberry tongue and macular rash
point Cera. The liver is not enlarged. Portion B 1037.What is the typical rash for chickenpox?
[duodenal probe] - 5 ml. Your diagnosis: A. * Vesicular, polimorphic
A. *Biliary tracts dyskinesia, hypotonic type. B. Hemorrhagic spots
B. Hepatocirrhosis C. Macular spots, localized in the fossae and
C. Acute colitis underarm
D. Chronic duodenum D. Maculopapular, confluent
E. Ulcer disease E. Ulcers of the skin
1031.A 4-month-old girl with blond hair and blue eyes 1038.The typical signs of croup, caused by influenza
has “mousy” odor of sweat and urine, delayed virus, are:
psychomotoric development. Mostly typical A. *Inspiratory dyspnea, barking cough, high
laboratory data for this disorder is fever
A. *Positive urine ferric chloride test B. Expiratory dyspnea, dry cough, normal
B. High level of oxyproline in urine temperature
C. High level of glycosaminoglycanes in urine C. Whooping cough
D. High concentration of chlorides in sweat D. Conjunctivitis and coryza
E. Low level of thyroid gland hormones in blood E. Tachypnea with moist rales
1032.A newborn girl has congenital lymphedema of the 1039.Dysentery is commonly characterised by:
hands and feet, short neck with loose skin, A. *High fever, vomiting, false urge to defecate,
antimongoloid slant of palpebral fissures, stool in the form of a spit mucus
KROK 2 – Question Bank 108
B. Watery, yellow, profuse stool 1045.A 2 year old boy has been vomiting intermittently
C. Hepatosplenomegaly and diarrhea for 3 weeks and has been irritable, listless, and
D. “Typhoid status” anorectic. His use of language has regressed to
E. Pea-soup feces speaking single words. In your evaluation of this
1040.A 6-year old asthmatic child is brought to the patient, the LEAST likely,diagnosis to consider is:
emergency room because of severe coughing and A. *Food allergy
wheezing during the prior 24 h. The child had B. Lead poisoning
been taking theophylline without relief. Physical C. Tuberculous meningitis
examination reveals a child who is anxious, has D. Brain tumor
intercostal and suprasternal retractions, expiratory E. Subdural hematoma
wheezing throughout all lung fields, and a 1046.Neonate of 5 days. What vaccination dose of BCG
respiratory rate of 60 breaths per minute. Initial vaccine [in мg] is necessary for vaccination of this
treatment may include the administration of child?
A. *Subcutaneous epinephrine A. *0.05 мg
B. Parenteral phenobarbital B. 0.025 мg
C. Intravenous fluids in the first 2 h to correct a C. 0.075 мg
water deficiency. D. 0.1 мg
D. N-acetyl cysteine and cromolyn by inhaler E. 0.2 мg
E. Parenteral gentamicyn 1047.A 39-year-old woman applied to a doctor with the
1041.A 1-year old infant is admitted for failure to complaints of monomorphous eruption on the skin
thrive. During the neonatal period he had an of the trunk and mouth mucosa. Pemphigus
exploratory laparotomy for intestinal obstruction. vulgaris has been diagnosed on the base of the
At 3,8, and 11 month of age, he had respiratory clinical picture. What cytological test verifies the
infections diagnosed as bronchitis. Physical diagnosis?
examination reveals a weight of 6,8 kg, thin A. *Cytodiagnosis after Tzanck
extremities with very little subcutaneous tissue, B. Test for LE-cells
and a protuberant abdomen. The essentials C. Detection of Lanhgerhans’ giant cells
diagnostic study in this child is: D. Detection of Meisner’s corpuscles
A. *Sweat electrolytes E. Detection of eosinophiles in the contents of the
B. Bronchoscopy bullae
C. Tuberculin skin test 1048.A 32-year-old man divorced, has an irregular
D. Serum immunoglobulin level sexual life. He complains of falling out of hair in
E. Skin test for milk allergy the region of eyelashes, eyebrows, scalp.
1042.Which of the following laboratory findings is Objectively: diffuse alopecia is observed, eyebrow
unusual in patients with simple (nutritional) margin is absent, eyelashes are stair-like [Pinkus’s
rickets sign].What investigation should be carried out first
A. *Hypercalciuria of all?
B. Hyperphosphaturia A. *Wasserman test, IFT
C. Elevated levels of serum alkaline phosphatase B. T. pallidum Immobilization Test [TPI]
D. Aminoaciduria C. Detection of the nasal mucouse for
E. Hypophosphatemia Micobacterium Leprae Hansen
1043.Schonlein-Henoch purpura is associated with all D. Consultation of neuropathist
the following common manifestations EXCEPT E. CBC
A. *Urticarial rash 1049.A triad of symptoms [“stearing spot”, “terminal
B. Hepatitis film”, “blood dew”] have been revealed in a
C. Arthritis patient. What disease should you think about?
D. Nephritis A. *Psoriasis
E. Abdominal pain B. Lichen ruber planus
1044.Thermal injury from immersion in hot water can C. Vasculitis
be prevented by setting water heaters at a D. Seborrhea
temperature no higher than: E. Ritter’s disease
A. *50.0°C 1050.The blood pressure of 120/80 mmHg is elevated
B. 36.0°C for children aged.
C. 42.5°C A. *4 years
D. 65.0°C B. 7 years
E. 80.0°C C. 10 years
D. 12 years
KROK 2 – Question Bank 109
E. 15 years B. Pallor
1051.Erb-Duchenne palsy is described best as: C. Fall in body temperature
A. *Weakness of an arm from a traction injury of D. Increased respiratory rate
the upper brachial plexus E. Metabolic acidosis
B. Weakness of a wrist and ipsilateral Horner's 1057.A primiparous woman whose blood type is 0-
syndrome positive gives birth at term to an infant who has
C. Pseudoparalysis of an arm caused by syphilitic A-positive blood and a hematocrit of 55 percent A
D. Osteochondritis serum bilirubin level obtained at 36 h of age is 12
E. Total ipsilateral arm weakness resulting from mg/dL. Which of the following laboratory
a fracture of a clavicle findings would be LEAST characteristic of ABO
1052.Neonatal seizures can develop as a result of each hemolytic disease?
of the following EXCEPT: A. *Negative direct Coombs' test
A. *Hypermagnesemia B. An elevated reticulocyte count
B. Anomalies of the central nervous system C. Fragmented red blood cells in the blood smear
C. Birth trauma D. Nucleated red blood cells in the blood smear
D. Hypocalcemia E. Spherocytes on blood smear
E. Hypoxia 1058.Which of the following is abnormal in a 2-day old
1053.A full term newborn infant is having episodes of infant?
cyanosis and apnea, which are worse when he is A. *Hematocrit 38 percent
attempting to eat, but he seems better when he is B. Vaginal bleeding
crying. The most important diagnosis to establish C. Breast enlargement
quickly is D. Bilirubin 3 mg/dL
A. *Choanal atresia E. Heart rate 140 beats per minute
B. Ventricular septal defect 1059.At 43 weeks' gestation a long, thin infant is
C. Ondine's curse (primary alveolar delivered who is apneic, limp, pale, and covered
hypoventilation syndrome) with "pea soup" amniotic fluid. The first step in
D. Sickle cell anemia the resuscitation of this infant at delivery should
E. Floppy palate syndrome be:
1054.A 2-week old infant has had no immunizations, A. *Suction of the trachea under direct vision
sleeps 18 h a day, weighs 3.5 kg, and takes 60 mL B. Artificial ventilation with bag and mask
of standard infant formula four times a day, but no C. Artificial ventilation with endotracheal tube
solid food and no iron or vitamin supplements. Of D. Administration of 100\% oxygen by mask
most concern is: E. Catheterization of the umbilical vein
A. *Caloric intake 1060.In an infant the most important noncardiac
B. Iron levels manifestation of digitalis toxicity is
C. Immunization status A. *Vomiting
D. Levels of vitamins A, C, and D B. Dizziness
E. Circadian rhythm C. Fever
1055.A full term infant is born after a normal D. Visual disturbances
pregnancy, delivery, however, is complicated by E. Urticaria
marginal placental separation. At 12 hours of age 1061.A newborn infant has mild cyanosis, diaphoresis,
the child, although appearing to be in good health, poor peripheral pulses, hepatomegaly, and
passes a bloody meconium stool. For determining cardiomegaly. Respiratory rate is 60 breaths per
the cause of the bleeding, which of the following minute, and heart rate is 230 beats per minute. The
diagnostic procedures should be performed first? child most likely has congestive heart failure
A. *Barium enema caused by
B. An Apt test A. *Paroxysmal atrial tachycardia
C. Gastric lavage with normal saline B. A ventricular septal defect and transposition of
D. An upper gastrointestinal series the great vessels
E. Platelet count, prothrombin time, and partial C. Atrial flutter and partial atrioven-tricular block
thromboplastin time D. Hypoplastic left heart syndrome
1056.An infant weighing 1400 g is born at 32 weeks' E. A large atrial septal defect and valvular
gestation in a delivery room that has an ambient pulmonic stenosis
temperature of24°C. Within a few minutes of 1062.A 4-year old child attends the kindergarten.
birth, this infant is likely to exhibit all the Complaints of the bad appetite, fatigue. Objective
following EXCEPT: examination: skin and mucous membrane are pale,
A. *Shivering child is asthenic. In the hemogram: hypochromatic
KROK 2 – Question Bank 110
anemia 1st., leucomoide reaction, of the Laboratory analysis reveals proteinuria and
eosinophile type. What pathology must be microhematuria.He is most likely to be affected by
excluded at first? A. *Anaphylactoid purpura
A. *Worm invasion B. Systemic lupus erythematosus
B. Lymphoprolipherative process C. Poststreptococcal glomerulo_nephritis
C. Hypoplastic anemia D. Polyarteritis nodosa
D. Duodenal ulcer E. Dermatomyositis
E. Atrophic gastritis 1068.Characteristics of hypenatremic dehydration with
1063.Following a chest x-ray that showed a right-sided Na+ in serum 170 meq/L and 10 percent loss of
tension pneumo_thorax, the patient in the previous body weight include all the following EXCEPT
question developed even worse respiratory distress A. *Low blood pressure
and is now deeply cyanotic in 80\% oxygen. The B. Convulsions
best course would be to C. Thirst
A. *Use needle and syringe to do emergency D. Lethargy
decompression E. Hyperirritability
B. Increase oxygen to 90\% 1069.On the third day of disease a 10 years old child
C. Request surgical consultation for placement of with acute respiratory infection developed
a chest tube productive cough. The percussion is without
D. Give intravenous bicarbonate pathologic features. The auscultation reveals
E. Follow blood gases bilateral rales over the lung surface. What
1064.A 6-year-old boy is brought to the emergency diagnosis should be made?
room with a 3-h history of fever to 39.5°C and a A. *acute bronchitis
sore throat. The child looks alert but anxious and B. relapsing bronchitis
has a mild inspiratory stridor. You should C. obstructive bronchitis
immediately D. Asthma
A. *Prepare to establish an airway E. Pneumonia
B. Obtain an arterial blood gas and start an IV 1070.An infant aged 1 year on the third day of common
line cold at night developed inspiratory stridor, hoarse
C. Order a chest x-ray and lateral view of the voice and barking cough. Physical examination
neck revealed suprasternal and intercostal chest
D. Examine the throat and obtain a culture retractions. There is a bluish skin discoloration
E. Admit the child and place him in a mist tent moistly seen over the upper lip. The respiratory
1065.After endotracheal intubation, it is appropriate to rate is 52 per min and pulse rate 122 per min. The
check the position of the tube by each of the body temperature is 37,50C. What disease does
following procedures EXCEPT the infant have?
A. *Instillation of saline while listening for A. *Acute infectious croup due to viral
bubbling sounds laryngotracheitis grade II of airway
B. Use of a magnet obstruction, incomplete compensation state
C. Fiberoptic endoscopy B. Acute laryngitis
D. Auscultation for symmetric breath sounds C. Bronchopneumonia without complications
E. Chest X-ray D. Acute bronchiolitis with respiratory distress
1066.A 7-day-old boy is admitted to the hospital for E. Acute epiglottitis
evaluation of vomiting and dehydration. Physical 1071.One week old male infant had one episode of
examination is otherwise normal except for vomiting yesterday and 2 episodes of spitting up
minimal hyperpigmentation of the nipples. Serum with poor feeding today. There is no history of
sodium and potassium concen_trations are 120 fever, diarrhea or coughing. His urine output is
meq/L and 9 meq/1 respectively. The most likely decreased. He was born at term weight 3.2 kg.
diagnosis is Weight now is 3.0 kg. Laboratory data: Na 128
A. *Congenital adrenal hyperplasia mmol/l, K 6.9mmol/l. What is the most probable
B. Pyloric stenosis diagnosis?
C. Secondary hypothyroidism A. *Salt wasting form of adrenal insufficiency
D. Panhypopituitarism B. Birth trauma of central nervous system
E. Hyperaldosteronism C. Gastroenterocolitis
1067.A 7-year-old boy has crampy aboominal pain and D. Pylorospasm
a rash on the back of his legs and buttocks as well E. Pylorostenosis
as on the extensor surfaces of his forearms. 1072.A 42-week-gestational-age, 3800-g, breast-fed,
female is noted to have persistent
KROK 2 – Question Bank 111
hyperbilirubinemia at 2 weeks of age. The infant 1077.Phenylketonuria was diagnosed in newborn male
has not gained weight since birth, has hoarse cry, during screening investigation. How long dietary
dry skin, hypotonia, an umbilical hernia, restriction of phenylalanine must continue?
constipation, and an anterior fontanel measuring 4 A. *8 – 10 years
- 6 cm. What is the most likely diagnosis? B. 1 year
A. *hypothyroidism C. 2 years
B. hereditary spherocytosis D. 5 years
C. neonatal hepatitis E. 6 months
D. biliary atresia 1078.A 9 year old female is referred to the hematology
E. galactosemia department with a chief complaint of acute onset
1073.The infant is born with perinatal asphixia. At birth of easy bruising and "rash" for 3 days. She had
he is apneic with a heart rate 70 per minute, so upper respiratory infection symptoms
bag-and- mask positive presurre ventilation with approximately 2 weeks ago. A diffuse petechial
100\% O2 was immediately instituted. After 30 rash is noted on her neck, trunk, extremities and
sec of ventilation the heart rate is not increased. groin. CBC shows Hgb 128 g|l, Hct 38.5, WBC
What is the most appropriate next step? 6,000x. Platelet count is low at 5,000. What is the
A. *chest compression most probable diagnosis?
B. tactile stimulation A. *Idiopathic thrombocytopenic purpura
C. to continue ventilation B. Bone Marrow Failure
D. intravenous sodium bicarbonate C. Hemophilia A
E. intravenous epinephrine D. Hemolytic uremic syndrome
1074.The boy is 10 years old. Two weeks ago he E. Acute lymphoblastic leycosis
suffered from acute tonsillitis. Now he complains 1079.Boy is 2 months old. He was born in September.
on common weakness, oedema of eyelids, ankle Не is on breast feeding. What should a daily doze
joints, headache, nausea. Skin is pale, appetite is of ergocalciferol be for prophylaxis of rickets?
reduced, daily diuresis is 600 ml. In urinalysis: A. *400-500 IU
protein – 0,066 g/l, WBC – 4-6, RBC – 40-45. B. 200-300 IU
The most probable diagnosis is? C. 300-400 IU
A. *Glomerulonephritis with nephritic syndrome D. 500-600 IU
B. Interstitial nephritis E. 100-200 IU
C. Pyelonephritis 1080.A 10 year old boy has the history Diabetes
D. Glomerulonephritis with nephrotic syndrome Mellitus type I during 6 years. He has headache,
E. Polycystic kidney disease nausea, sweet smell of his breath and he is
1075.A 1 year old boy does not walk, does not speak, breathing faster than usual. His lab studies show
not active. He holds head since 9 months. Physical Na 132 mmol/l, K3.3 mmol/l, glucose 28 mmol/l.
examination shows pallor, oedema, saddle nose, What treatment will you prescribe?
large tongue. Skin is dry, voice is gruff. Pulse rate A. *IV fluids and short active insulin (0.1 u/kg)
- 100 per 1 min. The teeth are absent. What B. IV fluids
diagnosis is the most probable? C. Short active insulin (0.1 u/kg)
A. *Hypothyroidism D. Combination of short active insulin and
B. Hirschsprung’s disease intermediate active insulin
C. Rickets E. Intermediate active insulin
D. Down’s syndrome 1081.A 2 – hour – old, 32 – week – gestational – age
E. Hydrocephalus infant develops progressive cyanosis, grunting,
1076.4300-g infant was born at term gestation to a nasal flaring, and chest retractions. Silverman
poorly controlled insulin-dependent diabetic scores were 4. The chest radiograph reveals a
mother. Initially, the infant did well, but at 2 hours ground glass–air bronchogram pattern. The infant
of age he was noted to be lethargic and then now requires oxygen therapy with continuous
develop tonic-clonic seizures. Blood glucose is 1.6 positive airway pressure to maintain adequate
mmol/l. What is the most likely reason of oxygenation. What is the most likely diagnosis?
seizures? A. *respiratory distress syndrome
A. *hypoglycemia B. intranatal asphyxia
B. hypocalcemia C. congenital pneumonia
C. hypomagnesemia D. pneumothorax
D. hyponatremia E. congenital heart disease.
E. pyridoxine deficiency 1082.A 7 year old male presents to physician with the
chief complaint of dark "cola colored" urine, facial
KROK 2 – Question Bank 112
puffiness and abdominal pain for the past 2 days. has mild tachycardia without murmurs or gallop
14 days ago he had a sore throat and fever. He has and mild tremor. Hyperthyroidism (Graves
had abdominal pain. His urine is dark. Urine disease) was diagnosed. What treatment will you
analysis shows an increased specific gravity, prescribe?
RBCs are too numerous to count. What is the A. *Propylthiouracil
most probable diagnosis? B. Surgical treatment
A. Glomerulonephritis. Nephritic Syndrome C. L-thyroxine
B. Glomerulonephritis. Nephrotic Syndrome D. Prednizolone
C. Acute heart failure E. Iodinated salt
D. Acute infection of urinary tract 1087.1 month old female presents to the intensive care
E. Hemolytic uremic syndrome unit with severe cyanosis, congestive heart failure,
1083.A 4 year old female has been limping with normal first sound, single second sound and an
swelling of her right knee for several months. insignificant one to two ejection systolic murmur.
Physical examination demonstrates swelling of her The electrocardiogram shows right axis deviation
right knee, flexion contracture of 10 degrees and and right ventricular hyperthrophy. The thoracic
flexion to 120 degrees. Lab. data: WBC 8 g/l, with roentgenogram shows cardiomegaly with narrow
45\% neutr., 47 lymphs\%, 8\% mon. Hgb 120 g/l. base and plethoric lung fields. What is the most
ESR 20mm/h. Rheum. factor neg., ANA 1:640 probable diagnosis?
speckled. What is the most probable diagnosis? A. *Congenital heart disease, right to left shunt
A. *Juvenile Rheumatoid Arthritis B. Congenital heart disease, left to right shunt
B. Rheumatism C. Pneumonia
C. Lupus D. Congenital lung malformation
D. Osteomyelitis E. Bronchiolitis
E. Infection Arthritis 1088.16 month old female presents with an acute onset
1084.The laboratory data of patient's hemoglobin as 70 of her hands and feet "drawing up. Both her hands
g/l, and the reticulocyte count as 1\%. The are flexed at the wrists with hyperextended fingers
published normal value for the reticulocyte count at the proximal and distal interphalangeal joints
is 0.7\% to 2.0\%, so the reticulocyte count is and flexion at the metacarpophalangeal joints.
within the laboratory's normal range. How would Neurologic exam reveals symmetric hyperreflexia,
you interpret this reticulocyte count? decreased muscle strength and tone. Lab.data: Ca
A. *This reticulocyte count value is normal for a 0,9 mmol/l, P 0.4 mmol/l. What is the most
patient with a normal hemoglobin, but for a probable diagnosis?
severely anemic patient, the reticulocyte count A. *Vitamin D deficiency, spasmophylia
should be high. B. Epilepsy
B. This reticulocyte count is normal, so the C. Acute infection of central nervous system
patient's bone marrow is making RBCs D. DiGeorge syndrome
adequately. E. Glycogenosis
C. This reticulocyte count is low. 1089.A 7 month old male delayed in psycho-motor
D. This reticulocyte count is too high. development from 6 month. He developed
E. This reticulocyte count is depend from pursuant to age up to 3 mo. His hair is lighter than
hemoglobin level in parents, eyes are blue. There are periodically
1085.Mother of a previously healthy 4 year old male cramps. It is marked the specific "mouth-like"
complains of cough and wheeze. Boy had playing odor of urine. The diagnosis of phenilketonuria is
with a small toy. During examination the right made. What is necessary to exclude from child
side of a chest show hyperresonance, diminished diet?
vocal resonance and poor air entry. What is the A. *Phenilalanin
most probable diagnosis? B. Metionin
A. *Foreign body aspiration C. Galactose
B. Asthma D. Glucose Maltose
C. Pneumonia 1090.A 8 year old boy has symptoms of polyuria,
D. Bronchitis nocturia during 2 months. He began to lose weight
E. Bronchiolitis over this same period. He has a noticeably sweet
1086.A previously healthy 14 year old female smell to his breath. His skin is warm to his wrists
complains of a fast heart rate, weight loss, and and ankles. A urine shows 4+ glucose and 2+
fatigue over the past 2 months. Her family history ketones. His initial lab studies show Na 132
is significant for a grandmother and aunt with mmol/, K3.3 mmol/l, glucose 23 mmol/l. A urine
Hashimoto thyroiditis. During examination she
KROK 2 – Question Bank 113
analysis shows 4+ glucose and 2+ ketones. What E. 3 years
is the most probable diagnosis? 1097.Child is 3 months old. At investigation in a
A. *Diabetes Mellitus, Type I, diabetic polyclinic rickets was diagnosed. The basic
ketoacidosis process characteristic for rickets is infringement of
B. Acute infection of urinary tract a metabolism.
C. Diabetes Mellitus, Type II A. * Calcium and phosphorus
D. Enuresis B. Calcium and potassium
E. Chronic lymphocytic thyreoiditis C. Calcium and magnesium
1091.The child is 6 years old. He suffers from nervous – D. Calcium and sodium
arthritic diathesis. What period is critical for E. Calcium and zinc
development of this diathesis? 1098.The child is three months old. He suffers from
A. * School rickets. What acid does the important role play in
B. Preschool an exchange of phosphorus and calcium,
C. Infancy influences on action of calciferol, function of
D. Neonatal parathyroid glands?
E. Perinatal A. * Citric
1092.Child is 2 months old. He is premature infant. B. Acetic
What should a daily doze of ergocalciferol be for C. Hydrochloric
prophylaxis of rickets? D. Phosphoric
A. *1000-1200 IU E. Sulfuric
B. 800-1000 IU 1099.The child is 7 months old. He suffers from
C. 1200-1500 IU spasmophilia. What infringement of a kind of an
D. 400-500 IU exchange is a pathogenic part at spasmophilia?
E. 1300-1500 IU A. *Phosphoric-calcic
1093.Girl is 4 months old. She was born in October. B. Phosphoric-potassic
She is on breast feeding. To what age should C. Phosphoric-sodium
prophylaxis of rickets carry out? D. Phosphoric-magnesion
A. *1,5 years E. Phosphoric-zinc
B. 2 years 1100.A 6-year-old boy with asthma has had mild
C. 2,5 years wheezing only four times since you began treating
D. 1 year him 3 months ago with Cromolyn inhalation twice
E. 6 months each day. For the past 2 days, he has again had
1094.Boy is 4 months old. Не is on breast feeding. mild coughing and wheezing. What should you
Determine a remedy for primary prophylaxis of recommend to treat acute attack of asthma?
spasmophilia. A. *inhalation Salbutamol
A. * Ergocalciferol B. inhalation corticosteroids
B. Calcium gluconate C. Loratadine
C. Calcium chloride D. Aspirin
D. Natrium chloride E. Theophylline
E. Calcium pantothenate 1101.The child is 6 months old. He suffers from
1095.Boy is 3 months old. He has signs of local lymphatic-hypoplastic diathesis. What is the main
manifestation of exudative catarrhal diathesis. pathogenic marker at this diathesis?
From what age should preventive inoculations do A. *Lymphocytosis
to this child? B. Lymphopenia
A. *From 3 months C. Neutrophylosis
B. From 6 months D. Neutropenia
C. From 8 months E. Monocytosis
D. From 10 months 1102.Full term newborn has developed jaundice at 10
E. From 12 months hours of age. Hemolytic disease of newborn due to
1096.Girl is 4 years old. She suffers from recurring Rh-incompatibility was diagnosed. 2 hours later
bronchitis. Through what time should prophylactic the infant has indirect serum bilirubin level
medical supervision of this child at absence of increasing 14 mmol/L. What is most appropriate
recurrences stop? for treatment hyperbilirubinemia in this infant?
A. *2 years A. *exchange blood transfusion
B. 1 year B. phototherapy
C. 2,5 years C. phenobarbital
D. 1,5 years D. intestinal sorbents
KROK 2 – Question Bank 114
E. infusion therapy 1108.Baby was born by vaginal delivery following a
1103.7 -year-old boy with chronic cinusitis and term gestation. Apgar score was 5 and 7. Physical
reccurent pulmonary infections has chest x-ray examination reveals scaphoid abdomen, slow
demonstrating a right-sided cardiac silhouette. respiration and reduced chest movement. Breezing
What is the most likely diagnosis? sounds are absent in a lower part of left lung.
A. *Kartagener syndrome Chest radiograph reveal shift of mediastinum to
B. cystic fibrosis right side, intestinal shadow on left side. What is
C. bronhiolitis obliterans the most likely diagnosis?
D. laryngotracheomalacia A. *diaphragmatic hernia
E. α-antitrypsin deficiency B. respiratory distress syndrome
1104.Parents of 3-year-old girl complain of rectal C. congenital bronhiectasis
prolapse and failure to gain weight in spite of a D. congenital pneumonia
good appetite. Patient has a history of recurrent E. cystic fibrosis
prolonged respiratory infections and frequent, 1109.A 2.9-kg term male infant is born to a mother who
bulky, greasy stools. Sweet chloride is 126 developed polyhydramnios at 34 weeks' gestation.
mmol/l. What is the diagnosis? At birth, the Apgar scores were 9 and 9. The infant
A. *cystic fibrosis develops choking and cyanosis with the first feed.
B. α-antitrypsin deficiency In addition, is unable to place a nasogastric tube.
C. Kartagener syndrome What is the most likely diagnosis?
D. celiac disease A. *esophageal atresia
E. Hirschprung’s disease B. choanal atresia
1105.2-year-old previously healthy boy had eaten C. laryngomalacia
peanuts and suddenly presents with an acute onset D. tracheal atresia
of cough, choking, and respiratory distress. E. respiratory distress syndrome
Physical examination reveals a RR of 45 and 1110.A 3 month child has vomiting, poor feeding, fever
wheezing, body temperature is normal. There is no up to 38°C for the 2 days. His stool is frequent,
history of asthma or allergic reactions, and no one with slime. Infant has lost 250 g of weight. He is
at home is ill. What is the most likely diagnosis? pale, mucous are dry, fontanel is “fallen”, turgor is
A. *foreign body aspiration reduced, the abdomen is moderately inflated. The
B. acute bronchiolitis diagnosis of enterocolitis was made, dehydratation
C. attack of asthma of 2nd degree. What solution is necessary to
D. acute laryngitis prescribe for rehydratation?
E. angioedema A. *Rehydron
1106.A 6-year-old girl has had a dry cough without B. 5 \% glucose solution
sputum for 2 months. The cough is getting worse C. Boiled water
after exercises and at night. Family history D. Tea
revealed that the parents have eczema. On E. Broth of a camomile (medical)
physical examination, you hear a wheeze in both 1111.The child is 11 months old. He suffers from
lung fields. She has none of the signs of chronic nervous – arthritic diathesis. The increased
lung disease. What is the most likely diagnosis? synthesis of what acid is pathogenically at
A. *bronchial asthma nervous- arthritic diathesis?
B. pertussis A. * Uric acid
C. foreign body aspiration B. Acetic acid
D. bronchiectasis C. Phosphoric acid
E. interstitial pneumonia D. Hydrochloric acid
1107.You are called to the delivery of a boy at 42 E. Sulfuric acid
weeks' gestational age with thick meconium 1112.What anatomic - physiologic feature of lungs
stained fluid. The obstetrician rapidly delivers the structure in the first years of life infants causes
infant and hands him to you for care. The boy is exactly segmental pneumonias?
hypotonic, cyanotic, apneic, and bradycardic. A. *Segments removing with soft connective
What is the most appropriate next step? tissue
A. *intubate the trachea and apply aspiration B. Left bronchus deviation at right angles
B. stimulate the infant to breathe C. Wide right bronchus which is trachea’s
C. administer epinephrine extension
D. provide bag-and-mask ventilation D. Elastic tissue hypoplasia
E. intubate the trachea and provide positive E. Reduced aeration and secretion evacuation
pressure ventilation
KROK 2 – Question Bank 115
1113.The child of 7 months old who is suffering from B. Big daily urine output
tetralogy of Fallot has admitted to hospital with C. Transitory low urine specific gravity
attack of dyspnoea and cyanosis. The respiration D. Small volume capacity of bladder
rate is 55 per minute, the heart rate – 120 per E. Rather low position of the kidney
minute. What is the treatment of this condition? 1118.A 10 years old child is ill with spastic cerebral
A. *IV promedol, anaprilin; oxygen supply palsy. Supported on trying to walk the child has
B. IV strophantin, glucose his legs flexed in hip and knee joints and
C. IV euphyllin, prednisone spastically scissored. The upper extremities
D. IV prednisone, furosemide movements aren’t disturbed. What type of
E. IV furosemide, albumine spasticity distribution is there?
1114.A 2.5 month girl has frequent vomiting A. *Lower paraplegic
irrespective of eat period, growing thin, weakness B. Tetraplegic
during last 1,5 months. She was born with C. Hemiplegic
bodyweight 3400. Present weight - 2900. Girl is D. Athetosis
pale, has penis-like clitoris, subcutaneous fat is E. Monoplegic
absent. The laboratory studies show K - 9.4 1119.A newborn aged 3 days with hyperbilirubinemia
mmol/l, sodium - 86 mmol/l. What is the most (428 mkmol/l) developed followed disorders.
probable diagnosis? From beginning there were severe jaundice with
A. *Adrenogenital syndrome (virilised and salt- poor suckling, hypotomia and hypodynamia. Little
wasting form) bit later periodical excitation, neonatal
B. Pylorostenosis convulsions and neonatal primitive reflexes loss
C. Pylorospasmus are noted. Now physical examination reveals
D. Enterocolitis convergent squint, rotatory nystagmus and setting
E. Partial intestinal ileus sun eye sign. How to explain this condition?
1115.The girl is 3 years old. She has admitted to A. * Encephalopathy due to hyperbilirubinemia
hospital with the parents’ complaint on poorly B. Skull injury
walking. The excessive development of shoulder C. Brain tumour
muscles is noticed, the lower extremities are poor D. Hydrocephalus
developed, muscle hypotonia. The borders of E. Spastic cerebral palsy
relative heart dullness are extended to the left on 2 1120.A 1.5 month male infant has frequent vomiting,
cm. There is systolic murmur in 2nd intercostal weight loss during last 3 weeks. X-ray
interspace on the right side of the sternum. Blood examination shows the extension of stomach, high
pressure on arms – 100/70, on legs – 40/20. Which level of fluid, the delay of contrast substance in a
diagnosis could be suspected? stomach for a long time (8 hours), which does not
A. *Coarctation of aorta disappear after atropin injection. What is the most
B. Ventricle septal defect probable diagnosis?
C. Atrium septal defect A. *Pylorostenosis
D. Tetralogy of Fallot B. Atresia of esophagus
E. myocarditis C. Cystic fibrosis
1116.A mother consults her 3 years old daughter by D. Sepsis
pediatrician. The child complains of hoarse voice, E. Hirshprung’s disease
barking cough, laboured breathing with difficult 1121.A girl aged 8 month has a large head with the
inspiration. The infectious croup due to acute circumference up to 48 cm corresponded a value
laryngitis is diagnosed. What anatomical feature over 95 percentile and big non bulged non pulsed
predisposes a child to laryngeal stridor? fontanel. The preliminary diagnosis is
A. *Narrow vocal slit hydrocephalus. What cerebral spinal fluid findings
B. Wide laryngeal lumen do you wait for in this case?
C. Watering-pot form of the larynx A. * Cells 2-3 in 1 mkl, protein 0,2-0,4 g/l
D. Poor vascularization of the mucous membrane B. Cells 200-300 in 1 mkl, protein 1,0-2,0 g/l
E. Diaphragmatic type of respiration C. Lymphocytes cells 500-600 in 1 mkl, protein
1117.The positive urine glucose test due to so-called 2,5-4,0 g/l
physiological glucosuria was revealed in a healthy D. PMNL (polymorphonuclear leukocytes) cells
1 month old infant on routine examination. What 50-100 in 1 mkl, protein 4,5-6,0 g/l
feature of urinary system in infant predisposes to E. PMNL cells 100-200 in 1 mkl, protein 5,0-6,0
this phenomenon? g/l
A. * Immaturity of glucose reabsorbtion in 1122.The child is 3 months old. He has admitted to
kidney hospital to diagnose the reason for cardiac
KROK 2 – Question Bank 116
murmur. Complaints of parents: low weight gain, physical examination. What hormone level are the
attacks of dyspnoea and cyanosis which amplify at most characteristic for this disease?
a physical load. Systolic murmur in 3rd intercostal A. * Free thyroxine and total serum
interspace on the left side, systolic murmur under triiodothyronine excess.
2nd intercostals interspace on the right side of the B. Free thyroxine deficiency
sternum, hypertrophia of right ventricle. What C. Serum thyroid-stimulating hormone excess
diagnosis should be suspected? D. Radioactive iodine uptake excess test
A. *Tetralogy of Fallot E. serum triiodothyronine deficiency
B. infectious endocarditis 1127.1 years old infant developed the signs of folic acid
C. ventricular septal defect deficiency anemia. What features of CBC are
D. hypertrophyc cardiomyopathy characteristic for this pathology?
E. atrial septal defect A. * hyperchromic and macrocytic RBC (red
1123.The child aged 12 yr complains of slight fever up bood cells), reticulocytopenia,
to 38.0(C, knee joints pain, a day before ankle thrombocytopenia, leukopenia
pain, tenderness in active and passive movements, B. hyperchromic and microcytic RBC (red bood
common weakness and cardiac pain. It is known cells), reticulocytopenia, thrombocytopenia,
he was ill with acute tonsillitis two weeks ago. leukopenia
Physical examination reveals left heart border C. hyporchromic and microcytic RBC (red bood
external shift, non frequent cardiac premature cells), anisocytosis, reticulocytosis,
bites. What disease should be suspected? thrombocytosis;
A. * Rheumatic fever D. normochromic and normocytic RBC (red bood
B. Now-rheumatic carditis cells), thrombocytosis, leukocytosis;
C. Rheumatoid arthritis. E. All answers are true.
D. Systemic lupus erythematosus 1128.1,5 years old infant has revealed iron deficiency.
E. Reactive arthritis What is the duration of treatment with oral iron
1124.A 7 years old boy is admitted to the hospital with preparations?
complains of thirst and polyuria. He is ill with A. *2-3 months
diabetes mellitus within 5 years. In the clinical B. 2 weeks
history there are 3 times happened diabetic comas. C. 21 days
Blood glucose is 15.54 mmol/l and in urine 5\%. D. 3-4 weeks
The retinal angiopathy signs has been revealed by E. 6 months
an ophtalmologist. What dose of insulin will be 1129.What is the most important suggestive clinical
administered to this child more likely? feature of CBC (complete blood count) and bone
A. *1,5 units/kg. marrow examination which make it possible to
B. 0,25 unit/kg suspect myelodysplastic syndrome in children?
C. 0,5 unit/kg A. *The high blood leukocyte count accompanied
D. 1,0 unit/kg by low platelet and RBC (red blood cells)
E. 2,0 units/kg counts associated with disorders of maturation
1125.A 12 years old girl suffers with diabetes mellitus bone marrow-derived cells
since 2 years of age. Clinical examination reveals B. High platelet and RBC (red blood cells)
the considerably enlarged abdomen. The slightly counts associated with bone marrow
tender liver is palpated 6cm lower of the costal hyperfunction
margin. The child has also Cushing type of C. High WBC count in peripheral blood normal
obesity, short stature, and pubertal delay. Blood or depressed bone marrow function
glucose is 17 mmol/l, glucosa in urine 4 \%. There D. Low RBC, WBC and platelet counts
are an increased levels of blood cholesterol and associated wiht disorders of maturation bone
ketoacidemia. What is the suggested diagnosis? marrow-derived cells
A. *diabetes mellitus I type, Mauriac’s syndrome E. Low RBC, WBC and platelet counts
B. Diabetes mellitus I type, Nobecurs syndrome. associated wiht bone marrow hyperfunction
C. Diabetes mellitus I type, ketoacidotic coma. 1130.8 years old girl had had a rheumatic fever
D. Cushing’s disease. manifested with chorea and carditis 6 months ago.
E. Cushing’s syndrome. She was treated in-patient department within 1,5
1126.The girl aged 14 years old complains of sleep months. Now she is under long term observation
disturbances, body weight loss, palpitations, by the rheumocardiologist in children out-patient
cardialgias, fatigue. 2nd degree thyroid gland department. The prevention therapy of rheumatic
hyperplasia and exophthalmus were noted by fever relapse foresees:
KROK 2 – Question Bank 117
A. *The administration of bicillin-5 every month E. Easy absorbed carbohydrates
during 5 years 1135.A 1 month male infant has symptoms of
B. The administration of bicillin-5 every month excitation. His head circumference is - 37 cm, the
during a year size of large fontanel is 2x2 cm. The child
C. The administration of bicillin-5 every month regurgitates after feeding by small portions of
during 3 years milk (changed and unchanged); stool is normal.
D. The administration of bicillin-3 every month Muscle tonus is normal. What is the most probable
during a year diagnosis?
E. The administration of bicillin-3 every month A. *Pylorospasmus
during 3 years B. Meningitis
1131.2 years old child has simple dyspepsia with nausea C. Pylorostenosis
and vomiting. There are no signs of dehydration. D. Microcephaly
The prevention of pathological fluid losses E. Craniostenosis
consists in the following administration: 1136.A 1 month female infant vomits frequently “by
A. * Oral rehydration solution (ORS) fountain” irrespective of eat period and has
B. Intravenous infusion 5\% glucose in water periodically liquid stool. She is dehydrated,
solution exhausted. There is clitoris hypertrophy. What
C. Starvation during 12 hours diagnosis is the most probable?
D. Antibacterial therapy A. Adrenogenital syndrome, salt-wasting form
E. Gastric lavage B. Pylorostenosis
1132.The child aged 12 yr complains of cramping pain C. Upper intestinal ileus
in the right hypochondrium which is easily D. True hermaphroditism
controlled with antyspasmotic preparations. E. Acute intestinal infection
During attacks nausea and less often vomiting 1137.A 2 year boy has subfebrile temperature, dry,
occur. Palpation of the abdomen reveals persistent, prolonged, attacked cough, frequent
tenderness in the gallbladder projection point. The breathing with hindered exhalation. The breathing
liver is not enlarged. What additional method of under auscultation is harsh, there are diffuse dry
clinical investigation is the most informative in sibilant rales. X-ray lung examination shows
this case? increased transparency. There is leukopenia in
A. *Ultrasound examination blood. What diagnosis is the most probable?
B. Esophagogastroduedenoendoscopy A. *Obstructive bronchitis
C. CBC (complete blood count) B. Pertussis
D. X-Ray upper Gastrointestinal (GI) series C. Pneumonia
E. Coprologic examination D. Rhinitis
1133.10-year old girl was admitted to the department E. Bronchiolitis
with symptoms of carditis. Well known that the 1138.A 1,5 year old boy has non-productive cough with
exacerbation of chronic tonsillitis was occurred 2 purulent sputum, dyspnea, retardation in physical
weeks ago. What etiological trigger of carditis is development, polyfecalia, increasing of sweat
the most possible in this case? chloride up to 150 mEq/l. The cystic fibrosis was
A. *streptococcus diagnosed. What treatment will you prescribe?
B. staphylococcus A. *Enzymes + antibiotics
C. pneumococcus B. Choleretics + adaptogens
D. klebsiele C. Н2-histaminic blockaders + hepatoprotectors
E. proteus D. Vitamins + antibiotics
1134.A 11 month female has poor appetite, stool with E. Vitamins + mucolytics
plenty of faeces, periodically vomiting after 1139.A 2 year old girl has symptoms of cystic fibrosis:
introduction of solid food during last months. relapsed pneumonia, secretion of a plenty purulent
Temperature is normal. Bodyweight is 7 kg. On green sputum, dyspnea, polifecalia, retardation in
physical exam there are pale, oedema on legs, physical development, increasing of sweat
enlargement of abdomen. Coprogram shows a lot chloride up to 120 mEq/l. What is the most
of greasy acids. The diagnosis of celiac disease suitable method of diagnosis?
was made, the aglutenic diet was prescribed. What A. *Direct detection of a gene by polymerase
is necessary to exclude from child diet? chain reaction
A. *Cereals B. Complete blood count
B. Milk products C. Karyotype
C. Meat D. Determination of sex chromatin
D. Egg E. Family tree
KROK 2 – Question Bank 118
1140.A 1,5 year old boy with poor activity does not ratio between the actual and average expected
walking, does not talking. Physical examination weight for his age is 14\%. The constipation and
demonstrates pale and dry skin, оedema, large little amount of stools with undigested bits are
tongue, saddle nose, gruff voice. His hairs are noted. The child is in the breast feeding. The total
thick and rough. Large fontanel - 3,0x 3,0 cm, day breast milk volume is not known. There is no
teeth are absent. What diagnosis is the most evidence of infection in this case. What is the most
probable? probable diagnosis:
A. *Hypothyroidism A. *Hyponutrition in development
B. Down’s syndrome B. Mucoviscidosis, intestinal form
C. Rickets C. Acute gastroenteritis
D. Growth hormone deficiency D. Chronic gastroduodenitis
E. Diabetes mellitus E. Dehydration
1141.Boy is 3 years old. Complaints: dyspnoe, fast 1145.What kind of breathing can be heard above the
tiredness, frequent episodes of respiratory diseases lungs in healthy children aged 7 years?
in history. Borders of relative heart dullness are A. *Vesicular
extended to the left, strengthening of the 2nd heart B. Puerile
sound in the 2nd intercostal interspace on the left C. Weakened vesicular
side, hard systolo-dyastolic murmur in the second D. Coarse
intercostal interspace on the left side and above E. Grunting
the clavicle (“machine noise”), which is conducted 1146.The girl is 5 years old. She has been hospitalised
on interscapular interspace. What is the most with the complaints on generalized oedema (face,
probable diagnosis? abdomen, lower extremities), reduce of daily urine
A. Open arterial duct volume up to 300 ml, loss of appetite. In urinalysis
B. aortal stenosis – protein – 3 g/l, RBC – 1-2, WBC – 6-7, specific
C. atrial septal defect gravity – 1027. CBC – Hb – 110 g/l, WBC – 10
D. ventricle septal defect G/l. What diagnosis is most probable?
E. Isolated stenosis of arteria pulmonalis A. *Glomerulonephritis with nephrotic syndrome
1142.The infant aged 3 days become jaundice. The B. Interstitial nephritis
course of pregnancy and delivery is without any C. Pyelonephritis
complications. The child was born with 3300 g of D. Polycystic kidney disease
body weight and 8 Apgar score. The general E. Glomerulonephritis with nephritic syndrome
condition of newborn is good. The infant blood 1147.The girl is 11 years old. She complains on pain in
group is O (I) Rh(-)ve, the mother’s blood group abdomen and lumbar region, headache, increase of
is A (II) Rh (+)ve. What is the most probable temperature up to 39 C, vomiting, reduce of
cause of this jaundice? appetite. Skin is pale, hot, respiration is normal,
A. * Physioligic jaundice heart rate – 100 per minute, positive Pasternatsky
B. Infant`s hemolytic caused by Rh – symptom. CBC: RBC – 3,9 T/l, WBC - 18 G/l,
incompatibility ESR-34 mm/hour. Urinalysis - protein 0,066 g/l,
C. Infant`s hemolytic anemia caused by ABO – WBC – 30-40, RBC – 1-2, bacteria - a lot of. Most
incompatibility probable diagnosis is?
D. Obstructive jaundice A. *Acute pyelonephritis
E. Jaundice due to septicemia B. Acute glomerulonephritis
1143.The girl of 11 years old. She is ill for 1 month. C. Cystitis
She has "butterfly"-type rash on face (spots and D. paranephritis
papules), pain and swelling of small joints on arms E. Polycystic kidneys disease
and legs, signs of stomatitis (small-sized ulcers in 1148.The girl of 12 years old is suffering from acute
mouth). CBC - Нb – 80 g/l, RBC – 2,9 T/l, WBC glomerulonephritis. Daily diuresis is 700 ml, heart
– 15 G/l, ESR - 40 mm/hour. Urinalysis - protein rate – 100 per minute, blood pressure – 130/95,
– 0,33 g/l. What is the most probable diagnosis? temperature – 37,3 C. In urinalysis – RBC – 30-
A. *systemic lupus erythematodes 40, WBC – 5-6, protein – 0,099 g/l. Which
B. juvenile rheumatoid arthritis, systemic type medicines should be prescribed in this case?
C. periarteriitis nodosa A. *Antibiotic, hypotensive, diuretic, vitamin C
D. Acute rheumatic fever B. Antibiotics + prednisone
E. dermatomyositis C. Antihistaminic + vitamins
1144.The infant aged 2 months complaints of D. Antiaggregants + diuretics
restlessness, subcutaneous fat wasting and E. Sulfanilamids + cytostatics
underweight. The deficiency calculated from the
KROK 2 – Question Bank 119
1149.The girl is 12 years old. Yesterday she was B. Elevation of blood pressure
overcooled. Now she complains on pain in C. Hypokalemia
suprapubic area, frequent painful urination by D. Osteoporosis
small portions, temperature is 37,8 C. Pasternatsky E. Cushingoid syndrome
symptom is negative. Urinalysis – protein - 0,033 1154.1,5 - years old infant was inoculated with live oral
g/l, WBC – 20-25, RBC – 1-2. What diagnosis is polio vaccine (OPV). On the 16th day after being
most probable? asymptomatic the child suddenly developed the
A. *Acute cystitis right low limb weakness and stopped to walk.
B. Dysmetabolic nephropathy How to comment this event?
C. Acute glomerulonephritis A. *Vaccine - associated poliomyelitis
D. Acute pyelonephritis B. Poliomyelitis
E. Urolithiasis C. Guillain - Barre syndrome
1150.The polycystic kidney disease was diagnosed at D. Landry syndrome
the boy of 3 years old. Mother complaints that the E. Polyradiculoneuropathy
boy has growth retardation, poor appetite, 1155.The child aged 3 years was admitted to the
vomiting. Skin is pale, turgor of soft tissues is pediatric department on the 3d day of illness. The
reduced, heart rate - 120 per minute, harsh general condition is moderate. Body temperature
breathing at auscultation, abdomen is enlarged, is 38,5oC. One time of vomiting, skin pallor, lost
soft. Biochemical tests – urea – 14 mmol/l, skin turgor, spasmodic colon descendens are
creatinine – 0,130 mmol/l, protein – 58 g/l. presented. The patient has stools with mucus and
Which condition has been developed? blood 9-10 times per day. What is the initial
A. *Chronic renal failure diagnosis?
B. Acute renal failure A. * Acute bacterial dysentery (Shigellesis)
C. Encephalopathy B. Enteric fever (Salmonellosis)
D. Interstitial nephritis C. Rotavirus gastroenteritis
E. Pyelonephritis D. Yersiniosis
1151.The boy of 9 years old. He is ill for 3 days. He has E. Escherichiosis
complaints on pain and restriction of movements
in right knee and left elbow joints, dyspnoea. He
was suffered from acute tonsillitis 2 weeks ago.
There are fever (38,5 C), oedema of joints,
extension of the borders of cordial dullness on 2
cm left, HR - 110 per 1 min, weakness of 1st
sound, "soft" systolic murmur on an apex. What
diagnosis should be suspected?
A. *Acute rheumatic fever
B. systemic lupus erythematodes
C. juvenile rheumatoid arthritis
D. Reiter’s disease
E. reactive arthritis
1152.The 7 years old boy developed an asphyxic attack,
expiratory wheezing and cough. In past history the
child has had relapsing rhinitis and red eyes
syndrome. His sister suffers with atopic
dermatitis. The correct diagnosis is:
A. * bronchial asthma;
B. acute bronchitis;
C. viral croup;
D. acute obstructive bronchitis;
E. pneumonia.
1153.The course of tonsillar diphtheria in a 2-years old
child was complicated with appearance of early
myocarditis. Prednisolone in a dose of 1,5 mg/kg
was introduced into therapy . Which one from
expected complications is not related with
prolonged glucocorticoid therapy?
A. *Hyperthermic reaction

You might also like